You are on page 1of 377
BAG CT oir ~ PGS. TS. VU DUONG THUY (Chi bién) - ThS. NGUYEN VAN NHO 40 NAM OLYMPIC TOAN HOC QUOC TE (1959 - 2000) (Tai ban lan. thit ba - c6 chinh li va b6 sung) DALHOC THAI NGu ves (eeceiene LIED. NHA XUAT BAN GIAO DUC Léi noi dau Ki thi Olympic Toan hoc Quéc té (IMO) dus t6 chic lan ddu tien tai Romania vao nam 1959. Ti d6 dén nay da té chtic duge 40 ki thi do nhiéu nude trén thé gidi ding cai. Chat lung va quy mé ki thi IMO ngay cing tng da gop phan tac dong tich cuc dén phong trao béi duéng hoc sinh gidi ton & tiing quéc gia riéng biét. Nudc ta da tham gia ki thi IMO lan dau tién vao nam 1974, dat thanh tich cao ngay ti ki thi dy va tit dé dén nay luén luén duoc ban bé Quéc té danh gid 1A mét trong nhung doan manh cua cac ki thi IMO. Dé tim hiéu vé cdc ki thi Todn Quéc té, ban doc c6 thé doc them mot sé tai ligu tiéu biéu sau: 1. Cae dé thi v6 dich todin cée nude (sach dich ctia cdc tac gid Lién X6, NXBGD xuat ban nim 1996, 2 tap). Sach gém cde bai thi v6 dich cua 19 nude, trong do cé Viét Nam. 2. Nhang bai thi hoc sinh gidi cde nuée va Quéc té, tap thé tac gid truéng DHSP Ha Néi I, GS. Doan Quynh va GS. Hoang Xuan Sinh cha bién. Sach tap hop mét sé dé thi cia cde nude, ngodi ra c6 gidi thigu cde dé thi du bi 6 IMO (xem muc 3), nhung khéng cé loi gidi. 3. Thi v6 dich todn Quéc té, Lé Hai Chau, NXB TPHCM, 1992. Sach gom cac dé thi cé kem theo Idi gidi cia cdc ki thi IMO tit 1981 dén 1991. Ngoai ra, ban doc cé thé tham khdo thém trén Internet - Amazon. 1. Otago, Booklets 1 - 15. The Tool-Chest, Australian Mathematical Olympiad Committee, 1989. 2. D. Shklaisky, N. Chentzov, I. Yaglom, The USSR Olympiad Problem Book, W.H. Freeman and Co., 1962. 3. Samuel L. Greitzer, International Mathematical Olympiads, 1959- 1977, in New mathematical library 27, Mathematical Association of America, Washington, 1978. 4, Samuel L. Greitzer (ed.), International Mathematical Olympiads, 1955-1977. MAA, 1979. 5, Murray S. Klamkin (ed.) International Mathematical Olympiads, 1978-1985, and Forty Supplementary Problems. MAA, 1986. Cuén sach 40 ném Olympic todn hoe Quéc té nay dude chung téi phan thanh 2 phan. Phdn 1 gém tt cd nhiing bai toan vé Hinh hoc (trit mét sé bai Hinh hoc té hgp chung téi dé lai cho phan sau) da ra trong cdc ki thi IMO tit nim 1959 dén nam 2000 (dé 1a giai doan 41 nam nhung nim 1980 khéng té chic), Phdn 2 véi sé lugng trang nhiéu hon, gom cac van dé con lai. Ngoai ra, 4 cudi méi phan con cé thém phan phu luc, gsm mét sé tu ligu vé ki thi IMO va mét sé dé thi v6 dich todn cia mot sé nude. Tai ligu tham khéo chinh cia ching téi 1a cdc Website trén Internet, két hop véi nhitng tdi ligu dé néu trén cing véi mét 86 tdi Lié khdc c lién quan. Nhan day, cho phép ching ti ngé Idi cam on TS. Nguyén Xuan My, khoa Toén - Co - Tin trutng DHKHTN thuéc Dai hoc QGHN, da cung c4p cho chung tdi bén tap tai liéu bing tiéng Anh nhan dé South Africa and the 33rd (34, 35, 36th) International Mathematical Olympiad. Cac tai ligu nay dang tai cdc ki thi tuyén chon hoc sinh gidi Quéc gia Nam Phi, cé kem theo nhing dé thi IMO 1992, 1993, 1994, 1995 vdi léi giai that dep cua nhiéu nha toan hoc. Chuing tdi hi vong cuén sdch nay sé dem dén cho ban doc yéu todn mét sé théng tin bé ich vé cdc ki thi IMO, cing nhu khoi day hing thu, nhiét tinh hoc tap va nghién cit Todn hoc etia cac ban. Ching téi xin don nhan va hoan nghénh moi ¥ kién x4y dung ciia ban doc dé lin tai ban tdi cudn sach sé tét hon. Moi gép y xin vui long chuyén dén: Ban Bién tgp - Chi nhdnh NXBGD tgi Da Nang 15. Nguyén Chi Thanh - Da Ning. Thang tu, nam 2001 PGS. TS. VU DUONG THUY ThS. NGUYEN VAN NHO LorNér DAU (COA BAN IN LAN THU HAD Nhan dip bé sich 40 NAM OLYMPIC TOAN HOC QUOC TE dugc tai ban, ching téi xin chan thanh cam on ban doc gan xa da thuc su quan tam va chuyén dén cho ching téi nhiéu nhan xét hitu ich. Trong lan tai ban nay, chung téi cho trich ding nguyén ban tiéng Anh kém theo méi bai toan thudc céc dé thi IMO. Viée ding song ngit Viét - Anh cho cac bai toan sé giup ban doc tién Igi trong viéc déi chiéu, so sdnh véi cac tai ligu tham khdo géc va kich thich ban doc trau dai thém ngoai ngit cia b6 mén Todn dé chudn bi tham dy cdc ki thi Quéc té. Ngoai ra, trong lin tai ban nay, 40 ndm Olympic Todn hoc Quéc té tap 1 va tap 2 duoc nhap lai thanh mét t4p duy nhat. Thang hai, ném 2002 LOLNéI DAU (COA BAN IN LAN THU BA) C&c tac gia xin chan thanh cam on ban doc yéu toan va quy thay cé gido 6 cdc trugng THPT chuyén toan trong cd nuéc da néng nhiét don nhan tap sdch cing nhu c6 nhiéu y kién déng gop dé tap séch due hoan thién hon trong mi ln in sau. Trong lan in thit ba nay, chung téi cé chinh li va bs sung thém 20 trang so véi lan in thit hai, trong dé cé dé thi va huéng dan gidi IMO 2001 tai Mi va IMO 2002 tai Anh. Thang mudi hai, ném 2002 PGS. TS. VU DUONG THUY ThS. NGUYEN VAN NHO , GIGI THIEU SG LUGC _ KI THI OLYMPIC TOAN HOC QUOC TE (Cae thing tin dudi diy duge trich hige tit "Tong quan vé THO + Overview of the IMO" ding trong Website cita hji Toan hye Canada, nude chit nha ding cai IMO nim 1985) Ki thi Olympic Todn hoc Quéc té (viét tat 1a IMO: International Mathematics Olympiad) duoc té chite hang nam cho hoc sinh cuéi bac phé théng. Vao nim 1959, tai Romania, lan dau tién IMO duge té chitc, danh déu mét bude phat trién tir cuéc thi mang tén Olympic khoa hoc Quéc té (International Science Olympiad) ma vao Vic dé Toan hoc chi 1a mét bd mén trong nhiéu mén thi. Thuc chat, trong lan té chic dau tién nay, IMO dugc hiéu nhu 1a cuéc tranh tai cla cdc hgc sinh xuat sic vé toan é chau Au. Mét doan dai biéu dén du IMO cé nhiéu hit 1a 8 ngudi, gm 6 thi sinh va 2 truéng phé doan. Trong 2 ngay thi lién tiép, cdc thi sinh sé lam hai bai thi, méi bai gdm 3 bai todn, mdi bai duge 7 diém téi da. Véi théi han khéng qué 4 thang, méi nuéc dude mdi sé gdi dén nhiéu nhit 1a 6 bai todn du tuyén. Tat cd sé dude tuyén chon thanh mét tap gdm khoang chimg 30 bai bai Ban Tuyén chon dé thi IMO cia nuéc chi nha (host country's competitions committee), va sé dude dé trinh lén Ban Giam khdo Quéc té (International Jury). Ban Gidm khdo Quéc té sé cé cudc hop truéc ki thi vai ngay nhim tuyén chon ra 6 bai ton cuéi cing lam dé thi. Tai day, ngudi ta sit dung 5 ngén ngif chinh 1a Anh, Phap, Diic, Nga va Tay Ban Nha. Nhiing nam gin day, nguti ta sit dung tiéng Anh 1a chinh, cdc thit tiéng 6 khac sé duge phién dich sang mdi khi cé nhu cdu. Riéng cac bai thi dude dich ra ting thd tiéng cia mai nuéc tham gia duéi su kiém soat cia Ban Giam khao Quéc té. Céc thi sinh t4p trung vé nuéc chi nha truéc dé vai hom dé chun bj tinh than va lam quen véi khi hau. Ho cn phai cé site khoé dé trai qua 2 ngay thi cing thing lién tiép nhau, mdi ngay 4 gid rudi khong gidi lao. Thong thudng, bai ton 1 dé nhdt va bai todn 6 thi kho nhat. Sau khi thi, trong lic Ban Giam khao chdm bai va chudn bi cong bé két qua (thong thuéng khoang 3 ngay), dy 1a thdi gian cdc thi sinh cam thay nhe nhang thoai mai véi nhitng chuong trinh gidi tri va giao leu van hod. Vé viée chdm diém, truéc hét, mdi Trung doan sé cham diém bai thi cho cac thi sinh cua nuéc minh, nhung khéng duge danh déu hodc viét gi lén cae bai thi dy. Sau d6, cdc Trudng doan sé lam viée véi Ban Cham diém (coordinators - Ban nay dutge nuée chi: nha chi dink) cing véi cdc truéng doan khac. Néu théng nhat y kién, cd hai phia cing ki vao va dé chinh 1a diém chinh thtic. Néu cé tranh cai khéng di dén thoa thudn dug thi vi Truéng Ban (Chief Coordinator) sé nd lic tim cdch hoa gidi. Trong trudng hgp vi nay cing bat luc, moi viée sé dug dé trinh lén Ban Gidm khdo Quéc té va thuéng thi ngudi ta sé gidi quyét bing mot cuéc bé phiéu. IMO don thuan 1a mét cuéc tranh tai cd nhan, né chua hé duoc tuyér. bé chinh thic ring dé 1a cuéc tranh tai dong déi. Tuy vay, mot sé doan da ty minh céng diém cho doan minh va cdc doan khac va cong bé thit hang toan déi khi vé nuéc. Néu cé mét ai dé néi ring nude A dimg thit 6 trong n quéc gia tham dy thi dé chi la danh gid cia cd nhan ho, chua bao gid dé 1a tin chinh thitc dugc céng nhan béi Ban Té chic IMO. Cac gidi thuéng chiém khéng qua nia sé thi sinh tham du, duoc chia lam 3 hang 1a huy chuong vang, bac va ding theo ti lé 1. 2: 3. Sé huy chuong vang chiém khong qué 1/12 téng sé thi sinh dy thi. Dé khuyén khich, ngoai cdc thi sinh nhan duc huy chuong, thi sinh nao dat diém téi da cho it nhAt mét bai todn sé duse cdp bing Tuyén duong danh du (Certificate of Honourable Mention) khéng kém theo huy chuong. Theo truyén théng, IMO két thc bing hai sy kién 1én 1a Jé trao gidi va budi tiée chia tay. Cac thi sinh gidi nhat sé tan huéng niém vinh quang va hanh phtic cia minh, va ho xtmg dang duce nhu vay, khi nhan huy chugng truéc déng dao moi ngudi. Cén buéi tiée sau cing, tét cd tran day tinh hitu nghi, bao trim lén dé 14 mét bau khéng khi vui tuoi vuct hin lén nhéng bién gidi chinh tri, ton gido, quéc gia. Thé gidi lung linh va mai mai tudi dep nay chi cn dong lai 6 nhing ban tay n4m chat nhing ban tay cla nhing nha Todn hoc da truéng thanh, dang phat trién cua hién tai ho&c tuong lai. Phiin 4 CAC BAI TOAN HINH HOC 10 KIEN THOC BO TRO Nhiing kién thie bé tré dude trinh bay dudi day bao gom cde khai nigm va dinh li ma da so khdng c6 mat trong chudng trinh THPT hién hank. Ching thue sv cin thiét cho nhing hoe sinh gidi todn d phé théng. Mot phn kién thie bé tra sé duve sit dung trie tiép trong sich nay, so cin lai li nhing dinh Ii todn hoe nai tiéng ma vige ching minh ching cing da la mét bai ton Ikhé d6i véi bie phd thing. Céc tée gid trude day da bi qua phin ching minh vi ¥ dé sit pham hode muc tiéu sit dung sich. day, ening tdi cd ging dua vio phin ching minh, nhung di nhién Khong chi tiét 6 da sé triding hop. Thue ra, moi diéu tu nd da ro rng va 06 thé né sé dude hoin thién chi tiét déi véi nhiing ban doc nio quan tim mét eich nghiém tic, Dinh nghia 1 Trong khéng gian, cho trong hé truc toa dé truc chudn hai vector a(m,n, p) va 5(q,r,s).Ta néi tich hitu huéng cia ching 1a vector ¢, ki hiéu [2,5], xdc dinh béi: ¢ =(ns— pr, pq-ms,mr—qn). Tinh chdt 1.1 Néu tam gidc ABC trong khéng gian cé céc dinh cé thanh phan toa dé nguyén thi tich hitu huéng cia c&p vector AB va AC cing cé thanh phan toa a6 nguyén. Chiing minh: Vi AB va AC cing cé thanh phan toa dé nguyén nén tit Dinh nghia 1 ta suy ra diéu phai ching minh. Tinh chdt 1.2 Dién tich tam gidc ABC la: S = 5 |(AB. AC]. Ching minh: Xem Hinh hoc 12, ban KHTN, tai liéu gido khoa thi diém, NXBGD, 1995, p. 61. Dinh nghia 2 Hai da gidc A,A,....A, va B,B,...B, dude goi la déng dang nhau néu ching c6 cac géc 6 cdc dinh A,, A,, ... A, bing cac géc tudng ting & céc dinh B,, B,,..., B, , dong thii: AjAz _ AoA3 _ _ AnAt BB, BB; _ B,B, Sé k dug goi 1a ti ding dang ciia hai da gidc. ‘Ti Dinh nghia 2, dé dang chimg minh duge: Tinh chét 2.1 Déi vii hai da gidc déng dang, ti cdc dung chéo tusng img bing ti ding dang. Hon nija, néu hai da gidc dy ngoai tiép thi ti s6 cdc ban kinh dudng tron néi tiép cing bing ti ding dang. Nguyén téc 3 (Nguyén tdc Dirichlet) Khong thé nhét n con thé vao m ling, véi n > m, sao cho mii ling chi chia mét con. ' Chi §: Cé nhiéu cdch phat biéu khdc nhau, nhung phat biéu trén =k. 11 la phé bién nhat. Ching han, trong chung trinh béi dung déi tuyén hoc sinh thi V6 dich Toan Quéc té, gido su Greg Gamble goi dé 1a Nguyén li Lé chuéng bé céu (Pigeon - Hole Principle: If 5 pegeons fly into 4 pigeon - holes then at least one pigeon - hole contains 2 or more pigeons). Nguyén tac nay thoat tréng don gidn nhung lai cé nhiéu img dung trong lap luan gidi ton. Trong Hinh hoc, ta cé nhing ménh dé tuong tg, ching han: Ménh dé : Néu trén doan thing cé dé dai I ta dat mét sé doan thing cé téng dé dai Ién hon / thi it nhat hai trong sé nhiing doan thing 46 phai cé diém chung. Dinh nghia 4 (trong tam cuia mét hé diém) Cho n bé (A;,m;), véii = 1, 2,..., n, trong dé A; 1a céc diém con m; la nhing sé thuc dugng. Ta néi trong tam cia hé n b6 (A;,m;) 1a mét diém T sao cho: m, TA +myTA3 +...+m,TA, =0. (Cé thé hiéu m, 1a cdc trong luong dat vao vi tri A. Khin = 3 va m, = mz = m; =1, ta gap lai khai niém trong tam cua mét tam gidc). Dinh nghia nay hop li, vi ta cé: Tinh chdt 4.1 ‘Véi moi n bé nhu da néi trén, trong tam luén luén tén tai va duy nhat. Chiing minh: Chon O 1a diém cé dinh trong mai phing, géc toa dé ching han, dé thay: my TA, +m TA] +..¢m, TA, 0 => (m, +m, +...+ m,)TO + mOA; +m,OA, +...+m,OA, =0 oT m +m, Nhu thé, T tén tai va duy nhit. Dinh nghia 5 Trong mat phing cho n diém. Ta néi bao [di céa hé n diém nay 1a da gidc 13i nhé nhdt chiia tat cd cdc diém dé, nghia 1a da gidc nay khong 12 chifa bat cut mét da gidc léi nao khac cing véi tinh chit dé. (Ta néi mét da gidc 1a [di néu kéo dai mét canh bat ki thi né sé khéng c&t bat cit canh nao khac). Chiu §: Khai niém hinh hoc nay duoc mé réng thanh khéi niém bao Iéi trong Giai tich hién dai, cu thé la 6 nganh Gidi tich 1éi. Ban doc c6 thé chimg minh duge ring bao léi cia mt hé hiu han diém luén luén tén tai va duy nhat. Cédec tinh chdt sau déy noi vé dudng trung tuyén cia mét tam gide. Tinh chét 6.1 Néu AM a trung tuyén cia tam gidc ABC thi AM < S(AB + AC). Chiing minh: Goi D 1a dinh thi tu cia hinh binh hanh BACD, dé ¥ AD = 2AM, ding bat ding thiic tam gic trong tam gidc ABD dé suy ra két qua. Tinh chét 6.2 Trong mét tam gidc, trung tuyén img véi canh dai hon thi sé ngan hon trung tuyén img véi canh ngan hon. Chung minh: Goi AM, BN 1a hai dudng trung tuyén cia tam gidc ABC, khi d6, 4p dung Dinh Ki cosin cho hai tam gidc ABM va ACM ta dé dang ching minh duge: AM? = 4 2b? 2c? —a?), BN? = Fa? +20? 6%), Tid: AM?- BN? = ia-oy >0, suy ra dpem. Tinh chdt 6.3 Didu kién At cé va dé dé tam gidc ABC c6 géc A nhon la trung tuyén AM Ién hon mét nia canh BC. Chiing minh: iBc = BM = CM thi Dinh Ii lin quan dén goc va canh déi cho ta: BAM > ABM va CAM > ACM. Ti d6 A = BAM + CAM >B+C, hay A > 90°. Néu AM < 13 ‘Tuong tu, néu AM > 5 BC thi tacd A < 90°. Tinh chét 7 Cho tam gidc ABC, mot dudng thing ct hai canh AB, AC 6 A’ va B' tung ting. Lic do: Srcy _ AB.AC' Syyc ABLAC * Chung minh: Ding cong thie $=} AB.ACsinA . Tinh chat 8 V6i S 1a dién tich mét tam gidc, R 1a ban kinh vong tron ngoai tiép, a, b, ¢ 1a d6 dai ba canh, ta 6: abc = 4SR. Chiing minh: ‘That vay, goi O 1a tam vong tron. Kéo dai AO gap ving tron tai K. Cho AH 1A duding cao cia tam gidc ABC. Khi dé dé thay hai tam gide ABH va AKC dong dang. Suy ra: AB_ AK c _2R BS SS hay “==. AH AC AH 6 Tudo abe = 2R.a.AH = 4SR. Dinh li 9 (Dinh li Sti-oa-to) Véi M 1a diém tuy ¥ trén canh BC cia tam gidc ABC, ta cé hé thie: AB? MC + AC?.BM - AM?.BC = BC.MC.BM. Chuing minh: Ké duéng cao AH cia tam gide ABC, hay sit dung hé thuc luong trong cde tam gide vuéng AHB, ACH va AMH dé ching minh ring AM? = MH? + AH?. Tit do: AB?.MC + AC?.BM - AM?.BC = (BM? +MH? - 2BM.MH).MC + (CM? - 2CM.MH +MH?).BM + AH?.MC + AH®.BM - MH?.BC - AH?.BC = BM2.MC + CM?.BM + MH?.(MC + MB) + AH®.(BM + MC) - MH?.BC - AH?.BC = BM.MC.(BM + MC) = BM.MC.BC. Dinh li 10 (Dinh li Céva) Goi E, F, G la ba diém tuong tig ndm trén céc canh BC, CA, AB 14 cia tam gidc ABC. Lic dé, ba dung thing AE, BF, CG cat nhau tai mot diém O khi va chi khi: AoE w GB’ ECFA Chuing minh: Phén thudn: Tit A va B, ké cdc dung song song véi BF, chting lan lugt cit CG va AE tai K, L tuong img. . CF_CO . LC_CO Ta co: —=—— va —~=—— FA OD AK OD (st dung Dinh li Thalés va cac tam gidc déng dang AKO, LCO). Tu dé ta duge: cate FA AK Céc c&p tam gidc ding dang LEC va OEB, AKG va BOG lai cho BE_ | 4 AS _ AK _ ca” ° BG ~ BO" AG BE CF_ AK BO CL Do dé: — =. GB EC’FA BO'CL “AK Phén dao: Gia sit ta 06 a a CE =1. Qua giao dibm oda ode dung thing AE va BF, ta ké dudng thing CC,, vdi C, nim trén canh AB. Khi dé, theo ching minh 3 phan thuan ta cé: AC, BE CF_,_AG BE CF C\B'EC'FA GB ECFA AC, _AG suy ra: GB Gs" , hay C, =G, ta cé diéu phai chimg minh. Chi 5: Dinh li Céva thudng duge sit dung dé chiing minh cdc dusng thang déng quy. Dinh li 11 (Dinh li Ménélaus) Cho tam gidc ABC. Mot dutng thing (d) bat ki c&t cac duéng thing BC, CA, AB lan luot tai P, Q, R. Khi do: 15 BI zl iz 3! @ AR QC BP Dao lai, gid sit cdc diém P, Q, R tuong img nim trén céc duing thing BC, CA, AB ciia tam gidc ABC sao cho (1) dugc thoa. Lic dé, P, Q, R thing hang. Chitng minh: @ Phdn thuén: Qua A, ké duéng thing song song véi BC, cit duéng thang (d) tai L. Sit dung R cdc hé thitc cba nhiing tam gidc ding dang KS .. dé ching minh: ee AL_AQ gp -CPAQ BR _BP _, BR AL_,_ cP CQ CQ.” AR AL AR BP Thay AL 6 dang thic thir nhat vao ding thitc thif hai ta duge diéu phai chimg minh. Phén dao: Gia st (1) due thoa. a C, la giao diém cia dutng thing QP va AB. Ta can ching minh C, = Bee tacé: BC, AQ PC BR AQ PC =l==—.= =. Dé chimg minh C, =R, ta dé ¥ ring phuong trinh Be ee) aX c6 khéng qué mét nghiém khi A B. That vay, chon géc toa dé la A, truc toa dé AB véi chiéu duong la chiéu tit A dén B. Cho toa dé cia X la x, toa 6 diém A laa, hic d6 phuong trinh trén tré thanh: x ma —EmMex= . x-a m-1 Tit dé ta 6 diéu phai ching minh. Chit §: Dinh li Ménélaus thuang duge sit dung dé ching minh tinh thing 16 hang cua cdc giao diém ciia mét sé duéng thang. Dinh li 12 (Dinh li vé duéng thang Simson) Ti mét diém P trén ving tron ngoai tiép tam gide ABC, ta lin lugt ha cdc duéng vung géc xuéng BC, CA, AB, ching tudng img gap BC, CA, AB tai A,, B,, C,. Khi dé, cdc diém Ay, B,, C, thing hang, va duéng thing tao béi 3 diém nay dusc goi la duéng thdng Simson. Ching minh: Khong mit tinh téng quat, gid st P nim trén cung AC cia vong tron ngoai tiép tam giac ABC. Vi: ABC + APC= 180°, ABC + A,PC, = 180° nén APC = A,PC,. Tid ta suy ra APC, = A,PC, va mét trong cdc diém A,, C, nim trén canh cia tam gidc, diém con lai nim trén phan kéo dai cia canh. Cac diém B, va A, nim trén ving tron dudng kinh CP, do dé A,B,C = A,PC. Twong ty AB,C, = APC,. Vi APC, = A,PC nén suy ra A,B,C = AB,C, , tic la A,, B,, C, thing hang. Dinh li 13 (Dinh li Ptolémé) Véi mét ti gide ndi tiép, tich cdc dudng chéo bing téng cia hai tich cdc canh déi. A ——=-— Cl MD AB Cing dé dang ching minh ring hai tam 7 gide BCM va ACD déng dang, do dé ta cé: BC _AC - BC.AD=AC.BM. BM AD Cong cac ding thitc da nhan dugc ta di dén: CD.AB+BC.AD = AC.BM+CA.MD = AC.BD, 17 olatocadce la diéu phai ching minh. Dinh li 14 (vé ti thé tich) Cho tit dién S.ABCD cé thé tich V. Mot mat phing tay y cdt 3 canh SA, SB, SC tuang ting tai cdc diém A’, B’, C’. Goi V’ 1a thé tich tit dign S.A'BC’ Laic dé ta 06: @ = SA°SB'SC’ V SASBSC Ching minh: ; Goi a 1a géc gitta SC va mat phdng (SAB), hién c nhién day cing 1a géc giita SC’ va mat phang (SA'B); goi f 1a géc dinh S ciia 3 1a géc dinh S cia Ny tam gidc SAB, day cing 1a géc dinh S cia tam gidc A Cc SA'B’ Dé thay: ¥ V =SCsina.di(SAB) , V'=SC'sina.di(SA'B'). Sit dung céc cong thitc: di(SAB) = 5SA8Bsin B, d(SA'B') = SSA‘ SB'sin B. ban doc dé dang di dén diéu cdn chiimg minh. Dinh li 15 (Dinh li Ménélaus trong khéng gian) Trén cdc canh AB, BC, CD, DA tao béi 4 diém A, B, C, D trong khéng gian, ta lan lugt ldy cdc diém M, N, P, Q. Didu kién At c6 va du dé M,N, P, Q déng phing la: MA qd) Chiing minh: * Gid su M, N, P, Q déng phang, ching xc lap mét mat phdng a. TitA, B, C, D, ta lan luot dung cdc mat phing song song véi a. Mat dudng thing A tudng ing cit cdc mt phing vita dung tai cdc diém A’, B,C’, D'va A cit mat phing a tai O. Dinh li Thales trong khéng gian cho ta: MA _OA’ NB_OB' PC_OC QD_OD' MB PD OD'’ QA OA” MB OB'’ NC OC’ ‘PD QA OB OC’ OD’ OA’ * Dao lai, gid sit ta c6: otanocacct a aw MB ‘NC PD QA Xét mat phng (MNP), gid sir mat phdng nay c&t canh DA tai Q’. Theo chimg minh é phan thuan ta cé: MANB PC QD MB NC PD QA Tw (1) va (2) ta 06: (2) QD _@w W Qa” Hé thiic nay chiing t6 ring D' tring D (xem ching minh 6 Dinh li 11); néi cach khac, 4 diém M, N, P, Q dng phang. Dinh li 16 (Dinh li Céva trong khéng gian) Bén trong ti dién ABCD, ldy tay y mét diém S. Cac mat phing (SCD), (SDA), (SAB), (SCB) lin lust cét AB, BC, CD, DA tai cdc diém tuong ting A’, B’, C’, D’. Laie dé: Déo Iai, goi A’, B', C’, D' la cdc diém tuong ting nim trén cdc canh AB, BC, CD, DA cia tit dién ABCD. Khi dé, 4 mat phing (ABC’), (BCD), Ching minh: Dinh li thuén: Kéo dai AS, cit mat phing (BCD) tai S'; DS’ cit BC tai B’, BS’ ct DC tai C’; B' va C’ lin lust chinh Ia giao diém cia mat phdng (SAD) véi CB va (SAB) véi CD; CS' cdt BD tai M. Kéo dai CS cét MA tai N. DN va BN cat AB, AD tuang ting tai A’, D’. Hai diém nay ln luot 1a giao diém cia mat phing (SCD) véi AB va (SCB) vai DA. Dinh li Ceva (xem Dinh li 10) trong cdc tam gidc BCD va ABD cho ta: CC DM BB" 1, 165) 19 AA’ BM DD! A'B' MD ‘D’ Nhan (1) va (2) vé theo vé ta duge: (2) A'B BIC C'D D'A Dinh li ddo: Hé thtic da cho ching td ring cdc diém A’, B', C’', D' ding phdng theo Dinh li Ménélaus trong khéng gian. Tu day dé dang suy ra diéu phai ching minh. CAC DE TOAN Va HUONG DAN Gidl phan Hinh hoc - trich tir cdc ki thi IMO tir 1959 - 2000 (C6 kém theo dé thi nguyén ban tiéng Anh) 21 Bai 1. (1959) Cho truée d6 dai cia canh AC, hay dung tam gidc ABC cé géc ABC = 90°va cé trung tuyén BM théa diéu kién BM? = AB.AC . Given the length AC, construct a triangle ABC with angle ABC = 90°, and the median BM satisfying BM? = AB.AC. Huéng dan: Gia st bai todn da gidi xong, ta dung dugc tam gidéc ABC théa diéu kién dau bai. Dat AC = c (a6 dai cho truéc). Vi c 2 nén dién tich tam gidéc ABC 1a: A BM = AM=MC= Ha BH 1 AC, tacé: = AILAC _ AH 2 2 suy ra AH = ; . Tu d6 , dé dung tam gidc ABC, ta dung giao diém B cia dudng tron dudng kinh AC = c va dudng thang song song vii AC, cach AC mét doan bing a Ro rang 1a bai todn luén luén cé hai nghiém hinh véi A, C cho truée. Bai 2, (1959) Cho diém M tay ¥ nim trén doan thing AB. Dung cdc hinh vuéng AMCD va MBEF 6 cing vé mét phia déi véi AB. Cac duéng tron ngoai tiép hai hinh vudng nay (cé tam P va Q tuong ting) cit nhau tai M va N. a) Ching minh AF va BC ct nhau tai N. b) Ching minh ring duéng thing MN di ngang qua mét diém cé dinh S (Ihéng phy thugc vao M). c) Tim quy tich trung diém cia PQ khi M thay déi. An arbitrary point M is taken in the interior of the segment AB. Squares AMCD and MBEF are constructed on the same side of AB. The circles circumscribed about these squares, with centers P and Q, intersect 22 at Mand N. a) Prove that AF and BC intersect at N; b) Prove that the lines MN pass through a fixed point S (independent of M); c) Find the locus of the midpoints of the segments PQ as M varies. Huéng déi a) Ta cé: ANM = ACM = 45° va BNM = MEB = 45° (theo tinh chat géc néi tiép) nén suy ra: ANM = BNM. Do dé ANB = 90°. Mat khac, N nim trén nua dung tron dudng kinh AC nén ANC = 90°. Ti day ta suy ra CNB=90°, nghia 1a ba diém C, N, B thing hang, hay N € BC. @) Tir tinh chat tif gide ndi tiép ta ducc NFM +NBM = 180°. Ngoai ra, dé thdy hai tam gidc vuéng CMB va AMF bing nhau nén ta cé AFM = CBM =NBM. Két qua la ta cé NFM + AFM = 180°; néi cdch khac, ba diém A, F, N thing hang, hay N « AF. c) Tir (*) va (**) ta két luan AF va BC ct nhau tai N. b) vi c): Ban doc tut gidi. Bai 3. (1959) Cho hai mat phdng (P) va (Q) khéng song song nhau, diém A nim trong (P) nhung khéng thuéc (Q), diém C nim trong (Q) nhung khéng thuéc (P). Hay dung cdc diém B, D lin lt thuéc (P) va (Q) sao cho tit gidc ABCD thoa man cac diéu kién sau: ABCD 1a tit gide phing cé AB song song véi CD, AD = BC va tén tai mét vong tron tiép xtic vdi cac canh cia ABCD. The planes (P) and (Q) are not parallel. The point A lies in (P) but not (Q), and the point C lies in (Q) but not (P). Construct points B in (P) and D in (Q) such that the quadrilateral ABCD satisfies the following conditions: it lies in a plane, AB is parallel to CD, AD = BC, and a circle can be inscribed in ABCD touching the sides. Huéng dén: Gia sit bai toan da duoc giai xong, goi (L) 14 giao tuyén cia (P) va (Q). Ré rang AB va CD phai song song vdi (L), vi néu khong thé thi AB, (L) va CD déng phdng. Goi H 1a chan dung vudng gic ha tit C xuéng AB. Gia sti vong tron ni tiép ABCD tiép x1ic véi cdc canh AB, BC, CD, DA lan lust tai M, I, K, N. - (L) H BMA Do ABCD ngoai tiép mét ving trn nén tif tinh chat tiép tuyén ta suy ra dude: AH = AM+ MH=AN+CK =AN+CI=AN+ND=AD. Nhu vay, AH = AD = BC. Tu phan tich trén, ta suy ra cdch dung B va D nhut sau: Dung giao tuyén (L) ctia (P) va (Q). Qua A va C, lin luot ké cde dudng thing (D) va (D’) song song véi (L). Goi H 1a chan dudng vuéng goc ha tit C xuéng (D). Dung hai duéng trdn tam C va tam A véi ban kinh bang d6 dai AH trong mat phing ((D), (D’)). Hai dudng tron nay theo tht ty ct (D) va (D’) tai B va D. Dé chinh 1a hai diém phai dung. Chitng minh: Dé dang. Dé nghi ban doc tu chimg minh. Bién Iuén: Bai toan cé hai nghiém hinh néu CH < AH, trong 6 mét nghiém hinh thoa diéu kién AB>CD va nghiém hinh kia thoa diéu kién AB < CD. Bai 4. (1960) Cho tam gidc vuéng ABC véi canh huyén BC cé dé dai a. Chia BC thanh n phan bing nhau, véi n 1a sé nguyén duong 1é. Khi dé, tam gide ABC duc chia thanh n tam gidc nhé va tam gidc nhé 6 chinh gia c6 goc tai dinh A bing a. Goi h 14 khoang cach tix A dén BC. Ching minh rang: Ina given right triangle ABC, the hypoteneuse BC, length a, is divided into n equal parts with n an odd integer. The central part subte- -nds an angle a at A. Let h be the perpendicular distance from A to-BC. Prove that: Iga = an’ —a Huéng dan: Goi M 1a trung diém BC, P va Q la hai diém chia nim hai bén né, véi P & gn B. Khi dé ta c6 a =PAQ = QAH-PAH (khong mat tinh téng quat, ta xem nhu ca P lin Q déu nim bén phai diém H, trong trudng hop nguoc lai, nghia 1a khi P hoac Q nim bén trai H, ta quy uéc ly goc am). 50 B Ding céng thitc ugng gide ig(x- y) = 2° —"& ta ca: 1+igx.tgy QH_ PH AH__AH _ (QH-PH)AH __PQ.AH__ }4QH-PH AH?+QH.PH AH? +QH.PH” AH? Mat hac, ta lai cé: Pq=~ , AH=h,QH=MH+MQ=MH+~, n 2n Iga = PH = MH -MP =MH -7 MH = VAM? - AH? = n nén thay vao ta dude diéu phai ching minh: ah Anh ee z z 7a" walt -e- 2] ane Iga = Bai 5. (1960) Dung tam gidc ABC biét cae dé dai duéng cao ké tit A, B va do dai dudng trung tuyén ké tir A. Construct a triangle ABC given the lengths of the altitudes from Aand B and the length of the median from A. 25 A Huéng dan: Gia st bai toan da giai xong, ta dung duge tam gidc ABC cé cdc duting cao AH =h, BI = k va trung tuyén AM = m (véi h, k, m 1a cac dé dai cho truéc). Ha MX 1 AC. ViM 1a trung diém BC, ding théi MX song song véi BI nén ré rang MX = Be. B Hl MEE C Tit phan tich trén ta di dén cdch dung: Ta bat dau bing cach dung tam gidc vuéng AHM vi cdc d6 dai AM = m va AH = h cho truéc. Tiép dén, ta dung duéng tron tim M ban kinh bing d6 dai cho truéc k/2, dudng tron nay c&t duéng tron ngoai tiép tam gidc vuéng AHM tai X. Hai duing thing HM va AX gap nhau & C. Goi B 1a diém déi xtmg cia C qua M. Ta due ABC 1a tam gidc phai dung. Ching minh: Theo cach dung trén, r6 rang AH = h ; M 1a trung diém AB nén AM la trung tuyén va AM = m. Ha dudng cao BI. Tam gidc AXM vudng tai X vi no néi tiép trong nita dudng tron dudng kinh AM. Suy ra MX / BI va vi vay BI = 2MX =k. Bién luén: Véi digu kién h < m, ta luén dung duc tam giéc vung AHM. Ngoai ra, dé dung dude diém X, dutng tron dudng kinh AM va dutng tron (M; ©) phi oft nau; muén thé ta phi eb B ), ta duge phuong trinh 2 1s? —45+1=0, phuong trinh bac hai theo s nay lai vo nghiém; diéu nay c6 nghia khong tén tai a dé V,=V, va khdng dinh & dé bai duge ching minh. b) Dat a =k, tacé phuong trinh (1+ 6k)s? +2(1-3k)s+1=0, dé 2 phuong trinh nay cé nghiém, ta phai c6 A’ =(1—3k)? -(1 +6h)20e>k>4. Vay gid ti hd mht ia Th =k ta ©, ting véi ssinar=5 vi OB =3r. - Dén day, viée dung géc a tré nén hoan toan don gian. Bai 8. (1961) Goia, b,c la cdc dé dai ba canh céa mét tam gidc cé dién tich 14 A. Chiing minh ring a? + 6? +c? > 4V3A. Khi nao ddu ding thiic xdy ra? Let a, b, ¢ be the sides of a triangle and A be its area. Prove that a? +b? +c? >4/3A. When do we have equality? Huéng dan: Ta cé cong thitc Heron: atb+c at+b-c a-b+ec bt+c-a 22 2 Do tinh chat tong hai canh trong mét tam gide luén luén lén hon carh thit ba, ta thay ring tat cA cdc thia sé trén déu duong. Vi vay, dp dung bat dng thttc Cauchy A= (xt y+2)? ey) xyz a cho ba sé x =(a+b—c), y=(a—b+c),z=b+c-—a ta duce: 4A= f(a+b+ca+b—c)a—b+cX(b+c—a) (a+b+c) (atb+e)" Nhuw vay ta dude mene . 33 dong véi téc dé khong déi doc theo chu vi hinh vuéng ABCD; Y cing chuyén déng cing téc dé d6 doc theo chu vi hinh vuéng B'C'CB. X xudt phat tir A di vé phia B con Y rdi tit B' dé di vé huéng C'. Tim quj tich trung diém XY. The cube ABCD.A'B'C'D' has upper face ABCD and lower face ABCD with A directly above A’ and so on. The point X moves at const- -ant speed along the perimeter of ABCD, and the point Y moves at the same speed along the perimeter of B’C'CB. X leaves A towards B at the same moment as Y leaves B’ towards C’. What is the locus of the midpoint of XY? Huéng dan: Ta ki hiéu O,, O,, O, lan lugt la tam cda cdc mat ABB'A’, BB'C'C va ABCD. Ta sé chiing minh ring quy tich cdc trung diém Z cia XY 1a dutng gap khtic 0,0,C0,0,. z Gia si A 1a géc toa dé; AB, AD, va AA’ Ia cdc truc toa dé x, y, 2 tuong img. Dat AB = AD = AA’ = 1. Chia théi gian diém X chay trén dung ABCDA lam 4 phan bing nhau va lay méi phan dy lam don vi théi gian. y Ta biét ring néu diém K chuyén dong thing déu thi su phy thude toa dé cia né vao thii gian la su phu thuéc tuyén tinh va ngusc lai, néu su phu thuéc toa dé cia diém K vao thai gian 1 tuyén tinh thi K chuyén dong thing déu. Néu T Ja trung diém cua doan OP véi: O = (%1,1,21) va P =(x2,¥2522) thi T= (2H +22 41 #2) 2 2 Ung dung cdc ménh dé da phat biéu trén, ta lap bang phu thuéc toa dé ciia cdc diém X, Y, Z vao¢ sau day: 34 inmate O AOC +180° -OAD -— ODA + 180° - ODC-OCD = 360° > adc- BAD BOD pico, A) (vi theo tinh chat tiép tuyén, AO 1a phan gidc cia goc BAD va OC 1a phan gidc cia géc BCD). 33 Do ABCD la tit gidc néi tiép trong (K) nén ta cd BAD +BCD = 180° va ADC + ABC =180°, vi vay ta duge: AOC -90° - 180° + ABC = 0 <> AOC = 270° - ABC. Ta suy ra cach dung: Truéc hét, ta dung cung chtta géc (270° — ABC) dung trén doan AC. Sau 46, goi O 1a giao diém cua cung nay véi dudng phan gidc cba géc ABC. Tiép dén, dung duéng tron (O) cé tam O va tiép xe véi BA, BC. Ti A ké tiép tuyén dén (0), tiép tuyén nay va (K) ct nhau tai D. ABCD 1a tit gide phai dung (néu diéu kién 6 phan bién ludn dude thoa). Viée chitng minh hoan toan don gian. Bign ludn: Ta sé ching minh ring diéu kién cdn va di dé bai toan cé nghiém hinh la AB + CD = BC + AD. ‘That vay, néu bai todn dung dudc, goi M, N, K, P la 4 tiép diém cia (O) véi AB, BC, CD, DA tuong img, theo tinh chat tiép tuyén ta cé: AB+CD = AM+MB+CK+KD = AP +BN+NC+DP=BC+AD. Dao lai, gid sit ta cd AB + CD = BC + AD, hay CD = BC + AD- AB, lic dé, ta dung D nhu da néi 6 phan edch dung. Tu C, ké tiép tuyén ca (O), c&t AD tai D’, ta sé chtmg minh D tring D’, nghia 1a cach dug cia ta thoa yéu cau dé bai. Theo tinh chit tiép tuyén ta c6: AB + CD' = BC + AD’ hay CD’ = BC + AD’ - AB. Tu dé : CD'- CD = BC + AD’ - AB - (BC + AD - AB) = AD'- AD = DD’ (hode -DD’). Diéu nay ching td D tring D’, diéu phai ching minh. A D A c c D Bai 14. (1962) Cho R, r lan luot 14 ban kinh dudng tron ngoai tiép va ndi tiép cda mét tam gide can. Ching minh ring khoang cach giita hai tam cia hai dudng tron nay la {R(R—2r). 36 The radius of the circumcirele of an isosceles triangle is R and the radius of its inscribed cirele is r. Prove that the distance between the two centers is (R(R~-2r). Huéng dan: Gia st ABC 1a tam gidc cé AB = AC, goi O va I tuong ting 1a tam cdc duéng tron ngoai tiép va ndi Dat d 1a khoang cach IO, ta quy uéc ring d > 0 néu O gan A han I, vad < 0 néu I gan A hon O. G day ta trinh bay va vé hinh khi O gin A hon I, trudng hop cin lai két qua van khéng thay déi véi quy ude trén. Goi a 14 géc OAB, M la trung diém BC. Ké TH 1 AB. Khi dé: A . HI HI r sing =—_= = , hay AI AO+OI R+d r=(R+d)sina ; (*) cos2ar = cosBOM = OM _ 01+ IM euia OB OB R hay r+d=Rcos2a. (**) B c Két hop (*), (**) va sit dung céng thie cos2a =1-2sin’@ ta tim duge hé thite: (d+ R+r)[d? - R(R-2r)|=0. Néu d > 0 nhu trudng hop ta dang xét thi hién nhién (d+ R +r) > 0, con néu d < 0 nhu trong hop con lai thi do ta luén c6. OI < OA nén ta cing c6 (d + R +r) > 0.Vi vay: [a? - R(R-2r)]=0 > d= JR(R—2r) (dpem). Bai 15, (1963) Cho diém A va doan thing BC, hay xdc dinh qug tich cdc diém P trong khéng gian sao cho géc APX bing 90°, vai X 14 mét diém nao do nim trén doan BC. Given a point A and a segment ist, determine the locus of all points P in space for which angle APX = 90° for some X on the segment BC. Hiuéng dan: Trudc hét, ta hay tim quy tich trong mat phing ABC. Qua A, vé mét dudng thing I tiy y; goi B,C, 1a hinh chiéu cia doan BC lén 1. Ta thay ring bat ki mét diém P, nao trong doan B,C, cing thuéc quy tich phai tim, béi vi néu goi X, 1a diém trong doan BC sao cho P, 1a hinh chiéu cia X, lén B,C, thi géc X,P,A hién nhién vudng goc, cé mét canh 37 qua A va mét canh cit doan BC tai X,. Bat ki mét diém N nao trén duéng thing J nhung 6 ngoai doan B,C, cing khong thuéc quy tich phai tim vi dudng vuéng goc véi J vach ti N khong thé cit doan BC. Nhu vay, trén dudng thing / thi quy tich phai tim 1a doan thing B,C,. Khi cho J quay xung quanh A thi cae diém B,, C, vach nén hai dung tron (O,), (O,) cd dudng kinh lan luct 14 AB, AC va doan B,C; sé quét nén mét hinh gém tat cd nhiing diém cia hai hinh tron (O,), (O,) - ké cd dudng tron bién - trit nhing diém 6 trong ca hai hinh tron, nén quy tich la mién c6 gach chéo trén cdc hinh a (trudng hop A & ngoai dung thing BC), 14 mién duce 6 dam trén cac hinh b, ¢, d (trudng hop AG trén dutng thing BC). Quj tich trong Ichong gian ¢6 duge bing céch quay quy tich trong mat phing trén day xung quanh dung n 6i tam O,O, cia hai hinh tron: quj tich 1a hai hinh cdu, ké cé mat cdu bién, trix phan khong gian chung bén trong hai hinh cau. Hinhe Hinhd Bai 16. (1964) Cho 5 diém trong mét mat phAng sao cho trong sé tat cd nhing dudng thing néi hai diém bat ki, khéng cé hai duéng thing nao tring nhau, song song hodc vudng géc véi nhau. 38 Qua mi diém, vé cdc dudng thing vuéng géc véi cdc dutng thing xée dinh béi hai trong s6 4 diém cin lai. Hay xéc dinh sé lén nhat cdc giao diém ma nhiing duting vudng géc nay cé thé giao nhau. 5 points in a plane are situated so that no two of the lines joining a pair of points are coincident, parallel or perpendicular. Through each point lines are drawn perpendicular to each of the lines through two of the other 4 points. Determine the maximum number of intersections these perpendiculars can have. Huéng dan: Goi 5 diém thoa man yéu cdu ctia dé toan 1a A, B, C, D, E. Xét cé - dinh mét diém, thé thi 4 diém cdn lai sé cho ta 6 dudng thing. Do dé co cd thay 6 dudng vudng gic di qua diém cé dinh dé. Nhu thé, tang cong cO 30 dung thing vudng géc. Tinh ném na, cif hai dudng thi c&t nhau tai mot diém, do vay, nhiéu lim 1a cé6 cd thay C2) = ae = 435 giao diém. Thé nhung, lai cé mét sé giao diém tring nhau. Cé ba nhom diém tring nhau. Dau tién, 6 dutng thing qua A (ching han) giao nhau tai mét diém (dé 1a diém A), trong khi tinh theo kigu trén 1a C2 = & =15 giao diém. Nhu vay ta mét di 5.14 = 70 diém. Tiép dén, cdc duéng thing qua C, D va E cing vung géc voi AB sé song song véi nhau, diéu nay loai thém 3 giao diém. Nhu thé, ta mat thém C3x3= he =10.3 = 30 diém. Thi ba, dudng thing qua A vuéng géc véi BC 1a dudng cao trong tam gidc ABC; ciing thé véi duéng thing qua B vuéng géc véi AC, dung thang qua C vudng goc véi AB, ching ding quy tai truc tam cia tam gide ABC. Véi tam gidc ABC nay ta phai loai di 2 giao diém. Nhu thé phai loai thém tt c&C3 x2 = 10.2 = 20 giao diém. Ro rang, cac giao diém tring lap néi trén déu phan biét nhau (do ba mién diém via xét rdi nhau), do vay, gid tri lén nhdt 6 thé cé cua sé cdc giao diém 1a 435 - 120 = 315. Bai toan dén day van chua thuc sy duge gidi quyét néu ching ta 39 Khong chi ra duoc mét trat tu dac biét vé cach sap xép 5 diém da cho sao cho sé giao diém thuc su la 315, nghia 1a cach s&p xép cdc diém dé bao dam duoc ring khong cdn mét giao diém tring l4p nao ngoai cdc tring lap ké trén. Dé nghi ban doc tiép tuc céng viéc nay hodc chi ra mét cach gidi khdc téi vu hon. Bai 17. (1964) Cho tit dién ABCD, D, 1a trong tam cia tam gide ABC. Nhang dudng thang song song véi D,D va lan lust qua A, B, C tung ing cat cdc mat phing (BCD), (CAD), (ABD) tai Ao, By, Co. Ching minh rang thé tich tit dién ABCD bing mét phan ba thé tich tit dién AgByCoDp. Néu D, la mét diém tuy y nim bén trong tam gidc ABC, liéu két qua nay cé con dung khéng? ABCD is a tetrahedron and Dp is the centroid of ABC. Lines parallel to DD, are drawn through A, B and C and meet the planes (BCD), (CAD) and (ABD) in Ag, Bo, and C, respectively. Prove that the volume of ABCD is one-third of the volume of AgB)CoDo. Is the result true if D, is an arbitary point inside ABC? Huéng dan: ‘Ay Ta Biai bai todn trong truéng hgp téng quat, khi D, la diém tuy y trong tam gide ABC. Gia st cdc dung thing song song véi DD va lan lugt qua A, B, C tuong ting cAt cde mat phdng (BCD), (CAD), (ABD) tai Ao, Bo, Co. : Trong tam gidc ABC, ta dung cac dudng thing Cc ADp, BDp, CD,; chiing lan lugt c&t cdc canh adi diéa tai A’, B', C’. Dé thay A,D 1a giao tuyén cua mat phang (DBC) va mat phang xdc dinh béi hai dudng thing song song AjA va D,D. Ta lai cé A’ 1a diém chung cia hai mat phdng nay. Do dé Ay, D, A’ thing hang. Hoan toan tuong tu ta cing c6 Bo, D, B’ thing hang va Co, D, C' thing hang. Tai cdc dinh A’, B', C’ ta dat lin lust cdc trong lugng la x, y, z sao cho trong tam cia ching 1a Dp. Ching han, ta dat AB'_z CA'_y BC'_x sao cho —— =, =? , ~_ C6 thé ta chi lay: BC x'A'B z’C'A y 40 x=, y= SA" apy aptez. A'B Tacs: Sace.-AB'AC ___ ___ yang tur ta cing e6: Sasc ABAC (x+y)(x+z) Spon a a Sav yy. : Sanco -(v+xXv+2) — Sapc (x +22 4+) pias: SABC ye Sanc (xtzKzty) (ytxy+z) (x4 yx+z) : 2xyz Cet yt zt)” Vi hai tw dién ABCD va A'B'C'D cé chung dudng cao nén Yeoeo _ Sxwe _ 2x2 aa * Vasco Sane (x+y ytzz+x)” a A'D _A'Do __* va tuong tu: DA, DA ytz : Bboy, CD12 DBy x-:'’DC, x+y" Do cdc géc tam dién D.A'B'C’ va D.A,ByCy bing nhau nén thé tich cac tit dién A'B'C’D vii AgB,CyD ti 1é véi tich cdc canh bén, tite 1a: Vere DB'DC'D _ xyz ) AgD-BoD.CoD (x+ y)y+2)(z+x) | Ngoai ra ta cé: *) Vagnoc ‘CD =2 Yn BCD ‘ABCD VngBoCoD (*) va (**) cho ta: » hay Va .p.cop = 2Vanco- Bay gid, ta chuyén ché cac trong lugng trong tam gidc ABC sao cho tai diém A’ cé trong higng 2, tai diém B’ cé trong lug xt va tai diém C’ eé trong luong = Mh 46, trong tam van la Dy. Tiép dén, ta dat tai cdc dinh Ao, Bo, Cy cdc trong lugng tugng img 3, 3%: laie 46, trong tm cia cXp diém (Ap, A’) 88 14 diém D, bai vi: yrz ApD _ADp _y+z_ 2 DA’ DoA' xX 2 Al Tuong tu nhu thé, trong tam cua mdi cAp diém (B,, B'), (Cp, C’) cing 1a diém D. Mat khdc, trong tam cia céc diém A’, B’, C’ la Dy va trong khéi cia n6 bing x + y+2z. Cdn trong tam cia cdc diém Ay, By, Cy nim trong mat phdng (AoB,C,), déng thdi cing nim trén d:ting thing D,D, tite 1 diém D,, nhung trong khéi ca n6 bing *”*? Dodo Dp _1 yg Dy _3. DD) 2. DD 2 Diéu nay c6 nghia la ti sé cdc dudng cao ha ti cdc diém Do va D lén mat phang (A,B,C) bing 3/2. Nhung cdc tit dign AjB,C,D va A,B,C, chung day A,B,C, nén ti day ta suy ra: VngBoCoDo _ VrgBoCoD 2 Theo chting minh trén ta lai 6 Vasngcgp = 2¥ancy» Vi Vay: Vn .BoCoD) = 3VaBcn> 1a diéu phai chimg minh. Céach khae: Bai todn trén c6 thé gidi dugc béng phuong phdp toa dé. Sau day la loi gidi vdn it: Chon D lam diém gée cia mét hé truc chudn sao cho céc vector DA, DB, DC 1a a, 3, c tudng img. Khi dé, vector DD, 1a pat+qb+re, véi p+q+r=1 vap,g,r>0. Vi vay, diém bat ki nim trén duing thing di qua A va song song vi DoD sé cé: DA =a+s(pa+gb+re), va diém nay nim trén mat phang (DBC) néu foe P Ma ta c6 Ay thod man diéu kién nay nén : DA, =- 26-7. P Pp Tuong tu, cdc vector DB, va DC, tuong img 1a: ~Pa-Te va -Pa-15 q q - Lf ‘Thé tich ABCD bing 4 axbe| va thé tich A,B,C,D, 1 42 Lebel [(r2 +a o(e F942 )oe(ae2 or] Suy ra ring ti sé thé tich ABCD va thé tich AgByCoDo Ia gid tri tuyét déi cua dinh thiic: ey pre gst + oe r va ta tinh duge gid tri nay 14 2+ p+q+r =3, diéu phai chimg minh. Bai 18, (1964) Cho tam gidc ABC cé dé dai ba canh 1a «, 6, c. Ta lan uot vé cdc tiép tuyén véi duéng tron néi tiép cha tam gidc nay song song véi 3 canh tam gidc. Mai tiép tuyén hgp véi hai canh kia cua tam gidc dé tao thanh mét tam gidc méi, nhu thé ta dugc 3 tam gidc mdi iao thanh. Lai vé 3 dudng tron néi tiép 6 3 tam gide méi dé. Hay tinh téng dién tich 4 hinh tron néi tiép néi trén. Triangle ABC has sides a, b, c. Tangents to the inscribed circle are constructed parallel to the sides. Each tangent forms a triangle with the other two sides of the triangle and a circle is inscribed in each of these three triangles. Find the total area of all four inscribed circles. Huéng dan: Goi cdc ban kinh cia 4 vong tron lin lugt 18 1,, r2, r5, 71 va S 1a dién tich tam giée ABC (xem hinh bén dui). Ta cd: £ [( ~at3te) va $= {p(p—a\\p- bp) - P 2 Ba tam gidc mdi dong dang véi tam gidc ABC nén: fe iy ty hy nly hh hy hh,” n trong dé h; (i = 2, 3, 4) la cdc dung cao cia céc tam gidc méi tuong ing 43 v6i h,, hy, h, la cde dudng cao cua tam gidc ABC ting véi 3 canh a, b,c. Khoang cach cua cdc dudng song song (ching han nhu A,A, va BC) la 2r,. Ta dé: fy = hy -2n hy =hy- 25 hy =h, - 2K; _— = A - 2 : 2n? _S 2aS? _S(p-a c Vay ry =, -—- = 2 - $ = See) hg =P pP2S p Tuong tu: 7 = St2=) > 5) . oe a P P Goi S,, Sp, Ss, S, 1a dién tich 4 vong tron, ta cé: S\ +S 4534S, =a(7 14772424774) = 2 92 2 2, 2 92 = Se Se=b) pe Pp p! p pt s? = rb? Ha? +96 +(P-0)"|= _ ala? +b? +c? at b-cat+c-b\b+c-a) = (at+b+c) : Tom lai, téng dién tich 4 hinh tron can tinh 1a: co n(a? +b +c? a+b-ca+c-b\(b+c-a) (at+b+ cp : Bai 19. (1965) Cho ti dign ABCD. Gi sit tir dién nay dugc chia lam hai phan béi mét mat phing song song véi AB va CD, khoang cach tit mat phing nay dén AB bing k lin khoang cach dén CD. Tinh ti sé thé tich cia hai phan dé. The tetrahedron ABCD is divided into two parts by a plane parallel to AB and CD. The distance of the plane from AB is k times its distance from CD. aA Find the ratio of the volumes of the two parts. Huéng dan: Ta gid sit AB va CD lan lugt cé dé dai laa va b, khoang cach giita hai dudng thing chéo nhau AB va CD 1a d, géc gitta hai duing thing nay 1a w. Goi (P) la mat phdng song song véi hai canh déi AB va CD. Gia st (P) cat tit dién theo thiét dién 1a tu gidc EFGL. Dé thay EFGL a hinh binh hanh. Goi MN 1a duéng vuéng géc chung cia AB va CD, giao diém cia MN véi (P) la I. Mat phdng qua I va AB sé cAt mat phdng (P) theo giao tuyén JK // AB. Dé thay ring B, J, N thing hang va N, K, A thing hang. Ta cé hai tam gidc JNK va BNA déng dang nhau, véi NI va NM 1a hai dudng cao ciia hai tam gidc dé. Ti gid thiét ta c6: NI_1 —— =~ véiNM=NI+IM; IM _ k NM_AB_a NI+IM IM — = = Sipe ites oe KJ KJ NI NI KJ GL _BJ_ IM k | DC BN MN. k+l Suy ra KJ = a Mat khac ta lai co ~~ = —~— dude GL = ZZ va a _ a IM = _ . Dung JX va KY song song véi IM; véi X, Y nam trén AB, suy ra cdc mat phing (XGL) va (YEF) vudng goc vdi (P). Dé thay ring khéi XGL.YEF 1a ling tru diing, do dé: Var.sor =Vxeu.ver +Varry +Vecix > Vo Se IK ; EF.YKJK, Ves yp at hb Rd a __ abd A XGLYEF 9 kal k+l k+l +1” Vacey +Veoix = 5 Sver “AY + $Sxau, .BX = : YK.EF(AY + BX) 3 =4M.GL(aB-Jk)=1 4. 2(« G }- oaks 6 6k+1 k+l 6(k +1? k+l ® abdk? | abdk®__abdk? Akay k+l? OeHIP abd(1+3k) 6k +1 ~ Vay ti sé thé tich cua hai phan do mat phdng (P) chia tit dién Nhu vay: Vare sat = G+k). Chimg minh tuong tu ta duge: Voy pgp = _ (Kk +3) Veevcr 3k+1) © Bai 20. (1965) Tam giac OAB cé géc O nhon, M 1a mét diém tuy ¥ trén AB, P va Q lan lugt 14 chan cdc dudng vudng géc ha tix M xudng OA va OB tuong wing. Tim quj tich truc tam H cua tam gidc OPQ. Quy tich cda H sé 1a gi néu M di d6ng trong mién trong cia tam gidc OAB? The triangle OAB has angle O acute. M is an arbitary point on AB. P and Q are the feet of the perpendiculars from M to OA and OB respectively. What is the locus of H, the orthocenter of the triangle OPQ (the point where its altitudes meet)? What is the locus if M is allowed to vary of the interior of OAB? Huéng dan: Goi X, Y lan lust 14 chan cdc dudng vuéng géc ha ti B, A tuong xing Sauc © 4AL-AM.sinA > AB.AC.sinA <> 4AL.AM > AB.AC <> 4AL.AM > (AM + BMXAL + CL) © 3AL.AM > AM.CL + BM.AL + BM.CL. pat k= BK 7 Ce = AM it ding thite trén tré thanh: CK AL BM Se q) nm Xét hoan toan tuong tu déi véi hai tam giac BKM va CLK, ta duce: Te B>kt+ +-, (2) 17 Soa (3) kok Cong (1), (2) va (3) vé theo vé ta c6 bat dang thiic: ob eee (4) Eom l:) om & Tuy nhién, theo bat ding thiic Cauchy ta lai c6: Tt tk lim 1 Loli k ol om k+l+m+—+—+-4+~—4--+—=k+—4+m+—+]4+-4+—4—4 - mililom k k m 1 itomk (5) 224242433)" an m Mau thudn gida hai bat ddng thiic (4) va (5) cho ta diéu phai ching minh. Bai 23, (1966) Trong khéng gian cho mét hinh ti dién déu. Chimg minh ring téng cdc khodng cach tit mét diém nao dé dén cdc dinh cia ti dién 1a nhé nhat néu va chi néu diém dé tring véi tam hinh cau ngoai tiép tit dién déu da cho. Prove that a point in space has the smallest sum of the distances to the vertices of a regular tetrahedron iff it is the center of the tetrahedron. Huéng dan: Gia su ABCD 1a tit dién déu cé O 1a tam hinh cau ngoai la diém tuy y trong khong gian. Goi X 1a trung diém CD. Cho P’ la chan duéng vuéng géc ha tit P xuéng mat phdng (ABX). Ta sé chtg minh ring néu P khong tring véi P’ thi PA+PB+PC+PD>PA+PB+PC+PD. p va P ntadorecedct AD That vay, gid str P khong tring P’, ta cd PA > PA vi géc PP'A vuéng; cing vay, ta cé PB > P'B. Do BX va AX vuéng géc CD nén (ABX) 1a mat phang trung truc céa CD. Suy ra P'CD 1a tam gic can tai P’, cin PCD thi khéng can tai P (néu nguse lai thi P trang P" béi khi dy P cing thuéc mat phing (ABX)). Mat khdc, PP' va CD nam trén hai mat phdng song song nhau nén khodng cach gitia P va CD cing bing khodng cach gitia P’ va CD. Diéu nay kéo theo PC + PD > P'C + P'D. Dé thay diéu nay, ta goi C' va D' lin lugt 1a cde diém déi xing cia C va D qua duing thing P'P, ta cé: PC +PD=PC'+PD>CD=PC'+PD=PC+PD. Tir do ta suy ra diéu phai chting minh. p Ti ching minh trén, ta suy ra duge ring néu P 1a diém sao cho PA + PB + PC + PD nhé nht, P phai nim trong mat phing (ABX); hoan toan tuong ty, D no cing phai nim trong mat phing (CDY), véi Y 1a trung diém AB. Suy ra ring P phai nim trén XY, dung néi trung diém cia c4p canh déi AB va CD. Lap lai li luan nay mét cach tuong ty nhu trén, P cing nim trén duéng néi trung diém hai c&p canh con lai céa tit dign ABCD; néi cach khac, P tring véi tam cia tit dién. Bai 24. (1967) Cho hinh binh hanh ABCD cé AB = a, AD = 1, BAD= a, tam gid ABD cé tat ca cde géc déu nhon (tam gide nhon). Hay ching minh ring cdc vong tron ban kinh 1 cé tam lan lugt 14 A, B, C, D sé phu kin hinh binh banh nay néu va chi néu cosa + V3sina 2a. The parallelogram ABCD has AB = a, AD = 1, angle BAD= a, and the triangle ABD has all angles acute. Prove that circles radius 1 and center A, B, C, D cover the parallelogram iff cosa + V3 sina >a. Huéng dan: Trude hét, ta ching minh bé dé sau day: Bé dé: Goi O 1a tam va R 1a ban kinh duéng tron ngoai tiép tam gidc ABC. Khi dé, cdc hinh tron tam A, B, C véi ban kinh x sé pha kin tam gidc ABC néu va chi néu x > R. Chuing minh: Ré rang cac hinh tron tam A, B, C ban kinh x phi tam gide ABC, muén vay né phai phi diém O, tam duéng tron ngoai tiép 50 x sateecasct tam gide ABC. Nhung khoang cach tit O dén A, B, C bing R, vi vay ta phdi cé x > R, do dé diéu kién can Ja hién nhién. Dado lai, gid st’ x>R, véi O, R la tam va ban kinh cia dudng tron ngoai tiép tam gidc ABC. Ta xét cic vong tron tam A, B, C ban kinh R. Khi 46, goi P, Q, R lan luot 1a hinh chiéu cda O xudng cdc canh AB, BC, CA tuong ung, thi hinh tron tam A ban B Q C kinh R sé pha ti gidc OAPR ; tuong tu nhu thé, hinh tron tam B ban kinh R sé phi ti gidc BPOQ va hinh tron tam C ban kinh R phu tit gidc CQOR. Néi cach khac, cdc hinh tron tam A, B, C ban kinh R pht kin tam gide ABC. Theo gid thiét, ta cd x>R nén hién nhién cdc hinh tron tam A, B, C ban kinhx phd kin tam gidc ABC. Diéu kién di da dude chimg minh. Tré lai bai todn: Hién nhién ring cac hinh tron don vj (cd ban kinh 1) tam A, B, C, D phi kin hinh binh hanh néu va chi néu 3 vong tron don vi véi tam 1a A, B, D phi kin tam gidc ABD. Goi R 1a ban kinh duéng tron ngoai tiép tam gide ABD, ap dung bé dé trén, diéu kién cdn va du dé 3 vong tron don vi véi tam 1a A, B, D phd kin tam gide ABD la: 12> R. “ ‘Ta cé: BD = 2Rsina , ngoai ra ta cing c6: BD? = a? +1-2acosa. Tu dé, 4R? sin? a= a? +1-2acosa. Do (*) nén diéu nay tuong duong véi: 4sin? a> a? +1-2acosa < 3sin? a> a? +1-2acosa+cos” a-1 <> 3sin? a > a? -2acosa+cos’ a. () Mat khdc, vi ABD 1a tam gide nhon nén chan dung cao H ké tir D xuéng AB phai nim trén doan AB, do vay ta suy ra cosa a cua bai toan tré thanh: V3sina >a—cosa © 3sin?@>a?-2acosa+cos*a. — (#**) (**) va (***) cho ta diéu phai ching minh. Bai 25, (1967) Ching minh ring néu mét tit dién chi 6 ding mét canh cé dé dai Ién hon 1 thi thé tich cia tit dign dé én nat 1a 1/8. Prove that a tetrahedron with just one edge length greater than 1 has volume at most 1/8. Huéng dan: Gia sé hinh tit dién ABCD cé canh lén nhdt 1a AB. Nhu vay, trong cdc tam gidc ACD va BCD, tat cd céc canh déu khong 1én hon 1; cdc chiéu cao tuong img AF va BE cua ching khong lén hon A trong dé CD =a <1. Chiéu cao cia hinh ti dién 1a 2 ASSAF < j1-< 4 (do tam gidc ASF vuéng tai S c6é AF 1a canh huyén). Thé tich cda hinh tit dién 1a: 2 BE.cp.As AC + AD (xét dinh A), BA > BC + BD (xét dinh B). Tit do suy ra : 2AB > AC + AD + BC + BD. “ Nhung trong cac tam gide ABC va ABD ta lai cé: AB 0 such that there exists a tetrahedron with k edges length a and the remainder length 1. Huéng dan: Ta xét tit dién ABCD. * Gia sir k = 1, AB cé dé dai a, cdc canh con lai déu cé dé dai bing 1. Khi dé, ta cé thé xoay cdc tam gide ACD va BCD quanh truc CD va lam thay déi AB. Trong qua trinh xoay nay, cdc tam gide ACD va BCD van déu véi canh bing 1, con AB thi thay déi tir d6 dai bé nbét 14 0 (khi A tring B) cho dén dé dai Ién nhit la lic A va B déi xiimg nhau qua CD, dé thay dé dai lén nht céa AB ide dé 1a V3. Ta sé loai trit hai gia tri cuc tri dé cda AB, khi ma tut digén ABCD suy bién thanh mét hinh phing, do dé, khoang xdc dinh cua a (hay diéu kién dé ton tai tur dién theo yéu cdu dé bai khi k = 1) la: 0—. 3 * Khi & = 2, c6 hai trutng hop xay ra: hoc hai canh c6 d6 dai a k& nhau (chung dinh), ho&c ching 1a hai canh déi nhau trong tif dién. 55 + Xét trudng hop thi nhdt, ta gid sit hai canh AC va AD cé dé dai a. Caing tién hanh quay quanh CD nhu trén, ta cé hai trudng hop A, B, C, D ding phang, do la: a) A va B nim khac phia déi véi CD. Luc dé, ta cd: a BH = 1.8. c yo4 2 AH=AB-BH= 1-22 =27%3, 2 2 sre anh WHS BR, D 4 4 c b) A va B nim cing phia véi CD. S| Dé dang tinh duce: a = ¥2+ V3. A H Nhv vay trong trutng hop nay ta thu dude két wt | es D + Xét trutng hop thif hai, gid st AB = CD = a. Ciing xoay nhut thé, ta duce a = 0 hic A tring B. Ta tang a dan dé dén khi tt dién ABCD tré thanh tit giéc ADBC, day la hinh vuéng cé canh bing 1 va do dé a = V2. Titdé ta duge: 0 =a. V2+ 3 * Voi k = 3, moi gid tri a > 0 déu chdp nhn duoc. Khi a <1, ta cd thé dung mét tam gidc déu cé canh bing a, va dung dinh thi tu cia tt dién sao cho dinh nay cach 3 dinh cia tam gidc dé mét doan bing 1. Khi a2>1, ta lai dung mét tam gidc déu cé canh bing 3, cn dinh thi tu thi cach déu 3 dinh cia tam gidc d6 mot doan bing a. Bai 30. (1969) Goi C 1a mét diém trén ntta dudng trin dung kinh AB, D 1a chan dudng vudng goc voi AB ké tix C. Goi (K,) 1A dudng tron néi tiép tam gide ABC; (K,) la dung tron tiép xtc vai CD, DA va nua dung tron; (K,) la dudng tron tiép xic véi CD, DB va mia duing tron. Chimg minh ring (K,), (Ke), (Kz) con c6 thém mét tiép tuyén chung nia ngoai AB. 56 C is a point on the semicircle diameter AB, between A and B. D is the foot of the perpendicular from C to AB. The circle (K,) is the incircle of ABC, the circle (K,) touches CD, DA and the semicircle, the circle (K,) touches CD, DB and the semicircle. Prove that (K,), (K,) and (K;) have another common tangent apart from AB. Huéng dan: Gid st O,, O,, O, tuong ving 1A tam cha 3 dudng tron (K,), (K,), (K,). Bai toan sé duge gidi xong néu ta ching minh duge ring O, nim trén dutng thing O,0,, bai vi lic d6, chi can lay anh déi xing cia AB qua duing thing 0,0, thi ta sé cé tiép tuyén chung thit hai cua 3 dung tron. Dat AB = c, BC =a, CA = b. Goi 7,7), 75 lan lugt la cdc ban kinh cde dutng tron (K,), (K,), (K,). Vong tron néi tiép tam gidc ABC tiép xtc véi AB tai P, AC tai Q va BC tai R. Khi dé, dé thay ACB = 90°, 0,QCR 1a hinh vuéng, AQ = AP va BP = BR, do dé ta cd: 7, = O,R = AC- AQ = AC- AP = b-AP, 7, = 0,Q = BC - BR = BC - BP = a—(c-AP), va cong vé theo vé ta dude: ce Oe tee ip b+e-a - Gia sit (K,) tiép xuc AB tai X, (K,) tiép xtc AB tai Y. goi O 1a trung diém AB. Xét tam gide vuéng 0,XO. Vi O, 1a tam ving (K,) tiép xtc voi nta dudng tron dutng kinh AB nén néu goi Z 1a diém tiép xtic nay thi dé thay 00, = OZ - 0,2 = OA- 0,2 = Son = AXD oO Y B Ta cing c6 OX = OD + DX = $-AD+ rn. Su dung hé thitc vé hai tam gidc dong dang ta dude: 2 ADCS AD AC AB ¢ Tu dé, 4p dung Dinh li Pythagore ta suy ra: es (g-2) = oe Qe) ee ace 57 khai trién va sip xép lai ta dudc hé thie: es 4 ne -anfe-®) [0 2-0. Nhung ABC 1a tam gide vuéng, do dé c? = a” +5”, suy ra: b2 = 7 2,2 Tit d6 ta duge: 2-2, © - 4% =o, phuong trinh bac hai theo c ¢ ry nay cho ta hai nghiém: 7 ry = a-*% (20) van = ~a+~ (<0). c c a e Vay r, = a——. Tuong tu: r, = b-—. Do do: c c O.X + O,Y =XY= 7 += at+b-c=2h, XP = AP - AX = AP - (AD - DX) = bte-a (8? btc-a atb-c a -(c-a)=- 2 2 fan =n. 2 c ' Tom lai, ta c6 duge XP = PY = PO, va XO, + YO; = 2PO,, nén ta suy ra O, nim trén dudng thing O,0,, dé 1a diéu phai chimg minh. Bai 31, (1969) Cho n diém trong mat phdng, véi 1 > 4 va khéng cé ba di (n—3)(n-4) 2 7m nao thing hang. Ching t6 ring cé it nhat tit gide Idi cé dinh nim trong sé n diém da cho. Given n > 4 points in the plane, no three collinear. Prove that there (n= 3-4) are at least convex quadrilaterals with vertices amongst the n points. Huéng dan: Truéc tién, ta xét 5 diém bat ki, khong c6 3 diém nao thing hang. Ta vach mat bao 1di tit 5 diém dé. Néu bao 18i nay cé hon 3 diém thi hién nhién cé it nhdt 1 tit gidc 1di. Néu chi gim 3 diém, chang han A, B, C, thi hai diém D, E cdn lai &t phai nim trong tam gidc ABC. Khi dé, c6 hai 58 dinh cua tam gidc ABC nim cing phia déi véi dung thing DE, va cing véi D, E, hai dinh dé tao thanh mét tit gidc Idi. Nhu vay, ménh dé can ching minh dung véin = 5 (dé y SaIC-4 =1). Xét n diém véi n> 5. Vi khong c6 3 diém nao thing hang nén sé. tt ca cde cach chon n diém nhu trén la: 5 _ n(n—1hn—2)(n -3)(n—4) d-“Meeaaea 120 Mii cach chon nay cho ta it nhat 1 tu gide 1éi; tuy nhién, bat ki tu gidc 1éi nao trong sé dé cing cé thé dude lap tit (n-—4) tap hop khac nhau gém 5 diém néi trén, do vay, cé it nhat 1 os _nn=In-2)in-3) n-4" 120 sé tat cA cdc tit gidc 14i dude thanh lap tit n diém da cho. Dé két thuic ching minh, ta sé ching té ring: n(n - Dn =2\(2= 3) =H ns s, 120 2 Diéu phai chimg minh nay tuong dung véi: n(n —1)(n—2) > 60(n—4) <> n(n—1)(n—2) -60(n—4) > 0. Dé dang thay ring cdc sé n = 5 va n = 6 1a nghiém cia phuong trinh n(n —1)(n— 2) - 60(n— 4) = 0, do dé ta cd thé phan tich thanh nhan titnhu sau: n(n—1)(n—2)— 60(n — 4) = (n—5)(n—6)(0 +8). Voi moi n nguyén duong va > 5, lap bang xét dau, dé thay déu cua (n—5)(n—6)(n+8) cing la dau cua (n—5)(n -6), va ta dugc (n-5)(n-6) = 0 khin = 5, 6; (n—5)(n—6)> Okhi n>6, suy ra diéu phai ching minh. Bai 32. (1970) Cho diém M tuy ¥ trén canh BC cia tam gide ABC. Goi r, 7, 1An luot 1a ban kinh cdc dudng tron néi tiép cla cdc tam gidc ABC, AMC, BMC. Goi g 1a ban kinh duéng tron bang tiép cia géc C (tite 1a dudng tron nim phia déi dién véi C so véi canh AB, tiép xtc véi AB va céc canh AC, BC kéo dai). Tuong ty, goi 4,92 tuong ting 1a cdc ban kinh cdc dudng tron bang tiép cia hai géc B va A. Ching minh ring Wd =" 92- M is any point on the side AB of the triangle ABC. r, r,, rz are the radii of the circles inscribed in ABC, AMC, BMC. q is the radius of the circle on the opposite side of AB to C, touching the three sides of AB and the extensions of CA and CB. Similarly, q, and q2. Prove that nd ="N92- Huéng dan: Ta bat ddu bing viéc tinh toan dé biéu thi r/q. Goi O la tam dudng tron néi tiép tam giac ABC va X 1a tam dung tron bang tiép cha géc C. Vi O 1a giao diém cia hai duéng phan gidc hai géc A va B nén néu goi M 1a tiép diém ciia dudng tron (0) véi AB ta cé: BM = OM.cotg > =reotg =, AM = OM. cote =reotg*, suy ra: c= AB=AM + BM = r{ootg +cotg 2). qa) Vi X nim trén phan gidc cac géc ngoai cia géc A va géc B nén xAB=90° -4 va xBA=90° -3 tiras: AM= XM cote 0° . *) 7 ate | BM= XM cote 90° . 4 ee 2 }. (2) suyrac= AB= AM + BM = ote +4 sta) va (2) ta dude: a IN > wo ot E +h _ ®, mt : Bo AB A | i =-s q cog *+ecotg 2 ig +1g® _ 2: L — Q igh ig® [4 2°2 B ic Li ludn va tinh toan nhu trén dugc lip lai hoan toan tuang tu déi véi cdc tam giac khac, ta dude: yA, CMA »_, B, CMB tg, 2 atgB ig CMB | a 828 2 gq, #28 2 ahung vi BE = 908 SE ay tg SP = nén: 2 60 nm_,A, CMA, B CMB A aa 82% 2 Be 9 ~Bo- Tir (3) va (4) ta suy ra diéu phai chimg minh. Bai 33. (1970) Ti dién ABCD cé géc BDC = 90° va chan dutng vudng géc ha tit D xuéng mat phing (ABC) tring véi truc tam cia tam gide ABC. Chimg minh ring: (AB+ BC+CA)* < 6(AD? + BD? +CD”). Khi nao dau dang thiic xay ra? B 85: (4) In the tetrahedron ABCD, angle BDC =90° and the foot of the perpendicular from D to (ABC) is the intersection of the altitudes of ABC. Prove that: (AB+ BC + CA)? < 6(AD? + BD? +CD?). When do we have equality? Huéng dan: ‘Truéc tién, ta sé ching minh ring CDA = 90°. ‘That vay, goi H 1a hinh chiéu cua D xuéng mat phing (ABC), gia sit CH ct AB tai E. Do AB | CEva AB 1 DH nén AB L (DEC), suy ra AB 1 DE. Tir dé, cdc tam B gidc vuéng BED va CEB cho ta: C BD? = DE? + BE? , CB? = CE? + BE*. BQ] Trit vé theo vé hai ding thie trén ta duge: i CB? - BD? = CE? - DE”. Nhung vi tam gidc BDC vuéng nén ta lai cé CB? - BD? = CD’, suy ra CE? = CD? + DE®, tiic la tam gidc CDE vusng tai D. Tom lai, ta cé CD L DB vaCD 1 DE nén CD 1 AD. Néi céch khac, CDA = 90°. Hoan toan tung tu ta ciing co ADB =90°. Tir dé, ta duge: AB? + BC? + CA? = 2(AD? + BD? +CD*). a) Mat khac, bat ddng thiic Cauchy cho ta: (AB+BC+CA)? = AB? + BC? +CA? +2AB.BC+2BC.CA + 2CA.AB <3(AB? + BC? +CA?). (2) Két hgp (1) va (2) ta duge (AB+ BC+ CA)” < 6(AD? + BD? +CD?). Dau ding thitc xdy ra khi AB = BC = CA. 61 Bai 34. (1971) Cho tit dién ABCD cé tat cd cde mat déu 1a cdc tam gide nhon. Lay cdc diém tuy y X, Y, Z, T tuong tmg nim bén trong cdc doan AB, BC, CD va AD. a) Gid sit ring DAB+BCD*CDA+ ABC, hay ching minh rng khéng thé cé mét dudng khép kin XYZTX nao cé chiéu dai cuc tiéu. b) Gia st DAB+BCD=CDA + ABC, khi 46, hay ching minh ring cé v6 s6 dung khép kin XYZTX cé chiéu dai bé nhat 1a 2ACsink, véi 2k = BAC+CAD+DAB. All faces of the tetrahedron ABCD are acute-angled. Take a point X in the interior of the segment AB, and similarly Y in BC, 2 in CD and Tin AD. a) IfDAB+ BCD *CDA+ ABC, then prove that none of the closed paths XYZTX has minimal length; 8) If DAB+ BCD = CDA + ABC, then there are infinitely many shortest paths XYZTX, each with length 2ACsink, where 2k = BAC +CAD + DAB. Huéng dan: Ta gid stt tit dién da cho cé cdc mat dugc lam bing bia cac-tong, va ta c6 thé roc tit dién nay theo cdc canh dé trai cdc mat dy ra trén mét mat phang. Gia stt ring khi dy ta duac hinh luc gide CAC'BDB’. Liic nay, dutng khép kin nhu da néi 6 dau bai 14 dudng néi Y trén B'C va Y' trén canh déi dién BC’ cia hinh luc gidc. Ro rang 1a dung nay cé dé dai cue tiéu khi né 1a duéng thang. Néu B‘C va BC’ song song véi nhau thi ta cé thé ldy Y tuy y trén canh dé, khi dé, dé dai dudng khép kin dat gid tri nhé nhat. Nhung néu BC’ va B'C khong song song thi dé dai duéng khép kin chi dat cuc tiéu 6 mt vi tri nao dé. Gid si CC’ < BB’. Néu géc CAC’ bén trong luc gidc bé hon 180°, cuc tiéu cia duing khép kin sé dat dude khi Y tring véi C. Diu nay khong thé dug vi dé bai ddi héi Y nim bén trong canh BC. Néu géc CAC’ bén trong luc gide lén hon 180°, cue tiéu cia dudng khép kin sé dat dugc khi X va T tring nhau tai A. Diéu nay cing 62 khéng duge phép nhu da néi. Dén day, bai todn rut lai 1a: tim diéu kién dé B’C va BC’ song song nhau. Dé thay diéu kién do 1a BCD + DCA +CAD + BAD + BAC + ACB = 360°, nhung vi DCA+CAD = 180° - ADC, BAC+ ACB = 180° - ABC nén ta cé diéu phai ching minh. Bai 35. (1972) Cho n Ja sé tu nhién lén hon 4, ching minh ring moi tit gidc ndi tiép déu cé thé duoc phan chia lam n tit gide ndi tiép. Given n > 4, prove that every cyclic quadrilateral can be dissected into n cyclic quadrilaterals. Huéng dan: Truéc hét, ta nhan xét ring tit gidc néi tiép ABCD luén ludn c6 thé duge phan chia thanh 4 tif gidc néi tiép. That vay, ta ldy P tay y bén trong tit gidc ABCD, goi K 1a mét diém trén canh AB. Néi PK. Tiép dén, ta ldy trén canh BC mét diém L sao cho KPL=180°-B (diéu nay chiing td tit gidc KPLB ni tiép). Ta lai ldy M trén CD sao cho LPM = 180° -C; sau 6, lay N trén AD sao cho MPN = 180° -D. Liic dé, dé dang chiing minh duge ring NPK =180°~A. Tuy nhién, ta con phai ching minh ring cac diém P, L, M lay nhy trén thod man ¥ dinh cua chung ta, nghia 14 né phai nim trén cdc canh nhu da néi. Diéu nay tuy thuéc vao cach lay diém cdc K va P ma phan trinh bay sau day sé ré. Ré rang 1a, néu ti gide néi tiép ABCD cé hai canh song song véi nhau (AB // CD) thi bai toan xem nhu dusc gidi quyét, béi vi lic dé, ta cb thé chia nhé ti gidc ABCD thanh n tuy ¥ tut gidc (cing néi tiép) bing n dung thing song song AB // CD. Vi vay, bai todn sé duge gidi xong néu ta chon cdc diém K va P thé nao cho mét trong cac tit gide néi tiép méi thanh lap c6 hai canh song song nhau. Diéu nay thi dé, bai K tuy y, do 6, ta lay PK // AD, lic nay, chic chin PL // CD vi: 63 KPL = 180° -B=D. Van dé con lai 1a xem xét sao cho cac diém K, L, M, N tong img ndm trén cdc canh AB, BC, CD, AD. Trude hét, xét K va L. K khéng thé nim trén AD vi ta cé PK // AD. Nhu thé, chi can ldy P dui gan diém D thi K sé nim trén canh AB (tranh tinh trang duéng PK c&t BC). Tuong tu nhu thé, ta cing ldy P di gn D dé bdo dam cho L nim trén canh BC. Lic dé, ta gid si ca M lin N déu nim trén AD, thé thi bing cach giit cho K cé dinh, ta kéo P tién vé gan sat véi CD, N sé dich chuyén lén CD, dé lai M nim trén AD. Bai toan da duge ching minh. 7 Bai 36. (1973) Cho OP;, OP;,...,OP),,; 1a cdc vector don vi trong mot mat phing. Cac diém P,, P,, ...., Pay. déu cing nim vé mét phia cia mét dudng thdng qua O. Ching minh ring: | OP; +OP; +...+ OP oq4) | 21 OP,, OP;,..., OP2,4; are unit vectors in a plane. Py, P,, . all lie on the same side of a line through O. Prove that |OP, + OP; +... + OPyq,i|21- Huéng dan: Ta tién hanh chimg minh bing quy nap theo n. Hién nhién ménh dé dung véi n = 1. Gia st ménh dé dung véi 2n - 1, tite 14 n6 dung véi hé vector OP;, OP;,..., OPy,_; . Ta xét hé vector OP; , OP3,..., OPp,,1 , va sip xép lai sao cho OP, , i = 1, 2,... 2n - 1, nim giita OP,, va OP2a.,. Luc dé, ro rang vector u= OP, + OP q,, co phuong nim trén phan gidc cla goc Pag OPanet , suy ra vector nay lap thanh mét goéc bé hon 90° véi vector v= OP, + OP, +.. + OP oq « (¥ chic chin nim gida OP,, OP,,_; nén suy ra n6 nim gitta OP, , OP, )- Theo gia thiét quy nap, ta cd | ar. Mat khéc, ding céng thie cosin ta duge jus {>| Do dé, ta suy ra ménh dé da cho duing véi sb 4 2n +1, diéu phai chimg minh. OP; = OP, =. con OP,,,,; 1a vector dan vi cé huéng tuy ¥. Bai 37. (1973) Hay chi ra mét tap hop cdc diém khéng ding phang, sao cho véi bat ki hai diém A va B cho truéc trong tap hop nay, tin tai hai diém C va D khac (cing trong tap hop dé) ma AB va CD song song véi nhau. 6 diag thuc, ta ldy: = OP, 5 =OP, =—OP, n+l Can we find a finite set of non-coplanar points, such that given any two points of this set, A and B, there are two others, C and D (of this set), with the lines AB and CD parallel and distinct? Huéng dan: Xét hai luc gidc déu cé chung duéng chéo chinh (dé 1a duéng chéo phan luc gidc thanh hai hinh thang can bang nhau), ching nam trén hai mat phing vudng géc nhau. Tap hgp céc dinh cia hai luc gidc nay chinh 1a tap hgp céc diém cn phai chi ra. That vay, néu ta ldy hai diém A, B cing la dinh cia mot trong hai luc gide dé thi ro rang c6 thé tim thay ngay C, D cing 1a hai dinh cua luc gidc nay sao cho AB // CD. Can néu ta lay A la dinh cia luc gidc nay va B la dinh cia luc gidc kia thi ta sé chon C 1a dinh déi dign trén dung chéo chinh CA thuéc luc gidc thi nhat, va ldy D 1a dinh déi dién trén dung chéo chinh BD thuéc luc gidc thi hai. Lic dé, dé dang ching minh duge ABCD 1a hinh thoi, va do vay, AB // CD, diéu phai ching minh. Bai 38. (1974) Chiing minh ring trén canh AB cia mét tam gidc ABC, co mot diém D sao cho CD 1a trung binh hinh hoc cia AD va DB khi va chi khi sinA.sinB< sin” € : Prove that there is a point D on the side AB of the triangle ABC, such that CD is the geometric mean of AD and DB if and only if sinA.sinB < sin” — Ghi chi: Ta néi CD 1a trung binh hinh hoc (tic trung binh nhan) cia AD va DB, nghia 1a hé thiic sau duge thoa man: CD? = AD.DB. searacosct 6 Huéng dan: Gia si D 1a diém nim trén canh AB sao cho CD? = AD.DB. “ Kéo dai CD, dutng nay ct dudng tron ngoai tiép tam gidc ABC tai E. Hé thite lugng trong dutng tron cho ta: CD.DE = AD.DB. : Vi vay, (*) xdy ra khi va chi khi CD = DE. A Liic d6, néu goi F 1a trung diém cung AB va FN 1a khoang cach ti F dén AB, CM 1a khoang cach tix C dén AB thi FN > CM Go) (vi CM ciing bing khoang cach tit E dén AB, ma FN lai 1a khoang cach lén nhat trong tét cd cdc khoang céch tit mét diém bat ki trén cung AB dén AB). Ta lai co FAB = FCA = S , suy ra: FN= ANAgS =f ag , CM = BC.sinB = a.sinB, véic = AB, b = CA vaa = BC, ki hiéu nhw thudng lé. Nhu vay, tit (**) ta duge £.tg©> a.sinB, nhung tit Dinh ii ham sin ta cd a = ¢ 4 nen 22 sinC diéu kién trén tré thanh Cc. sin - sinC c 2sinA.sinB © sinA.sinB< sin? —. 2cos 2 7 Diéu dao lai dé dang bing cach di ngugc lai qua trinh trén. Bai 39. (1975) Cho tam gidc ABC bat ki. Ta dung bén ngoai tam gidc dé cdc tam gidc BCP, CAQ, ABR sao cho: PBC = CAQ = 45°, BCP = QCA =30° , ABR = BAR =15°. Chiing minh ring QRP = 90° va QR = RP. Given any triangle ABC, construct external triangles ABR, BCP, CAQ on the sides, so that PBC = CAQ = 45°, BCP = QCA =30° , ABR = BAR =15°. Prove that angle QRP = 90° and QR = RP. ‘se eannocntctt Huéng dan: A Q Trude hét, dat BC = a, CA = b, CA = b, sit dung Dinh li ham sin trong tam gide AQC, R ta tinh dude: oS B Cc 2sinl0S° 2cos15 Tuong ty ta cing cé: a c P PB= —"__ , AR=RB = 7: 2cos15 2cos15 Ti dé, dang Dinh li ham cosin ta duse: ppre @ +67 —2accos(B + 60°) RQ’= BP +c? ~2becos( A+ 60°) 4cos? 15° 4cos? 15° ee thé, dang thiic QR = RP tuong duong voi: a@? +c? ~2accos(B+60°) _ b? 4cos” 15° 4cos? 15 <= a” — 2ac cos( B + 60°) = b* — 2be cos(.A + 60°) <9 a” —accos B + V3acsin B= b? — becos A+ V3bcsin A. Nhung theo Dinh If ham sin, asin B = bsin A nén dang thic trén tuong dusng véi: a(a—ccos B) = b(b-ccos A). (*) Mat khac, dé dang ching minh duc rang: b=ccos A+acosC < b—ccos A =acosC, a=ccosB + bcosC a—ccosB=heosC, nén (*) hién nhién ding. Diéu nay 6 nghia la ta da ching minh duge QR = RP. Hoan toan tudng tu, ta cing tinh dudc PQ’, va suy ra PQ? = 2RP*, tit d6 c6 diéu phai chting minh. Céch khée: Dang lugng gidc, ta tinh dude: ag = (6 =v2)ac vayac | BP= a QC = AQ V2, vo BP Ra= nb « en Lan lust xét cdc tam gide ARQ, BRP, CPQ ta cé: 67 RQ? = AR? + AQ? - 2AR.AQ.cos(A +60°) = AB?(2 - V3) + AC?(2- 2) - 2AB.AC(2~ V3).cos(A +60°). Thay cos(A +60°) = eos - 8 sinA va thuc hién mét sé bién déi, cudi 2- A cling ta dude: RQ? = “—*= (a? +b? +c? + SV3). Bang 24ch tién fa tuong tu, ta cing 6: Rp? = 2 Be +b? +07 + SV3), QP*= (2- y3) (a? +8? +c? +S V3). Tir dé suy ra diéu phai ching minh. Bai 40, (1976) Mét tit gidc 1éi trong mat phdng cé dién tich 32, téng cia hai canh déi dién va mét dung chéo bing 16. Hay xdc dinh nhiing dé dai cé thé cé cia duéng chéo kia. A plane convex quadrilateral has area 32, and the sum of two opposite sides and a diagonal is 16. Determine all possible lengths for the other diagonal. Huéng dan: Gia str ring x 1 d6 dai cua duéng chéo AC ma ting cia né véi hai canh déi dign AD, BC cia tit gidc 1a 16. Ha DK 1 AC va BH 1 AC. Dat y = BH + DK, goi S la dién tich tit gidc, ta co: S = *. A ét, AD + BC = (16—x), ma vi canh huyén 1a canh Ién nhat trong mét tam B gidc vuéng nén y cé gid tri lén nhat 1a (16 — x), diéu nay xay ra khi K tring A, H tring C. Luc dé, tit gidc da cho cé AD va BC vudng géc vdi AC (xem hinh vé bén trai). Mat khéc, ta co: x(16-x) _ 64~(x-8)" _ 64 a va dng thtic xay ra khi x = 8. Vi theo gid thiét, dién tich tit gidc la 32 nén ta c6 x =8. Ta chi cé thé dat dude diéu nay khi Theo gia tl =32, y=BH+DK=8. Laie nay, dé thdy d6 dai dudng chéo kia la BD = BI + ID = 2.4v2 = 8v2. Bai 41. (1976) Mét hinh hép chit nhat cé thé dugc lp day bing nhing hinh lap phuong cé canh bing don vi. Ngoai ra, ngudi ta cé thé sip xép cdc hinh lap phuong cé thé tich bing 2 chéng lén nhau, canh song seng véi canh ca hinh hép, va lap day duoc 40% hinh hop. Hay xc dinh kich thudéc c6 thé cé cia hinh hép da cho. A rectangular box can be completely filled with unit cubes. If one places as many cubes as possible, each with volume 2, in the box, with their edges parallel to the edges of the box, one can fill exactly 40% of the box. Determine the possible dimensions of the box. Huéng dan: Ta goi k 1a cin bac ba cia 2. Véi moi sé tu nhién n, ta ki hiéu n’ 1a s6 ty nhién én nhat thod man: n‘k <1. Goi kich thuéc ca hinh hop chit nhat 1a a, 6, c. Hién nhién a, b, c 1a nhitng sé ty nhién. Vi ngudi ta cé thé sip xép céc hinh lap phuong cé thé tich bing 2 ching lén nhau, canh song song véi canh cia hinh hép, va ldp ddy dude 40% hinh hop nén ta can phai cé: 40%.k.k.k.a’.b'.c' = a.b.c, hay 5a’.b’.c' = a.b.c. (*) Dé dang thay ring, véi n lan lut bing 1, 2, ..., 10, ta c6: I =0, 2'=1,3'=2, 4’ =3, 5'=3, 6 =4, 7 =5, 8'=6, 9 =7,10'=7. Ciing dé thay n’k > n-2. Nhung 6? > 0,4.8°, suy ra: (n'k)? > (n-2)° > 0,4.n', voi moi n>8. Ta cé thé kiém tra truc tiép duge ring (n'k)? > 0,4.n? voi n = 3, 4,5, 6, 7. Vi vay, suy ra a = 2 (khéng thé cé a = 1, <3 1 = 0). Ti (*), ta phai 6b hay c chia hét cho 5. Ta gia stf ring mot trong hai canh dé bing 5. Vi 5’ = 3, nén néu canh cdn lai cé dé dai n thi ti (*) ta phi ta phai cé: 5.2'5'n' =2.5.n eon = En. (*) Ta cé thé kiém tra dé dang ring = < $ , va suy ra: Fak sien, Vn27, do dé ta duge > En Wn27. (a) Bat dng thiic (***) mau thudn véi (**) nén viée cdn lai 1a kiém tra khi n <7. Tuy vay, vi waZn a sé ty nhién nén n 1a béi ca 3. Do 6 chi can kiém tra véi n =3 van = 6. Ca hai két qua nay déu chép nhan dude. Do vay, kich thuéc cua hinh hép da cho la 2x3x5 hoic 2x3x6. Cuéi cing, véi mét canh bing 2, ta cdn phai tinh dén kha nang mét canh bing 10 tré lén (nhu trén, c6 mét canh chia hét cho 5). Gia sit canh dé 1a m ( m 1a béi céa 5). Dé dang kiém tra dugc ring mo Goi m r la canh cén lai. Te (*) ta suy ra r phai thod man: — < 4. Két hop véi r nhiing tinh toan vé n’ véi n nhé 6 trén, ta suy ra r = 2. Vaya = b = 2. Nhung gid day ¢ phai thod diu kin ’ = : c. Digu nay khéng thé duse vi ta phai co St >1. Tém lai, kich thuéc cia hinh hép da cho 1a 2x3x5 hoe 2x3x6. Bai 42. (1977) Vé phia bén trong hinh vuéng ABCD, ta dung cdc tam gidc déu ABK, BCL, CDM, DAN. Ching minh ring cdc trung diém cia KL, LM, MN, NK va cdc trung diém céa AK, BK, BL, CL, CM, DM, DN, AN tao thanh mét da gidc déu 12 canh. Construct equilateral triangles ABK, BCL, CDM, DAN on the inside of the square ABCD. Show that the midpoints of KL, LM, MN, NK and the midpoints of AK, BK, BL, CL, CM, DM, DN, AN form a regular dodecahedron. Huéng dan: Ta dang phuong phap toa dé dé giai bai toan nay. Goi O 1a tam hinh vuéng, lap hé truc xOy sao cho cac diém A, B, C, D lin lugt cé toa dé 1a (1,1), (-151), (-1,-1), (4-1) Khi dé, dé dang tinh dugc toa dé cdc diém K, L, M, N lan lust 1a: (0,-2k), (2k,0), (0, 2k), (-2k,0), voi 70 A Tir do, ding céng thtic tinh toa dé trung diém, ta tinh duge toa dé cdc trung diém E, F, G, H tuong tmg cia KL, LM, MN, NK 1a: (kk), (kk), (-h ke) (“kk Suy ra ring cdc khodng cach tix E, F, G, H dén O déu bing nhau va bing: D Vi? 44? =kv2, déng thai cac vector géc O, diém mut tuong img 1a E, F, G, H hop véi truc hoanh céc géc lan lugt 1a 315°, 45°, 135°, 225°. Tiép dén, tit toa d6 da xdc dinh duge cua cc diém trén, ta dé dang tinh duge toa dé tugng ting ctia cdc trung diém P, Q, R, S, T, U, V, X cia cdc canh AK, BK, BL, CL, CM, DM, DN, AN lin lugt la: ADB) (TMS > (-h,-j),(h,- j)sCi,— h),G, A), 6 day: h=- pict-iii. Sorkin din. 70. Xe dat dani 1 ¥3-1 1-33 =ky2. WF fooling) gt Cac diém nay cing 1a ddu mut cia nhing vector géc O lan ligt hop véi truc hoanh céc géc tuong tmg 15°, 165°, 105°, 255°, 195°, 345°, 285°, 75°. Tiép dén, ta can xét cdc géc cua tam gidc vudng 0 ba canh la k, h, j. Géc x gitta h va k cb i h sinx=>, cosx=—. k k Do dé sin2x = 2sinxcosx = ye => Suy ra x = 15°. Nhu vay, 12 diém néi trén céch déu géc O va 1a ddu mit cia nhiing vector géc O hop véi truc hoanh cac géc 15° + 30n, voi n=0, 1, 2, 1, Tu dé, suy ra ring ching lap thanh mét da gidc déu 12 canh. (digu phai ching minh) 7 Bai 43, (1978) Cho P la mét diém cé dinh nim bén trong mét hinh cdu cho truéc. Ba doan thing PA, PB, PC,vuéng géc nhau ting déi mét, cé ba dau mit A, B, C nim trén mat cau. Goi Q 14 ddu mut thi hai cia dung chéo PQ cua hinh hép chit nhat ma céc canh 1 PA, PB, PC. Tim quy tich cba diém Q khi ba diém A, B, C chay trén mat cdu. P is a point inside a sphere. Three mutually perpendicular rays from P intersect the sphere at points A, B and C. Q denotes the vertex diagonally opposite P in the parallelepiped determined by PA, PB, PC. Find the locus of Q for all possible sets of such rays from P. Huéng dan: Theo bai ra ta cé PQ= PA +PB+ PC. Goi G 1a trong tam cia tam gidc ABC thi GA+GB+GC=0 nen PA + PB + PC = 3PG + (GA + GB+ GC) = 3PG. @ Do dé PA =3PG, vi thé quy tich céa Q 1a anh cia quy tich cha G qua phép vi tu tam P ti sé bing 3. Phan thudn: Tit (1) ta cé: 9PG" =PA’+PB’+PC’ +2PA.PB+2PBPC+2PCPA. (2) Do PA, PB, PC vuéng gic véi nhau ting déi mat nén 9PG? = PA? + PB? +PC?. (2) 2 =? 2 ee ee ee Mat khac: PA +PB +PC =(PG+GC)? +(PG+GA)? +(PG+GB)* 4 =3PG’ +GA’+GB +GC’ +2PG(GA +GB+GC). Do GA+GB+GC =0 nén ta duce —2 2 ~22 2 ~2 3.2 x2 PA +PB +PC =3PG +GA +GB +GC . (3) Hoan toan tudng tu thay P bing diém O vao dang thtic (3) ta c6: OA? + OB? +0C? = 30G? + GA? +GB? +GC?. (4) Tir (2’), (3) va (4) ta suy raz 6PG? = GA? +GB? +GC?, 3R? =30G? +6PG?. Tid R? =OG? +2PG?, do vay OG & = AQ AB AC Do AP = AQ nén suy ra AO’ IA trung truc cia PQ. Goi giao diém cia AO’ véi PQ 14 I. Vi PQ // BC, tam gide ABC can tai A, nén tit AO'1a trung truc cia PQ ta suy ra dugc AO’ la trung truc cua BC. That vay: PQ/BC=> AO’ vuéng géc véi BC. Goi giao diém cia AO’ véi BC 1a H, ta cé: Pr _ AU _ IQ _ BH=Hc BH AH IC Do dé AO’ Ia trung truc cia BC va AO’ di qua tam O cua dudng tron ngoai tiép tam gidc ABC. Kéo dai AO’ ct (O) tai K ta cé: ABK = 90°, O'PA = 90°. MX Vay O'PIBK = O'BK = PRB. BA Le Do O'P = O'K nén O'PK = O'KP. K Suy ra: PRB=PKA.=> PB. KB. PA KA’ Xét hai tam gidc vudng déng dang API va AKB ta cé: KB _ Pl PB _ Pl peal. KA PA PA PA Tir dé PIB = PBI. Mat khac PIB = IBC (do PQ / BC). Suy ra PBI = IBC. Ta da cé PAI=IAQ (tam gidc can APQ cé AI 1a trung truc va phan gidc). Vay I 1a tam dudng tron néi tiép tam gide ABC. Bai 45, (1979) Cho hai dutng tron giao nhau trong cing mét mat phang. Goi A 1A mét trong hai giao diém dé. Xudt phat tix A, c6 hai diém chuyén dong véi téc dé khéng déi, mai diém chuyén déng trén mot duéng tron, theo cing mét huéng. Sau mét ving chuyén déng, cd hai diém cing tré vé A. Ching minh ring cé mét diém cé dinh S trén mat phing sao cho hai diém chuyén déng néi trén luén luén cach déu S. Two circles in a plane intersect. A is one of the points of intersection. Starting simultaneously from A two points move with constant speed, each traveling along its own circle in the same sense. The nm n TA two points return to A simultaneously after one revolution. Prove that there is a fixed point P in the plane such that the two points are always equidistant from P. téng dan: Gia sé B 1a giao diém thit hai cia hai duéng tron. Goi C va D lin lugt 1a tam cia hai duéng tron. Lay E trén (C) va F trén (D) sao cho EF 1 AB tai A. Gid si P, Q tuong ting 1a hai diém chuyén dong trén (C) va (D) nhu da néi 6 dau bai, ta cé: ABP =! ace. AEQ = +ADQ. \ E 1 w tric 1a ba diém P, B, Q thing hang. Ta cing cé: trung truc cia PQ. Ma S cé dinh nén diéu nay cé nghia la P, Q luén cach déu diém S cé dinh trong suét qua trinh chuyén déng cia ching (diéu QR Given a plane (K), a point P in the plane and a point Q not in the Nhung theo gid thiét, P, Q chuyén déng cing chiéu va cung tré BPE = 180° - BAE = 180° -90° = 90°, BOF = 180° - BAF = 180° - 90° = 90°. phai ching minh). Bai 46. (1979) plane, find all points R in (K) such that the ratio OP+PR vé A mét lic nén ACP= ADQ, suy ra: ABP = eT Tw dé ta dude: ABQ = 180° - AFQ = 180° - ABP , ny) Néi cach khée, EP va FQ vung géc véi PQ. Liic nay, ta goi S 14 trung diém EF va G 1a trung diém PQ thi SG la dung trung binh cia hinh thang vung EPQF nén SG la duéng Cho mat phing (K) va mét diém P trong mat phing dé. Q 1a diém ngoai (K). Hay tim cae diém R trong (K) sao cho ti sé QP+PR at gid tri lon nhit, OR Huéng dan: Ta xét diém R trén dung tron tam P trong mat phdng (K). is a maximum. Goi X la chan dung vuéng géc ha tix Q xuéng (K). * Gid sit P khéc X. Ta nhan thy ring khi R chuyén déng trén dudng tron tam P thi QR sé nhén gid tri nhé nhdt khi R 6 cing phia véi X so véi P va ding thai R nim trén duting thing PX (hic dé, oe dat gid tri lén nhét, vi PR thi bing ban kinh dung tron, con QP khong déi). Nhan xét nay gitp ta di dén két luan ring diém R lam cho a dat gid tri lén nhdt At phai nim trén tia PX. Q Goi S14 diém trén dudng thing PX vé phia déi dién véi X qua diém P sao cho PS = PQ. Liic dé, véi R trén tia PX, ta cé: : px R QP+PR _ PS+PR_ SR _ sinROS QR QR OR sinQSR (ép dung Dinh Ii ham sin cho tam gide SQR). Nhung géc QSR oé dinh nén sinQSR khéng afi, do do, tisd PP+PR gat cuc dai khi sinRQS = 1 <> RQS =90°. Tom lai, néu P khac X thi tén tai duy nhat diém R xdc dinh nhy trén thoa man yéu cau cua dé bai. * Néu P tring X, ta van cé thé chon R sao cho RQS = 90°, va khi dy diém R cing thod man dé bai. Tuy nhién, hic nay R khong con nim trén tia PX nila, né c6 thé nim bit cit dau trén dung tron tam P ban kinh PS = PQ. Bai 47, (1981) Cho P 1 mét diém nim trong tam gidc ABC va D, E, F theo thir ty la hinh chiéu vudng géc cua P trén BC, CA, AB. Tim tt cd nhimng diém P sao cho ting BC, CA, AB PD PE ‘PFE dat gid tri cuc tiéu. P is a point inside the triangle ABC. D, E, F are the feet of the perpendiculars from P to the lines BC, CA, AB respectively. Find all P which minimise: 76 BC CA AB —+——+4+—., PD PE PF Huéng dan: B Goi ba canh cia tam gide ABC 1a : AB =a, AC =b, BC=c E D va chon ba bién x=PF,y=PE,z=PD. Ta phai tim diém cuc tiéu cla ham A c é Truéc hét, ta nhan xét ring theo dé bai, diém P nim trong tam gidc ABC nén goi S 1a dién tich tam giac ta c6: S=Sppc +Sapc + Sapp - Suy ra: ax + by + cz = 28 (S la hing sé). (*) Gidi han vj tri cba diém P trén doan MN song song véi canh BC ta duige cde gid tri cz va c/ = khong di vai moi vay bai todn quy vé tim diém cuc tiéu ca ham sé chi cé 2 bién: 4, b xy Nhung cac bién nay lai lién hé véi nhau bdi diéu kién (*) c6 dang ax + by = D, trong dé D = 2S - cz khong déi. Vay ta phai tim sé cue tri cia ham sé m trén doan nay. Nhut A= 4S 4 x Dé xét ddu cia dao ham fi Gt 7 +7, each tién nhat 1a x? (D-ax) aye —yxry). D-ax Bay gid ta thay ring f'\(x) = 0 khi x = y, tite la khi x = ; va day 1a diém cuc tiéu. Tw dé, theo tinh chét duing phan diém cua MN véi phan gidc géc A, day 1a diém duy nhét dé gid tri cba f(%) trén khoang MN dat cuc tiéu. Ac, ta dude diém P, la giao Vay diém cuc tiéu cda ham sé f chi c6 thé 1a diém thudc dudng phan gidc AK. B Néu P khong thuéc AK thi ta ké qua P dung song song véi BC va ta tim /> giao diém P, cia dung thing nay [ro v6i AK. Ta chi can ching minh ring A N c f (Po) < f (P). Véi cde diém cua phan gic AK ta c6 x=y. Theo diéu kién (*) 28 — . : ta tim dugez = 29 @* 5) ya niu thé véi cde diém eta doan AK ta 6: e ath +cotgB+ cotg) ( cosA 1-cosB , cosB_1-cosA =r —— +-——— + + —— sinA sinB sinB sinA a ols + aq) "OA+OB=AB. sinA cosA Vay AD + BC =AB. Bai 54, (1985) Cho tam gide ABC. Mét duing trdn tam O di qua cac diém A va C va lai cit cdc doan AB va BC theo thit tu tai hai diém phan biét K va N. Gia sit cdc dutng tron ngoai tiép cua cdc tam gide ABC va KBN cit nhau tai dung hai diém phan biét B va M. Chiing minh géc OMB vuéng. A circle center O passes through the vertices A and C of the trian- -gle ABC and intersects the segments AB and BC again at distinct points K and N respectively. The circumeircles of ABC and KBN intersect at exactly two distinct points B and M. Prove that angle OMB is a right angle. Huéng dan: Goi P 1a giao diém cae duéng thing AC va KN. Do KMA =BMA-BMK = BCA -BNK = KPA, nén bén diém M, P, A, K nim trén mét duéng tron. Ngoai ra: B AMP = AKP =2- ACB =2- AMB (tut gidc ACNK ni tiép). Vay digém M nim trén doan BP. Ta dit BO = 6, PO = p va goi R 1a ban kinh dung BM.BP = BK BA = 8? - R®; p PM.PB=PKPN =p?- Céng ting vé hai dAng thiic nay ta duce: BP? = 6? + p?- 2R?, khi d6 2 Oe t-R?) (p2-R?)_ BP BP Bee ope 2 -O +P a P ) <5? — p? = BO? -Po?. Tid6OM 1 BP. Bai 55. (1986) Trong mat phing cho tam gidc A,A,A, va diém P,. Dat A, = A,5 véi moi sé nguyén s > 4. Ngudi ta dung day diém P,, P,, P,..., sao cho P,,, la anh cia diém P, qua phép quay quanh diém A,,, theo chiéu kim déng hé véi géc quay 120°, véi k = 0, 1, 2,... . Ching minh ring néu Pyog, = P, thi tam gidc A,A,A, la tam gidc déu. Given a point P, in the plane of the triangle A,A,Ay. Define A, = A,s for all s>4. Construct a set of points P,, P,, Py ... such that P,,, is the image of P, under a rotation center A,,, through an angle 120 clockwise for k = 0, 1, 2, ... . Prove that if Pisss = Po, then the triangle A,A,A; is equilateral. Huéng dan: Truéc hét, ta ching minh ménh dé sau: “Néu thuc hién vai phép quay vdi tong cdc goc 1a i cla 360° thi phép bién hinh thu dude trong mat phdng la phép tinh tién". Chitng minh: Gia sit ring qua phép bién hinh néi trén, diém M bién thanh diém M, va diém N bién thanh diém N,. Ménh dé sé duge ching minh néu ta chting minh due ring MM, = NN; - Xét vects MN, sau mdi phép quay, dé dai cia né khong déi, nhung phuong cua no thay déi theo goéc quay. Sau phép quay cudi i cing phugng cua né6 tring véi phuong ban dau, vi thé vecto cd dude MN, iN, bing vects MN, vay tit gidc MNM,N, 1a hinh binh hanh, suy ra diéu khing dinh trén. Tré lai bai todn: Néu diém P, khong tring véi diém P, thi sau ba phép quay lién tiép theo gc 120° trong mat phing ta sé duge phép tinh tién theo vects — P,P) va 1986 phép quay sé cho ta 1986 : 3 = 662 phép tinh tién nhu thé, tic 14 PpPjog¢ = 662.PpP, - Vay diém Prog chi tring véi diém P, trong truéng hop diém P, tring véi diém P,, tite la phép tinh tién duge thyic hién theo vects khéng. Ta lay diém A, thay cho diém P,, thé thi diém P, cing tring véi Ay. 8 Bay gid ta dé y dén diém P,. Diém A, phai nim 6 dau dé cho phép quay quanh né theo chiéu kim ding hé bién diém P, thanh diém P,? Ti cic khodng céch bing nhau ctia diém A; téi cdc diém A, va P,, suy ra diém A, phai nim trén phan gidc géc A,A,P, véi géc P,AgA, bing 120°. Tur dé, ta dude tam gidc A,A,A; 1a tam gidc déu, vi A,A2A3 = AA3A, = 60°. Bai 56. (1987) ‘Trong tam gidc nhon ABC, dudng phan gidc trong cua géc A cit BC tai L va cdt dudng tron ngoai tiép tam gidc tai N. Tu L ta ha cac duéng vuéng géc LK va LM theo thit tu xuéng cdc canh AB, AC. Ching minh ring dién-tich tit giac AKMN bing dién tich tam gidc ABC. In an acute-angled triangle ABC the interior bisector of angle A meets BC at L and meets the circumcircle of ABC again at N. From L perpendiculars are drawn to AB and AC, with feet K and M respectively. Prove that the quadrilateral AKNM and the triangle ABC have equal areas. Huéng dan: Dién tich tam giée ABC bing LAB.ACsina, trong dé a=BAC, cn dién tich tit gidc AKNM bing 3 AN-KM (vi cdc duting chéo AN va KM vuong géc). Ta chi can ching minh ring: AB.AC.sina = AN. KM. Xét hai tam gide ACL va ANB, ta c6 CAL=NAB vi AL la phan gide va ACL =ANBvi cing chin mét cung AB cia dung tron ABC. Vay hai tam gidc ACL va ANB 4Ang dang nhau, suy ra: AB _ AL hay AB.AC = ALAN. e B _ AN AC Chi cdn phai nhan xét ring KM = AL.sinc (KM la N day cung cia dudng tron duéng kinh AL ma gic KAM Ia géc néi tiép bing a). Bai 57. (1988) Trong mat phng cho hai dudng tron déng tam, ban kinh R, r (R >r). P la diém cé dinh trén dudng tron ban kinh r con B 1a diém chuyén 8 déng trén dudng tron ban kinh R. Dudng thing BP cdt duéng tron ban kinh R 6 diém thi hai C va dudng thing 1 vuéng géc véi duéng thing BP tai P ct duéng tron ban kinh r 8 diém tht hai A (néu / 1a tiép tuyén ca dung tron thi A =P). a) Tim tap hop cdc gia tri cia bidu thite: BC? +CA? + AB?. b) Tim quj tich trung diém eda AB. Consider two coplanar circles of radii r, R (R > r) with the same center. Let P be a fixed point on the smaller circle and B a variable point on the larger circle. The line BP meets the larger circle again at C. The perpendicular to BP at P meets the smaller circle again at A Gif it is tangent to the circle at P, then A = P). a) Find the set of values of AB’ + BC? + CA’. b) Find the locus of the midpoint of BC. Huéng dan: a) Ta biéu thi ting T = AB? + BC? + CA? qua dé dai cdc doan AP, CP va PD bing cach ap dung dinh Ii Pitago va cdc ding thtic CP = DB, BC = CP + PD + DB. Ta dugc: T = (AB? + (PD + DB)")}+(4CP* + 4CP.PD + PD*)+(CP? + AP?) = (2AP* + 2PD*) + (6CP* + 6CP.PD). Theo dinh Ii Pitago, biéu thitc trong ddu ngodc thi nhdt bing Sr’, va theo dinh li vé cat tuyén trong dutng tron thi CP(CP + PD) = R? - P, vay biéu thie trong déu ngodc thit hai bing 6R - Gr”. Ta dude T = 6R ? + 2r? la mét dai lugng khéng déi va tap hgp cdc gid tri cia biéu thie BC? + CA? + AB? chi gém mét s6 duy nhat 1a GR? + 2r’. b) Goi O 1a tam duéng tron va M 1a trung diém cia AB. Thé thi OM sé la duéng trung binh cua tam giéc ADB va ta dé dang thay ring diém M la anh cua diém C qua phép vi tu ti sé 1/2 véi tam vj ty la diém ndm trén OP va chia né theo ti sé 1/2. Néu diém B chay trén duing tron Ién thi diém C cing chay trén CK dudng tron nay, cin diém M chay trén dutng tron vi tu véi né cé ban kinh R/2 va tam 1a trung diém cia OP. Vay quy tich trung diém M cia AB 1a dudng tron ban kinh R/2 cé tam 1a trung diém cia OP. Bai 58. (1988) Cho tam giéc ABC vuéng tai A, D 1a chan dudng cao qua A. Dudng thing néi tam cdc dudng tron néi tiép cdc tam gidc ABD va ACD c&t AB, AC lan lust tai K va L. Goi S va T lan lugt 1a dién tich cdc tam gidc ABC va AKL. Ching minh S > 27 . ABC is a triangle, right-angled at A, and D is the foot of the altit- -ude from A. The straight line joining the incenters of the triangles ABD and ACD intersects the sides AB, AC at K, L respectively. Let S, T be the area of the triangle ABC and the area of the triangle AKL respectively. Prove that S>2T. Huéng dén: A Lay cdc diém K,, L, lin luot trén AB, AC sao cho AK, = AL, = AD. Néi KL, va ké phan gidc cdc goc BAD va K, DAC theo thi ty cdt K,L, tai ma Tab AAK,M = AADM (c.g.c), suy ra ADM = AK\M. Tam gidc AK,L, vuéng can nén AK,M =45°, tir dé ta cé ADM =45° nén DM la phan gidc géc ADB, do dé M 1a tam dudng trin néi tiép tam gidc ABD. Tuong ty ta cd N 1a tam dudng tron néi tiép tam gidc ACD. Vay K, tring véi K, L, tring vdi L. Ta co: T=Sqe = AK AL; =3AD?, 14 1 1 2 1 == = + 2 =<: 2T AD? AB? AC? AB.AC S Vay S227. Bai 59. (1989) Cho tam giac ABC cé ba géc nhon. Cac duéng phan gidc trong ctia cac géc A, B, C c&t duing tron ngoai tiép tam gide ABC theo thi ty 87 tai A,, B,, C,. Goi A, 1a giao diém cia phan gidc trong AA, véi cde phan gidc ngoai cia cac géc B va C. Cac diém B, va C, dugc dinh nghia mot cach tudng ty.. Chimg minh rang: a) Dién tich tam gidc AgBC, bing 2 lan dién tich luc gidc AC,BA,CB,. b) Dién tich tam gidc A,B,C, khéng nhé hon 4 lin dién tich tam gidc ABC. In an acute-angled triangle ABC, the internal bisector of angle A meets the circumcircle again at A, Points B, and C, are defined similarly. Let Ay be the point of intersection of the line AA, with the external bisectors of angles B and C. Points By and Co are defined similarly. Prove that a) The area of the triangle AgByCo is twice the area of the hexagon AC,BA,CB,. 6) The area of the triangle AgByCo is at least four times the area of the triangle ABC. Huéng dan: Y a) Goi I la giao diém cae phan gidc Cat ca tam gidc ABC. Ta ching minh ring SK TA, = A,Ap. @ x Do A,iB= 445 va B,Ba, = 2 +4 nén ‘A, 7, 2 22 A, iB =IBA, vi thé IA, = A,B. Cac diém A,, B, va C, la giao diém cde phan gide cia cdc B g6c ngoai tuong ting cua tam gidc ABC, nén AA, B,B, C,C 1a nhiing dudng cao cia tam giac A,B,C,. Vay: AA,B=5—A,iB, A,BA, = 5A. Tit do A,B = A,A, va (*) da duge chimg minh. Tit (*) ta suy ra Siq,n =Sp,a.p- Chiing minh tuang ty ta dude cdc ding thic nhu vay déi véi nim tam gide dinh I con lai do luc gidc AB,CA,BC, duge phan chia thanh 6 tam giac, suy ra dpcm. b) Goi H 1a truc tam tam gidc ABC va cac diém X, Y, Z déi ximg cia H theo thit ty qua BC,, AC, AB. Ba diém X, Y, Z nay nim trén dudng tron ngoai tiép tam gidc ABC. Vi A, la diém chinh gitta cung BC nén Sya,c > Spxc, Tuong ty Sa,pc 2 Saye; Suc,a 2 Spza- Vay: Sac,pa,cn, 2 Sazexcy = 2(Spuc + Sea + Sanp)=2-Sanc- Bai 60. (1989) Mét ttf gidc 1éi ABCD cé ca tinh chat sau: a) Cac canh AB, AD va BC théa man AB = AD + BC; b) Cé mét diém P bén trong tit gide cach dudng thirg CD mét khodng cich h sao cho AP =h + AD, BP =h + BC. x 1 1 1 Ching minh rang: — > ——— +. eminene Tn JAD VBC Let ABCD be a convex quadrilateral such that the following properties is satisfied: a) The sides AB, AD, BC satisfy AB = AD + BC. 6) There exists a point P inside the quadrilateral at a distance h from the line CD such that AP =h+ ADand BP=h+BC. 1 1 1 Show that: — 2 ——= + ==. vh~ VAD BC Huéng dan: Tit gidc ABCD va diém P théa min ~~ diéu kién cua bai todn duce biéu dién & hinh / bén. Dé dang thdy ring véi AD = R va BC = | r cho truéc, dai lugng h sé cé gid tri én nl khi dudng thing CD 1a tiép tuyén chung cua hai dung tron tam A va B. Vi thé trong trudng hgp nay chi can chimg minh ring: 1 1 1 = p= te. vh VAD VBC Gia si PM 1 CD. Theo dinh li Pitago ta cé: CD=y(R +7)? -(R-1)? =2V Rr va CD=CM+MD = f(r +h)? —(r— hy)? + VR +h)? —(R—AY = “) = 2Vrh +2URh. pads VRr =Vrh-+VRh hay 1 LRA fh fh VRr R r 1 tact. 1 va VR vr’ Suy ra: Bai 61. (1990) Cho A, B, C, D 1a bén diém phan biét thuéc mét duéng trén sao cho cdc doan thing AB va CD cat nhau tai mét diém E. Trén doan thing BE ly diém M khac B va E. Tiép tuyén tai E cda dudng tron di qua ba diém D, E, M cit cdc duéng thing BC va AC theo thit tu tai F va G. Tinh tisé ES theo = AM. EF AB Chords AB and CD of a circle intersect at a point E inside the circle. Let M be an interior point of the segment EB. The tangent at E to the circle through D, E and M intersects the lines BC and AC at F and G respectively. Find lug in terms of += 2M. EF AB Huéng dan: Truéc hét, ta chimg td dé dang ring diém E nim giita cdc diém G va F. Néi D véi cdc diém A, M va B. Vi CEF=DEG=EMD va ECF = MAD nén hai tam giée CEF va AMD déng dang. Tu dé: CE.MD = AM.EF. @ =MBD va CGE = CEF- GCE = EMD - MBD = BDM nén hai tam giac CGE va BDM cing déng dang. Do vay: EG.MB = CE.MD. (2) Tir (1) va (2) ta cé: EG.MB = AMLEF, tic la EG_AM_ tAB oft EF MB (I-).AB 1-7" Chi ¥: Néu diém M nim giita cdc diém A va E thi cé thé adi chd Mat khéc, vi ECG vai trd céa cdc diém A va B, F va G va dat ¢' = 1-¢. Bang li luan twong tu ta cé: — GE AM 1-1" Bai 62. (1991) Cho tam gidc ABC va diém I 1a tam dudng tran néi tiép tam gidc. Cac dutng phan gidc trong cua cdc géc A, B, C lan lugt ct cdc canh déi dién tai A’, B’, C’. Ching minh ring: 1. _ALBLCL 8 4 AA‘\BB'CC' 27 Given a triangle ABC, let I be the incenter. The internal bisectors of angles A, B, C meet the opposite sides in A’, B', C' respectively. Prove that: 1. ALBLCI . 8 4 AA‘BBYCC' 27° Huéng dan: Trude hét ta hay ching minh Al BI Cl <8 AA' BB! CC’ 27 Muén vay, ta tim ti s6 « theo ba canh cua tam giac ABC. Ta biét ring dé dai phan gidc AA’ cla géc A theo cdc canh ké cia géc 1a 2be.cos : AA'= ——2 b+c (chting minh bing cach ding: Sagc = Spun + Sanc)- Mat khac, ta lai cd: A ) cos. 2 be (ching minh bing dinh li ham sé cosin), vi vay: wb A Al p-a, «098 9 _(p-aylb+c) _(p-aXb-+e) _ b+e AA aoe Bae 2be.cos? A ~~ 2p(p-a) atb+c™ Tuong ty ta cing gos: BL. exci Cue tanh BB’ atb+c’CC’ atb+c Nhu vay ta duge: AL BI Cl _(a+byb+eNera) AA'BB'CC (abc)? 91 Ap dung bat ding thiic Cauchy ta cé: sf a+b | bre 4 ota 2 Verne rater) 3lat+b+e atb+e at+bte a+bt+e To ee a AA'BB'CC’ 27 Bay gid ta ching minh ti 6 —ALBLCI_ 1 AA'BB.CC 4 Muén vay ta nhé lai rang: a) Néux+y=x,+y, va |x- y|<|x, - y\| thi Sry b2>c. Vi —-—— > 0, =c>|a—d| va 2 2 2 at+b+c_latb-c | n ——— >|--.—— —¢}] nen 2 1 LC] _ (at+b+c)?-a-b?-c3 -BB'.CC’ 3(a+b+c)> arbecdi (arbeell 2 2 at+b+ce) (a+b+ey (rey (rey a 2 2 al 3(a+b+ey _ Vay —ALBLCL 1 . AA'BB'CC' 4 Tit (1) va (2) suy ra dpem. Bai 63. (1991) Cho tam gidc ABC va mét diém P nim trong tam gidc. Chimg minh ring cé it nhdt mét trong ba géc PAB,PBC,PCA nhé hon hodc bing 30°. Let ABC be a triangle and P an interior point of ABC. Show that at least one of the angles PAB,PBC,PCA is less than or equal to 30°. (a+b+cy -( > > > 92 Huéng dan: Dé cho gon ta dat x= BAC, y = ABC,z = ACB, 1 = PAB,u =PBC,v=PCA. A Do x, y,z la ba géc cia tam giéc ABC ta sit dung 2 bat ding thiic lugng gidc quen thuéc sau day: sinx.sin y.sinz < 33 5 () 8 cot gx + cot gy + cot gz > V3. (2) Trong trudng hop gécz ta, ta cé thé B Cc viét (2) duéi dang: cotgx + cotgy + cotgz 2 2eotg*2” + cotge 2cotg == cot (x+y), hay néu dat m = cotg=7¥ oa thi ta cé: 2 eee >3m+e 3. 2m 2 2m Xét tam gidc PAB, theo dinh li ham sé sin ta cé = sing | PB sint PB esinG@ey) fC sin(x—t) Titdé ta duge: PC sinu PA sinv sin(x —f)sin(y—w. sintsinu Tuong ty, Vi see) =sin x(cot gf —cot gx) nén ta cé: sii 1 sin xsin ysinz_ (cot gt — cot gx)(cot gu — cot gy)(cot gv —cot gz) = Ap dung bat ding thiic Cauchy, ta cé thé viét: (cot gt —cot gx)(cot gu —cot gy)(cot gv—cot gz) < s(setattcotae toot gy otros gouge) 3 Tir dé, dp dung bat ding thic (1) 6 trén ta dude: cot gf + cot gu + cot gv—cot gx —cot gy —cot gz > 2V3. Két hop véi (2) ta duge: cot gf + cot gu +cot gv >33 . Vay mét trong ba géc ¢, u, v ma cé cotg > V3 thi géc dé phai nhd hen hodc bing 30°. Ta cé dpem. 93 Bai 64. (1992) Goi (L) 1a tiép tuyén cda dudng tron (C) va M 1a mét diém trén (L). Hay tim quj tich cac diém P thod man tinh chat: tén tai hai diém R, Q trén (L) sao cho RM = QM va tam gidc PQR nhan (C) lam dudng tron néi tiép. Lisa tangent to the circle (C) and M is a point on L. Find the loc- -us of all points P such that there exist points Q and R on L equidistant from M with (C) the incircle of the triangle PQR. Huéng dan: Cho X 1a giao diém cia (C) va (L), O 1a tam cia (C). Gid sit XO cAt (C) tai diém thit hai Z; Y 1a diém trén QR sao cho M 1a trung diém XY. Goi (C’) 1a dudng trén tiép xuc véi canh QR, cac dudng thing PQ, PR nhung 6 vé phia khac véi (C) so véi canh QR (tite dudng tron bang tiép géc P cua tam gidc PQR). Gia st (C’) tiép xtc vi QR tai Y’. i bién (C) thanh (C’), tiép tuyén véi (C) tai Z bién thanh dung thing QR, suy ra Z bién thanh Y’. Ta sé chimg minh ring QX = RY. That vay, hién nhién X va Y' 1a chan cdc dudng vudng géc vdi QR ha tit O va O' tung ting. Ta cing cé 0Q0'= ORO'= 90°. Tu d6 suy ra hai tam gide QY'O' va OXQ déng dang nhau, do dé: ) Ov" = OX Phép vi ty tam P, ti sé Yo xQ° Cing vay, hai tam gidc RXO va O'YR dong dang nhau nén: RX_O'Y' : XO YR “ Suy ra: QY'.XQ = Y'0.0X = RX.Y'R. Tirdd ta duge: 2X - _QX ____RY'__RY' os ox- py. RX QR-QX QR-RY QY' Mat khac, QX = RY vi M vita Ja trung diém cia XY, vita la trung diém QR, do dé ta cé Y tring Y'. Nhu vay, diém P di dong nbung luén oF luén nim trén tia YZ (dé thdy Z cd dinh, Y cd dinh nén tia YZ cé dinh). Dao lai, lay diém P bat ki trén tia YZ, thi bing cach li luan tuong ty nhu trén ta cing cd QX = RY. Nhung M 1a trung diém XY nén suy ra M a trung diém QR, nhu thé P 14 diém ctia quy tich. Tém lai, quy tich cha P la tia YZ. Bai 65. (1993) Cho D 1a diém nim bén trong tam gidée nhon ABC sao cho ADB = ACB+90° va AC.BD = AD.BC. a) Tinh ti sé ABCD AC.BD b) Ching minh ring hai tiép tuyén tai diém C cia dutng tron ngoai tiép cac tam gidc ACD va BCD vudng géc nhau. Let D be a point inside the acute-angled triangle ABC such that ADB = ACB + 90°, and AC.BD = AD.BC. AB.CD AC.BD (8) Prove that the tangents at C to the circumcircles of ACD and BCD are perpendicular. (a) Calculate the ratio Huéng dan: a) Lay diém B’ sao cho CB = CB’, BCB'=90° va B' nim khéc phia cia A so véi canh BC. Dé dang kiém tra duge ring cdc c&p tam gidc ADB va ACB’, DAC va BAB’ ding dang véi nhau. Ti dé suy ra: AB_AB' CD _ BB’ BD BC’ AC AB’ va diéu nay kéo theo ABCD _ BBY jg . AC.BD BC b) Goi XD 1a tiép tuyén eta dudng tron ngoai tiép tam gidc ADC tai D, véi XD nim trong géc ADB. Tusng ty nhu thé, ldy YD 1a tiép tuyén tai D cua dudng tron ngoai tiép tam gidc BDC. YB’ Lic dé, ADX = ACD , BDY = BCD, do dé ADX + BDY = ACB, suy ra XDY = ADB~( ADX + BDY) = ADB- ACB = 90°. Néi céch kha, hai tiép tuyén tai D néi trén vudng géc véi nhau. Tit day, goi (L) la duéng thing néi tam cia hai dudng tron ngoai tiép céc tam 95 gidc ADC va BDC, bing phép déi xtmg qua (L), ta suy ra hai tiép tuyén tai C cing vung géc nhau. Bai 66. (1993) Cho ba diém P, Q, R trong mat phing, ta dit m(PQR) la dé dai bé nhat cia 3 dudng cao cla tam gidc PQR, va quy uéc rang m(PQR) = 0 néu P, Q, R thing hang. Ching minh ring véi 4 diém bat ki A, B, C, X ta c6: m(ABC) < m(ABX) + m(AXC) + m(XBC). For three points P, Q, R in the plane, define m(PQR) as the minimum length of the three altitudes of the triangle PQR (or zero if the points are collinear). Prove that for any points A, B, C, X: m(ABC) < m(ABX) + m(AXC) + m(XBC). Huéng dan: Trudc hét ta cé cdc chi ¥ sau: (1) m(ABC) bing 2 lan dién tich tam giac ABC chia cho canh dai nhat cua tam gidc dé. (2) Bat ki doan thing nao nim bén trong mét tam gidc cing ngin hon canh dai nhét cia tam gidc dé. Tuy thudc vao vi tri tuong déi cia X déi véi tam gidc ABC, ta cé 3 truéng hgp I, II, Il dé xét sau day: Trusng hop I: Xét cdc dudng cao céa tam giéc ABC va ABX. Ha XY va AB vuéng géc xuéng AB. Ré rang XY > CD. Tiép dén, ké AW 1 BX va AE | BC. Néu AW ct BC tai F, thi AW > AF > AE. Tuong tu nhu thé cho cdc duéng cao ha ti A. Nhu thé, moi duéng cao cia tam gidc ABX déu dai hon dung cao cua tam gidc ABC. Do dé: m(ABC) < m(ABX) < m(ABX) + m(AXC) + m(XBC). xX x 2x A DY B A B Truéng hap I: Khéng mit tinh téng quat, ta gid sit AB 14 canh dai nhat trong tam gidc ABC. Liic dé: d(ABC) = dt(ABX) + dt(BCX) + dt(CAX). Chia vé theo vé dang thc trén cho : canh dai nhat cia tam giac ABC (canh c nay luén ngan hon 5 AB), ta duge: m(ABC) < m(ABX) + m(AXC) + m(XBC). Trudng hop IL: Dé J: m(ABY) = 0 (A, Y, B thing hang), ta c6: m(ABC) < m(ABY) + m(BCY) + m(CAY) (theo truéng hgp II) < m(ABX) + m(BCX) + m(CAY) (theo truéng hop 1). A = B Bai 67. (1994) Cho tam gidc can ABC véi AB = AC. Gid st: i) M ld trung diém BC va O JA diém nim trén duéng thing AM sao cho OB 1 AB. ii) Q 1a diém tuy ¥ trén canh BC, khong tring véi B va C. iii) E, F 1a hai diém tuong img nim trén cdc dudng thang AB, AC sao cho E, Q, F la 3 diém phan biét va thing hang. Ching minh ring OQ L EF khi va chi khi QE = QF. ABC is an isosceles triangle with AB = AC. Suppose that i) M is the midpoint of BC and O is the point on the line AM such that OB is perpendicular to AB. ii) Q is an arbitary point on BC different from B and C. iii) E lies on the line AB and F lies on the line AC such that E, Q, F are distinct and collinear. Prove that O@ is perpendicular to EF if and only if QE = QF. Huéng dan: Truéc tin ta gid st: OQE = 90°. Khi d6, cdc tit gic EBQO va QCOF néi tiép dugc (hinh a). Suy ra: OFQ = 0CQ = OBQ = OFQ. Tirdé ta cd AOEQ = AOFQ (g.c.g.), do vay QE = QF. TWlaingc eae 97 d ANY We Hinh a. Hinh b. Do lai , gid sit QE = QF, nhung OQE #90°. Trén AB va AC ta lin lugt lay E’ va F’ sao cho E’, Q, F" thing hang va OQE' =90°. Khi dé, theo chiing minh trén ta cd E'Q = F'Q (hinh b). Tu dé, AE'QE = AFQF va do vay QFC = QE'B. Diéu nay kéo theo AB // AC. Ta cé diéu mau thuan, suy ra diéu phai ching minh. Bai 68.(1995) Cho A, B, C va D 1a bén diém phan biét trén mot dudng thing va dude sip theo thi tu dé. Cac dudng tron dudng kinh AC va BD c&t nhau tai cdc diém X va Y. Dudng thing XY cét BC tai Z. Cho P 1A mét diém trén dudng thing XY khac Z. Dudng thing CP cat dudng tron dudng kinh AC tai C va M, dudng thing BP cit duéng tron dudng kinh BD tai B va N. Ching minh ring cdc dung thing AM, DN va XY déng quy. Let A, B, C, D be four distinct points on a line, in that order. The circles with diameter AC and BD intersect at X and Y. The line XY meets BC at Z, Let P be a point on the line XY other than Z, The line CP intersects the circle with diameter AC at C and M, and the line BP intersects the circle with diameter BD at B and N. Prove that the lines AM, DN, XY are concurrent. Huéng dan: a) Xét truéng hop P# X,P#Y. Do XY la truc ding phuong cia hai dudng tron duéng kinh AC va BD nén PM.PC=PN.PB. a) Déng thdi, XY 1 AD kéo theo PZD = 90°. Gid st AMO XY = {K}. Vi M nim trén dutng tron dudng kinh AC nén AMC = 90° va KMC = 90°. Vay KMC = KZC = 90°. Suy ra 98 Teton att bén diém K, M, C, Z cing thuéc mét dudng tron ma ta goi dudng tron dé la (v). Xét phuong tich cua diem P doi vdi vong tron (v) ta cé: PM-PC = PK. (2) Tuong tu, néu DNAX PN.PB (3) Ti (1), (2) va (3) ta co PK.PZ = PK'.PZ. Do P#Z nén K' = K. Vay AM, DN, XY déng quy. b) Truéng hgp P = X. Khi dé M=X va N=X. Do vay AM, DN va XY dong quy. c) Trudng hop P = Y dude xét tuong ty nhu (b). Bai 69.(1995) Xéc dinh tat cd cde sé nguyén >3 sao cho tén tai n diém A,, A, A, trong mat phing va cac sé thuc r,, r»,..., 7, thda man hai diéu kién sau: i) Khéng cé 3 diém nao trong sé cdc diém Aj, A,,.., A, thing hang. ii) V6i mdi bé 86 (i, j,k) (ISi< jf 3 for which there exist n points Ay, ... A, in the plane, no three collinear, and real numbers ry, ..., 7, such that for any distinct i, j, h, the area of the triangle A;A; Ag is, +1, + Tp Huéng dan: Ta thay n = 4 théa man cdc tinh chat (i) va (ii) cba bai ra. That vay, chi cdn chon 4 diém Ay, A,, As, A, 1a 4 dinh cua hinh vuéng canh 1 va bé 4 sd ye ba =1,2,3,4. Ta sé chitng minh n>5 khéng théa man cdc diéu kién cia bai todn. Ta thay ngay, chi can xét truéng hgp n = 51a di. Ta cé cdc nhan xét sau day. A Nhén xét 1. Néu 4 diém A, A, Ay, Ala 4 dinh cia ti gidc li AjAjA,A, thi renee A ‘That vay, theo gia thiét (xem hinh vé), tir hé thic dt(AjA jAy ) + dt(A;Ag Ay) = dt(A; AA, ) + dt(Ay AjA;) tacd rer tr tnt tr ar try tm tr; ty hay 99 Rtn arytn. Nhén xét 2. Néu A, nim bén trong tam gidc A,A,A, thi an trtrjp+hH =0. That vay, ta cé: dt(A,A Ay) + dt(A,A jA,) + dt(A, AA, =dt(A,AjA,), hay TAT AMAT, AT HT AT HM ST tT; HM. Tit day ra c6 ngay hé thtic can chimg minh. Tré lai bai todn: Xét bao Idi cia 5 diém A,,..., Ag. Cé cdc trudng hgp sau day: 1) Bao li la tam gide Aj,A;,Aj,, con A;,,Aj, nim trong tam gidc dé. Theo nhén xét 2 thi 3n, +1, +74, +7, =0, ab) Bn, try +H, +N, =0- ' (2) (1) va (2) kéo theo 37;, +37, +27, +2r,, +2r;, = 0 va dan dén diéu v6 Ksau: dt(A;, A,,Aj,)+dt(A;, A; Ai, ) +dt(A,,A;,Aj,)+dt(A,,Ai,A 2) Bao li la tet gide Aj,Aj,Aj,Ai, va Aj, ndm trong tit gidc dé. is) is Khéng mit tinh ting quat, coi Aj, thuée AA;,A,,Aj;,- Khi dé H+, =Vin 1%, 5 4 2 My => , Th thigh My _ theo nhén xét 1 thi: 3) Bao ldi la nga gidc A\A,AsA,As. Do cdc tif gidic AyAsAyAs va AgAsA,As la tit gide Idi, nén ty atts SPnHn. ry $14 = 13 +15 Tuong tu, ta cing c6: 7) =1y =73 = 14 = 75 Nhu vay, dt(AjAzA3)=dt(A,AA4)=dI(AjA2As) va As, Ay As thing hang (v6 li). Két luan: n = 5. Bai 70. (1995) Cho ABCDEF 1 mét luc gidc léi cd AB = BC = CD, DE = EF = FA va BCD = EFA = 60°. Cho G va H la 2 diém nim bén trong luc gidc sao cho AGB = DHE = 120°. Ching minh ring 100 AG +GB+GH +DH+HE2>CF. Let ABCDEF be convex hexagon with AB = BC = CD and DE = EF = FA, such that angle BCD = angle EFA = 60°. Suppose that G and H are points in the interior of the hexagon such that angle AGB = angle DHE = 120°. Prove that AG + GB+GH + DH +HE>CF. Huéng dan: Trude hét, ta ching minh Bé dé sau: Cho tam gide déu ABC néi tiép trong duéng tron tam O va M la mét diém trén cung nh BC, khi dé: MA = MB + MC. A ‘That vay, ldy N trén AM sao cho MN = MB thi BN = BM va ABN=MBC Suy ra AABN = ACBN. Tit dé AN = CM va do dé AM = AN+NM-=CM+BM. » ~- Tr Iai bai todn: Tix BC = CD va gée BCD = 60° ta suy ra BD = BC = AB. Tuong ty, tit EF = FA, EFA = 60° ta cé AE = EF = DE. Vay BE 1a truc déi xting cla tit gidc ABDE. Lay K déi xing véi C, L déi xing véi F qua BE thi hai luc gic ABCDEF va DBKAEL la anh ca nhau qua phép déi xting trén va vi vay KL = CF. Vi AKB, DEL la cac tam gidc déu va AGB = DHE =120° nén theo bé dé trén thi KG = AG + GB,HL = DH + HE. Do dé AG+GB+GH+DH +EH =KG+GH+HL2>KL hay AG+GB+GH +DH+HE2>CF. Dau dang thiic xdy ra khi K, G, H, L thing hang. Bai 71. (1996) Goi P 1a mét diém nim trong tam gic ABC sao cho APB - ACP = APC ABC. Cho D, E 1a tam cdc vong tron néi tiép tuong img cia céc tam gidc APB, APC. Ching minh ring cdc dudng thing AP, BD, CE ding quy tai mot diém. Let P be a point inside triangle ABC such that APB— ACP = APC- ABC. Let D, E be the incenters of triangles APB, APC, respectively. Show that AP, BD, CE meet at a point. 101 Huéng dan: ‘Trudc hét, ta c6 bé dé sau day: Bé dé: Goi P 1a mét diém nim trong tam gidc ABC va X, Y, Z lan luot la chan cdc dudng vudng géc ha ti P tuong ting xuéng BC, CA, AB. Khi dé: PA = ve va APB- ACP = XYZ. sinA Chitng minh: Dé ¥ ring tit gidc AYPZ néi tiép va tit dinh Ii ham sin ta c6 ding thitc thi nhat: paj AY___ AY _ YZ sinAPY sinAZY — sinA Dang thiic thit hai duge ching minh nhu sau: XZY = X7P + YZP = XBP + YAP = 90° —XPB + 90° — YPA = 180° - (360° — APB- XPY) =-180° + APB + (180° — ACB) = APB- ACB. Tré Iqi bai todn: Ta c6: APB- ACB= XYZ va APC- ABC = XYZ. Suy ra tam gide XYZ can, véi XY = XZ. Ti do: PC.sinACB = PB.sinABC, AB_ AC nhung AC.sinACB = AB.sinABC, do dé ta duge: —— = —— PB PC" Gia st BD c&t AP tai W. Khi dé, theo dinh li vé dutng phan gidc AB_ AW AW _AC 06 ——— =—— _, suy , ti dé CW ciing la phan gidc cia géc PB PW’. PW PC’ eee ACP, ta cé diéu phai chtg minh. B x c Bai 72. (1996) Cho da gidc 16i ABCDEF thoa diéu kién AB / DE, BC // EF, CD // FA. Goi Ry.R-,R, lan lugt la bin kinh cdc vong tron ngoai tiép cha cdc tam gidc FAB, BCD, DEF tuong ting; ki hiéu p 1a chu vi da gidc. 102 Ching minh ring Ry+ Ro + Rp > 7 Let ABCDEF be a convex hexagon such that AB is parallel to DE, BC is parallel to EF and CD is parallel to FA. Let R4,Rc.R, denote the circumradii of triangles FAB, BCD, DEF, respectively, and let p denote the perimeter of the hexagon. Prove that R4+ Re +R 28. Huéng dan: Tw céng thie: M B Cc N pe sinA sinB_ sinC D tasuyra 2R,= 2, A sinA 2 =P, sinC Q F E P 2Ry ED sinE (dé don gian, ta goi A, B, C, D, E an lugt 1a cde géc 6 dinh cua da gidc léi da cho). Mac di diéu nay ding khi FAB >120°, nhung trong truéng hop tang quat, diéu dé khong dung khi BE BALA, sinA do dé, truéc tién ta phai néi ring da gidc da cho phai thoa man diéu kién trén. Kéo dai BC va FE, tit A va D, ké nhiing dudng vuéng goc véi ching, ta duce hinh chi nhat MNPQ nhu hinh vé. Ré rang ta c6 BF > MQ = NP. Mat khac: MQ = AB.sinB + AF.sinF, NP = CD.sinC + DE.sinE (48 ¥ ring hai géc bi nhau thi cé sin bing nhau). Suy ra: 2BF > AB.sinB + AF.sinF + CD.sinC + DE.sinE. Hoan toan tuong tu ta ciing c6: 2BD > BC.sinB + CD.sinD +AF.sinA +EF.sinE, 2FD > AB.sinA + BC.sinC +DE.sinD +EF.sinF. Tir day ta duge: a ae 2 sinA sinC sinE 103 an (Sn sinB +BC sinB , sinC +cp/ sine , sinD sinE sinA sinc sinE sinA sinC pe( SE. SnP), mr( Ste» Snk sap SIDE, sin) sinA sinE sinC sinE sinA sinC Tiép dén, tit gid thiét cdc cdp canh déi cia da gidc song song, ta suy ra dude cdc cp géc bing nhau: A = D, B = E, C = F. Ti day, dé y dén cong thite x os 1 9 (ddu bing xéy ra khi va chi khix = 1), ta suy ra {22 KD }=2eva cé diéu phai ching minh. sinA sinC » sinE Bai 73. (1997) Cho tam gide ABC cé géc A bé nhat. Hai diém B va C chia vong tron ngoai tiép tam gidc ABC thanh hai cung (chifa A va khong chia A). Goi U 1a mét diém nim trén cung BC khéng chia A. Cac dutng trung truc cla AB va AC lan lugt cdt dudng thang AU tai V va W tuong ting. BV va CW cat nhau tai T, ching minh ring AU = TB + TC. The angle at A is the smallest angle of triangle ABC. The points B and C divide the circumcirele of the triangle into two arcs. Let U be an interior point of the arc between B and C which does not contain A. The perpendicular bisectors of AB and AC meet the line AU at V and W, respectively. The lines BV and CW meet at T. Show that AU = TB + TC. Huéng dan: Kéo dai BV, duéng nay cit vong tron tai diém thit hai X, kéo dai CW, cat vong tron tai diém thi hai Y. Tit tinh cht déi xing (dé ¥ cdc dudng trung truc cla tam gidc ABC giao nhau tai tam vong tron) ta co AU = BX va AU = CY. Ta cling cé AX = BU va AY = UC. Suy ra XY = BC. Ti dé BYC = XBY va suy ra TY = TB. Vay: AU = CY = CT + TY = CT + TB. Bai 74, (1998) Cho tit gide 181 ABCD cé hai duéng chéo AC va BD vudng gic véi nhau, céc canh déi AB va CD khong song song nhau. Goi P 1a giao diém hai dudng trung truc cia AB va DC, biét ring P nim bén trong ABCD. 104 Ching minh ring ABCD néi tiép duoc khi va chi khi hai tam gidc ABP va CDP cé cing dién tich. In the convex quadrilateral ABCD, the diagonals AC and BD are perpendicular and the opposite sides AB and DC are not parallel. Suppose that the point P, where the perpendicular bisectors of AB and DC meet, is inside ABCD. Prove that ABCD is a cyclic quadrilateral if and only if the triangles ABP and CDP have equal areas. Hung dan: Gia s& AC va BD gap nhau tai X. Goi M, N la chan cac dutng vudng gc ha tit P tuong ting xuéng AC, BD. Ta sé biéu dién dién tich cdc tam gidc ABP va CDP thanh céc thanh phan thuan Igi dé ching minh. Ta cé: D dt(PAB)= dt(ABX) + dt(PAX) + dt(PBX), dt(PCD)= dt(CDX) - d(PCX) - dt(PDX). A Nhu vay, d(PAB) = d(PCD) tuong x dung vii: ae dt(ABX) - dt(CDX) + d(PAX) _ Ys + dt(PBX) + dt(PCX) + dt(PDX) = 0. (*) 7 Mat khae, hai tam gidac ABX va CDX B vuéng nén: dt(ABX) = SAX. BX va dt(CDX) = — SCXDK, Ta lai c6: AX © AM -MiK~ AM-PNBX-BN-PM, CX=CM+PN,DX=DN+ PM. Ngoai ra: dt(PAX) + dt(PBX) + dt(PCX) + dt(PDX) = = dt(ACP) + dt(BDP) = 5 (ACPM + BD.PN). Tir €6, (*) cho ta AM.BN = CM.DN. (**) Bay gid, dua vao két qua phan tich trén ta sé gidi bai toan. Gia st ABCD 1a tit gidc néi tiép, khi dé, dé thdy P chinh 14 tam vong tron ngoai tiép va ta cé: AM = CM, BN = DN. Nhu thé, diéu kién (**) due thod va do dé hai tam gidc PAB va PCD cé dién tich bing nhau. Dao lai, gid st hai tam gidc PAB va PCD cé dién tich bing nhau, tic 1a (**) due thod man. Néu PA > PC thi AM > CM. Nhung tit cach 105 dung diém P ta cé PA = PB va PC = PD nén PB > PD, suy ra BN > DN. Tit dé AM.BN > CM.DN, mau thuan véi (**). Vay PA khéng thé Ién hon PC. Hoan toan tuong tu ta ciing cé PA khong thé nhé hon PC. Suy ra PA = PC. Diéu nay kéo theo PA = PB = PC = PD, tiic 1 tit gidc ABCD néi tiép duge trong mét ving tron. Bai 75. (1998) Goi I 14 tam ving tron néi tiép tam gidc ABC, vong tron nay tiép mic véi cde canh BC, CA, AB tai K, L, M tuong tg. Qua B, ta ké dung thing song song véi MK, dudng nay cdt LM va LK lan lugt tai R va S. Chiing minh RIS1a géc nhon. Let I be the incenter of triangle ABC. Let the incircle of ABC touch the sides BC, CA, and AB at K, L, and M, respectively. The line through B parallel to MK meets the lines LM and LK at R and S, respectively. Prove that angle RIS is acute. Huéng dan: Ta sé chimg minh: RI + SI? - RS? > 0, khi dé, ti dinh li cosin ta dé dang suy ra két qua. Vi BI vuéng géc véi MK nén né cing vuéng géc véi RS. Do dé: RI? = BR’ + BF va IS? = BI? + BS*. Hién nhién ta cé RS = RB + BS, do vay: RS? = RB? + BS? +2BR.BS. Ti dé ta dude RI? + SP? - RS? = 2BI? - 2BR.BS. Xét tam gide BRM. Ta e6: RBM = 90° -4 , RMIB= 90° -4 nén NS Cc MaB= 90° ©. Dang dinh li ham sin ta duse: BR = cos feos ©. 2 BM 22 Tuong tu, xét tam gidc BKS ta ciing cé: Bs = ose feos . BK 2222 Do dé BR.BS = BM.BK = BK’, va cui cing suy ra: RI? + SI? - RS? = (BI? - BK?) = 21K? > 0, dpem. Bai 76. (1999) Tim tt ca cdc tap hop hitu han S trong mat phing cé it ahat 3 diém, sao cho véi bat ki 2 diém phan biét A, B thuéc S, dudng trung truc 106 ciia AB cing 1a truc déi xting cua tat cd cdc diém trong S. Find all finite sets S of at least three points in the plane such that for all distinct points A, B in S, the perpendicular bisector of AB is an axis of symmetry for S. Huéng dan: Cac tap hop S cdn tim 1a tat cd nhiing da gidc déu n dinh, véi n > 2. That vay, xét tap S gom nhimg diém thoa man yéu cdu bai todn. Gia stt Aj, Ao, As, ... A, 1a nhing dinh cia bao 1di cla S. Ta sé ching t6 nhing dinh nay tao thanh da gidc déu & canh. A,,, phai nim trén trung truc cia A,A,,. (néu khong, anh déi xtmg cia né qua trung truc cia A,A,,. 86 nim ngoai bao ldi). Suy ra ring tat ca cdc canh clia da gide do bing nhau. Tuong tu, A, va Aj, phai 1a anh déi xting cia nhau qua trung truc cia A,A,, (néu khéng, mét trong hai diém dé sé nim ngoai bao 1éi). Do dé, tat ca cdc géc déu bing nhau. Vay A,A;...A, la da gidc ddu. Bay gid, bit ki truc déi xing nao cho cdc diém thuéc S cing phai la mét truc déi xtmg cho céc diém A,, i = 1, ..., &, ta suy ra ring truc nay phai di qua tam C ctia da gidc déu k canh A,A,...A,. Gia sit X 1a mot diém tuy y nim bén trong da gidc A,A,...A,. Khi 6, X phai nim bén trong ho&c ngoai tam gidc A,A,,,C. C phai 1a tam vong tron ngoai tiép AA,X (vi C 1a giao diém 3 dutng trung truc cia tam gide dé, mi dung trung truc nay 1a mét truc déi xtmg cla S), do vay, X phai nim trén vong tron tam C, vong tron nay qua A, va A,,,. Thé nhung, tat ca cdc diém cia tam gidc A,A,,,X thyc su nim bén trong dudng tron, ngoai trix hai diém A, va A,,.. Vi vay, X khong thé nao nim bén trong da gidc 16i A,A,...Ay. Ta c6 diéu phai chimg minh. Bai 77. (1999) Cho hai ving tron (C,) va (C,) nim bén trong va tiép xuc voi vong . tron (C) theo thi tu tai M va N. Gia sit (C,) di qua tam cia (C,). Dung néi hai diém caung cia (C,) va (C,) ct (C) tai A va B. Cac dutng thing MA, MB ct (C,) tuong ting tai E va F. Chiing minh ring dudng thang EF 1a tiép tuyén ciia (C,). The circles (C,) and (C,) lie inside circle (C), and are tangent to it at Mand N, respectively. It is given that (C,) passes through the center of (C,). The common chord of (C,) and (C,), when extended, meets (C) at A and B. The lines MA and MB meet (C,) again at E and F. 107 Prove that the line EF is tangent to (C,). Huéng dén: Goi O, 0, O, , 4,7),7;1an lust 1a tam va ban kinh cua cdc vong tron (C), (C,) va (C,) tuong ting. Gia sit EF c&t 0,0, tai W, dat O,W = x. Ta cdn chtg minh x =r, . Chon hé truc truc chudn cé géc 18 O,, 0,0, 1a truc hoanh, gid sit O cé toa dé (a,b). Dé ¥ ring néi chung, O va M khéng nim trén 0,0,. Gia sit AB cat 0,0, tai V. 2 Dé théy 0,V = 2 «Ching han, goi X 1a mét giao diém cua (C,) 7 va (C,), ¥ la trung diém O,X. Khi dé, cdc tam gidc O,YO, va XV O, ding 0,V _02Y dang, suy ra —— = —— ., 0,X 0,0 Phép vi tu tam M, ti sé ~, bién O, thanh O, bién EF thanh AB. 4 Do dé EF | 0,0,. Cang vay, khodng cach tit O, dén EF bing 7) lin r khoang cach tit O dén AB, suy ra: 2K Bay gid ta can xac dinh a. Bang cach tinh khoang cach tit O dén O, va O, ta nhan dugc hai phuong trinh chia avab sau day: (r-ny =) -ay +87, (r=ny =a? 48. 2 C7 -9="{o-Z} (*) r 7 DB ~ : s a yy m ty Khit b tit hai phuong trinh trén ta cé: a=Feerore., " aT Thay vao (*), ta nhan dude x = r,, dpem. Bai 78. (2000) Cho 2 duing trn c&t nhau tai M va N; C va A la 2 diém trén dudng trdn thi nhat va B, D 1a 2 diém trén duing tron thit hai sao cho AB la tiép tuyén cia cd hai duéng tron. Diém M nim giita C va D, trén dudng thing CD, va AB // CD. Cac day cung NA va CM, NB va MD cit 108 nhau tai P, Q tuong wing. Hai tia CA va DB gap nhau tai E. Chimg minh ring PE = QE. AB is tangent to the circles CAMN and NMBD. M lies between C and D on the line CD, and CD is parallel to AB. The chords NA and CM meet at P; the chords NB and MD meet at Q. The rays CA and DB meet at E. Prove that PE = QE. Huéng dén: Vi CD // AB, ta cé6 EBA =BDM; mit khac, theo tinh chat tiép tuyén, ABM = BDM, suy ra AB la phan gidc EBM. Tung tu, ta cing cé BA la phan gidc géc EAM. Tir dé, E déi xting véi M qua dudng thing AB. Néi cdch khéc, EM_L AB va suy ra EM.LCD. @ Goi X la giao diém cia MN va AB. Theo tinh chdt tiép tuyén, ta cé XA? = XN.XM = XB’, suy ra XA = XB. Nhung theo Dinh li Thales: S.hCUrrC MA NP NM MQ Nhu vay, MP = MQ. (2) 3 Ta (1) va (2) ta duge EP = EQ (dpem). Bai 79. (2000) Cho tam gidc nhon A,A,A. Vai i = 1, 2, 3, ta goi K, la chan dutng cao tuong ting ha tit dinh A,, va goi L; 1a diém tiép xiic cba ving tron ndi tiép tam gidc véi canh déi cia dinh A; Qua duéng thing L,L,, ta ldy dutng thing déi xing véi dutng thing K,K,. Tuong tu nhu thé, ldy cae duéng thing déi ximg cia K;K,, K,K, lan lugt qua cac dudng thing L,L5, L,L, tuong ting. Ching minh ring 3 giao diém cia 3 dudng thing méi nay tao thanh mét tam gidc cé dinh nim trén vong tron néi tiép cha tam gidc A\A,Ay. A,A,A; is an acute-angled triangle. The foot of the altitude from A, is K, and the incircle touches the side opposite A; at L,. The line K,K, is reflected in the line L,L» Similarly, the line K,K, is reflected in LL, and K,K, is reflected in LsL,. Show that the three new lines form a triangle with vertices on the incircle. 109 Huéng dan: Goi O 1a tam vong néi tiép. Gia sit duéng thing qua L, va song song véi A,A, c&t A,O tai X. Ta sé ching minh ring X 1a anh déi xting cua K, qua duing thing L,L,. Goi B, 1a giao diém cia A,A, va A,O. Khi dé, A,K, vudng goc voi K,B, va OL, vuéng géc L,B,, do dé hai tam gidc A,K,B, va OL,B, déng dang nhau. Suy ra: A Kyl) _ A,O L)B, OB," Nhung OA, la phan gidc géc A,A,By, do dé: A,O _ AzA3 OB, B)A;" Lay B’, trén dudng thing A,O sao cho L,B, = L,B', (B', khac véi B,, trix khi L,B, vudng géc vdi dudng thing d6). Lic do, L,B', X=A3B2A2. Ciing thé, vi L;X // A,A, nén ta c6 L,XB', = AjA2B2. Tit do, cdc tam gidc L;XB’, va A,A,B, déng dang. Suy ra (vi B,'L; = B,L,): A,A3_ XLy _ XLy BA; By'L, ByL)- K,L, _ XL, L,B, BL do dé: A,A\A3 = A,LX =L)XK +L)K,X = 2L)XK,, suy ra Nhu thé, ta da ching minh duoc , suy ra KyL, = XL, L, XK, = Saray =A,A0. L.X / AgA,, do dé ta c6 K,X // OA. Nhung OA, 1 LoL, nén K;X L LyL, va vi vay X 1a anh déi xting cua K, qua dung thing L,L5. Tiép dén, ta c6K,K,A, =A,A, Az, vi ching bing 90° -K,K,A,= 90° K,A,A2= A,ApA3 (a8 y A,A.K,K, néi tiép trong ving tron duéng kinh A,A,). Do 46, anh déi xing cua K,K, qua dutng thing L,L, 14 mét dudng thing qua X va tao vi L,X mét goc bling A,A,A3; néi cach khac, d6 la dudng thing qua X 110 va song song véi A,A,.Goi M, 1a anh déi xtmg cia L; qua dung thing AO. Ta c6: M,XL, =20XL, =2A,A,0=A,A,Aq (8 ¥ A,A, // L,X). Do dé, MX // A,Ag; néi cach khac, M, nim trén dutng thang anh néi trén (nh déi xing cla K,K, qua dung thing LL,). Tuong ty, ta cing cé M, nim trén dudng thing anh nay, cé nghia ring duéng thing M,M, la anh déi xing cua K,K, qua LyLy. Cung vay, hoan toan tudng tu, ta cling ching minh dude duéng thing M,M, la anh déi xing ca K,K; qua L,L, va dutng thing M,M, la anh déi xing cia K,K, qua L)L,. Néi cach khdc, giao diém cia 3 duéng thAng anh (déi xving) 14 tam gidc M,M,M,, suy ra diéu phai ching minh. li Phu luc MOT SO DE THI VO DICH TOAN CAC NUGC VA KHU VUC (Phan Hinh hoc) 112 Bai 1. (Thi v6 dich Bungari, ving I - 1997) Trong méi mat bén cua mét hinh chép tu gidc, ta gid str néi tiép dude mét dudng tron. Hai mat bén ké nhau bat ki cia hinh chép nay tiép xuic véi nhau. Chimg minh ring bin tiép diém cia bén dudng tron néi trén véi cdc canh day ca hinh chép 1a cdc dinh cia mot tit gide ndi tiép dude. Huéng dan: Gia str cdc tiép diém trén cdc canh bén OA, OB, OC, OD lin lust 1a A’, B’, C,, D'; cdc tiép diém trén cdc canh day AB, BC, CD, DA lan luct la: M, N, P, Q. Suy ra: AA'= AQ= AM, DD'=DQ=DP , titdé AQAMva AQDP 1a cac tam gide cin. Do vay ta cé: @, = 90" - BAD. 20 Q, =90" ADE Q; = 7 (BAD- ADC). Tuong tu ta cé: Ny =FBeD -CBA). Tu dé: O, +N; =180°. Vay tii gide MNPQ Ia néi tiép (dpem). Bai 2. (Dé thi vé dich todn Lién X6, 1982) Trén dudng tron day cua mét hinh try diing, ta ldy hai diém xuyén tam A va B; trén dudng tron day thit hai ta ldy diém C (C khong nim trén mat ph&ng (AOB), véi O la trung diém cia truc hinh tru). Ching minh ring téng cdc géc nhi dién cia géc tam dién véi dinh O va cdc canh OA, OB, OC bing 360°. Huéng dan: Goi C' 1a diém déi xing céia C qua tam O. Khi dé, néu 6 géc tam dién OABC cé cdc géc nhi dién véi cdc canh OA, OB, OC lan luct bing a, B, y thi d géc tam dién OABC'' cé cac géc nhi dién vdi cae canh OA, OB, OC’ lin lust bing 180°-a@, 180°- 8, 7. Goi 11a tam dudng tron dudng kinh AB, hic dé, trong tit dién OABC;, cdc goc nhi dién véi canh Lob pace M3 OA va OC’ bing nhau (vi ti dién nay nhan mat phan gidc cua goc nhi dién canh OI lam mat phing di xing); ngoai ra, trong tit dién OBIC’, cdc géc nhi dién véi canh OB va OC’ bing nhau. Ti dé ta dude: (180° —a) + (180°- 8) = 7 eat B+ y =180° Ge Bai 3. (CHDC Dic, vong 4, 1980) Cho 4 hinh cau cé cing ban kinh r va chung dugc sip xép sao cho déi mét tiép xtc véi nhau. Ta dung 4 m&t phing sao cho mdi mat phing déu tiép xtc véi ba hinh cdu va khong cit hinh cdu con lai. Bén mat phang dé tao nén mot tit dién déu. Hay tinh thé tich eda khdi ti dign dé theo r. Huéng dan: Goi M,, M2, M, va M, la tam cia 4 hinh cau da cho. Dé dang thay ring dé 14 4 dinh cia mét tit dién déu cé canh bing 2r va do dé cé chiéu prev. Goi 0 14 tam eda tit dign déu M.M,M,M,. Dé dang thay rang tit dién da cho déng dang phéi canh vi tit digén M,M,M,M,, hon nita O chinh 1a tam déng dang. Ta goi cdc dinh cia tit dién da cho 1a A,, A, Ay va A, sao cho trong phép bién déi déng dang néi trén M, bién déi thanh A, (i = 1, 2, 3, 4). Goi k 1a ti sé ding dang dé, ta tim ti sé ding dang. Truéc hét, ta dé ¥ ring hai mat phiing (M,M,M,) va (A,A;A,) song song véi nhau va cé khoang cach ding bing ban kinh r. Goi G 1a tam cia mat (M,M,M,) va G’ la tam cia mit (A,A,A,). Khi dé, ta cé ti sé déng dang k = OG’ : OG. Ta lai cé 0G =5h =" 6 og. ocrraft 8), r cao i hase 6 va thé tich av Vay k=1+V6. Do dé thé tich cia tit dién AyA,AsA, 1a: Vay = Frid +16). Bai 4. (Cuéc thi Todn hoc Miia Déng, Varna, 1999) Cho tam gidc ABC véi vong tron ngoai tiép cd tam O va ban kinh R. Dudng tron néi tiép tam gidc ABC cé ban kinh r va tiép xtc véi AB, 114 sw etn BC, CA tuang ting tai cdc diém C,, A, va B,. Cac duéng thing néi trung diém cac cp doan thing AB, va AC,, BA, va BC,, CA, va CB, g4p nhau tai cdc diém C,, A, va B,, Ching minh ring dudng tron ngoai tiép tam gidc A,B,C, cing cé tam O va ban kinh bing R + 7 Huéng dan: Trude tién ta chimg minh ring hinh chiéu B, cda B, lén AC 1a trung diém cia AC. Goi B, va B, lan luot 1A trung diém AB, va CB,. 6 day ta cing sé ding cdc ki hiéu quen thudc cho cdc dé dai lién quan dén tam gidc ABC. Ta cé: Tur dé ta dugce B,B; p-a_ AB, B,B, = B,B, = 224 = ABs | a3 va BBs 2 2 do dé AB, = CB,, Vi vay ta cé: B,O = B,B, + B,O = Bete + ReosB “) 7 (6 day a,B,y lan luot 1a sé do 3 géc tuong ung A, B, C cua tam gidc ABC). Dé két thuc, ta sé chimg minh ring vé phai cia (*) bing R+ 7 rdi tuang ty A,O = C,0 = Ree. Dé lam didu nay, ta sit dung bién déi tuong duong: (P=NP-a) , ReosB = R+— 2r 2 Ss Ss Sb 28 2 —2 = R(l-cos £) > —"—_ = 2Rsin? £ °p-b) 2p : e 2p(p-6) 2 115 a2 B ota aint B cog B= m2 p-b . sin Bai 5. (Cuéc thi Todn mua Xuén, Kazanlak, 30 - 3 - 1999) Mét vong tron tiép xuc ngoai véi vong tron ngoai tidp tam gidc ABC va véi hai tia AB, AC tuong ting tai cdc diém M, N. Ching minh ring tam vong tron bang tiép géc A cia tam gidc ABC nim trén doan thing MN. sin B= 2sin£ cos 8. Huéng dan: Goi O 1a tam vong tron ngoai tiép tam giac ABC va L la tam ving tran tiép xtic ngoai véi (O) va hai tia AB, AC. Gid sit R, 7, 1dn lust 1a ban kinh cdc vong tron ngoai ép, bang tiép géc A cia tam gide ABC. ‘Truéc tién ta tinh bin kinh p cia vong tron nay. Ti tam giéc OAL ta c6 AL=—2-,AO=R,OL= R+p, nor 24 sin * Dinh li ham cosin cho ta: 2 2Rpcos Bee (R+p)? =R?+ 2 ne sin sin 2 sau khi don gian, ta dugc: pcos” = 2Rsin® sin“ +.c0sB-© = 2Rsin*cosB cos© : 2 2 2 2 2 2 Mat khae, vi A_ [(p—bXP-2) 4. B P(p-b) be 2 ac” cos = |PP=) | abe =4RS 2 ab nén ta duge: p=—" 4: Goi 114 giao diém cia AL vi MN, T 1a hinh cos’ 2 chiéu cia I lén AB. Do AL L MN nén: rr AEM _ ALAM.LM _ ALLM _1 44.4.2 AM — AM.AL AL 2 116 Ngoai ra, I nim trén phan gidc géc A nén ding thc trén chimg t6 I chinh 1a tam vong tron bang tiép géc A cia tam gidc ABC, ta cé diéu phai ching minh. Bai 6. (V6 dich todn Quéc gia Ba Lan, 5 - 1999) Cho tam giac nhon ABC cé cac dinh A, B, C lan luot nim trén cdc canh B,C,, C,A, va A,B, cua tam gidc A,B,C, va ABC = A,B,C,, BCA= B,C,A,, CAB= C,A,B,. Ching minh ring cdc truc tam cia hai tam gidc ABC va A,B,C, cach déu tam vong tron ngoai tiép tam gidc ABC. Huéng dan: Goi H 1a truc tam tam gide ABC. Vi CHB = 180°- CAB= 180° C,A,B, nén ta cé A, nim trén vong tron (K,) ngoai tiép tam gidc BHC. Tuong tu, B, va C, tuong ting nim trén cdc vong tron (K,) va (K;) ngoai tiép cac tam gidc CHA va AHB. Ti dé: B,HC, =B,HA +C,HA =B,CA +C,BA =2B,A,C,, C,HA, =2C,B,A,, A,HB, =2A,C,B,, va ta suy ra H 1a tam vong tron ngoai tiép tam gidc A,B,C,. Tu cdc dinh cia dudng thing ABC, ta lan lugt ké cdc duéng thing song song véi cdc canh déi tuong tng, ki hiéu cdc giao diém cia 3 dung thing nay 1a Ay , By va Cy. Vi: AgBpCo = AyB,C; , BoCoAg = B,C, A; , CoAgBy = CA, By nén ta suy ra ring AjH, B,H, CoH cé dé dai lan luot bing duéng kinh cdc vong tron (K,), (Ky), (K;) néi trén. Ro rang, tén tai mét phép hop gitia mét phép quay va mét phép vi tu c6 cing tam H sao cho qua phép vi tu nay, tam gide A,B,C, bién thanh tam gide AjByCo. Nhu thé, truc tam H, cia tam gidc A,B,C, bién thanh truc tam H, cia tam gidc AgB,Co. Vi vay HH,Hy= HA; Ay = 90°. Cuédi cing, dé két thiic, ta sé chtmg minh ring tam © cia vong tron ngoai tiép tam gidc ABC la trung diém HH. That vay, anh cua tam giac ABC qua phép vi ty tam 1a trong tam cia tam gidc ABC, ti sé —2, 1a tam gidic AgByCy. Do d6 MHy =-2MH va tit MH =-2MO, ta thu duge: OH, =-OH. 117 Bai 7. (Ki thi Olympic 30-4 lan thit 5, khéi 10, VN, 4 - 1999) Goi R, r, p lan lugt 1a ban kinh dudng tron ngoai tiép, ban kinh duéng tron ndi tiép, nvta chu vi cia tam gidc ABC. Chimg minh ring : wf + on) + wea(t + cost) (1 + cosS) 5 2 2 2 2 2 . P(R+r)+ RAR +r) pR Huéng dan: Ta cé vé trai va vé phai cia bat dang thic cdn chimg minh lan lust bing [14 +e +e] +| sind sing. sin] 2 2 2 2 pR+ pr+4R? + Rr _ pR [ va r ,4R+r aes a - Re Ane . x a A r Ta sé ching minh rang: sin —- +sin—+sin— 21+—. That vay: 2sin % — 2eos2*E > 2cosB+€ cos B= .COs = cosB +cosC. . B Tuong tu:: 2sin 2 2 cosA + cosC » 2sin—2cosB+cosA . Suy ra: 2 . . —c r na a 2 cosA +cosB + cosC =1+—. (ddu ” = * xdy ra khi va chi khi A = B = C) Ta sé chting minh: eA sige sige eames = . qd) 222-5 Ta cé: 2 AB. Co: r r ( Ses . 1) B,R BR LB, BB, DC A,B, : va mat khéc ta edng eo EF __DF (2) EB, A,B, Tit cdc hé thiic (1) va (2) ta suy ra diéu phai ching minh. Bai 10. (Trai Rhodes *, Nam Phi, thang 4 - 1995) * Diy Ia cude thi duge 3 chic béi Rhodes University him kiém tra Iai di ngi lin cudi cling truée hic doi Nam Phi lén dung dy IMO 1995 tai Canada. Da sé cde bai thi trong dgt nay duge trich tit cdc dé thi IMO va vi dich quée gia trude d6 cia cic mide. Riéng bai hinh hoe nay Nam Phi da trich ti cue thi vé dich Toa Quée gia Uc nim 1985. Trong cing mét mat phing, cho (K) va (k) la hai dung tron co ban kinh tuong wing 1a R va r. Gid sit (K) va (k) cit nhau tai dung hai diém S va T. Tiép tuyén vdi (k) tai S cit (K) 6 B, va B thi nim trén tiép tuyén chung cia (k) va (K). Ching minh ring néu ¢ 1a géc trong cia hai tiép tuyén tai S vdi (k) va (K) thi ta cé: r = (2sin ty. R 2. Huéng dan: Gia st tiép tuyén chung tiép xuc véi (k) tai A va (K) tai B. 120 Cho tam ciia (k) va (K) ln lugt 14 O va E. Kéo dai SM, c&t (K) iai P. Ta c6 SBM= BSM= 90°—¢, suy ra ABS= ¢ vi ABM=99°. Do BA va BS cing 1a tiép tuyén véi (k) nén SB = AB, B BAO = SBO= é. Vi vay, xét tam gidc OBA ta cé: (k) Theo tinh chat tiép tuyén, Kk) BPS= 4, vi thé tam gic SBP cho ta sing = = 2k Ta dé ta duce: =-2" SB otg9 sing, R SBIR ©2 va suy ra diéu phai chting minh, véi chu y ring Y cos? sing = 2sin~.cos—. $ 2 2 Cie bai sau diy duge trich tit céc ki thi William Lowell Putnam tit 1997 dén mim 2000, Day 1a cupe thi dinh cho siah vién cic nude Mi vi Canada, nhung abiog bai hinh hoc trich & diy phit hdp vai hoe sinh gidi Todn phé thing & me ta Bai 11. (Cuéc thi William Lowell Putnam, 1997) M6ét hinh chit nhat HOMF cé HO = 11 va OM = 5. Mét tam gidc ABC nhan diém H lam truc tam, O lam tam dung tron ngoai tiép, M 1a trung diém BC va F la chan dung cao ké tit A. Tinh dé dai doan BC. Huéng dan: Trong tam G cia tam gidc ABC nim trén dutng thing HO (duéng thing Euler), va trong tam nay cing rim trén AM, cach A va M theo ti sé 2:3. Do vay H ciing cach A va F theo ti sé 2:3, suy ra AF = 15. Cac tam gidc vudng BFH va 3) AFC déng dang vi: HBc= 7 -C=CAF, do dé ta 6 05 Br =a hay BF.FC = FH.AF = 75. Ta lai cé: 121 BC? = (BF + FC) = (BF -FC)’ +4BF.FC, nhung BF - FC = BM + MF -(MC- MF) = 2MF = 22, do dé: BC = ¥22? +4.75 = V784 = 28. Bai 12, (Cuéc thi William Lowell Putnam, 1998) Mét hinh nén trdn xoay cé chiéu cao bing 3, cé day 1a hinh tron cé ban kinh 1. Mét hinh lap phusng néi tiép trong dé sao cho mét mat thi nim trén mat phing day, 4 dinh cia mat déi dién cua hinh lap phuong thi tua trén m&t nén. Hay tim chiéu dai cua canh hinh lap phuong. Huéng dan: Ta xét mat phing chia truc hinh non va hai dinh déi dién cua day hinh lap phuong. Mat phing nay sé cit hinh lap phuong theo thiét dién 1a hinh chit Q P nat MNPQ (xem hinh) cé mét canh bing MQ = s, canh kia bing MN = sV2, véi s 14 d6 dai canh cua p hinh lap phuong. “MON ho aa a hepa 7 Mat phang ndi trén cing c&t hinh nén theo thiét dign 1a tam gidc SAB. Cac tam gidc déng dang AQM va ASO cho ta: 2 SD 9v2-6 Se »suy ra s= : 3 1 7 Bai 13. (Cuéc thi William Lowell Putnam, 1999) Cho tam gide ABC cé AC = 1, géc ACB = 90°, va géc BAC= f. Goi D la diém nim gida A va B sao cho AD = 1; E 1a diém nam giita B va C sao cho EDC = f. Dung vuéng géc véi BC tai E cit AB tai F. Hay tinh lim EF. f70 Huéng dan: Xét tam gidc ACD, ta cé: ACD = ADC = 90° -£, suy ra DCE= £. ‘Tit dé, ap dung Dinh li ham sé sin cho tam gidc CDE ta duge: CE _sinf DE sin f 2 122 tisé nay tién dén 2 khi f —> 0. Ta lai cé: A BAD= 90" - f, EBB= 90° -L. De E 2 Do dé ee ee tién dén Lkhi f 0. BE cos B E Cc 2 EB 1, EB 1 EF _EB a Se ee EF LEB do a6 EF tién dé mre ec 2 Be 3 SAC BC Aci a Bai 14. (Cuéc thi William Lowell Putnam, 2000) Cho A, B, C 1a cdc diém cé toa dé nguyén va nim trén mét vong tron ban kinh R trong hé truc Decartes vuéng géc. Ching t6 ring cé it nhdt mét trong cdc kho&ng cach AB, BC, CA Jén hon 1 RB. Huéng dan: Nhu Ié thutng, ta ki hiéu dé dai ba canh cia tam gidc ABC laa, 8, c. Trude hét, ta sé ching minh ring abe = 4SR, vdi S 1a dién tich tam gidc, R 1a ban kinh vong tron ngoai tiép. ‘That vay, goi O 14 tam vang tron. Kéo dai AO gap vong tron tai K. Cho AH 1a dung cao cua tam gide ABC. Khi 46 dé thdy hai tam gidc ABH va AKC déng dang. Suy ra: AB_ AK c _2R oS = * hay AH AC AH Bb Ti d6 abe = 2R.a.AH = 4SR. Bay gid ta dé y dén gia thiét ring cdc diém A, B, C cé toa dé nguyén. Xét hai vector 1a hai canh cla tam gidc c6 géc cing mét dinh. Ré rang mét atta dé Ién tich hitu hung ciia hai vector nay 1a dién tich tam gidc ABC, nhung cac thanh phan toa dé cia ching 1a sé nguyén nén suy ra dé lén cua vector tich cing vay, nghia 1a phai lén hon hode bing 1. Suy raS 24, dodo abe 22R>R. 7 Tit d6, it nhdt 14 mét trong 3 canh a, 6, c phailén hon R3. 123 Cage thi USABTS chon tai nding Ton hoc cia Mi (USA Mathematical Talent Search) dude gido sf George Bezsenyi t6 chile lin dau tién vio nim 1988, duéi sq tai tro cia Hoe viga Ki thudt Rose-Hulman va COMAP (Consortium for Mathematics and It's Applications - Hi Toin hoc va ting dung). Day la mot chuyén mye thudng xuyén cia t8 bio mang tén Consortium, duige COMAP xuit bin. Dui day, ching ti trich gidi thigu cimg céc ban hai bai Hinh hoe ti! cuge thi USA Mathematical Talent Search nim hoc 2000 - 2001. Bai 15. (USAMTS, ving 2, 2000 - 2001) Trong tam giée ABC, nhu hinh bén, cdc T doan PQ, RS, TU tuong ung song song véi cdc canh AB, BC, CA; chung giao nhau tai X, Y, Z. Hay xdc dinh dién tich tam gidc Q ABC néu mai mét trong cdc canh PQ, RS va t R > TU chia tam giée ABC thinh 2 phan cé dign tich bing nhau va néu dién tich tam gidc XYZ bing 1. Dap sé cua ban nén dat duéi dang a+bv2, véia, b la cdc sé nguyén duong. Huéng dan: Dat AB = x. Vi PQ chia tam gidc ABC thanh 2 phan cé dién tich bing nhau nén (33) =}: suy apg. Vi cdc tam gide PCQ, UTB va ASR ding dang nhau, hon nia, lai 6 cing dién tich, nén ching bing nhau. Ti dé: reece 2xoxv2 yy. xV2 _4x=2xV2 _3xv2-4x 2 2 2 2 Hai tam gidc XYZ va ABC déng dang nén: dt(XYZ) -() _(3¥2-4) _ 17-122 dt(ABC) (AB) | 2 >) 2 Ma dt(XYZ) = 1 nén ta cd: dt(ABC) = 34424V2. 2 17-12¥2 — Bai 16. (USAMTS, ving 2, 2000 - 2001) Chimg minh ring trong mét da dién, tén tai 2 dinh sao cho ching cé cing sé cdc canh gip nhau tai Huéng dan: Truéc hét, trong mét da dién, mai dinh phai cé it nhat 3 canh gap nhau (ttic la phi cé it nhdt 3 canh xuat phat tit cing mét dinh). 124 Gia si da digén c6 n dinh. Néu nhu sé cdc canh gap nhau tai céc dinh 1a nhiing sé khdc nhau (ting déi mét) thi cé mét dinh cé sé canh gap nhau dn nhat la n + 2 (3, 4, .... n+2). Thé nhung, da dién cé n dinh thi tai mai dinh sé khéng thé cé nhiéu hon n—1 canh gap nhau, vi mdi canh 1a mét té hop cua 2 dinh phan biét. Mau thudn nay chiing té khong thé nao sé céc canh gap nhau tai cdc dinh 14 nhiing sé khac nhau. Néi cach khac, phai tén tai 2 dinh sao cho chung cé cing sé cdc canh gap nhau tai mdi dinh dé. Bai 17. (USAMO. Dé thi vé dich Todn todn nude Mi , 1997) Cho tam gidc ABC, bén ngoai tam gidc nay, vé cdc tam gidc can BCD, CAE, ABP. Chiing minh céc doan vuéng géc ké ti A, B, C tuong ting xuéng EF, FD, DE bing nhau. Huéng dan: Truéc tién, ta ching minh ring véi 4 diém tuy ¥ W, X, Y, Z trong mat phing, WX | YZ néu va chi néu WY? - W2? = XY? - XZ?. ) Dé chimg minh, ta xét hé truc vuéng géc sao cho W =(0, 0), X=(1, 0), ¥ = (x9), Z= (x2,y2)- Khi dé, (*) tr than xf + yf —x} —y} =(x,-1)? + yf —(%2 -1)? - 93, tuang dudng véi x, = x2. Didu nay dung khi va chi khi YZ vudng géc véi truc hoanh WX. Bay gid, goi P 1a giao diém hai dung vuéng géc ha tit B va C tudng ting xuéng FD va DE, tit ménh dé trén suy ra PF? - PD? = BF* - BD?, PD? - PE? = CD? - CE?. Tu gid thiét vé cdc tam gidc can ta cé AF = AF, BD = CD, CE = AE, do 6, PF? - PE? = AF? - AE”, suy ra PA cing vuéng gic véi EF, la diéu phai chiing minh. Bai 18. (Dé dé nghi cho IMO cua Canada, 1993) Cho tam gidc ABC cé ban kinh dudng tron ngoai tiép bing 1. Goi r la ban kinh dudng tron néi tiép ABC, con p 1a ban kinh dudng tron néi tiép cda tam gidc truc tam A'B'C’ (nghia 1a, AA’, BB’, CC’ la cdc dung cao cia tam gidc ABC). Ching minh 1 2 psi-3dery. 125 Huéng dan: (i) Xét tam gide ABC, ta 06 cosA + cosB + cosC = Leb ler (R=). (dé ching minh cong thic trén, ding Dinh li ham sé cosin va cdc cong thie tinh dién tich S = pr =e vip Ta nda chu vi) (ii) Tiép dén, ta chimg minh p = 2cosA. cosB. cosC. Dé y, true tam H cia tam gide ABC ciing 1a tam ving néi tiép cia tam gidc A'B'C’. Tit H, ké HX vudng géc xuéng A’C’ thi ta co HX = p. Tam gidc A'BH cho ta: tgA'BH =HA'/BA’, suy ra HA' = ccos Big(90° - C) = ccos Boot gC . Mat khéc, tit gide CA'HB' néi tiép nén XA'H = HA'B'=B'CH, tirdé: sinXA'H = sin(90° - A) = HX/HA' = p/ HA’. Vay: p=HA'‘sin(90° - A) = c.cosB.cotgC.cosA = — cosA.cosB.cosC = 2 cosA.cosB.cosC. sinC (iii) Tiép dén, dé ¥ ring ta 06 2cosA.cosB.cosC + cos*A + cos’B + cos?C = 1. That vay, diéu nay xdy ra vi ta co: 4cosA.cosB.cosC = 2(2cosA.cosB).cosC = 2(cos(A+B) + cos(A-B)).cosC = 2cos(A+B).cosC + 2cos(A-B).cosC = cos(A+B+C) + cos(A+B-C) + cos(A-B+C) + cos(A-B-C) 1 - cos2A - cos2B - cos2C = 2 - 2cos*A - 2cos"B - 2cos*C. (iii) Sau cing, sit dung bat ding thie Cauchy-Schwarz ta duge: 5 cosA + cosB + cosC)*< cos"A + cos”B + cos’C. Tir do: p+3d +1)?= 2cosA.cosB.cosC + i cosA + cosB + cosC)? S 2cosA.cosB.cosC + cos?A + cos”B + cos’C = 1, va suy ra bét dang thiic cdn chiing minh: p <1- ia +n. 126 Phiin 2 CAC BAI TOAN SO HOC - DAI SO - GIAI TICH HINH HOC TO HOP 128 KIEN THOC BO TRO Nhing kién thie bé tré dude trinh bay dudi diy bao gim cde Khai niém va dinh li ma da s6 khéng cd mat trong chudng trinh THPT hién hinh. Ching thuc sv cdn thiét cho nhiing hoe sinh gidi todn d phd thong. Mot phin kién thitc bé trg sé duge sit dung true tiép trong sich nay, s6 con Iai la nhiing dinh li toan hoc néi tiéng ma vige chiing minh ching cing da 1a mot bai toan Ikhé déi vdi bie phd thing. Céc tée gid trude diy di bo qua phin chting minh vif dé si! pham hode mye tiéu sit dung sich. 6 day, chning tai cS ging dia vio phan ching minh, nhuing di nhién khéng chi tiét 6 da sé trudng hop. Thuc ra, moi dieu tu né da ro rang va c6 thé né sé duge hoan thién chi tiét déi vdi nhiing ban doc nio quan tim mét céch nghiém tic, _ 1.86 NGUYEN 1.1. Phép chia cé du Dinh li 1 Véi hai sé nguyén bat ki a va b, b+0, luén tén tai duy nhdt mét c&p sé nguyén q var sao cho a = bq +r va O 0 vab > 0. Liic dé: Néua =0 thiq=O0var=0. Néua>0,b>0vaa 0, b> 0 va a > b, xét day 86 b, 2b, 3b, ... , ta tim due sé qb sao cho qb < a<(q+1)b. Dat r=a—bq,tacd a=bq+rva O 0. Lic dé, dé y —a>0 va ta dp dung truéng hgp 1 cho -a>0. Truong hop 3: a <0 hoic a> 0 vab <0. Lic dé, dé y —b>0 vata ap dung cac trudng hgp 1 va 2 cho —b>0. Nhu vay, ta ching minh dugc su tin tai cia c4p sé nguyén q var nhu da néi. Dé chimg minh tinh duy nhit, ta gia su con cé thém cdp q’ va r' thod man Dinh Ki, ta cé: a=bq+rva 01q'-q| <1. én day, nhé ring |q'-q| la mét sé ty nhién, ta suy ra q = q' va 6 diéu phai chimg minh. 1.2. Uéc chung Ién nhat (UCLN) va béi chung nhé nhat (BCNN) Xem nhu ban doc da biét r6 vé khai niém uéc sé va béi sé, 3 day chiing téi nhic lai cdc dinh nghia sau: Dinh nghia 2 Sé nguyén d duge goi 1a UCLN cia n sé nguyén aj,a2,...,a, néu d 1a uéc chung cia aj,a2,:..,a, va néu e 1a mét uéc chung khac cia ching thi e|d. Ta thudng ki higu d =(a),a9,...,a,). 129 sn tatoc pet Dinh li3 UCLN cia n sé nguyén khac 0 luén tén tai (va di nhién duy nhat). Ching minh: Dinh li nay cé phan ching minh kha dai, ban doc quan tam c6 thé tim thdy 6 cdc gido trinh Sé hoc 6 bac dai hoc (kién thitc dude str dung hoan toan cé thé hiéu duce é trinh dé khd gidi bac Phé théng). Luge dé ching minh nhu sau: Gia sit ta cé cdc sé nguyén khac 0: a),29,...,a,- Dat 7 A= {x k= Sz; 2; <7. isl Ton tai phan ti bé nhat d trong A. Khi dé, d sé 1a UCLN cia cac s6 a) ,a2,...,a,. Dé ching minh d chia hét cdc sé aj ,a3,...,a,, Dinh Ki 1 duge st dung, sau cing, ching minh ring néu c|a;, i = 1, 2... n thi clk véi moi k € A, suy ra c|d. Dé tim UCLN cta 2 sé nguyén a, b, ta cd thuat toan Euclide sau da; Bé dé 4 (co sé cia thuét todn Euclide) Néu a = bq +r thi (a, b) = (b, r). (Euclid of Alexandria, sinh khodng 325 BC, mat vao khoang 265 BC tai Alexandria, Egypt) Chiing minh: Dat d= (a, b) va u=(b, r). Tu dla va d|b, ta suy ra: d|(@—bq)=r, do dé ta cé d|r va db, vay dla uéc chung cia b va r, suy ra dju, dou = (b, r). Hoan toan tudng tu, ta cé thé suy ra u|(bq +r) va tit dé ta duge uld. Vayu=d. Tu bé dé trén, ta cé thudt todn Euclide nhu sau: Dé don gian, ta trinh bay véi a, b nguyén dusng. Chia a cho b, ta dudgc thuong q, va s6 du 1; Chia b cho 1, ta dude thuong q, va sé du r,; Chia 1, cho ry, ta duge thuong q3 va sé du 5... Khi tiép tuc qua trinh trén, ta dude mét day gidm b, 1), 1), 13... sn teannecqact Day nay dan dén 0, va dé la cdc sé ty nhién nén ta sé thyc hién khéng qua b phép chia. Thuat todn sé két thtic sau mét sé hau han buéc va bd dé trén cho ta: (a, b) = (b,1)) =... th, Dinh nghia 5 Ta ndi b 1a béi chung nhé nhat (BCNN) cia n sé nguyén khac 0: ay,22....,4, néu b la béi chung cia a,,ay,...,4, va néu e 1a mét béi chung khac cia ching thi ble. Ta thuéng ki hiéu d = [a,,a2,....a,]. Dinh li6 Néu x, y la hai sé nguyén khac 0, BCNN ctia chting luén luén tén tai va bing 2 (xy) Chiing minh: Do (x, y) la UCLN etia x va y nén hién nhién va — déu ey (x,y) 1a nhiing sé nguyén. Tit do, b = la béi chung cia x va y, bai vi: XY) y b=x. = . (x,y) x,y) Tiép theo, ta gia sit c 1a béi chung khac cua x va y, suy ra tén tai sé nguyén m sao cho c = xm va ta cé y|c nén: y | c = y xm @& y) &y) & y) (x.y) Nhung i = 1 nin al hay m=u—, véi Gy) (xy) & y) “Goy) u 1a 6 nguyén nao dé. ee ev c= xu—— = Gy) (x, y" hay cA boi cia b= CS, suy ra didu phdi chitng minh. xu) Bing quy nap, ban doc cé thé mé réng dinh Ii trén dé cé dugc khdng dinh vé su tin tai BCNN cua n sé nguyén khac 0. 1.3. Phép chia hét va sé nguyén té Ciing nhu trén, xem nhu cac ban da biét ré cdc khdi niém va tinh chat thong thudng. 131 Nhic lai ring hai sé nguyén a va b dudc goi la nguyén 16 cing nhau, ki hiéu (a, b) = 1, néu usé chung lén nh4t cia ching la 1. Cac sé a, b, n, m... dugc sit dung duéi day déu 1a cdc sé nguyén. Néu m chia hét cho n ta cing néi n chia hét m. Dé dang ching minh cdc tinh chat don gian sau day: (1) Néu d chia hét a va d chia hét b thi d chia hét a+b. (2) d chia hét b néu va chi néu d chia hét -b. (3) Cho d chia hét a. Khi dé, d chia hét b néu va chi néu d chia hét a+b. (4) UCLN (a, b) = UCLN (a, a+b). Ti dé, (a, b) = 1 khi va chi khi (a,a+b)=1. Dinh li 7 (Dinh li co ban vé sé nguyén t6) Moi sé nguyén dusng n, n > 1, déu c6 thé duce viét mét cach duy nhat (khong tinh dén viéc sip xép cdc nhan tt) dudi dang: n=py'py Pes véi k ,e; 1a sé ty nhién va p; 1a cac sé nguyén té thod man: 1 m, sao cho méi léng chi chita mét con. Chi ¥: Cé nhiéu cach phat biéu khdc nhau, nhung phat biéu trén 1 phé bién nhdt. Chang han, trong chuong trinh béi dudng déi tuyén hoc sinh thi V6 dich Todn Quéc té, gido su Greg Gamble goi dé 1a Nguyén li Lé chuéng b6é cdu (Pigeon - Hole Principle: If 5 pegeons fly into 4 pigeon-holes then at least one pigeon-hole contains 2 or more pigeons). 135, 1.8. Mét sé cong thiic vé té hgp va nhi thie Newton (Sir Isaac Newton, 1643 - 1727, nguéi Anh) ar ta dé dang ching minh duoc cac cong thitc thudng xuyén duge sii dung sau day: Ch=Cy*, Ch+Ch! =Chty. Bang quy nap, sit dung cac cong thiic vé té hap néu trén, ta cing chung minh dugec céng thiic nhi thie Newton: n (a+b)? = )Cha" "bk. k=0 Tw cong thiic tinh té hop Ck = 1. S6 THUC 2.1. Mét sé kién thiie co ban Dinh nghia 16 (cén trén vd can dudi) Cho tap A cdc sé thuc. Sé thuc a duce goi 14 can trén cia A néu moi x € Atacéx < a. Dinh nghia tuong tu cho can duéi cua A. Nguyén li supremum Moi tap con A khac rong va bi chan trén (tuong ting, bi chan dudi) cia R déu cé can trén bé nhat (tuong ting, cin dudi lén ohat). Nguyén li Archimede (Archimede, 287 - 212 B.C., Hy Lap) Véi moi sé thuc r, tén tai sé ty nhién n sao cho r xeEZ; 3) x=fxJo0 fx}+ by) +y}< beh. Chitng minh: Ban doc ty ching minh cdc tinh chét trén. Ching han, é day ta ching minh tinh chat 6. Theo tinh chit 4., ta c6: x+y—l<[x+y] 2[a}.[2b] > 2[b], nén ré rang bat dang thitc (1) dung. 2) 11. Khi 6: (2a]= 2[a]+1, va vi [2b] > 2[b], nén bat ding thue (1) dung. Ménh dé dude chimg minh. 2.3. Ham 16i Dinh nghia 22 Ham f(x) xac dinh trén [a, b] duoc goi 1a /6i trén khoang dé néu vdi moi x,.x7 €[a,b] ta co: (222) FOG) + £00) | 2 2 dau dang thiic xdy ra khi va chi khi x, =x . Ta cing ndi ham f Jém trén [a, b] néu bat ding thiic trén cé chiéu nguéc lai. Tu dinh nghia trén, ngudi ta chimg minh duce mét trudng hop dac biét cia bat ding thic Jensen sau day (Johan Ludwig William Valdermar Jensen (1859 - 1925), nguéi Dan Mach, mét ki su va 1a mot nha toan hoc nghién ctu nhiéu vé Giai tich, Dai s6, cae déng gop cia ong rat quan trong cho nganh Gidi tich Idi - Convex Analysis). Mot trong cdc phuong phap chig minh 1a ding phuong phap quy nap kiéu Cauchy: truéc tién, gid sit céng thite ding vdi n, ching minh cong thie dting vdi 2n, réi tit dé, chiing minh céng thite dung cho n+ 1. 138 Dinh li 23 (Truéng hop dac biét cia bat déng thc Jensen) Néu ffx) la ham di trén (a, b| thi véi moi x, .x9,....x, €[a,b] ta cd {= +XQ+X3 tt Xq \< £ (xy) 0X2) +... + (XQ) 2 j 2 dau ding thic xay ra khi va chi khi x, =x 2.4, Mét sé bat ding thie Dinh nghia 24 Ta néi trung binh céng (hoac trung binh sé hoc- arithmetic mean) cua cac sé thuc a; .a.....a, la: trung binh nhan (hoac trung binh hinh hoc - geometric mean) cia cdc sé thuc khéng am aj,a9,...,a, 1a: faja9...a, , va trung binh diéu hod (har- «monic mean) cita céc s6 thue ay .a2 a, la: Dinh li 25 (Bat deing thitc vé trung binh cong vé nhan) Véi cdc sé thuc khong 4m a),a9.....a, ta c6: +248 a;tay 7 rae =. n dau bang xay ra khi va chi khi a, =a = Ching minh: Xét ham f(x) =Inx. Dé thay f(x) la ham léi trén mién xac dinh cua nd. Tit bat ding thuc Jensen (Dinh Ii 1) ta suy ra Ina, +Ina+...+Ina, < In2! +azt..tay, : n n tu dé suy ra bat ding thiic cin chimg minh. Ban doc dé dang ching minh trudng hop dau bang xay ra. Bat ding thiic trén cin dug goi la bat ding thie Cauchy (Augustin Cauchy, nha toan hoc Phap, 1789 - 1857). Hé qua 26 (Bat dang thiic gilda TB nhén va TB dieu hoa) Gia sit x, ,x2,....X_ 1a cde sé thuc duong. Khi d6: _-_ Dau bang xay ra khi va chi khi x, = x9 Ching minh: e p 1 ot Ap dung Dinh li 25 cho cac sé thuc duong —, —. a ea Dinh li 27 (Bat déng thite Cauchy - Schwarz) Cho cdc sé thuc a) ,a3,....a, Va bj ,b2,....b,- Khi dé: (aj +a} +...4.a2 (bp +b3 +...+b2)> (ayb, tab +...+a,b,)”. Dau ding thtic xdy ra khi va chi khi b, = kaj. duong nao dé. Chi ¥: Mot sé noi goi bat ding thie trén 1a bat ding thic Schwarz (Hermann Amandus Schwarz, 1843 - 1921), trong khi mét sé ndi khac, dic biét 1a Nga, thi goi la bat ding thtic Cauchy - Buniakowski (Buniakowski, 1804 - 1889). Cauchy da dé cap dén vao nim 1821, Buniakowski thi vao nam 1859, con Schwarz 14 nim 1884. Déi khi, bat dang thiic trén con dude goi 1a Cauchy-Buniakowski-Schwarz. (cf. S.M. Nikolsky, A Cours of Mathe- -matical Analysis, V.1, Mir Publishers, Moscow, p. 183). Ching minh: Mot trong nhiing cach ching minh bat dang thi trén 1a dat f(x)= Sax +b; 5 ist va dé dang bién dii ffx) thanh f(x) = Ax? + 2Bx +C, trong dé, 0 a ? B= Diajb, vaC= b,?. 1 isl Tit dé, ta di dén diéu phai chimg minh bing cach dé y ring f(x) 20, Vx B?-AC<0. Dinh li 28 (Bat dang thie Bernoulli) Gia sit x,,X2,....X, 14 cdc s6 thuc cing dau va Ién hon 1. .N, vdi k 1a sé thue 140 Khi dé ta cé: [[arver ds. (aniel Bernoulli, nha toan hoc Thuy si, 1700 - 1782) Ching minh: Pavers nal Dé thy bat dang thtic cdn chiing minh ding véi n = 1, 2. Gia st Ta 06: []0+x;)= Os xenf [rs od né ding véi n, nghia 1a isl n+l n+l =| 1+ ox; saan 2]1+>, G day, x, ,x2,...,.x, la cde sé thyc cing déu va Ién hon —1 nén a Xnst Xj 20. isl Tit dé ta c6 diéu phai ching minh. Hé qua 29 Néu a> -1 thi véi moi sé tu nhién n ta déu cé: (l+a)" >1+na. Chiing minh: Dé thay day la truéng hop dic biét cia bat ding thitc Bernoulli ndi trén. Da sé cdc sdéch phé théng thudng trinh bay bat dang thiic Bernoulli duéi dang nay (xem Dai s5 10, Ban KHTN, tai liéu gido khoa thi diém, NXB Gido Due, 1995). Duéi day 1a bat ding thitc duge st dung 6 Bai 137 (1995). Dinh li 30 (Bat dang thite T-sé-bu-sep) Cho hai day hitu han cac sé thuc aj,a9,...,a, Va b,,b9,...,b, Khi dé: a) Néu ca hai day sé trén cing tang hoc cing gidm thi ta co x b; Ya oi 141 a) Néu mét trong hai day sé trén tang va day sé cdn lai gidm thi tacd dab, x oi n n Dau dang thie 6 a) va b) xdy ra khi va chi khi hodc a, =a) =...=a,, hodc bj =by =...=by- Ching minh: Chig minh hoi dai, dé nghi ban doc tham khdo, ching han, GS. Phan Diic Chinh, Bat déing thile (séch ditng cho hoc sinh cdc lép chon va cdc ldp chuyén Todn), NXB. Gido Duc, 1994, p. 53 - 56. T-sé-bu-sep (P. L. Chebyshev, 1821 - 1894), nha toan hoc Nga c6 nhiéu déng gop dang ké cho Li thuyét 36 (Theory of Numbers). Tén cha ng cé khi duge mét sé sich tiéng Anh phién thanh Tschebycheff, do vay, ki hiéu T,(x) dé chi da thifc Chebyshev rat thuing dude ding trong cdc séch vé Phuong trinh vi phan (differential equations). IIL MOT S6 CHU ¥ VE DA THUC Tinh chdt 31 (vé nghiém hitu ti cia da thite nguyén) Cho da thiic khac 0 vdi cac hé sé nguyén, n > 0: f(x) =ag +ajxX+...4a,x". Khi dé, néu f(x) cé nghiém hiv ti thi né sé cé nghiém hitu ti dang 5 vi Tlag va slay. Ss Ching minh: Gia sit da thite f(x) c6 nghiém hau tic = m/q, ta goi dla UCLN cua m va q, ta cé m= rd, q = sd, khi dé: c = r/s, véi (r,s) = 1. Vi f(c)=0 nén ta suy ra: ags" tayrs"! +..4a, yr stage" =0. Ti ding thtic nay ta suy ra s|a,r" va rjags". Mat khae, tit (r, s) = 1 ta duge (s,r")=1 va (r,s")=1. Dén day ta suy ra s|a,r", diéu nay kéo theos|a, va rlagr", tit dé suy ra r|ag, do vay ta dude diéu phai ching minh. 142 Hé qua 32 Néu tat ca cdc hé sé cia phuong trinh x" +a,x""! +..ta, =0 déu 1a cdc sé nguyén thi tat cd cdc nghiém hiu ti néu c6 cua phuong trinh nay cing déu 1a cdc sé nguyén va 1a mét trong céc ude sé (am, dung) cua a, . Tinh chdt 33 (Tiéu chudn bdt khé quy Eisenstein - Eisenstein’s irreducibility criterion ) Cho da thiic vdi cc hé s6 nguyén f(x) =ag +a)x+...+a,x" théa man diéu kién: tén tai mét sé nguyén té p sao cho Piao, Play.-. Plants nhung p” khéng chia hét ay va p khong chia hét a,. Khi dé, da thiic f khéng thé phan tich duc thanh tich cia 2 da thiic véi hé sé nguyén. Chiing minh: Gia sit ngude lai ring f(x)=g(x)-h(x), trong dé g(x) va h(x) la cdc da thiic véi hé sé nguyén khong tam thudng: h(x) = bg +b)x+...+ b,x", B(x) = Co +E)X +... x™ Ta cé O 5 . Khi dé (2) duge thod man. Déi véi diéu kién (3), ta 06 thé viét : x>V2x—1. Vi x2, cd hai vé cia bét ding thiic nay déu khéng 4m, nén co thé binh phuong cua hai vé va ta duoc x? 22x -1e> x? -2x+120 (x-1)? 20, bat ding thiic cudi cing duing. Tom lai, diéu kién cia bai ton 1a: 1 25. 4) x25 (4) Bay gid ta hay giai lan lugt cdc phugng trinh da cho. a) Binh phuong hai vé cia phuong trinh, ta duge x+V2x—1 +x —V2x—1+2yx? -(2x-1) =2 hay sau khi don gian: x +|x—1|=1, tic 1a ta c6: |x —I]=1-x =-(x-1). pé ding thic nay xdy ra, ta thay ring: x-1<0 hay x <1. Két hgp véi diéu kién (4), ta thdy ring nghiém cia phuong trinh (a) 18 : 1 2 b) Cang bién déi nhu trén, ta dusc: 1 2 x. Két hop véi digu kign (4), ta di dén x = Nhumg gid tri nay khéng thoa man (5). Vay phuong trinh (b) vé nghiém. c) Cang bién déi nhu déi véi phusng trinh (a), ta duge x +[x-1|=2 hay Ix-Il=2-x. Nhu vay ta c6 2> x.Véi diéu kién nay, ta 6 thé binh phugng ca hai vé dé di dén phuong trinh tuong duong x?-2x+1=4-4x+x", hay 2x =3,vay x= . Nghiém nay thod man diéu kién 22x va (4). Vay phuong trinh (c) ¢6 mot nghiém JA x = 2. Bai 3. (1959) Cho cc sé thuc a, b, c va phwong trinh bac 2 theo cosx: acos” x+bcosx +c =0. Hay xAy dung phuong trinh bac 2 theo cos2x ma nghiém cba né van 1a gia tri x. Hay so saénh cdc phuong trinh theo cosx va cos2x véi a=4,b=2,c=-1. Let a, b, ¢ be real numbers. Given the equation for cos x: acos’ x+bcosx+c=0, form a quadratic equation in cos2x whose roots are the same values of x. Compare the equations in cos x and cos2x for a = 4, b = 2,c =-1. Huéng dan: Ta cé cos2x = 2cos” x —1, thay vio va bién déi ta cé: a? cos? 2x + (2a? + 4ac — 2b?) cos2x + (4c? +4ac—2b? +a7)=0. Véi cdc gid tri a, b, c cho truéc, phucng trinh da cho va phuong trinh trén tuong duong, Gidi ra ta duge cdc nghiém oe [ Bai 4. (1960) Xac dinh tat cd cdc sé 6 3 chit sé, chia hét cho 11, sao cho thuong sé trong phép chia sé dy cho 11, bing téng binh phuong cac chit sé cia sé dy. Determine all 3 digit numbers N which are divisible by 11 and where N/11 is equal to the sum of the squares of the digits of N. Huéng dan: Goi sé phai tim 1a abc, thi bai todn dua vé viée gidi phuong trinh: 100a+10b +c =11(a? +b? +c), q@) trong dé a,b,c 1a nhiing sé nguyén, cé gid tri 1a mét trong cac 86 0,1,2,....9, ngoai ra az 0. Dé y ring: 100a +10b+c =99a +1 Ib+a—b+c (2) nén tit (1) va (2), suy ra ring a—b+c phai chia hét cho 11. Nhung -8 0, ta phai ed: HAA gy AN oy, Vi n 1a mét 86 nguyén khéng 4m, nén phi cd n = 0. Khi dé c =0, tirdd suy ra a =b=5. Nhu vay trong trudng hgp nay, s6 phai tim 14 550. 2) k =1. Tit(1) va (2), ta c6: 99a +1 1b +11=11(a? +b? +c”) hay 9a+b+1=a2 +b? +c?. (5) Nhé ring a—b+c=11, tic la b=a+c-11. Thay gia tri nay vio (5), ta duge: 10a+¢—-10=a? +c? +(a+c—11)” hay 2a? + 2c? + 2ac — 32a — 23¢ +131=0. 6) Tu day suy ra ring c 1 mét sé 1é. Gia si c = 2n+1. Thay gid tri nay vao (6), ta di dén: a? —(15—2n)a + 4n? -19n +55=0. Vay fea : 4tV31 Tam thi bac hai —12n? +16n+5 6 nghiém la 4-H oo 44 VII oy , do do 6 6 Vi n nguyén va khong 4m, nén n =0 hodc n=1. P ONS ee ad a ears la mét sé vé ti, diéu nay trai véi dieu kién a 1a mot sé nguyén. Fi -2+ Néu n=1, thi a= !5 243 pie ta: - hoic a=5, khi dé c=3 va b=-3. Nhung b phai la mét sé nguyén Ihéng am, vay khéng thé chon cac gid tri nay; -hofc a=8, khi dé c=3,b=0, va ta di dén s6 803. 153 ‘Tém lai cdc sé phai tim 14 550 va 803. Bai 5. (1960) Véi nhiing gid tri thuc nao cia x thi bdt phuong trinh sau day dude nghiém dung: <2x+9? aL (l-vi+2x)? For what real values of x does the following inequality hold: 2 _ (—V1 42x)? Huéng dan: Dé vé trdi cia bdt phuong trinh cé nghia, ta phai dat diéu kién 14+2x>0 va 1-V142x #0, tite la x 2-5.x 40. mM 2x 2x(1+ V1+2x) Sa = = - (It V1 4 2x) ta duc 1-V14+2x 1-(14+2x) ( ) . 4x? 2 = (14 V4 2x) =2+2x+2v1+2x, (l-vi+2x)? va bat phudng trinh da cho cé thé viét duéi dang 242x4+2V14+2x <2x+9, hay V1+2x 4a? 2442 Joab? 304 —3h4 Vay | +b + 10a“b* ~3a" -3b ; 4a 4a _a?+b? _ V10a7b? —3a4 — 364 Aa 4a [ 2a x 3 2 a? +b? DE x,y,z dudng, can phai c6 x+y>0 hay >0. Vay a> 0. Véi diéu kién nay thi x va y duong, bii vi xy>0. 7 a?—b? Tirdidu kién z= >0,véi a>0 ta suy ra ring a a? >b?, hay a>|b]. wM Dé x #y, ta thdy ring cdn phai cé : 10a7b? —3a‘ —3b4 >0. (2) ie Dat t=—, a ~3t* +10t? -3>0, hi 10+ ere [FeO 0. (3) ay — 3( BA a) )> thi theo (1), ta cb 1>t>0, va cé thé viét (2) dudi dang : 155 Vit20,nén t+V3>0,t+-L>0, vavitct,nén t-V3<0, v3 va bt phuang trinh (3) tuong duong vai t— 5 , tie la a>|b|>—= (a>0). 3B >0. Nhu vay i>t>k hay 1M 4 v3 a Bai 7. (1961) Giai phuong trinh cos" x —sin" x =1, vdin 14 sé tu nhién. Solve the equation cos" x —sin" x =1, where n is a natural number. Huéng dan. Vi cos?x +sin?x =1 nén phuong trinh vé nghiém khi n#2 va 0<|cosx|,|sinx|<1. Néu n = 2 va 0<|cosx|,|sinx|<1 cing dé thay phuong trinh v6 nghiém vi vé trai luén bé hon 1. Nhu vay phuong trinh chi c6 nghiém khi sinx = 0 hodc cosx = 0. Tu day ta cé: e nchin:x=kz,keZ. © nlé cosx =0 va sinx =1, cho tax = 5 +2ke hove cosx =1 va sinx =0, cho ta x =2kz. Bai 8. (1962) Tim sé nguyén duong n nhé nhat cé cdc tinh chat sau day: a) Viét duéi dang thap phan, sé dy cé tan cing bing 6. b) Néu bé chit sé 6 cudi cing va dat chit sé 6 lén truée cc chit sé con lai, sé duge mét sé lén gap 4 lan s6 kan dau. Find the smallest natural number with 6 as the last digit, such that if the final 6 is moved to the front of the number it is multiplied by 4. Huéng dan: Gia thit sé phai tim 1a mét s6 c6 n+1 chit sd. ‘Theo diéu kién a), sé dy 1a: a,a3...a, 6. Theo diéu kién b), ta cd: 6a,a2..a, =4.a)a)..a,6. @M Dat a=ajay..a, , thi: aa7..a,g =10a+6, 6a,a2...a, =6.10" +a, va ding thic (1) tr thanh 6.10° +a =4(10a+6) hay 2(10"-4)=13a. (2) Dang thiic nay ching td vé trai chia hét cho 13, nhung vi 2 va 13 1a hai 156 s6 nguyén té cing nhau nén 10"—4 chia hét cho 13. Vi cdc phuong trinh (1) va (2) la tuong duong nhau, nén bai todn quy vé tim sé ty nhién n nhé nhdt dé 10"-4 chia hét cho 13, khi d6 sé tim ra s6 a va sé phai tim 1a 10a +6. Ta hay cho n lan lust bang 1,2,... thi 10" —4 sé lan lugt 06 cdc Bid tri 6, 96, 996, 9996, 99996,.... Bang cach thi truc tiép, ta thay ring sé dau tién chia hét cho 13 trong cdc sé dy 14 99996. Khi dé a = 15384. Vay sé phai tim 1a 153846. Bai 9. (1962) Xéc dinh tt cA cdc sé thc x thoa man bat phuong trinh: Vi-x—Veti>$. Find all real x satisfying: J3— x —Jx+1> *: Huéng dan: Dé vé trai cua bat phusng trinh da cho 1a cé nghia, ta phai dit diéu kién: 3-x >0 va x +120, nhu vay -1 Jal. (2) Vé phai la khong am, vi vay Tax20 hay xsl. (3) Véi diéu kién (3), c6 thé binh phuong hai vé cua bat phuong trinh (2) va dude bat phuong trinh tuong dudng: 99 x 4x? >x+l hay 4x 2_gx 43350, (4) 16 16 Tam thite bée hai 4x? —8x +05 hai nghiém IA 1 By vay nghiém Bh Bley cda bat phuong trinh (4) 1a x <1-—— ,14+~—— 157 Két hgp véi cdc diéu kién (1) va (3) ta thay ring bat phung trinh da cho c6 nghiém -11 hay |x|>1. Hon nia, vé trai khong 4m, nén vé phai cing khong Am, tic la: x > ax? -1 a 4) Tw day ta thdy ring x > 0. Binh phuong ca hai vé céa bat ding thitc (2), ta duce: x?>4(x?-1),hay 4>3x?,va nhé ring x>0, vay 22> 3x. Thanh thi dé vé phai cia (1) cé nghia va khong am, ta phai cd 2 2x21. (3) v3 Dé gidi phuong trinh (1), ta hay phan biét cdc truéng hgp sau day: 158 1) p<0. Khi dé vé trai cda (1) luén luén cé nghia, ta cé thé binh phuong hai vé cua (1) va duge x? —p=x?—4xvVx?—144(x?—1), hay ~p=4x?—4—4xyx?-1, (4) Cé6 thé viét phuong trinh nay dudi dang — p = 4Vx? -1(Vx? -1-x), hay, béi vi x21 theo didu kién (3), nén x =[x}= Vx? = (x? -1) 41, —p=4vx? -1(Vx? -1— fx? +0. 6) Vé trai cua (5) 14 —p>0, trong khi dé vé phai (néu né 6 nghia) luén luén nhé hon hay bing 0, vay phuong trinh (5), tuc 1a phuong trinh di cho, la vé nghiém. 2) p>0. Khi do dé vé trai cia phuong trinh (1) c6 nghia, ta can phai dat diéu kién x? > p hay (vi theo (3), x>0): xp. (6) Nhu vay, nghiém cia phuong trinh (1) phai thoa man dong thdi cde diéu kién (3) va i Thanh ae a) Néu Jp Gi , tic A p >$, thi cdc diéu kién (3) va (6) mau thudn véi nhau, do 7 ‘hung trinh (1) v6 nghiém. b) Néu Jp< a , thi c thé théng nhat cdc diéu kién (3) va (6) duéi dang sau (ki hiéu max{l, /p} chi sé lén nhat trong cac sé 1 va yp ): 2 rR —= 2x 2>max{l, : (1) B yp} Véi diéu kién (7), ta c6 thé bién déi phuong trinh (1) va di dén phuong trinh (4). Viét phuong trinh dy dudi dang 4xvx?-1=4(x?-1)+p, thi ca hai vé déu khéng 4m, do dé cé thé binh phuong hai vé va di dén phugng trinh tuong duong 16x?(x? ~1) =16(x? -1)? +8p(x?-1) +p’, hay 16(x? —1)—8p(x? ~1)—p? =0; ti'dé suy ra x? — ee Py 4p? 8(2-p)— 8(2-p)” p,nén 4~p>0 va 2-p>0.Vay x= Theo (7), a Dé hoan thanh phép ching minh trong trudng hgp nay, ta phai chiing té ring gid tri cha x da tim duge thod man diéu kién (7), hay cing vay (vi 4—p>0 va 2—p>0): Sox? > maxtlp. 9) ) Bat ding thie : >x? tuong duong véi 2 2 2 -_ fy 7 3 8(2-p) 3 8(2-p) Tam thiic bic 2 6 vé phai cia (10) ¢6 hai nghiém 1a —4 va ‘, do dé bat <= 8(2—p) > 3p” <> 0>3p” +8p—16. (10) phuong trinh (1) 6 nghiém —41. eee ¥) Cuéi cing, ta hay kiém nghiém digu kién x? > p, tic la a >p. -P) Bién déi, bat ding thiic nay tudng duong véi (3p- 4)? > 0. Tut cdc két qua nay, ta thdy ring diéu kién (9), va do dé diéu kién (1), dugc thod man. Nhu vay, c6 thé két luan: -Néu p <0: phuong trinh vé nghiém. -Néu 0,X3,X4,X5) cia hé phuong trinh: X5 +X2 = Yr Xy +43 = yr2 Xy +4 = Yo3 xy +X5 = xq X4q +X) = x5, 160 trong dé y 1a tham sé. Find all solutions (x,,x2,%3,%4,%5) to the system of equations erie areca H+ = xy +Xy 3 +X5= X4+% =X. where y is a parameter. Huéng dan: Céng ca 5 phuong trinh cia hé, ta duge : 2(xp XQ +X3 +X 4 +X5)=V(Kp+XQHXZ+XZ+Xs). (DY Dang thiic nay chi dung trong hai tring hop : 1) y =2. Khi do hé da cho cé thé viét duéi dang fxs —xy = xy - [rr ¥2 = 2 ~%3 4X2 —X3=X3—-Xy [3X4 =X4— 4X5 ify 7X5 =X5 —X- Dat x5—x, =t. Thé thi Xs =x +l Xq = (X4—X5) +5 =H +20 X3 = (13 —x4) +4 =, +31 Xq = (x2 —43) 443 = +40 Xy = (x) - x2) x2 =H, + St. Tit day suy ra t=0 va xy =x2 =x; =X4=X5(= sé tuyy). 2) Néu y #2, thi theo (1) ta phai co (2) Xp +X_ +X34Xq+Xs5 =0 Tir ding thiic nay va tit cdc phudng trinh cia hé da cho, ta suy ra: Wx, = YVR = WOXy +5) = Wey + 5 = (4 +45) + (4 +01) = Xp + (Hy +3 +84) = Xp — (Xz +X5) =X YX hay (y? +y—1)x, =0. Twang tu, ta cing c6 : 161 Natdencesctt ( +y-Ix,=0 ( b) Gia str y? +y-1=0 (tite la y= Tu phugng trinh thi hai céa hé, ta suy ra: x3 =-x,+yx2. (3) Cong hai phuong trinh dau ctia hé, ta duge : Xp +X_ +X3 4X5 =y(X, +X2), hay, do (2): x4 =—y(x; +x). (4) Hon niia, cé thé viét phuong trinh dau cia hé duéi dang: X5 = YX; —X2- (5) Ta hay ching té ring trong truéng hop nay, nghiém cia hé phuong trinh 1a :x,,X tuy ¥, cn x3,X4,X5 dude xdc dinh lan lust béi cac cong thitc (3), (4), (5), nghiém dy thod man diéu kién (2). Qua vay : a) Phuong trinh dau duge nghiém diing do (5). ®) Phuong trinh this hai cing dugc nghiém dung do cé (3). y) Do (3) va (4), cing véi gid thiét y? + y—1=0, tacé: Xq +Xq =X2 —y(X +X) = —yx, -(y 1x2 = yxy +Y?x2 = ¥(—Ky + YQ) = YXQ. Vay phuong trinh thi ba duc nghiém dung. 5) Do (3), (4) va (5), cing véi gid thiét y? +y-1=0, tacé: X3 4X5 =(—Xy + XQ) + (yx —X2) = (Y=) +X2)= 70K] +X2) = YRg- Vay phuong trinh thit tu duoc nghiém dung. e) Do (4), (5), cing véi gid thiét y? + y-1=0, tacé: Xqg tx) = —y(x) + X2)+ a a -y)x, —yX2 = 7x1 —YX_ = ¥(¥X1 —X2) = YX5- Vay phuong trinh this nim dude nghiém dung. Cuéi cing, tit (3), (4) va (5), ta thdy ring: x) +xX2+X3+X4+X5= Xy +Xq +(-X, + ¥XQ)— (XK, + XQ) + (yx, — XQ) =O. ‘Tom lai, cdc két qua tim duge nhu sau : -1+75 1) Néu y# va y #2, thi hé phuong trinh cé nghiém : 162 Hb atnnee pte Xp =Xq =X3 =Xy =Xs <0. 2) Néu y =2, thi hé c6 nghiém: x, = x = x3 =x = x5 (s6 tuy y). 3) Néu y = ae , thi hé cé nghiém: x, tuy y, x2 tuyy, X3 = Oy + XQ, Xy = — YO HQ). XS = MY —Xp- Can dé y ring trong truéng hgp nay, vi cc diéu kién cla dau bai sé khong déi néu ta hodn vi ving quanh ede chi sé eiia ede dn, nan eiing c6 thé thuc hién phép hodn vi dy déi véi biéu thtic cua nghiém. Ching han: x, tuy y, x, tuy ¥, x) = —Xq + yx5.X2 = —Y(Xq +.X5).x3 = WY —N5- Bai 13. (1963) Chiing minh ring cos” cos 7% +005 eu Prove that cos” —cos* +cos = i Huéng dan: Xét phuong trinh x" +1=0, nghiém céa phuong trinh nay la ix ida il3x Cuca. hay ching minh ring téng cdc nghiém nay bing 0. Dac biét, ta suy ra ting cde phan thye cia chung bling 0. Nhung cos” Fi = 1, ein cf cp cén lai bang nhau, do cos(2 - x) =cosx . Tit dé: on | on, | cos— +cos-—+cos: - 7 7 y Sau cing, dé y rang cos(7-x)=—cosx , cos =-00s 2 »tadi dén diéu phai ching minh. Bai 14. (1963) C6 5 hoc sinh A, B, C, D, E trai qua mét ki thi va két qua xép hang duce danh sé tit 1 dén 5. Ngudi ta dy doan ring két qua cuéc thi sé theo this ty A, B, C, D, E. Thé nhung lai khéng cé hoc sinh nao dat két qua vi tri nhu da dy dodn va cing khéng cé hai hoc sinh nao dat két qua ké nhau (tht ty gan nhau) nhu dy doan. Vi du, hai hoc sinh C va D khdng dat két qua tuong ting la 1, 2 hay 2, 3, hay 3, 4, hay 4, 5. Mot du doan khéc cho ring thi tu xép hang 14 D, A, E, C, B. Két qua sau khi 163 thi, cé dting 2 hoc sinh da dat vi tri nhu du dodn va cing da cé dugc hai c&p hoc sinh khac nhau xép thi tu ké nhau nhu da du dodn. Hay xc dinh két qua xép hang cia 5 hoc sinh d6. Five students A, B, C, D, E were placed 1 to 5 in a contest with no ties. One prediction was that the result would be the order A, B, C, D, E. But no student finished in the position predicted and to two students predicted to finish consecutively did so. For example, the outcome for C and D was not 1, 2 (respectively), or 2, 3, or 3, 4 or 4, 5. Another prediction was the order D, A, E, C, B. Exactly two students finished in the places predicted and two disjoint pairs predicted to finish consecutively did so. Determine the outcome. Huéng dan: Ta bat dau ti dy dodn thit hai. Theo dy doan nay, cdc c&p hoc sinh khac nhau chi cé thé 1a DA va EC, DC va CB hay AE va CB. Ciing theo du doan dé, cé ding hai hoe sinh da dat duc vi tri nhu du dodn, do vay, thit tu xép hang la mét trong nhiing kha nang sau: DABEC, = (1) DACBE, (2) EDACB, (3) AEDCB. (4) Dén day, ta xét két hop véi dy doan ban dau. Két qua (1) khong thé xdy ra vi AB da roi ding vi tri lién tiép nhu du dodn ban dau. Két qua (2) cing khéng xdy ra béi C xép dung vi tri nhu du doan ban dau. Cang vay, két qua (4) khéng xay ra vi néu thé thi A lai xép dung vi tri cua dy doan ban dau. Tém lai, E, D, A, C, B xép theo thi ty tugng img tir 1 dén 5, dé 1a két qua sau ki thi. Bai 15, (1964) a) Xée dinh tat cA cdc sé nguyén dugng n sao cho 2"-1 chia hét cho 7. b) Chiing té ring voi moi sé nguyén duong n, sé 2" +1 khong chia hét cho 7. a) Find all natural numbers n for which 7 divides 2" -1. b) Prove that there is no natural number n for which 7 divides 241. Huéng dan: Trudc hét, ta hay dé y rang néu cho nan lust bing 1,2,3,4,5,6 thi 2® sé lan lugt 1a 2,4,8,16,32,64, do dé phan du cia phép chia 2" cho 7 sé lan lugt 14 2,4,1,2,4,1. Diéu dé goi y ring phin du cia phép chia 2” cho 7 sé lan lugt 1a: 2,4) néu n lan lugt bing 3k +1, 3k +2,3k (knguyén dung). Qua vay, néu n= 3k thi: 28 =23* =g* = (741k =7m41, néu n=3k +1 thi: 2" = 23+! = 23.2 = 2(7m+1) =14m+2, va cudi cing néu n= 3k +2 thi: 2" =3°**? — 23k 4 = 4(7m+1) =28m+4. Tit két qua nay ta suy ra ring: a) Sé 2"-1 chia hét cho 7 khi va chi khi n cé dang 3k, 1c 1A khi n 1a mét béi cua 3. b) Phan du cia phép chia 2" +1 cho 7 lan lugt 1a 2, 3, 5, khi n ln lugt bing 3k ,3k +1, 3k +2; vay 2" +1 khéng bao gid chia hét cho 7. Bai 16. (1964) Ki higu a,b,c 1a d6 dai cdc canh cia mét tam gidc. Ching minh ring: a2(b+c—a)+b?(c+a—b)+c7(a+b—c) <3abe. Suppose that a, b, c are the sides of a triangle. Prove that: a*(b+c—a)+b?(c+a—b)+c7(a+b-c) < 3abe. Huéng dan: Dé ¥ : abc—a?(b +c—a) = a(be —ab—ac +a”) =a(a—bya—c), abe - b?(c +a —b) = b(b—c)(b—a), abe - c?(a + b—c) = c(e -a\(c-b). Vi vay bat dang thie phai ching minh tuong ducng véi bat ding thitc : a(a—b)(a—c)+b(b—c)(b—a)+c((c—aXc—b) 20. (*) Ta hay chimg minh ring bat d&ng thic (*) van dung (va do dé, bat dang thie da cho 1a ding) véi gid thiét rong hon: a,b,c 1a ba sé khéng 4m. Qué vay, vé trai ciia (*) khong thay déi khi thay déi vai tro ctia a,b,c vi vay ta cé thé gid thiét a>b2>c2>0. Khi dé c6 thé bién ddi vé trai cua (*) nhu sau: a(a—b)(a—c) + b(b—c)(b—a) +. c(c—a)(c—b) = =a(a—b)[(a—b) + (b—c)]—_b(a — b(b—c) + c(a—c)(b—c) =a(a—b)? +a(a —b)(b—c) — b(a —b)(b—c) + c(a—c)(b-c) =a(a—b)? +(a—b)?(b-c) + c(a—c\(b—c). 165 Vé cudi cing 14 khong Am vi ta da gid thiét a>b>c20. Bai 17. (1964) Méi nha bac hoc trong sé 17 nha bac hoc déu viét thu cho cdc bac hoc kia. Ho chi trao déi véi nhau vé ba vAn dé. Mai cap hai nha bac hoc chi trao déi véi nhau vé mét van dé. Ching minh ring cé it nhdt ba nha bac hoc trao déi véi nhau vé cing mét van dé. Each pair from 17 people exchange letters on one of three topics. Prove that there are at least 3 people who write to each other on the same topic. Huéng dan: Xem mét nha bac hoc A nao dé. Vi A viét thu cho 16 nha bac hoc cén lai vé ba van dé (goi 1a cde van dé I, II, ID, nén A phai trao déi it nhat véi 6 nha bac hoc vé cing mét van dé. Gia thit dé 14 mét van dé I, va 6 nha bac hoc dy la B, C, D, E, F, G. Séu nha bac hoc nay cing trao déi véi nhau vé cdc van dé I, II, III. Néu c6 mét cap nao dé, ching han cp B, C, trao déi véi nhau vé vin dé I, thi bai toan duge ching minh: céc nha bac hoc A, B, C cing trao déi nhau vé van dé I. Ta hay xét trudng hop B, C, D, E, F, G chi trao déi véi nhau vé cdc van dé II va III. Vi B trao déi véi 5 ngudi kia vé hai van dé, nén B phai trao déi it nhat véi 3 ngudi cing mot van dé. Gid thi dé 1a van dé TI va 3 nha bac hoc dy 14 C, D, E. Nhé lai ring C, D, E chi trao déi véi nhau vé cae van dé II va III. Néu cé hai ngudi nao dé, ching han C, D, trao déi véi nhau vé van dé II, thi bai todn duse ching minh: cdc nha bac hoc B, C, D cing trao dai véi nhau vé van dé II. Kha nang cén lai 1 céc nha bac hoc C, D, E chi trao déi véi nhau vé van dé III: trong truéng hgp nay, bai todn cing dude ching minh. Bai 18. (1964) Cho 5 diém trong mét mat phdng sao cho trong sé tét cd nhing dung thing néi hai diém bat ki, khong c6 hai dutng thang nao tring nhau, song song hodc vuéng géc véi nhau. Qua diém, vé cdc dutng thang vuéng géc véi cdc dudng thing xdc dinh béi hai trong sé 4 diém cén lai. Hay xdc dinh sé lén nhat céc giac diém ma nhing duéng vuéng géc nay cé thé giao nhau. 166 5 points in a plane are situated so that no two of the lines joining a pair of points are coincident, parallel or perpendicular. Through each point lines are drawn perpendicular to each of the lines through two of the other 4 points. Determine the maximum number of intersections these perpendiculars can have. Huéng dan: Goi 5 diém thoa man yéu cdu cia dé toan Ia A, B, C, D, E. Xét 6 dinh mét diém, thé thi 4 diém con lai sé cho ta 6 dudng thing. Do dé co c& thay 6 duéng vuéng géc di qua diém cé dinh dé. Nhu thé, tang cong c6 30 dudng thing vung géc. Tinh ném na, ct hai dudng thi cAt nhau tai mét diém, do vay, nhiéu lim a ¢6 ed thay C2y = a2 = 435 giao diém. Thé nhung, lai cé mét sé giao diém tring nhau. C6 ba nhém diém tring nhau. Dau tién, 6 duéng thing qua A (ching han) giao nhau tai mét diém (dé 1a diém A), trong khi tinh theo kiéu trén 1a C2 “2 =15 giao diém. Nhu vay ta mat di 5.14 = 70 diém. Tiép dén, cc dudng thang qua C, D va E cing vuéng géc véi AB sé song song véi nhau, diéu nay loai thém 3 giao diém. Nhu thé, ta mat thém cna 582 =10.3 = 30 diém. Tuit ba, dudng thang qua A vuéng gic véi BC 1a dung cao trong tam gide ABC; cing thé véi dudng thing qua B vuéng géc vi AC, dutng thing qua C vuéng géc véi AB, ching déng quy tai truc tam cia tam gidc ABC. Véi tam gidc ABC nay ta phai loai di 2 giao diém. Nhu thé phai loai thém tt c&C3x2= 10.2 = 20 giao diém. Ro rang, cac giao diém tring l4p néi trén déu phan biét nhau (do ba mién diém vita xét ri nhau), do vy, gid tri lén nhat cé thé cé cuia sé cdc giao diém 1a 435 - 120 = 315. Bai ton dén day van chua thuc su dude gidi quyét néu chung ta khong chi ra dude mét trat tu dac biét vé cach sp xép 5 diém da cho sao cho sé giao diém thuc sy la 315, nghia 1a cach sp xép cdc diém dé bao dam dugc ring khong cdn mét giao diém tring lAp nao ngoai cdc tring 167 lap ké trén. Dé nghj ban doc tiép tuc cong vide nay hoac chi ra mét cach gidi khac téi wu hon. Bai 19. (1965) Tim tat ca cac gid tri x thudc [0,27] sao cho: 2cosx s| vi+sin2x —Vi-sin2x |s2. Find all x in the interval [0,27] which satisfy: 2cosx <| Vi+sin2x -—Vi-—sin2x |s2. Huéng dan: Dat y=| Vitsin2x - V1=sin2x |, hie dé: y? =2-2|cos2x|. ¢ Néucosx > 0, do gid thiét 2cosx <2, ta suy ra | cos2x |=| 2cos? x -1|=1—2cos? x, va y = 2cosx, trong trudng hop nay cac bat dang thiic da cho 6 dé bai tré rE do vay ta duse: xe[%. AES ml. © Néu cosx < 0: thanh cosx < oe 1 * Gia str |cosx |< tacd v2 cosx xe _e 1 one : v2 27-4) L4°2 5D 1 . * Gia su |cosx|>——, ta cd |cosx | Ra |cos2x|=2e08?x 1 y=2Isinx|=> xe EF] d ‘Tém lai, bai todn cé nghiém x %. ml, Bai 20. (1965) Cho hé phuong trinh: 4 yX1 +ay2X2 +a43x3 =0 2X1 +499X2 +43X3 =0 31X, +439X2 +433x3 =0, vdi cdc hé sé thod man cdc diéu kién sau day: a) 411,822,433 dung; 168 b) tat cd cdc hé sé cin lai déu Am; ¢) trong méi phuong trinh, téng cdc hé sé 1 duong. Chimg minh rang nghiém x, =x7 =x3=0 1a nghiém duy nhat cua hé phuong trinh dy. The coefficients a,; of the following equations Qy)X) +. A4QXq +.443x, =0 4X1 + GgX2 +473x; =0 €3)Xy + G39Xy + 433x3 =0 satisfy the following: @) 41,477,433 are positive; 6) other a, are negative; c) the sum of the coefficients in each equation is positive. Prove that the only solution is x, = x7 = x; =0. Huéng dén: Gia thir (x,,x2,x3) 14 mét nghiém cia hé phuong trinh da cho. Tir diéu kign cia bai todn, ta thay ring néu cn thi sé thay déi céch danh sé cae x1,x,X3, nén cé thé gid thiét ring: bx] [xo] va fxs] 2 xs]. Phudng trinh thit nhat eda hé cé thé viét duge dudi dang: py = —AyQX2 —A43X3, do dS [ayyx[=[ay2xz +213X3] < [ajax] +lay3xs]- Vi ay, >0, aj) <0, a); <0, nén ta c6: ay ei] $—ay2)xa]—ay3)xs] $—ay2)xa|—ay5|x), hay (aj) +4y2 +)3)/x| $0. Nhung aj) +442 + a)3 > 0, vay |x,|=0, vado do |x.|=0,|x3|=0, tic la: xy =x2 =x; =0. Bai 21. (1965) Tim bén sé thuc x,,x2,X3,x4 Sao cho méi sé cong véi tich cdc s6 cdn lai déu bing 2. Find all sets of four real numbers such that the sum of any one and the product of the other three is 2. Huéng dén: Bai todn quy vé phép giai hé phuong trivh : 169 Xp $XQX3Xq=2 XQ +XyX3xX4=2 a X34+X)X2X4=2- Xq+X)X9x3 =2 Dé y ring néu x, =0 thi ta sé cé: X2X3Xq =2, Xp =X3 =Xq Diéu nay vo ly. Vi vay x, #0, va vi vai tro cla x),X2,x3,x4 ld nhu nhau, nén ta cing c6 x7 #0,x3 #0,x, 40. Thanh thi nhan phuong trinh tht i (i= 1,2,3,4) eda hé (1) véi x;, thi ta duge hé tuong duong: xX? +x)X9X3X4 = 2x; (i =1,2,3,4). DAt p=x)X2Xx3Xq, thi ro rang mdi x; 1a nghiém cia phugng trinh bac hai x? ~2x +p=0. Nhu vay méix; chi cé thé léy mét trong hai gid 1+ J1—p. Do dé, chi c6 thé xdy ra mét trong 3 trutng hop sau day: 1) Ca4s6 x; déu bing nbau, tiic la: x, =x2 =X3 =xq =t. Khi dé hé phuong trinh (1) thu vé mét phuong trinh: t34t-2=0 hay (t-1(t? +t+2)=0. - vi Patea-(ted] +] >0, nén ta phai of t=1, tite la: X) =X2 =X3=xX4=1. 2) Trong 4 sé x;, chi cé 3 sé bing nhau. Khi dé, c6 thé gid thiét: X, =X2 =X3 =t#x,. Hé phuong trinh (1) tré thanh: pres =2 (2) U4+x4=2. Lay phuong trinh thit nhat trit cho phuong trinh thit hai, ta duge : (t-P)+(t? —1)x4 =0 hay (t? -1)(x4-t)=0. Vi x4 #t, nén ta phai cé t? =1. Do dé: ~ hoac t=1, tit phuong trinh thi hai cda hé (2) ta suy ra x,=1, nhung diéu nay mau thudn vdi gid thiét t# x4. - hoac t =—1, ti phugng trinh thi hai cua hé (2) ta suy ra x4=3. 170 Vai tro cua 4 s6 x; 14 nhu nhau, nén trong trudng hop nay ta di dén két ludn: trong 4 sé x;(i =1,2,3,4), 3 s6 bing —1, sé cn lai bing 3. 3) Bén sé x; lap thanh hai c&p sé bing nhau, tic 1a chdng han: Xy =X2 =t¥ x3; =x, =u. Khi dé hé (1) tré thanh: t+? =2 2 . (3) utut’ =2 Lay phuong trinh thi nhat trix cho phuong trinh thi hai, ta di dén: (t—u)(I-tu)=0. Vi t#u, nén tu=1. Ding thdi trong hé (3) ta cing cé (t+u)(I+tu)=4. Va vi tu=1, nén t+u=2. Nhu vay t va u la cae nghiém cia phugng trinh bac hai z” —2z+1=0, phuong trinh nay chi c6 nghiém z =1, tic la t =u =1.Nhung diéu nay mau thuan véi gid thiét teu. Téng két lai cdc truding hop da xét, ta di dén két ludn: hé (1) c6 cdc nghiém sau day : a) X}=X2=X3=X4=1; — b) 386 x, bing -1, sé con Jai bing 3. Bai 22. (1965) Cho n diém trong mat phing véi n > 2. Ching minh ring cé nhiéu nhit Ja n chp diém cé khodng cach eve dai. Given n > 2 points in the plane, prove that at most n pairs of points are the maximum distance apart (of any two points in the set). Huéng dan: Truéc hét, ta dé ¥ ring néu cé hai doan thang cé dé dai bing nhau d ma khéng cAt nhau thi ta cé thé tim hai dau mit trong 4 dau mit céa hai doan thing dé dé tao thanh mét doan thing cé dé dai lén hon d. That vay, gid sit hai doan thing dé 1a PQ va RS, véi PQ = RS = d. Néu diéu ngua lai véi khdng dinh trén xy ra, ta cé ge PQR > 90° (vi néu khéng thi PR > PQ = d). Tuong ty, ta chimg minh dude tat cd cde géc ca tit gidc déu bé hon 90°, diéu nay mau thudn. Ta ding két qua trén dé ching minh bai todn. Gid st ring cé hon n c&p diém cé khoang cach cuc dai (1a d). Nhu thé, dé thdy ring tai mét dinh A nao dé phai cé 3 doan AB, AC, AD cé dé dai d (cue dai). Gia sit AC nim gia AB va AD. Khi dé, C khéng thé néi dude véi mot diém nao nia, mau thudn nay cho ta diéu phai chtmg minh (that vay, gid sit C néi 171 véi X, theo ménh dé trén, suy ra CX phai cit cd AB lin AD, mau thuan!). Bai 23, (1966) ‘Trong mét dé thi cé ba bai todn A, B, C. C6 25 thi sinh, méi nguéi déu da gidi duoc it nhdt mét trong 3 bai do. Biét ring: ¢ trong sé cdc thi sinh khéng gidi duge bai A thi sé thi sinh da gidi dude bai B nhiéu gdp hai lin sé thi sinh da gidi duge bai C; ¢ sé thi sinh chi gidi dugc bai A nhiéu hon sé thi sinh giai dugc bai A va thém bai khac 1a 1 ngudi; « s6 thi sinh chi gidi dude bai A bing sé thi sinh chi gidi dude bai B cong véi sé thi sinh chi giai dugc bai C. Hii cé bao nhiéu sé thi sinh chi gidi dugc bai B? Problems A, B and C were posed in a mathematical contest. 25 competitors solved at least one of the three. Amongst those who did not solve A, twice as many solved B as C. The number solving only A was one more than the number solving A and at least one other. The number solving just A equalled the number solving just B plus the number solving just C. How many solved just B? Huéng dan: Goi a 1a sé thi sinh chi gidi duc bai A, b 1a sé thi sinh chi gidi duge bai B, ¢ la sé thi sinh chi gidi dugc bai C, d 1a sé thi sinh gidi duge 2 bai B va C nhung khong gidi duge bai A. Khi dé, sé thi sinh gidi due bai Ava thém it nhat mét trong hai bai B va C 1a 25-a—b-c-d. Cac diéu kién 6 dé bai cho ta: b+d=2(c+d), a=1+25-a—b-c-d, a=bt+e. 4b+0=26 . =6 Tir cdc ding thic trén ta duge { > . d=b-2c>0 [c=2 Vay sé thi sinh da gidi dugc chi méi bai B 1a 6. Bai 24, (1966) Chiing minh ring véi moi sé ty nhién n va moi sé thyc x sao cho sin2"x #0 (n=1,2,...) ta cd: 1 ——+— Pe sin2x sin4x sin2"x =cotgx—cotg2"x. 172 Prove that we have 1 —— + +... + sin2x sin4dx sin2"x for any natural number n and any real x (with sin2"x non-zero). Huéng dén: = cot gx —cot g2"x Ta co: cosy 2cos*y 1 1 ‘siny > 2sinycosy - 2sinycosy > in2y" Tiép tuc, ta ding quy nap dé chimg minh dAng thuic & dé bai. Hién nhién ding thic dung vi n = 1 (ly x = y 6 (*). Gid sit ding thc dung vdi n, tit (*) ta chon y =2"x va dude: cotgy —cotg2y = “ Faye, TONE x eotea x ; suy ra két qua ding véi n +1, diéu phai chimg minh. Bai 25. (1966) a},4,43,a4 14 bén sé thye khdc nhau cho truéc. Giai hé: fas —ay)x, +a, ~as)x3 +f —aylxy =1 faz ~ajps, +fa2 ~ajlx; +|az—aglxg =1 las ~aypey +a ~ag|x, + [a3 —aglx4 =1 faa ~ae, +la4 -aq|x +4 —a3|x; =I. Solve the system of equations Jar = ay|xy +a, -a;|x, +|ay —a4|xq =1 jar - an +e. -ay|r, +|a -aglxq =1 los - ay) +a, -a))x) +|a3 -ag|x4 =1 , lay ~ayfx, +a, -a)|x, + [aq —a5|x3 =1. where a; are distinct reals, i= 1, 2, 3, 4. Huéng dan: Dé y ring néu thay déi vai tré hai chi sé (cia cdc hé sé va céc dn 173 s6) thi hé phuong trinh khéng thay déi. Vi vay cé thé gid thiét: ay >a, >a; >a,. Khi do hé phuong trinh tré thanh: (a —a2)X2 +(a —43)x3 + (ay —a4)xq =1 (a; —a2)x) + (az —83)x3 +(A2 —ay)xy =1 (ay ~43)x) + (az —a3)X2 + (a3 —ag)xy =1 (ay — a4); + (82 —4)X2 + (a3 —44)x3 =I. Bién déi bing céch tris cdc phuong trinh trén cho nhau ta dude: (a, —ay\(-x, + X2 +X; 4X4) =0 (ap —a3)(-Xy —X2 +X3 + x4) =O (a3 —a4)(—X —X2 —X3 +X4)=0, thanh thi hé da cho tuong duong véi hé: —X)+X2+X3+x4=0, () —X1—X2+X3 4x4 =0, (2) —X1—X2—-X3 4X4 =0, (3) (a, —a4)X; + (a2 —a4)x2 +(@3 —a4)x3 =1. (4) Céng (1) véi (3), thi suy ra: x; =X4. Céng (1) véi (2), ta duge: x; =x3 +Xx4, vay x; =0. Cong 2) vi (3), ta duge: x) + x2 = xy, vay x2 = 0. 1 ay—ay Mang cfc gid tri nay vao (4), thi duge: x, =x, = Nhu vay véi gid thiét a, >a, >a; >a4,thi hé phuong trinh da Aan 1 AoA eee a cho c6 nghiém: x2 = x3 =0,x,;=x4=—-~ --. Néu gia thiét trén khong ay—a4 dung, ma ta cé ching han: a, >a4 >a3 >a), thi do nhan xét & dau bai gidi, ta thay ring nghiém cia hé phuong trinh la : 1 X3=X4=0, 7]. = 27a Bai 26. (1967) Cho cac sé ty nhién k, m, n sao cho m + k + 1 1a sé nguyén té Ién hon n + 1. Dat c, =s(s+1). Ching minh ring tich 86 (mst —k MEms2 — Ck (men —Ck) 174 chia hét cho ¢,c...c,- Let k, m, n be natural numbers such that m + k+ 1 is a prime greater than n+ 1. Let c, = s(s +1). Prove that: mmr ~ Ck MCms2 — Ck) (Cman — 4) is divisible by the product ¢\cy-.<- Huéng dén: Truc hét, hay ching minh: c, —c, =(a—bXa+b+l). Tir dé suy ra ring (Cm41—Cj (Cm42 —Ck)---(Cman —Ck) 1a tich ca n sé nguyén lién tiép (m-k +1)....,(m—k-+n) nhan vdi tich cia n sé nguyén lién tiép (m-k+2),....(m-k+n+l). Dang céng thtic té hgp dé ching minh tich thi nhat chia hét cho ni, tich thif hai chia hét cho (n +1)!, dé y dén gid thiét m +k + 11a 86 nguyén té lén hon n + 1. Cuéi cing, hay ching minh ¢)c9...c, =n'(n+1)!, ti dé suy ra két qua. Bai 27. (1967) Xem day sé {c,} dude xde dinh bii: n =ap +a5 +...+ag, voi moi n= 1, 2,3, ..., trong d6 a),a,...,ag 1a nhiing sé (thy) khong déng théi bang khéng. Biét ring trong cdc sé hang cia day sé {c,}, c6 vo sé sé hang bing khong. Tim tat cd cdc s6 n sao cho c, =0. ,A7,.-- = 12,3, ag are reals, not all zero. Let c, = ay’ +45 +...+ag forn Given that an infinite number of c,, are zero, find all n for which c,, is zero. Huéng dé Cac sé aj,a9,...,ag khong déng thii bing khéng, nén Cy, = ark - adh +. 402 > 0 (k =1,2,3,...)- Vi vay néu c,, =0 thi m phai 14 mét sé 1é. Néu a; > 0 (i=1,2,...,8) thi a™>0 khi m 1é, ti day suy ra c,, >0 khi m Ié (vi cé it nhdt mot sd a, > 0), do dé c, #0 véi moi n, trai véi gid thiét. Tuong tu, cing khéng thé xay ra trudng hop a, <0 véi moi i=1,2,...,8. Thanh tht trong cdc 86 a; , 6 nhitng sé duong va cé nhiing sé 4m. C6 thé coi ring 175 sé) thi hé phuong trinh khong thay déi. Vi vay cé thé gid thiét: a) >az >a; >a4. Khi dé hé phuong trinh tré thanh: (a, —a2)xX2 + (a, —43)x3 + (ay —a4)xy =1 (ay —a2)x) + (a2 —a3)x3 + (a2 —a4)x4 (a —a3)xy + (@2 ~a3)X2 +(a3 —a4)xy =1 (ay —44)x) + (a2 — 44 )X2 + (3 —a4)x3 = 1. Bién déi bing cach trix cdc phusng trinh trén cho nhau ta duge: (a, —a2)(—x) + Xz + x3 +x4)=0 (az —a3)(—x, —X2 +X34+X4)=0 (a3 —ag)(—x, Xp — x3 +X4) = 0, thanh thi hé da cho tuong dung véi hé: —X1)+X2+X3+x,=0, () , Q) , @) (a, —a4)x; + (@2 a4 )X2 +(a3 —a4)x3 =1. (4) Céng (1) véi (3), thi suy ra: x, =X4. Cong (1) véi (2), ta duce: x, =x3 +X4, vay x3 =0. Xp XQ 4X3 4X4 = Xj —X_—X34+Xy= Cong 2) véi (3), ta duge: x; +X2 =X4, vay x2 =0. Mang ede gid tri nay vao (4), thi duoc: x) =x4 = ay aq Nhu vay véi gid thiét a, >a) >a; >a4,thi hé phuong trinh da cho c6 nghiém: x2 = x3 =0, x; =xX4=- . Néu gid thiét trén khong a, —aq dung, ma ta cé ching han: a, >a4 >a3 >a), thi do nhan xét 6 dau bai gidi, ta thay ring nghiém cia hé phuong trinh 1a : 1 3 2X4 20, x ==. Pa | Bai 26. (1967) Cho céc sé ty nhién k, m, n sao cho m + k + 1 1a sé nguyén té lén hon n + 1. Dat c, =s(s+1). Chting minh ring tich sd (Cit ~ Ck MCm+2 — Ck )--Cmn Ck) 174 chia hét cho c,cp...c,- Let k, m, n be natural numbers such that m + k+ 1 is a prime greater than n + 1. Let c, = s(s +1). Prove that: (Cmnsi ~ Ck ME m42 ~ Ck )---(Cmn — 4) is divisible by the product ¢C2..,- Huéng dén: Truédc hét, hay ching minh: c, —c, =(a—b)(at+b+1). Ti dé suy ra ring (Cm4|~Ck (Cms2 —Ck)-(Cmen Ck) 1a tich cia n sé nguyén lién tiép (m—k+1),..., (m-k+n) nhan voi (m-k+2),...,(m—-k+n+1). Ding céng thuc té hgp dé ching minh tich thit nhat chia hét cho ni, tich thit hai chia hét cho (n +)!, dé ¥ dén gia thiét m + k + 11a 86 nguyén té én hon n + 1. Cudi cing, hay ching minh c¢,c2...c,=n!(n+1)!, ich cha n sé nguyén lién tiép tir dé suy ra két qua. Bai 27. (1967) Xem day sé {c,} dude xac dinh béi: c, =a] +a} +...+ag, véi moi n=1, 2,3, ..., trong d6 aj,29,...,4g 1a nhitng sé (thuc) khong déng théi bing khong. Biét ring trong cdc sé hang cia day sé {c,}, cd v6 sé sé hang bing khéng. Tim tat cd cdc sé n sao cho cy, Q,,Qy,...,4g are reals, not all zero. Let c, = aj +a} +...+ag forn = 1, 2, 3, .... Given that an infinite number of c,, are zero, find all n for which c,, is zero. Huéng dan. Cac sé a),a9,....ag khong déng thi bing khong, nén Cay =apk +a3k +403 > 0 (k =1,2,3,...). Vi vay néu c,, =0 thi m phai 1a mét sé 1é. Néu a; > 0 (i=1,2,...,8) thi a™>0 khi m lé, tit day suy ra c,,>0 khi m 1é (vi cé it nhat mét sé a; > 0), do dé c, #0 véi moi n, trai véi gid thiét. Tuong tu, cing khéng thé xy ra trudng hop a; <0 véi moi i =1,2,...,.8. Thanh thit trong cac sé a;, cé nhiing sé dugng va cé nhing sé am. Cé thé coi ring 175 aj 2a22...2a7 2ag, nhu vay a, 1a sé duong lén nhat trong tat cd cdc sé a; duong va ag 1a sé 4m véi tri sé tuyét déi 1én nhdt trong tat cd cdc a; 4m. Ta hay ching td ring |a,|=lagl, tite 1 a, = —ag. Qua vay, gia thit |a,/>|ag!. Ta cd: marfi(s) (2) } w " Cm ag ay jas) 1 Vi ay | 8 <1, nén véi m kha Ién, ta sé c6 ching han m Néu a; <0 thi [a;j<|ag|, do dé ta cing c6 a 1a tri da néi cia m. Thanh thir néu m Ié va m kha lén, thi trong ting sé. trong ngo&c vuéng & vé phai cia (1), khong cé qua 7 sé hang 4m, mbi sé hang dy cé tri sé tuyét déi nhé hon 1/8, tit dé suy ra ring téng sé trong ngoac vudng 1a sé khac 0, va do dé c,, #0. < je vii ede id ‘Tom lai ta da ching minh ring néu ja,|>{ag|, thi khi m 1é va m kha lén, ta cé c,, #0; nhu vay trong day sé {c,} chi cé mét sé hitu han nhiing sé hang bing khong, trai vdi gid thiét. Tuong tu ta cing ching minh duge néu |a,|<|ag| thi ciing di dén két qua trai véi gid thidt. Thanh thi a, = —ag, do dé khi c,, #0 lé ta cd ay =~ag’, va Cm, =ay taj +...+ay' (m 1a). Tiép tuc lap luan nhu trén véi cép a2,a7, ta ching minh dude ring a) =—a7, sau dé lai suy ra: a; =—ag, ay =—as. Nhu vay 8 sé a, chia ra lam 4 ep sé trai ddu nhau va ta cé: c, =0 khi n 1é. Bai 28. (1967) Trong mét cudc thi ddu thé thao, ting sé huy chuong 1a m, duge phat trong n ngay thi ddu. Trong ngay thit nhdt, ngudi ta phat 1 huy chuong va mét phan bay s6 huy chung con lai. Ngay thi hai, ngudi ta phat 2 huy chugng va mét phan bay sé huy chuong cin lai. Nhiing ngay cén lai dugc tiép tuc va twong tu nhu thé. Ngay sau cing, con lai n huy chuong dé phat. Héi c6 tat cd bao nhiéu huy chuong duge thudng va da phat trong bao nhiéu ngay? 176 Ina sports contest a total of m medals were awarded over n days. On the first day one medal and 1/7 of the remaining medals were awarded. On the second day two medals and 1/7 of the remaining medals were awarded, and so on. On the last day, the remaining n medals were awarded. How many medals were awarded, and over how many days? Huéng dan: Gia sit sé huy chuong con lai khi bit du ngay thi ddu thir 1a m,. Khi dé, m; =m, m, =n va véi moi k n-6 nénn=6 va m = 36. Bai 29. (1968) Tim tat cd cdc sé nguyén dudng xsao cho tich céc chit sé cia né (viét duéi dang thap phan) bing x? - 10x -22. Find all natural numbers x the product of whose decimal digits is x? -10x-22. Huéng dan: Gia thi tén tai mét sé x thod man diéu kién cia bai todn. Vi tich cdc chi sé cha x 14 mét s6 nguyén khéng am, nén 0 0 nén x—(5+ V47)> 0, vay ta phai co x2>5+V47 =I1,... . @ Gid thit sé x 6 dang x=a,a,_4..a;aq , trong dé ag,ay,....a, 1a nhiing sé nguyén, vi 0 0, nén x (2) Sé x 1A nguyén, nén ti (1) va (2) ta suy ra duge ring x =12 (néu né tén tai). Thi lai: (12)? —10.12—22 = 144-120 - 22 = 2=1.2, do dé bai todn 6 nghiém x =12. Bai 30. (1968) Cho hé phuong trinh cia cac dn sé x),x3,... ax? + bx, +¢=x ax} +bx2 +¢=x; ax? +bxy_) +C=Xq ax? +bx, +¢=X),, trong dé a,b,c 1a nhiing sé thuc va a # 0. Ching minh ring: a) hé khéng cé nghiém thuc, néu (b—1)? — 4ac <0, b) hé cé mét nghiém duy nhat, néu (b—1)? —4ac=0, c) hé cé hon mét nghiém thuc, néu (b—-1)? — 4ac > 0. a, b, ¢ are real with a non-zero. x, Xp ... , X_ satisfy the n equations: ax} + bx, +0 = x7 2 axz + bx, +0=x3 ax?) +bx, 1) +C=Xq ax? +x, +0=%,, Prove that the system has zero, 1 or >1 real solutions according as (b-1)? -4ac <0, (6-1)? —4ac =0 or (b-1)" -4ac > 0. Huéng dan: Truéc hét ta hay xét truéng hgp a > 0. Dé todn goi ¥ xét tam thiic bac hai : f(z) =az? +(b-l)z+c. 178 sre saanrec ade Bién ddi hé phuong trinh da cho thanh hé tuong duong ax? +(b-I)x, +0=X2—x, ax} +(b-I)x2 +¢=x3—x2 a ax? +(b-1)x_ +0=X4—X_ Dat yy = Xz —X 1, ¥2 = Xz —X7,Y3 =X 4 —XZo-Yn =X] Xp» thi hé (1) c6 thé viét duoc duéi dang: y; =f(x;) (i=1.2,...,n), (2) va dé y ring Vit yo +.-+Yq =0. (3) a) Néu (b~1)? —4ac <0, thi ta cd f(z)>0 véi moi z (vi a>0), nén y;, >0(i=1,2,...,n), nhung diéu nay mau thudn véi (3). Vi vay trong truéng hgp nay, hé (2) khong thé cé nghiém thuc. . b) Néu (b—1)? —4ac=0, thi tam thuc f(z) co mét nghiém duy nhit 2, “1, vanéu z#2, thi f(z)>0. Tw (2) ta théy ring y; 20 (/=1,2,...,n), do d6 dang thtic (3) chi xay ra néu : y, = yz =...y, =0, vi vay ta phai c6 1-b 2a c) Néu (b-1)? —4ac > 0, tam thitc f(z) cd hai nghiém phan biét: 1-b-Y(b-1)? ~4ae £(x;) =0 ((=1,2,....m), hay xy =X) =...=Xq = 4 > 225 2a RG rang khi dé hé (1) (tite 1 hé da cho) duge nghiém dung néu ta léy x, =X7=...=X,=2), ho&e x;=X)=..=X_=Z), Nhu vay trong trudng hop nay, hé da cho it nhdt 1a hai nghiém da néu. Truéng hop a <0 ciing dude xét tuong tu. Bai 31. (1968) Cho ham sé f:R* ->R théa diéu kién: véi a 14 1 sé duong, ta cé: f(x-+a)= 51 F002 00.¥x eR*. Ching t6 ring f 14 ham tudn hoan. Cho vi du vé mét ham nhu thé (khéng phai ham hing) khi a = 1. Let f be a real-valued function defined for all real numbers, such 179 oo that for some a > Owe have f(x-+a)=>+ Vf0)-72).vx€ R*. Prove that f is periodic, and give an example of such a non-constant f for a = 1. Huéng dan: Tit diéu kién da cho ta suy ra: f(x+ay2tVeeR" tirds fade pane’. Do vay: f(a +2a)= 3 + Yf0c+a)— 2(x4a) = f(x +ayl—f(x+a)) (x) + £2(x) = $+ #00-4] =£(x). 2 Vay f tuan hoan véi chu ky 2a. 1khi0R, f(x)= . 2 2 Akhilsx <2 Luc dé, stt dung diéu kién f(x + 2)= f(x) ta xac dinh duge ham f trén ca tap hop R’. Bai 32, (1968) Véi moi sé ty nhién n, tinh ting: [as n+2 [4 n+2* —— |4)——" J4] Jt] i 2 4 8 eel trong dé, ki hiéu [x ] chi phan nguyén cia sé thuc x. For every natural number n evaluate the sum ntl n+2 n+4 n+2* — | +] =] 4] oe] eo 2 4 8 7a where [x] denotes the greatest integer < Huéng x41 Véi moi sé thuc x ta cé: [x ¥{s}+[ 22"). “ That vay, néu x = 2n + 1 +t, véin la sé nguyén va 0k? Sk n? —2nk + 2k? =(n—k)? +k? >k? >1, nhu vay sé z duge phan thanh tich cua hai sé nguyén dugng én hon 1, tic la z kh6ng phai 1a mét 86 nguyén t4 Bai 34. (1969) Cho ham sé f(x) =cos(ay +1) +L coslay +x+htas + tt pyc +x), véi a; 14 cdc hing sé thuc va x la bién thyc. Néu f(x,)=f(x2)=0, chimg minh ring x, —x, la béi cia 7. Let the function 181 S (x) = cos(a, #34 5 costa +x) + Tas + tnt sppeoalty +x), where a, are real constants and x is a real variable. If {(x,)=f(x2)=0, prove that x,—X> is a multiple of x. Huéng dan: Trudc hét, ta thdy f khong déng nhat bing 0, béi vi: f(-a,) =1+Scoslay ~a))+...>1 “24 ~..>0. Sut dung cong thiic khai trién cho cos(x+y), ta thu duge: 4 a 1 1 . f(x) =beosx + csinx, véi b=cosa, + 00882 tot TOS n» va : is i c¢=—sina; ——sina, —...-—sina,. 22 get Ta cé b va c khong déng théi bing 0, vif khéng déng nhat bing 0, do do f(x) = Vb? +c? .cos(d +x), véi cosd = . — c va sind = —-———.. b2 +c? Tir d6, nghiém cia f(x) =01A mz + 5 -d, suy ra diéu phai chtmg minh. Bai 35. (1969) Cho n diém trong mat phing, véi n > 4, trong sé dé khong cé 3 diém nao thing hang. Ching minh ring cé it nhdt (n-3)(n—4)/2 tit gidc ldi tao thanh cé dinh nim trong sé n diém dé. Given n>4 points in the plane, no three collinear. Prove that there are at least (n—3)(n—4)/2 convex quadrilaterals with vertices amongst the n points. Huéng dan: Truéc tién, ta xét 5 diém bat ki, khong cé 3 diém nao thing hang. Ta vach mot bao Idi tit 5 diém dé. Néu bao 1di nay c6 hon 3 diém thi hién nhién cé it nhat 1 tit gidc 16i. Néu chi gém 3 diém, ching han A, B, C, thi hai diém D, E cén lai At phai nim trong tam giac ABC. Khi dé, cé hai dinh cia tam gidc ABC nim cing phia déi véi duéng thing DE, va cing véi D, E, hai dinh dé tao thanh mét tif gidc lai. Nhu vay, ménh dé cdn chiing minh dung véi n= 5 (dé y G-AC~9 1), 182 Xét n diém vdi n>5. Vi khéng cé 3 diém nao thing hang nén sé tat cd cdc cach chon n diém nhu trén 1a: 5 = Ma=Din=2}(n—3)(n=4) 120 Mi cach chon nay cho ta it nhat 1 tt gide léi; tuy nhién, bat ki tit gidc 1éi nao trong sé dé cing c6 thé duge lap ti (n—4) tap hop khac nhau gém 5 diém néi trén, do vay, c6 it nhat 1 es _n(a=In- 20-3) n-4" 120 sé tat cA cdc tif gidc 14i duge thanh lap tit n diém da cho. Dé két thtic ching minh, ta sé ching t6 ring: n(n =in=2Kn=3) , (N=3)(N-4) sg, 120 2 Diéu phai chtmg minh nay tuong duong véi: n(n —1)(n — 2) > 60(n — 4) <> n(n -1)(n -2)- 60(n — 4) > 0. Dé dang thay ring cdc s6 n = 5 van = 6 la nghiém cia phusng trinh n(n—1)(n—2)—60(n—4) =0, do dé ta c6 thé phan tich thanh nhan ti nhu sau: n(n —1)(n -2)—60(n -4) = (n—5)(n—6)(n +8). Voi moi n nguyén duong va > 5, lap bang xét dau, dé thdy déu cia biéu thie (n—5)(n—6)(n+8) cing la déu cia (n—5)(n—6), ti dé ta duoc (n—5)(n—6)=0 khi n = 5, 6; (n—5)(n—6)> 0 khi n>6, suy ra diéu phai ching minh. Bai 36. (1969) Cho cac sé thuc x, ,X2,.¥1,¥2>Z1>.Z thod min cac diéu kién X1 >0,x2 >0 va xy; —27 > 0, x2y2—23 >0. Ching minh ring 8 — (x) + x21 +¥2)- (2 +22)" MYL - ZF Z2Y2-73 Thiét lap digu kién can va du dé xdy ra déu dang thi. Given real numbers x, ,X2,¥\,¥2,2,,22, satisfying x, >0,x. >0, x11 — 27 > 0, x2y2 -23 > 0, prove that: ea see (yn ty) tz.) 2 2292-23 Give necessary and sufficient conditions for equality. 183 Huéng dan: Xét cdc tam thiic bac hai : P(X) =x X? —2z,X+yy, Py(X)=x.X? -22z,X + yp, P(X) = Py(X) +P (X) = (x) + XQ)X? ~ 2(2y +22)K + (Y1 +Y2)- Dat Dy = x1y1—21 5 Dz = X2¥2 —23, D= (x1 +X2MY1 +¥2)— (2 +22)" thi P,,P,,P cé biét sé lan luot lA — D,, -D va D. Dé y ring P(X) =x) (x-2J eee x if ' 2 X11 ~ 21 x] vi vay véi moi gid tri X, ta déu co P,(X)> = Pi. du ding xX thtic chi xay ra khi X = 2). Tuong tu: P,(X)> 22, ddu ding thute chi x x2 xdy rakhi X= 72 ,va P(X)> , ddu ding thie chi xdy ra khi X2 Xy +X X= Z+Z2 x, +Xx2 Nhung vdi moi gia tri X ta cd P(X)= moO +P,002 ot +22 x2 nén khi X= 2!*?2 thi duge —P 2PL Dr. a X1 +X Xj+X_ Xp XQ d&u dang thiic chi x4y ra néu 2! = 22 (p(x) va P(X) dat gid tri nhd xX, X2 nhat tai cing mét diém X) . Tur (1) suy ra (sti dung bat ding thtic Cauchy cho 2 - Bo COB oy era{ PeP2] 2fx1x2- 2 {Pr Da Bo: xy XZ Xy X2 yoo (2) DD, Dy D2 Dé & (2) c6 dau ding thiic, thi (1) phai cé dau ding thiic va khi dp dung bat dang thtic Cauchy, ta phai cé d4u dang thitc, tric 1a : 184 Z: t=, x) =X2,D, =D2, x. hay x) =X, =Y2.21= Bai 37. (1970) Gia st a,b,n 1a nhiing s6 nguyén Ién hon 1. Cac sé a,b 1a co sé ca hai hé dém. Cac sé A, va B, 6 cing cach biéu dién x,x, 1X0 trong cdc hé dém véi co sé a va btuong ting, ngoai ra x, #0 va X,-1 #0. Goi A,_; va B,_; 1a cdc sé suy ra tit A, va B, sau khi xod X,- Ching minh ring a > b khi va chi khi Ana . Baa An B, Suppose that a, b, n are intergers which are greater than 1. We have 0b iff A B, mle nl Ay By Huéng dan: Theo dinh nghia, at cé: at n k k k Avi das yxy - == k=0 = k=0 Bat ding thiic trong dé bai tuong duong véi n-!l nl Yx,a" Yx,b* nr ——~ < <"—\, hay —*, tiicla Yx,b* >x.a* Yx,b* k=0 k=0 k=0 x,a" > Xo +xya+x.a7 +..4x,a" X,b" k=0 (1) X_b" Xq+Xyb+X2b7 +... Ta hay ching minh ring voi gia thiét x,_,#0,x, #0, thi bat ding thuic (1) tuong dugng véi a > b. Muén vay, truéc hét ban doc hay chimg minh ménh dé sau: néu A,B,C,D 1a 4 sé duong thi cde bat ding thitc 185 Ce Are C <= va <= D B+D D A B tuong duong nhau. Bay gid ta dé y ring bat ding thtic a > b tuong duong véi: laa? ~C _véi vé sé chi sé n. The real numbers ag,a),42,-..,a,... satisfy 1=a9 Sa, Sa) S...S a, S.... @ .. are defined by by = (1-2-4 1. . kal a Vay a) Prove that 0C for all sufficie- -ntly large n. bby, Huéng dén: a) Véi moi k>1, tadéucd 1q. Day sé a, = ae (n=0,1,2,...) thoa man diéu kién P 1=ag C, nén a n> néu n di lén thi sé c6 b, >C. Bai 39, (1970) Tim tat cd cde sé nguyén dung n cé tinh chdt sau: 06 thé chia tap hop 6 s6 {n,n+1,n+2,n+3,n+4,n+5} thanh hai tap hgp, sao cho tich tat ca cdc sé cia tap hgp nay bing tich tt ca cdc sé cha tap hop kia. Find all positive integers n such that the set 187 {n,n4+1n+2,n4+3,n+4,n+ 5} can be partitioned into two subsets so that the product of the numbers in each subset is equal. Huéng dan: Ta hay dé ¥ ring trong 5 sé nguyén lién tiép phai cé mét (va chi mét) sé chia hét cho 5. Vi vay néu tap hgp 6 sé {n,n +1.,....n+5}c6 tinh chat da néu trong dau bai, thi trong tap hop dy phai cé ding hai sé chia hét cho 5, di nhién dé phai 1a cdc sé n va n+5, con cae sé n+in+2,n+3,n+4 thi khong chia hét cho 5. Mat khac, néu trong 6 s6 cia tap hgp trén chia hét cho mét s6 nguyén té p>7, thi 5 sé con lai sé khéng chia hét cho p, va tap hop khéng c6 tinh chat doi hoi. Ti day dic biét suy ra ring cdc sé n+1,n+2,n+3 va n+4 chi chia cdc thia sé nguyén t6 2va 3, tic la: n+1=2%3h, n+2=2%23h, n+3=2535, n+4=2%34, trong dé kj,1;,....k4,l4 1a nhing sé nguyén khong 4m. Néu n+1 (va do dé n+4) chia hét cho 3, thi n+2 va n+3 khéng chia hét cho 3, vay I) =1;=0 va n+2=2",n+3=2", nhung nhu thé thi n+2 va n+3 1a hai sé nguyén lién tiép ma lai la hai sé chin, diéu nay v6 Ii. Lap lun tugng ty, ta thdy ring néu n+2chia hét cho 3, hodc néu n+3 chia hét cho 3, thi ta van gip mau thudn. Mau thudn dy ching té khong cé sé nguyén dugng n nao thod man diéu kién bai todn. Bai 40. (1970) Trong m&t phing cho 100 diém phan biét sao cho khéng cé 3 diém nao thing hang. Chiing minh ring trong sé cdc tam gide dude tao thanh ti 100 diém dé, cé khéng qué 70% cdc tam gidc nhon. Given 100 coplanar points, no 3 collinear, prove that at most 70% of the triangles formed by the points have all angles acute. Huong dan: Ti 4 diém phan biét khong cé 3 diém nao thing hang, nhiéu lam 1a co 3 tam gide nhon. Ti két qua nay, suy ra véi 5 diém phan biét 188 khéng cé 3 diém nao thing hang, ta nhan dugc 10 tam gide va cé khong qua 7 tam giac nhon. Véi 10 diém phan biét khéng cé 3 diém nao thing hang, sé cuc dai cdc tam giac nhon tao thanh 1a: sé cdc tap con 4 diém nhan cho 3 réi chia cho sé cdc tap con 4 diém chiia 3 diém cho truéc. Trong khi dé, sé tdt cd cdc tam gidc tao thanh cing cé biéu thie tuong tu nhu-trén nhung thay vi nhan 3 ta nhan cho 4. Do vay sé cac tam gidc nhon chiém khong qua 3/4 sé tat ca cdc tam gide (déi véi 10 diém). Li luén tuong tu, ta xét 100 diém phan biét sao cho khéng cé 3 diém nao thing hang. Sé cuc dai cdc tam gidc nhon tao thanh 1a: s6 cde t4p con 5 diém nhan cho 7 réi chia cho sé cdc tap con 5 diém chtta 3 diém cho trutéc. Trong khi dé, sé tat cA cdc tam gidc tao thanh cing cé biéu thie tuong tu nhu trén nhung thay vi nhan 7 ta nhan cho 10. Do vay sé cdc tam gidc nhon chiém khéng qua 7/10 sé tat cA cdc tam gidc tao thanh, diéu phai ching minh. Bai 41. (1971) Xem ménh dé S(n): "Voi moi sé thite a),43,...,4,, déu xdy ra bdt ding thite: (a, — a2 (ay — a3)... ag) + (2 — a) ay —a3)...(ay —a,) + s+ (Aq — 84 )(y —2).--(Ay ~Ay_y) 20". Chiing minh ring ménh dé ding khi n =3 va n=5, ménh dé sai véi moi gid tri khac cua n (n> 2). Let S(n) be the proposition that (ay 42 (a1 - 93)---(4y ~ an) + (42 ~ & (2 — 43).-.(4 — Aq) + +4 (Aq — 4 XAq — 42)--(Aq — 4p) 20 happens for all real a,,@3,...,d,. Prove that S(n) is true for n = 3 and 5, but for no other n > 2. Huéng dan: a) Ta hay xét trudng hop n =3. Cac sé ay,a7,a3 c6 vai tro nhu nhau, nén cé thé gid thiét a, >a, >a. @ Khi dé dé dang bién déi: (a —a2)(ay — a3) + (a2 — aj (az — 3) + (@3 — a) (a2 -a) = (a) —a9)” + (ay ~a3 lag a3). 189 Biéu thie cuéi cing khong 4m do gid thiét (1). Vay ménh dé ding khi n = 3. b) Ta xét trudng hgp n= 5. Ciing nhv trén, cé thé gid thiét a, 2a 2a; 2a42as. (2) Thé thi: (a; —42)(a1 — 43 )(a1 — a4 May — a5) + (@, — 4) a2 — 3 (Az — a4 (a2 — a5) = (a —2)[(@1 —23 ay — 44 )(ay — a5) ~ (42 ~ 43 (ay — a4 (ay ~a5)]. (3) Do (2) ta c6 a; —ay > 0, va biéu thi trong ngo’c vudng cing khéng 4m béi vi: aj —a3 2a, —a; 20, a) -a4 2a) —a4 20, aj—as 2ay-a5 20. Nhu vay biéu thie 6 vé tri cita (3) la khong Am. Ta lai c6, do (2): (a3 —a) (a3 a2 a3 —a4 (a3 —as) = (aj — a3 (a2 —a3)(a3 — a4 (a3 —a5)20. (4) Cuéi cing, (aq —a1)(@4 — a2 Mag — 23 aq — 25) + (As — 24 (as ~ a2 Mas —a3)(as a4) = (a4 —45)[(@1 — a5 (az — a5 )(a3 — a5) —(a; —4)(a2 a4 )(az3—a4)]. (5) Theo (2), ta cb ay—as >0 va biéu thic trong ngodc vuéng cing khéng Am bdi vi: ay —a5 2a) —a, 20, az —as Day —ay 20, a3 —as 2az-a4 20. Vay biéu thitc 6 vé trai cia (5) 1a khong 4m. Thanh thi ting cia cdc vé trai cua (3), (4), (5) khong 4m, va ménh dé dung khi n = 5. c) Ta hay xét truting hgp n chiin (n> 2). Lay a, =-I,a) =a; = Khi dé, vi n chiin, trong ting 6 vé trai cia bat ding thtic, chi cé sé hang dau tién la khdc 0 , sé hang nay bing (a — a9 )(a —a3)..(ay—a,) =(-1)"! <0. Vay bat ding thiic khéng ding, tic 1a ménh dé khong ding néu n chin. d) Cuéi cing xét trudng hop n Ié véi n> 7. Lay a, =1, ay =a3 =a4 =2, as = ag =...=a, =0. Khi d6, trong ting néi trén, ciing chi cé sé hang dau tién 1a khdc 0 va bang (a; —a2)(a, —a3)(a, ~ a4)(a, —as)...(a, —a,) =-1<0. Vay ménh dé cing khong dung trong trudng hgp nay. Bai toan dude chimg minh. 190 Bai 42. (1971) Ching minh ring ta cé thé tim duge mot tap vo han cdc sé nguyén dudng cé dang 2" -3, trong dé n 1a sé nguyén duorg, sao cho méi cp sé bat ki trong tap hop dé déu nguyén té cing nhau. Prove that we can find an infinite set of positive integers of the form 2" -—3 (where n is a positive integer) every pair of which are relatively prime. Huéng dan: Bai todn duce gidi néu ta ching minh dugc ring hé cé mét tap gém r phan tt nhu da ndi 6 dé bai thi ta mé réng ra duoc mét tap nhu thé ma cé r + 1 phan tit. Gia sit ta c6 tap hop gom r s6 nguyén t6 cing nhau déi mét: 2"! —3, 2"? -3, ..., 2" -3. Dat m = (2™ —3)(2" —3)...(2™ -3), ta di tim sé n=n,,, sao cho 2"-3 nguyén té cing nhau véi tat cd cdc nhan ti cia m, diéu nay cé nghia 2" -1 chia hét cho m. Trong cac sé 2°,2',...,2™, chic chin phai cé 2 sé cd quan hé déng du mod m, nghia Ja tén tai m,,m2, m, >m, dé 2™ =2™2 (mod m), do m 1a sé 1é, ta suy ra 2™~™? -1=0 (mod m). Vi vay, ta chon n=n,,, =m, —my, suy ra diéu phai ching minh. Bai 43. (1971) Cho A = (a;;), véi i, j=1,2, .... n, la mt ma tran véi cdc hé sé. aj 1a nhing sé nguyén khong 4m. Véi moi i, j ma aj= 0, téng cac phan tit 6 hang thit i va cét thet j bé nhdt la bing n. Chimg minh ring ting cac phn tit cia ma tran cé gid tri bé nhdt 1a n?/2. Let A = (ay), where i, j = 1, 2, ..., n, be a square matrix with all aj, non-negative integers. For each i, j such that ay = 0, the sum of the elements in the ith row and the jth column is at least n. Prove that the sum of all the elements in the matrix is at least n? /2. Huéng dan: Goi x la téng céc phan tit cia hang hodc cot ma ting nay bé nhat (so véi céc hang hay cét khac). 191 Néu x2 thi coi mbu bai todn gidi xong, do vay ta gid sit x <>. Khéng mit tinh ting quat, ta gid st dé la mét hang cd téng cdc phan tir bé nhdt (do cé thé bién déi hang va cét), va goi dé ia hang H. Néu sé cac phan tit khac 0 trong hang H nay la y thi ta cd ¥< 5s do méi phan tit Ichdc 0 bé nhét 1a 1. Bay gid ta chuyén sang tinh téng cdc c6t bing cach dya vao hang nay. Ré rang y cét nay (mdi cét cé chifa phan ti khac 0 cua H) cé téng méi cét bé nhat 1a x (theo dinh nghia trén cla x). Ta ciing c6 (n - y) cot (mii cét chia phan tit 0 cba H) cé téng méi cét bé nhat Ja n - x (chu ¥ gid thiét), Tir dé ta suy ra téng cdc are titcda ma tran it hat phai la 4 (2=2xyn-2y) | n? 2 7. Ban doc cé. thé xét nhu anit. vi du véi ma tran aij) ma xy +(n—x)(n—y)= aj, =1 néu i va j cling chin hodc cing lé, | ay = 0 trong truéng hop khac. Bai 44. (1972) Tir cdc s6 10, 11, 12, ..., 99, ta thanh lap mét tap con S tay y gom 10 sé phan biét. Chimg minh ring ti tap hop S nay cé thé trich ra duge 2 ta» con rdi nhau (c6 giao bing réng) ma téng gid tri cc phan tit é hai tap con dé bing nhau. Given any set S of ten distinct numbers in the range 10, 11, ... , 99, prove that we can always find two disjoint subsets from S with the same sum. Huéng dan: Sé cdc tap con khac réng cua tap hgp S 1a 2'°-1=1023. Trong tat cd cdc tap con dé, téng gid tri cdc phan ti 6 mii tap hop lén nhat 1a 90 + 91+... + 99 = 945. Tir dé, suy ra ring trong sé 1023 tap con dé phai tén tai 2 tap A va B khdc nhau sao cho tng gid tri cdc phan ti cda A bing téng gid tri cdc phin ti cia B. Dat M = A\ B va N=B \ A. Dé dang chting minh duge ring M, N la cdc tap con khdc réng, rdi nhau cua S va tong gid tri cdc phan ti cia M bing téng gid tri cdc phan tit cua N. 192 Bai 45. (1972) Véi moi sé nguyén khong 4m m va n, chting minh ring: (2m)!(2n)!_ m!n!(m+n)! 1a mét sé nguyén dung. Prove that for any non-negative integers m and n, _QmiQn) min'(m+n)! is a positive multiple integer. Huéng dan: Dé gidi bai toan, ta chi viéc ching t6 véi moi sé nguyén té p, sd cc thita sé p chifa trong tich (2m)!(2n)! khéng nhé hon sé cdc thita sé p chifa trong tich_m!n!(m+n)!. Nhu da biét, sé cac thiia sé p chia trong tich (2m)!(2n)! la: 2m 2m 2m 2n 2n 2n Sp =| — | +) +] sy [tt] | a |] es P Pp P P pv P Cén sé céc thifa sé p chiia trong tich m!n!(m +n)! bang : {2} (5}(s}—{2h{3}[3}- P pe] LP P} |p P m+n m+n m+n [o=}-f3)-fez}- P Pp” P Bat dang thitc S, >S, suy ra tit bat ding thie: FERS pT pe} Lp yl p* | véi moi k. Bat dang thie cudi cimg 1a hé qua cia mot ménh dé vé phan nguyén tang quat hon (xem phan kién thitc bé tro). Bai 46. (1972) Cho n 1 sé ty nhién Ién hon 4, ching minh ring moi tit gidc ndi tiép déu cé thé duge phan chia lam n ttt gide ndi tiép. Given n > 4, prove that every cyclic quadrilateral can be dissected into n cyclic quadrilaterals. Huéng dan: ‘Trudc hét, ta nhan xét ring tit gidc ndi tiép ABCD luén luén co ae Teamrec act 193 thé duoc phan chia thanh 4 tit gidc néi tiép. That vay, ta ldy P tay y bén trong tit gidc ABCD, goi K la mét diém trén canh AB. Néi PK. Tiép dén, ta ldy trén canh BC mét diém L sao cho KPL=180°-B (diéu nay ching té ti gide KPLB -néi tiép). Ta lai dy M trén CD sao cho LPM =180°—C; sau dé, lay N trén AD sao cho MPN =180° -D. Lic 6, dé dang ching minh duce ring NPK = 180° — Tuy nhién, ta con phai ching minh ring cdc diém P, L, M ldy nhu trén thoa man ¥ dinh cia chung ta, nghia 1a né phai nim trén cdc canh nhw da néi. Diéu nay tuy thudc vao cach ly diém cdc K va P ma phan trinh bay sau day sé ro. Ro rang 1a, néu tit gidc néi tiép ABCD cé hai canh song song vdi nhau (AB // CD) thi bai toan xem nhu duc giai quyét, bai vi hic dé, ta c6 thé chia nhé tit gidc ABCD thanh n tuy ¥ tit gidc (cing néi tiép) bing. n dudng thing song song AB // CD. Vi vay, bai todn sé duge gidi xong néu ta chon cdc diém K va P thé nao cho mét trong cdc tif gidc ndi tiép méi thanh lap cé hai canh song song nhau. Diéu nay thi dé, béi K tuy y, do dé, ta ly PK // AD, hic nay, chic chin PL // CD vi: KPL = 180° -B=D. Van dé cin lai la xem xét sao cho cdc diém K, L, M, N tuong ing nim trén cdc canh AB, BC, CD, AD. Trude hét, xét K va L. K khong thé nim trén AD vi ta c6 PK // AD. Nhu thé, chi cin ldy P di gan diém D thi K sé nim trén canh AB (tranh tinh trang duéng PK cit BC). Tuong ty nhu thé, ta cing lay P di gin D dé bdo dam cho L nim trén canh BC. Lic dé, ta gid sit cd M lin N déu nim trén AD, thé thi bing cach git cho K cé dinh, ta kéo P tién vé gan sat véi CD, N sé dich chuyén lén CD, dé lai M nim trén AD. Bai toan da duge ching minh. Bai 47. (1972) Cho f va g la cdc ham sé xac dinh trén R, nhan gid tri trén R. Véi moi x va y, gid sit f(x + y)+f(x—y) = 2f(x)g(y). Ngoai ra, f khong déng nhat bing 0 va [f(x,)|<1,Vx €R. Chimg minh ring: |(x)|<1,Vx eR. fand g are real-valued functions defined on the real line. For all xand y, f(x+y)+f(x—y)=2f(x)g(y). f is not identically zero and [f()| 1. Lay bat ki x véi {f(x} > 0, ta cd: 2k >|f(x + y)] + [f(x — y)]2 iF + y) + £(x - y)| = Ag(y)IP Co), do dé |f(x)| < kg(y)|- Noi cach khac, k /|g(y} 14 mét cn trén cia |F(x}; ma lai bé hon k. Diéu nay mau thudn. Bai 48. (1972) Tim tat cd cdc nghiém thuc duang ciia hé: (x7 —x5X5)(X3 ~x5x5) <0 (23 x 4x x5 —x4x1) SO (x5 —x5x2)(x} —X5x2) <0 (x5 — 1x3 0x3 — xx) $0 (x3 ~ x9x4)(x? —x9x4) $0. Find all positive real solutions to (x7 = x5%5)(x3 — x55) $0 (3 — x1 (x} —xyx1) $0 (x} —x5x2 MxF —45%2) <0 (x9 = 1143) —xpx5) <0 (x3 ~x2xq)(xP - p44) $0. Hung dan: Truéc hét, ta cé nhan xét ring hé da cho kh6ng thay déi néu ta thuc hién mét phép hodn vi vong quanh cua cac sé x4,X7.X3,X4.%5- Nhu vay, néu (a,b,c,d,e) 14 mét nghiém, thi ching han (b,c,d,e,a) cing 14 mét nghiém. Dé tim tat cd cdc nghiém cia hé, ta hay phan loai cdc nghiém theo hai loai : I. Trong nghiém (x),x2,X3,X4,X5) €6 it nh&t mét sé bing 0. II. Trong nghiém (x),x2,X3.X4,X5) khong cé sé nao bing 0. Ta hay xét ting trudng hop. 1. Do nhan xét da néu, ta hay tim nghiém trong dé x, =0. Tir bat phuong trinh thd hai va thi tu, suy ra: 195 XQX3=X4Xx5 =0. Khi dé cdc bat phuong trinh thit nhét, thit ba, thi: nim tré thanh 0 > —x3X5(x3 — xgx5) = —x3x 5x5 + x9xF = x33, 0 > x3x2 — x9x3x5— xQKIX5 + X3K3 = XGNZ + XGKF, 0 > —x9x4(x3 — xgx4) = —X gx 4X3 + x9x9 = x3x5. Nhu vay: xX4 =X2X5 =X3X4=X3X5 =0. Thanh thi x;x; = 0 vdi i# j G,j=1,2,3,4,5). Diéu nay xay ra khi va chi khi trong nm s6 x; , c6 bén sé bing 0, con sé kia thi tuy y. Il. Trong trudng hop nay, ta hay nhan xét thém ring néu (X1,%,%3,X4.%5) la mot nghiém, thi (x),(x2,tx3,tx4,tx5) cing 14 mot nghiém, vdi t tuy y. Bay gid ta hay dé y ring theo bat phuong trinh thi nhat, tich X3X5 phai nm gitia hai sé duong x},x3, vay x3x5>0, ttic lA x3,x5 06 cing dau. Tuong ty, theo bat phuong trinh thit ba, suy ra ring x5,x> cd cing dau. Lai theo bat phuong trinh thit nim, suy ra ring X2.X4 CO cing dau. Cuéi cing, theo bat phuong trinh thif hai, suy ra ring x4,x; 6 cing ddu. Thanh thé cA 5 sé x1,x2,X3,X4,X5 c6 cing ddu, va do nhan xét vita néu & trén, ta c6 thé gid thiét ring x; > 0 (i=1,2,3,4,5)- Dé tiép tuc tim nghiém, ta cé thé gid thiét (do nhan xét dau tién da néu) ring x, 1a sé nhé nat trong cdc s6 x1,x2,x3,%4.%5- Tw cac bat phuong trinh thé nhat va thé nam, suy ra: x X. x X3X5 SX}, X9Xq Sx}, hay -2< 72 <=5_ @ a2 5 Aa Ta hay so sinh x va x5. Hai kha nang cé thé xay ra: © x2 x5. TH(1) tacd: x) Sx3 x7 2, do dé, (1) tuong duang véi: x 197 t? -2+at+b=0 hay 2-t? =at+b, - 2 _ 9,2 tits h? —2]=Jace ils va? +b? Vi? +1, suy ra a? +b? 2 : vel (m-3)? oe a. Bing céch khio sét ham f(m) = &"—3)- m>5, ta tim duge gid m Dat m=t? +1 thi m>5 va a? +b? > tri nhé nhét cha a7 +b? 1a thi m=5e||=2e)x|=1eoa=4% vi bas. Tom lai, min(a? + v?) Bai 50. (1973) Mét ngudi linh 1am nhiém vu ra min, anh ta can phai dd cing khap mét khu vuc c6 hinh tam gidc déu, May dd min (detector) c6 ban kinh dé hiéu qua bing mét nia chiéu cao tam gidc dé. Ngudi linh bat ddu tir mét dinh cua tam gidc. Héi anh ta nén di theo con duéng nao dé cho dé 1a con dudng ngfin nhat ma van dd khdp dugc cd mién tam gidc? A soldier needs to sweep a region with the shape of an equilateral triangle for mines. The detector has an effective radius equal to half the altitude of the triangle. He starts at a vertex of the triangle. What path should he follow in order to travel the least distance and still sweep the whole region? Huéng dan: Khéng mat tinh tng quat, ta gid str ring khu vue cdn ra min 1a tam gidc déu ABC cé canh bing 1, va ngudi linh khéi sy ti dinh A. Anh ta cdn phai ra dén hai dinh kia cua tam gidc. Do dé, con dutng anh ta di chuyén phi giao véi hai dutng tron tam B, C, ban kinh aD Gia si dudng di cha ngudi linh cdt hai dung tron tam B, C néi trén tugng ting tai X va Y, va gid sit anh ta di dén X trudéc, dén Y sau. Dé dudng di ngin nhit, rd rang cdc dudng tu A dén X va tit X dén Y phai la nhiing dudng thing. Hon nia, dudng di ngin nhat tit X dén dudng tron tam C phai nim trén dutng thang XC va cé dé dai 198 AX + XC Vi lé dé, ta di tim diém X sao cho AX + XC cuc tiéu. Xét diém P, giao cia dudng cao BK ké tit B véi dudng tron tam B ban kinh 7 Ban doc cé thé ching minh duoc ring néu P’ 1a mét diém tuy y (khac P) nim trén dudng thing qua P vuéng gic véi BK, ta luén 6: AP’ + P'C > AP + PC. Bay gid, véi X nim trén dudng tron tam B néi trén, goi P’ 1a giao diém cia AX véi duéng thing qua P vuéng goc BK, ta dude: AX + XC > AP’ + P'C > AP + PC. Nhu vay, ta chon X tring véi P xdc dinh nhu trén. Van dé cin lai la kiém tra xem ba hinh tron tam A, X, Y (xdc dinh nhu da néi trén), ban kinh J3/4, cé phd duge tron ven tam gidc hay khong. Diéu nay ro rang, do hinh tron tam X da phu duge gan tron tam gidc, ngoai trix. mét phan nhé gan diém A va mét phan nhé gan diém C, ma cdc phan nay thi phi duc béi cdc hinh tron tam A va Y (ban kinh V3/4). Bai 51. (1973) Goi G 1a tap hop tat ca cdc ham f :R > R, véi f khdc ham hing, va f co dang f(x)=ax+b, a,beR. Néu f,geG thi f.geG, véi dinh nghia f(x) =f[g0]. Khi feG thi f'(x) = x8, dod6 f'eG. Ta néi x; 1a diém bat dong cua f néu f(x-) =x; . Ta gid sit moi f eG déu c6 diém bat dong. Chimg minh ring tét cA cdc ham trong G déu 6 chung mét diém bat dong. G is a set of non-constant functions f. Each f is defined on the real line and has the form f(x) = ax + b for some seu! a, &. If f and g are in G, then so is fg, where fg is defined by f.g(x) = f[g(x)]. If fis in G, then so is the inverse f~'. If fix) =ax+ b, then f-\(x)=*—. Every fin @ nas oe a fixed point (in other words we can find x,such that f(x )=x,). Prove that all the functions in G have a common fixed point. Hung dan: Dé théy f(x) =ax+b 06 diém bat dong rs néu al. -a 199 Con néu a = 1, do gia thiét ring f cé diém bat déng nén ta phai cé b=0, hic dé, moi diém déu la diém bat dong cua £. Gia stt trong G cé hai ham f(x) =ax+b va g(x)=ax+b'. Lic 6 b=b’. That vay, theo dé bai, ham nguoc cua fa h(x) = ab , ta c6: aa ha(xyax+ Pb, aa Theo gia thiét, h.g eG nén ta phaicé b'=b. Bay gid, cho f(x)=ax +b, g(x)=cx+d 1a hai ham tay y thudc G. Khi dé: f.g(x)=acx +(ad +b) , gf(x)=acx +(be+d). Theo ching minh trén ta phai cé ad +b = be +d, suy ra: b c l-a 1-d° Néi cach khéc, f va g cé chung mét diém bat déng, suy ra diéu phai chiing minh. Bai 52, (1973) Gia thit a),a2....,a, la n sé duong da cho truéc, q 1a mét sé da cho trude véi 0 0 vdi moi i, j=1,2,....n Va Cy, =I. (1) Diéu kién b) cua bai toan dude viét duéi dang: Cea FC 41.282 FF Chen C14] +CK 282 +L pny q bat ding thie nay sé dude thoa man néu ta cé: Ck qs s— (2) CK q véi moi j= 1,2, ...,n va moi k =1,2,...,.n—1, va trong tat ca cc bat ding thric (2) ting véi j=1,2,...,n, cd it nhdt mét bat dang thiic thyc su. as 1 Trong (2), lay j=k thi ta c6 theo (1): 9S CKaik a tir day ta thy ring nén chon Cy 41,, =4. Véi cach chon nay, dua trén (2), ta thay -k os Prk va ring c6 thé chon C424 = 47 5--Cnk =4 ss =g*-! Chak =G5-5C.K =o Thanh thi c6 thé viét (*) duéi dang by =a) +a) +q7a3 +...+q" "a, by =qaj +a) +qa3 +...4q" 7a, b3 = qa, +qa2 +a3+...+q" a, by =a" 1ay +q" ay +q" a3 +..4aq, *) tite 1a by =q*!a, +q* 7a, +... +. gay) tay + ayy +. +q™*a,. Vi O\C3K1}.2** is not divisible by 5 for any non- k0 negative integer n. Huéng dan: Dat. x = vB , ding khai trién nhi thie Newton dé bién déi: (1+x)?"*! =A+Bx, *) 0 trong d6 B= )°C3Kt}.2°* . Tuong ty: k=0 (i-x)?"*! =A-Bx. c*) Nhan vé theo vé (*) va (**) ta duge: 7°°*! = 8B? - A?. Mat khdc, 72°*! = 42 (mod 5). Do vay, néu B 1a béi ca 5 thi A? =+2(mod 5), diéu nay khong thé xdy ra. Bai 56, (1974) Mét ban c3 8x8 6, duoc chia thanh p hinh chit nhat réi nhau (duing ké phan chia doc theo cdc dutng ranh giéi giifa cdc 6 ban cd), sao cho méi hinh chit nhat cé sé cdc 6 tring bing sé cac 6 den, va cic hinh chi nhat nay cé téng sé 6 vudng (6 mai hinh) khéc nhau. Tim gid tri Ién nhit cé thé cé cia p va tap céc kich thuéc c6 thé cé cba nhiing hinh chi nhat. 204 An 8x8 chessboard is divided into p disjoint rectangles (along the lines between squares), so that each rectangle has the same number of white squares as black squares, and each rectangle has a different number of squares. Find the maximum possible value of p and all possible sets of rectangle sizes. Huéng dan: Diéu kién mdi hinh chit nhat cé sé cdc 6 trdng bang sé cde 6 den c6 nghia 1a méi hinh chit nhat cé mét sé chin cdc 6 vudng. Ta cé: 2+ 4+ 6 +84 10+ 12+ 14 + 16 = 72 > 64, do dé phai co p < 8. Cé 5 kha ning phan sé 64 thanh ting cia 7 sé khong bing nhau: 24446484104 12422; 24446484104 14420; 24446484104 16+ 18; 24446484124 14418; 24+4+6410+12+14+ 16. Kha nang dau tién bi loai bé vi mét hinh chit nhat c6 22 6 vuéng thi phai cé mét canh dai hon 8 (don vi 6). Cac kha nang con lai chap nhan due, cu thé, su phan chia nhu sau: 22222224 22222224 11111554 Did vio 6 4 11111554 Trt’ 6 6 4 33333664 33333774 22222222 2,22 292 22 2 11111155 111211155 11111155 33333766 33333766 44444444 22222227 22222227 pore rrpras 11111144 11111144 33333344 33333366 55555566 111121111 Presa 22222227 2222222 7 33333366 33333366 44444555 44444555 Bai 57. (1974) Véi moi sé thuc dusng a, b, c, d, hay xac dinh nhing gid tri co thé nhan duge cia biéu thie a b c + tar . atb+d a+b+ce b+c+d atc+d Determine all possible values of a b c d + + + a+b+d at+b+c bt+c+d atct+d for positive reals a, b, ¢, d. Huéng dan: Goi S 1a biéu thic 6 dé bai, ta sé ching minh ring S cé gia tri nam giita 1 va 2. That vay, truéc hét ta cd: a b c d S= . + + > atb+d atbtc btc+d atct+d a b c d +—____ + —__——__ + _——__ = 1. atb+c+d at+btctd atbt+ctd atbtct+d Khong mit tinh téng quat, ta gid sit a 14 sé lén nhdt trong 4 sé thuc dugng a, b, c, d. Khi dé, <1 va: atbt+d 206 b c d bo. c d + + < + + 7 at+b+c b+c+d a+c+d bt+c+d b+c+d b+e+d do vay S< 1+ 1=2. Tom lai, tacé61i, >1, —— 0, >0 atb+d btet+d atb+te atc+d do dé S — 2. Mat khac, néu dat a = 1, c= d va cho b, : rat bé (tién din dén 0 mét cach lién tuc), thi S —> 1 béi vi ta c6: a >i, ci 0, 20, —— a+b+d b+c+d atb+e atc+d Ta tinh lién tuc, ta suy ra S cé thé nhn bat ki gid tri nao trong khoang m6 (1, 2). Bai 58. (1974) Cho da thitc P(x) cé bac d > 0 va 6 cac hé sé nguyén. Goi n 1a sé t4t cd cdc nghiém nguyén phan biét cia hai phugng trinh P(x) = 1 va P(x) = -1. Ching minh ring n < d+ 2. Let P(x) be a polynomial with integer coefficients of degree d > 0. Let n be the number of distinct integer roots to P(x) = 1 or P(x) = -1. Prove thatn < d+ 2. Huéng dan: Xét hai da thite A(x) va B(x), véi cdc hé sé nguyén, chung giéng nhau hoan toan, chi trix hai sé hang tu do 1a khac nhau, hai sé hang nay hon kém nhau 2 don vi. Goi r va s 14 cdc nghiém nguyén tuong ting cia hai da thie, tic 1a A@)=0, () Bis)=0. (2) Khi dé, trit (1) cho (2) ta duge mét ting cia cdc hang tit co dang a(r' —s') va cong thém cho 2. Méi hang tif nay chia hét cho (r—s), do dé 2 phai chia hét cho (r—s). Ti d6, suy ra r vas hon kém nhau 0, 1 hodc 2 don vi. Bay gid, ta 4p dung nhan xét nay dé giai bai todn. Gid st r la nghiém nguyén bé nhat trong tét cd cdc nghiém nguyén ca hai phudng trinh P(x) = 1 va P(x) = -1. Ta biét ring da thiic bac d cé khéng qua d nghiém phan biét, do 6 né cing cé khéng qué d nghiém nguyén phan biét. Theo nhan xét 207 trén, néu r 14 mét nghiém nguyén cua phudng trinh nay va s 1a mét nghiém nguyén cua phuong trinh kia thi r va s khdc nhau 0, 1 hodc 2 don vi. Nhung ta cé s 2 r, do dé ta dude s = r,s =r+ 1 hoics=r+2. Do vay, ta suy ra ring phuong trinh thif hai chi cé thém vao nhiéu nhit 1a 2 nghiém phan biét nia. Noi cdch khae, ta cé diéu phai ching minh: n...2Xq, Yj 2Y2 2 ~-.2Yq- Chting minh ring néu day céc sé Z,Z2,-5 Zq 1a mét hodn vi bat ki ca cdc sé y, yz... Yq thi ta c6: A n 2 Xi -¥) < Ne; -2)’. isl isl Let x, 2x2 >...2X, and y, >y7>...2y, be real numbers. Prove that if 2), 22,..., Zq is any permutation of the y,,¥2,.-s Yq» then we have S(x,- 94)? <3; - 2). i=l fl Huéng dan: True hét, ban doc dé dang chting minh dusc ring néu ta c6 x>x' va y>y' thi (x-y)? +(x-y')? <(x-y')? +(x-y)?. Suy ra, néu i p. Huéng dan: Ta cé dinh sé p>1. Véi diéu kién Oap- Lic dé, vdi moi a, € B, ma m >q, ta c6 a, = aq (mod a, ), do d6 ay, = Xap + aq, Voi 86 tu nhién x nado do. Bay gid, dat y = 1, ta cé diéu phai ching minh. Bai 61. (1975) Goi A la ting cdc chit sé (trong hé thap phan) cia sé 4444" va goi B la ting cac chit sé cua sé A. Hay tim tng cdc chit sé cia sé B. Let A be the sum of the decimal digits of 4444" and B be the sum of the decimal digits of A. Find the sum of the decimal digits of B. Huéng dan: Dat X = 4444) khi do, X phai cé sé cdc chit sé (thap phn) bé hon sé 4x 4444 = 17776. Vi vay, gid tri lin nhat cia sé Ala 9x 17776 = 159984. Suy ra ring gid tri lén nhat cia sé B 1d 5x9 = 45, diéu nay cé nghia ting cdc chit sé ciia B 1a sé chi cé mét chit s6. Ta c6 4444=~-2 (mod 9) nén tit 6 suy ra X = (-2)"“4 (mod 9). Mat khae, (-2)° =1 (mod 9) va 4444=1 (mod 3), do dé: X = -2 =7 (mod 9). Vay dap sé cdn tim cia bai toan 1a 7. Bai 62. (1975) Xac dinh tat ca cdc da thuc hai bién P(x, y) sao cho ca 3 diéu kién sau day duge théa man: 1) Tén tai sé nguyén duong n va moi sé thuc t, x, y: P(tx,ty) =t"P(x, y). 2) Véi moi sé thuc x, y, 2: Ply + z, x) + Pz +x, y) + P(x +y,z)=0. 3) PQL,O)=1. 209 swenmcesee Find all polynomials P(x, y) in two variables such that: D P(tx,ty) =t" P(x, y) for some positive integer n and all real t, x, y. 2) For all real x, y, 2: Ply + 2, x) + Plz +x, y) + P(x +y, 2) =0. 3) PU, 0)=1. Huéng dan: Diéu kién (1) thudng duoc goi 14 tinh thudn nhat bac n cia P(x, y). Xét trudng hop n = 1, 2, 3, ta dé dang tim thay cdc da thiic tuong img théa didu kién d& bai la: x —2y, (x+y\x-2y), (x + y)?(x-2y). Ta cing dé dang kiém tra duc ring véi moi n, da thite (x+y)"(x -2y) théa diéu kién dé Bay gid, tit diéu kién (2), cho x = y = z ta dude P(2x, x) = 0, nén da thie P(x, y) théa diéu kién dé bai luén nhan (x —2y) lam mét nhan ti. Lay x= y = 1,2 = —2, diéu kién (2) cho ta P(1,—12" -2)=0, do 6, (x + y) 1a mt nhan tit cda da thie khi n > 1. TAt cd nhiing didu trén ggi y ring nghiém téng quat cia bai ton la: (x + y)" (x — 2y). Ta sé ching minh diéu nay. Tw (2), cho y = I—x, z = 0, ta duge: P(x, 1-x)=-I-P(I~x, x), dac biét, P(O, 1) = —2. Bay gid, cho z = 1-x-y, ta duge: P(I—x, x)+P(I-y,y)+P(x+y.1-x-y)=0, suy ra {(x+y)=f(x)+f(y), 6 day ta dat {(x)=P(I-x,x)-1. Bang quy nap, ta dé dang ching minh duce ring, véi moi sé nguyén m va moi sé thuc x, ta c6 f(mx) = mf(x). ‘Tir dé, suy ra ring véi moi r,s ta c6 {2)-i0. (}-£0. 2 Ls Ss. s Nhung P(0, 1) = -2, do dé f(1) =-3, vay f(x) =—3x véi moi sé hitu ti x. Nhung f(x) 1 ham lién tue nén f(x)=-3x véi moi sé the x. Véi a, bla cée sé thye tiy ¥ via +b+0, ta dat x= 2 khi dé, a Pla, b) = (a+b) P(I-x,x) =(a+6)"(SR 1) =r e—20. atb Dé y ring khi a +b = 0, vdi n> 1, tit tinh lién tuc ta cing c6 P(a, b) = 0. Tom lai, nghiém téng quat ciia bai toan 1a: 210 ca inmecase P(x, y) = (x +y)"(x —2y). Bai 63. (1976) Cho cde da thiic P, (x), k =1,2,3,... xc dinh bii: Ph(x)=x?-2, Pig =PY(Pi(x)), 1= 1,2, Ching minh ring P,(x)=x cé tat ca cdc nghiém déu Ia cdc sé thuc phan biét. nhau. Let Pi(x)= x? -2, Pry = B(PQ)for i= 1, 2, 3, Show that the roots of P,(x) =x are real and distinct for all n. Huéng 2 DAt x =2cost, ta thu hep viée xét nghiém cia phuong trinh trén doan -—2n-2. Nhung 6° > 0,4.8°, suy ra: (n’k)? > (n-2)° > 0,4.n°, voi moi n>8. Ta cé thé kiém tra truc tiép dugc ring (n’k)® > 0,4.n? véin = 3, 4, 5, 6, 7. Vi vay, suy ra a = 2 (khong thé 6 a = 1, vi I’ = 0). Tir*), ta phai cé b hay c chia hét cho 5. Ta gid st ring mét trong hai canh d6 bang 5. Vi 5’ =3, nén néu canh cdn lai cé dé dai n thi ti (*) ta phai ta phai c6: §.2'.5'n' =2.5n on" Ce) et aas oy 2k 6 Ta cé thé kiém tra dé dang rang Se 2 en nee do dé ta duge: n'>Sn,vn27. er) Bat dang thic (***) mau thudn véi (**) nén viée con lai 1a kiém “712. yoo nego tra khi n<7. Tuy vay, vi n’= 3" la sé tu nhién nén n 1a béi cua 3. Do d6 chi cdn kiém tra véi n = 3 va n = 6. Ca hai két qua nay déu chdp nhan duge. Do vay, kich thudéc cda hinh hép da cho 1a 2x3x5 hokc 2x3x6. Cuéi cing, véi mét canh bing 2, ta con phdi tinh dén kha nang mét canh bing 10 tré Jén (nhu trén, cé mét canh chia hét cho 5). Gia st 212 canh dé 1a m ( m 1a béi cia 5). D8 dang kiém tra dude ring ™ > —"-. Goi m oe ri canh cin lai. Tit (*) ta suy ra r phai thod min: © < . Két hgp vai 7: nhing tinh todn vé n’ véi n nhé 6 trén, ta suy ra r = 2. Vay a = b = 2. Nhung gid day ¢ phai thod digu kién c’ = fe. Didu nay khong thé duse vi ta phdi ob tk >1. Tém lai, kich thuée cda hinh hép 4a cho 1a 2x3x5 hotic 2x3x6. Bai 65. (1976) ' Hay xac dinh sé 1én nhat la tich ca cdc sé nguyén duong sao cho cdc sé nguyén dudng dé cé ting bing 1976. Determine the largest number which is the product of positive integers with sum 1976. Huéng dan: Gia st ta tim dude sé N lén nhét 1a tich ca m sé nguyén duong nj, sao cho m sé nguyén dugng nj, d6 cé ting bing 1976 (m= 1, 2, ... m, cdc ny c6 thé trang nhau). Khi dé, trong cdc sé n, , khong thé cé sé lén hon 4, vi néu ngude lai, gid sit tén tai t dén,> 4, ta thay sé n, béi hai sé 3 va n, —3 thi ting cia m + 186 nj,...,3,....0, —3,...,m van la 1976, trong khi dé ta lai nhan dugc mét tich Ién hon, diéu nay trai vdi gid thiét N lén nhat. Trong cdc sé n, dé cting khéng cé sé nao bing 1. That vay, néu c6 sé bing 1 thi ngoai ra cing c6 s6 r > 1 (béi néu khéng thi tich chi bing 1), khi d6, ta thay 1 va r bing sé r + 1 thi duge cdc sé van cé ting 1976, nhung tich lai lén hon N, do ta cé r+ 1 > Lr, mau thudn! Ngoai ra, néu trong cdc sé ny dé cé sé 4 thi ta cing cé thé thay 86 4 béi hai sé 2 va tich cing nhu ting sé khong thay déi. ‘Tém lai, cdc sé n, cdn tim gom toan cac sé 2 va 3. Ta cé: 1976 = 3x 658 + 2. Vay s6 N tim duge la: N = 2x38, Bai 66. (1976) Cho sé nguyén duong n va m = 2n. Véi moi i, j thoa diéu kién 213 I0 nén suy ra x; +Xx7 +x; <0. Cing vay, Vi X5+X6 +..+X]) <0 Va xX) +X +...+X1,>0 nén xy >0-. Gia su day da cho cé nhiéu hon 16 phan ti thi Ii ludn tusng tu nhu trén ta sé duge x5 >0, x4 >0, x7 >0. Thé nhung Xp HX t..4X7 <0, Ks +Xe+..4+%1) <0, va X;+X2+...+X,;>0 nén ta cing suy ra x5 +X¢+X7 <0, mau thuan! Vay sé cdc sé hang cé trong day 44 nhai <16. Ta sé chimg td ring 16 1a dap sé cua bai ton, muén vay ta di tim day gém 16 sé hang thoa diéu kién dé bai. Gia str cé mét day gom 16 sé hang nhu thé, ma don gidn nht, ting cia 7 s6 hang lién tiép ta -T, téng cia 11 sé hang lién tiép 1a 1, ta dé dang thiét lap cdc phugng trinh va tim duge day 5,5, -13,5,5,5, -13, 5,5, -13, 5,5, -13,5,5,5. Tom lai, gid tri lén nhdt cia sé cac sé hang c6 trong day thod man yéu cu dé bai 1a 16. 215 Bai 68. (1977) Cho n 1a sé nguyén lén hon 2, goi V, 1a tap cde sé nguyén cd dang 1 + kn, vdi k nguyén duong. Mot sé m thude V,, dugc goi la bt khd phén (indecomposable) néu né khéng thé biéu dién duge thanh tich cia 2.86 thuéc V,. Chung minh ring tén tai mét phan ti cia V, sao cho phan tu do cé thé biéu dién duoc thanh tich cde phan ti bat kha phan cia V,, bing nhiéu hon mét cach (cdc biéu dién chi khdc nhau vé thi ty nhan ti thi duoc xem 1a nhu nhau). Given an integer n > 2, let V,, be the set of integers 1+ kn for ka positive integer. A number m in ¥., is called indecomposable if it cannot be expressed as the product of two members of V,. Prove that there is a number in ¥,, which can be expressed as the product of indecomposable members of ¥., in more than one way (decompositions which differ solely in the order of factors are not regarded as different). Huéng dan: Ta xét cac sé a = b = n-1,¢ =d = 2n-1. Dé dang thay ring a? © V,,, ngoai ra, ta cé a 1a phan ti bat kha phan cia V,, vi nd bé hon (1+n)’, tic 1a bé hon binh phuong phan ti nhé nhat cia V,. Gid sii ac = 2n? —3n+1 (thude V,,) khong bdt khd phan, lic dé, taco kkin+k +k’ = 2n-3, véi cdc sé nguyén k, k’ >1. Nhung k, k’ khong thé > 2, vi néu khong thé thi vé trai cia dang thiic trén sé qua 1én. Con néu k =k’ = 1 thisuy ran = 5. Tuong ty nhu vay, néu c?(thude V, ) khong bdt khd phén thi ta seco kkn+k+k' = 4n—4. Lic dé, néu k =k’ = 1 thin = 2, diéu nay khéng thé vin > 2 theo gid thiét, Néu k = 1, k’ = 2, ta phai cd n =i. nén cing khéng duge. Néu k = 1, k' = 3 thin = 8. Cac kha nang khdc lam cho vé trai cia ding thu trén qua lén. Nhu vay, tif cac phan tich trén ta di dén: * Néu n khac 5 hodc khac 8, ta chon s6 (n—1)?(2n-1)”, sé nay thuée V,, va cé thé phn tich dude bing 2 cach thanh tich cae phan ti bat khd phan cia V,, nhu sau: 216 (n=1)? x (2n—1)?, (n=1)(2n-1)x(n-1)(2n-1). Nhung dé y ring diéu nay khong thuc hién dugc cho n = 5 va cho n = 8, vi 16.81 = 36.36 nhung 36 khéng phai phan tu bat kha phan (6x6); 49.225 = 105.105, nhung 225 = 9x 25. * Néun =5, ta chon 3136: 3136 = 16.196 = 56.56. *Néun=8, ta chon 25921: 25921 = 49.529 = 161.161. Bai 69. (1977) Cho a, b, A, B 1a cdc hing sé thyc va f(x) = 1-acosx — bsin x — Acos2x — Bsin2x . Gia sit £(x) > 0, Vx ER. Chimg minh ring a? +b? <2 va A?+B? <1. Define f(x) =1-—acosx—bsinx— Acos2x-Bsin2x, where a, b, A, Bare real constants. Suppose that f(x)>0 for all real x. Prove that a? +b? <2 and A?+B' <1. Huéng dan: Goi y va z 1a cde sé thuc sao cho b a : cosy === , siny= > Va? +b? Va? +b? A B VA? +B?” cosz= sinz= VA? +B? ' Khi dé, biéu thie f(x) tré thanh: f(x) =1—ccos(x —y) -Ccos2(x -2), véi c=va?+b?,C=VA74+B?. Tir d6, f(z)+f (4 +z)20 cho ta C= VA? +B? <1 va (ra}etrthe cho ta c= b? <2, diéu phai ching minh. Bai 70. (1977) Cho hai sé nguyén duong a va b. Dem a? +b? chia cho a + b ta duc thuong 1a q va phan du la r. Xic dinh tat cd cdc cp s6 a, b sao cho q? +1r=1977. Let a and b be positive integers. When a? + b* is divided by a + b, the quotient is q and the remainder is r. Find all pairs a, b such that q?+r=1977. 217 Huéng dan: 2 Taco a 4b?2 2 d0a6q 282°. suy rar <2 Mat khac, sé binh phuong lén nhat nhé hon 1977 1a 447 = 1936. Ta cé 1977 = 447 +41. Sé lén nhdt ké theo cho ta 1977 = 43 +128. Nhung 128 > 2.43, do dé ta phai c6 q = 44, r = 41. Tit dé suy ra a? +b? = 44a +b)+41, do vay: (a — 22) + (b—22)? = 1009. ‘Thi lai, ta tim thdy 1009 = 28? +157. Tir d6, ta tim dugc cdc sé a, b 14 50, 37 va 50, 7. Bai 71. (1977) Cho ham f xéc dinh trén tap cdc sé nguyén duong va cing nhan gid tri nguyén dung. Gia sit véi moi n ta cé: (n+1)>f(F(n)). Ching minh ring f(n)=n véi moi n. The function f is defined on the set of positive integers and its values are positive integers. Given that (n+1)> f(f(n)) for all n, prove that f(n) =n for all n. Huéng dan: Truéc tién ta sé ching ti ring f(1)r thi f(r) 1 thi f(m) > fis), véi s=f(m—-I), nén f(m) khéng thé la phan tit nhé nhét cta tap {f(1),f(2),f(3),..}. Nhung tap nay bi chin duéi béi 0, nén chdc chin né phai cé phdu tu bé nhét. Suy ra phan tu nay 1a f(1). Vay S, ding. Gia si S, ding. Lay m>n+1, khi d6 m-1>n, do dé ta cd f(m-1) > f(n) (vi S, dung). Nhung ciing tir S, ta co: f(n)>f(n-1)>...>F(), do vay, ta duge f(n)>n-1+f(l)2n. Suy ra f(m—-1)2>n+l, tu do f(m) > f(n +1). Tw day suy ra ring Sas ding. Vay S, dting véi moi n. Néi céch khac, néu nm-+1, thé thi f(f(m)) > f(m+1), diéu nay mau thudn. 218 Suy ra f(m) f(m—-1) >...> FQ) nén f(m)>m_véi moi m. Suy ra {(m) =m véi moi m. Bai 72. (1978) Cho m va n 1a nhiing sé tu nhién véi n > m2>1. Trong cach viét thap phan, ba chit sé cudi cing cia sé 1978" theo thit tu bing ba chit sé cudi cing cia 1978". Tim cc sé m va n sao cho tong m+n cé gid tri nhé nhat. m and n are positive integers with m 978" = 978" (mod 1000), hay 978™(978"~™ — 1) = 0(mod 1000) <> 2"(978""™ —1) =0 (mod 1000) , (1) do 978 = 2.3.163. Ta thay 1000=27.5°, ma 978"-™ -1 lé nén suy ra 2™ phai chia hét cho 2°. Vay m>3. Ngoai ra,(I) tuong duong véi 2™-3(978""™ —1) chia hét cho 2°.5? =1000. (2) Vi m2>3 nén chia cd hai vé clia (2) cho 23 ta cé: 2™-3(978"-™ _1) chia hét cho 5°, hay 978"-"—1 chia hét cho 5°, (3) vi 2 va 3 nguyén té cing nhau. Muén cé (3) truéc tién ta tim m va n dé 978"-" —1 chia hét cho 5. Ta thay 8! tan cing la 8, 8° tan cing la 4, 8° tan cing la 2, 8* tan cing 1a 6, 8° tan cung 1a 8, ... Nhu vay chu ki cua sé chit tan cing cla luy thita cua 8 1a (8,4,2,6). Mudén cho 978""™ —1 chia hét cho 5 thi cha sé tan cing cta 978""™—1 phai la 5 hodc 0, nhu vay chi c6 8** véi k =1,2,3... thi 8"* —1 mdi tan cing bing 5. Suy ra n-m=4k véi k =1,2,3... Tiép theo, ta cing cé thé Ii ludn dé ching minh dude ring n—m=100p (p=1,2,3,...). (4) Két hgp véi diéu kién m > 3 ban dau suy ra: Muén cho 219 978" =978" (mod 1000) thi m+n2100p+62106, vi p21. Vay gid tri nhé nhat cia téng m+n dé 1978™ cé ba chit sé cudi cing theo thi ty bing ba chit sé cudi cing cua 1978" 1a 106. Khi do m=3 va n=103. Bai 73, (1978) Biét ring tap hop tat cd nhiing sé nguyén duong la hap cia hai tap hgp con khéng giao nhau {F (1), f(2),-.-»F(n),...}, {8(),8(2),.--.8(1),...} trong dé: f(1) < f(2)<...< f(n)<..., g(l)< g(2)<...< g(n)<... va g(n) = f(f(n))+1 véi moi n 21. Hay xac dinh f (240). The set of all positive integers is the union of two disjoint subsets {FDS (2oer SM o-- bs {81,8 (2)s-- 8()---} > where f(I)< f(2)<..1 nén f(f(1))>1, suy ra g(I)>2. Nhu thé, 1 khéng thuéc G nén 1 thuéc F, 1 phai 14 phan tit bé nhat ciia F, do d6 f1) = 1. Ti dé g(1) = 2. Ta khéng thé cé dugc hai sé lién tiép n va n + 1 trong G, béi vi néu g(m) = n + 1 thi ta suy ra f(k) = n (voi k nao dé), khi dé, n vita thuée F vita thuéc G, mau thudn. Dac biét, ta co 3 thudc F (vi theo trén 2 thudc G), do dé f(2) = 3. Gia stt fln) = k. Laie dé g(n) = fk) + 1. Suy ra |Neqys4 OG [=n (ki higu IAI chi sé phan ti cia tap hifu han A). Nhung ta 06 [Neayy1 OF =k nén suy ran + k = f(k) + 1, hay f(k)=n+k-1. Ti d6 g(n)=n+k, do vay (n + k + 1) phai thudéc F va ta suyra f(k+I)=n+k+1. Nhu thé, néu biét fn) = k, ta cé thé tinh duge f(k)=n+k-1 va f(k+l)=n+k+l. Két qua nay cho ta tinh duge: f(2) = 8, f4) = 6, 7) = 11, 112) = 19, 20) = 32, (33) = 53, £154) = 87, 88) = 142, (143) = 231, 232) = 375. Day tinh trén (sit dung f(k +1) =n+k +1) khong gidp gi dugc dé tinh f(240), ta lai cé ging vdi f(k) =n+k-I: f(3) = 4, f4) = 6, 16) = 9, R9) = 14, £114) = 22, R22) = 35, {(35) = 56, (56) = 90, f(90) = 145, £145) = 234, ciing sé khéng tinh dude (240) néu tiép tuc di xa hon. Tuy nhién, lai cé gang, véi két qua f(56) = 90, ta ding céng thiic f(k+1)=n+k+1 thi duge (91) = 147, £148) = 239, (240) = 338. Vay ta dude 240) = 338. Bai 74. (1978) Cho {a,} la mét day cae sé nguyén duong phan biét (k =1, 2, ...). n a Chimg minh ring véi moi n>1 ta cé: re) 2dE {a} is a sequence of distinct positive integers. Prove that for all positive integers n, we have ye > xh kak” 4k Huéng dan: A Ki higu Slay aa~-ty)= véi {a;} thoa diéu kién a,eN*,a; #a; néuisj. Ta nhan xét ring néu i>j ma a, j(vi a; >a;). Vay néu thay aj= ajay = aj thi: S(Qj 258 jy--8j>-58p) > S(ay...,a,--5a', aes Ay) Tw dé, goi P = max(aj,...,a,) thi bing cdc thay thé trén ta cé: S$(Qq,--.8}>-94q) > S@y,--»P)» trong dé tit a, dén p 1a mét su s&p xép tit bé dén lén. ‘i Do dé, 48 chimg minh S(a),..,2,) 2 ys ta sé chimg minh: k=l a S(ay.....p) > >t . kel Nhung ro rang a, >k, vi tir a,dén p la mot su sAp xép tit bé dén l6n, dén day ta dé dang suy ra diéu phai ching minh. Dau bing xdy ra khi va chi khi a, =k (k =1.2.....m). Bai 75. (1978) Mét hdi quéc té cé héi vién thuée 6 nuéc hdc nhau. Dai.h sach cac héi vién gdm 1978 ngudi duce danh sé theo thi ty 1, 2, ..., 1978. Chiing minh ring cé it nhdt mét héi vién ma sé thit ty bing tang cdc sé thit tw cia hai héi vién thude cing mét nuéc véi héi vién dé, hose big hai lan sé thi ty céa mét hdi vién thudc cing mét nude véi hdi vién do. An international society has its members from six different countries. The list of members has 1978 names, numbered 1. 2, ... , 1978. Prove that there is at least one member whose number is the sum of the numbers of two members from his own country, or twice the number of a member from his own country. Huéng dan: Gia si 6 nude dé 1a A, B, C, D, E, F. Sé héi vién tang cong 1a 1978 ngudi. Nhu vay tén tai 1 nude cé sé héi vién lén hon hodec bing [| +1=330, 6 ta gia st nuéc dé la A. Ta dénh sé 330 hi vién trong sé héi vién cia nude dé 1A a4,a,...0339 Voi ay j) cing bing sé thi ty cua mét héi vién nao dé trong 1978 hdi visn nay. Bay gid ta gidi thiét trai lai: khéng c6 nuéc nao cé héi vién c6 sé thif ty bing téng cdc sé thit tu cia 2 héi vién nuée dé, hodc bing hai lan sé thi ty cla mot hdi vién nude dé. Ta nhan xét ring: néu a > b 1a hai sé thi ty cia hai héi vién thi a-b2>1 cang la sé thit tu cda 1 hdi vién nao do trong 1978 ngudi nay, tuc 1d khéng tén tai a,b,c 1a ba sé thi ty héi vién cua mot nbc dé cho a—b=c (bcé thé bing c). Xét 329 higu ay —ay,a3 —a),....8339 —a). Cae hiéu nay doi mét khac nhau va theo digu gid str ciia ta, cdc hiéu dé cing khc ca cac sé 222 thi ty céa héi vién thudc nude A. Vay 329 hiéu nay 1a sé thit ty cla 329 hi vién thuéc 5 nude cdn lai. Suy ra_ ton tai [7h =66 hiéu 1a sé i thi tu cia cdc héi vién thudc mét nuéc, gid str dé 1a nude B va 66 hiéu dé ting véi cdc sé thif tu b),b2,....bg, cla cac hdi vién cha B: by i; j, i=1,2,....66) cing 1 sé thit ty héi vién cha mét nuéc nao dé (do 1 jsi, j= 1,2, .. 16) theo gid thiét khong phai la sé thit ty cia héi vién nude C, cing khong thé la sé thi tu cia cdc hdi vién nude B, cing khéng thé 1a sé thit tw cua cdc hdi vién nude A .v.v... Cudi cing, ching minh hiéu f, —f, khong thé 1a sé thi ty ca hoi vién mot trong 6 nuéc nao ca. Nhung theo dé bai, né lai 1a sé thit tu cia mét trong 1978 héi vién tic phai thuée mét trong 6 nuéc. Dé 14 diéu mau thudn, ching té gid thiét cia ta 1a sai. Vay phai tén tai mét nude trong dé sé thi ty cha mét héi vién bang tang sé thif ty cia hai héi vien khdc cua nuéc dé hodc bing hai lin s6 thi ty ca mot héi vién khac cia nude dé. Bai 76. (1979) Cho m va n la céc sé nguyén duong sao cho: moj tjtl i +. + . n 2°93 4°” 1318 1319 Ching minh ring m chia hét cho 1979. Let m and n be positive integers such that: m 1,11 1 I —=l--+2-—+..-—_ + . n 234 1318) 1319 Prove that m is divisible by 1979. Huéng dan: 1 1 1 Toi es 234 1318 1319 wad I 1 11 Sl+itit—+..¢ 4+ ~~~ 4+ + 23 4 1318 1319 24 17 1 1 =o t+ tt. 660 661 1319 Dé y ring 660 + 1319 = 1979, ta tach ting cap cac sé hang 6 téng trén va tinh nhu sau: a, 1979 1 1 1979 660 1319 660.1319’ 661 1318 661.1318" 1 . 1 1979 662" 1317 662.1317" C6 mét sé chin cac c4p dugc tach nhu trén, do vay, ta cé thé tach thanh cdc c&p tAt cd cdc sé hang cua tng trén, va mdi cAp duge tinh nhu trén co dang 1979/k, vdi k khéng chia hét cho 1979 (vi 1979 la sé nguyén té va do dé tich cac sé bé hon 1979 khéng chia hét cho 1979). Ta c6 téng cdc sé dang 1/k néi trén 14 mét phan sé cé mau khéng chia hét cho 1979. Tit day, ban doc cé thé tiép tuc suy ludn dé di dén diéu phai ching minh. Bai 77. (1979) Cho hinh lang tru cé day trén va day duéi 1a cdc ngi giac A\A2A3A4As va B,BB;B,Bs. Mai canh cia hai ngii gidc nay cing nhu méi canh trong 25 canh A,B; (i, j = 1,-..,5) déu duge té mau dé hoac xanh. Biét ring bat ki tam gidc nao tao thanh tit 3 dinh cia ling tru ma ca 3 canh déu duge t6 mau thi phai c6 2 canh cé mau khdc nhau. Ching minh ring tat cd 10 canh cia hai ngi gidc (8 day trén va day duéi) déu cé cing mét mau. A prism with pentagons A\A7A3A,4A; and B,B2B3B,Bs as the top and bottom faces is given. Each side of the two pentagons and each of the 25 segments AiBj is colored red or green. Every triangle whose vertices are vertices of the prism and whose sides have all been colored has two sides of a different color. Prove that all 10 sides of the top and bottom faces have the same color. Huéng dan: * Trude tién, ta ching minh ring t4t cA 5 canh cia ngi gide A\A2A3A4As déu c6 cing mau. That vay, gia sit diéu ngugc lai xay ra, khi dé, tén tai hai canh, chdng han A,A2, A,As, cé mau khac nhau. Ta xét 5 canh A,B;, j = 1, 224 2, ..., 5. Vi chi cé 2 mau xanh va dé nén it nhat trong ching cé 3 canh cing mau, ta gid sit 3 canh nay cig mau xanh, va A,A, cing cd mau xanh. Gia stf dé 1a 3 canh A,B), AjB), A;By . Lic dé, xét cde tam gide A\A2B,, AjA2Bj, AjA2B,, ta c6 3 canh A2B;, A2Bj, A2B, phai cing mau 46. Hai trong 3 dinh B;.B,,B, phai ké nhau, néu hai dinh nay tao thanh canh son mau dé thi cing véi Az, ta duge tam gide c6 cd ba canh cing mau dé, con néu 2 dinh dé tao thanh canh sdn mau xanh thi ching sé cing vdi A, tao thanh tam gidc cé cd 3 canh mau xanh, diéu nay mau thudn véi gid thiét ban dau. Nhyw vay, tat cd 5 canh cia ngii gide AyA2A3AyA5 déu co cing mau. * Tuong tu nhu trén, ta cing chiing minh duge tat cd 5 canh cia ngu gidc B,B2B3B,4Bs cing déu co cung mau. * Van dé con lai 14 ching minh mau cac canh ctia hai nga gidc trén giéng nhau. Gia sit ngudc lai, chang han, 5 canh cua nga giac A,A2A3A4A5 6 mau xanh va 5 canh cia ngi gidc B,B2B3;ByB; cé mau dé. Ré rang 3 trong 5 canh A,B,, i= 1, 2, ..., 5, phai co cing mau. Gia sit 46 14 mau dé, thé thi li luan nhu trén, 2 trong 3 dinh B, dé phai la hai dinh ké nhau, tit dé, hai dinh nay tao véi A,thanh tam gidc cé c& 3 canh mau dé, mau thuan! Nhu thé, 3 trong 5 canh A,B;, i= 1, 2, ....5, phai cé cing mau xanh. Li lun hoan toan tuong ty, 3 trong 5 canh AB;,i= 1,2, ...,5, phai cé cing mau xanh. Trong sé 3 canh A,B; (tap hgp 1) va 3 canh A,B; (tap hgp 2) vita chi ra 6 trén, chic chic phai cé cing mét B, nao dé chung 6 2 tap hap 1 va 2. Khi a6, B, nay sé tao vi A, va A, thanh mét tam gidc cé ca 3 canh mau xanh, diéu nay mau thudn véi gid thiét ban dau. Ta cé diéu phai ching minh. Bai 78. (1979) Tim tt cd nhing sé thuc a sao cho tén tai 5 sé thuc khong 4m X1,%25X3.Xq.%X5 thoa man cac hé thie: ‘Sh lainnacenct 225 > > > a i =... el kel kel Find all real numbers a for which there exist non-negative real numbers X1,X75X3,X4.Xs satisfying: 3 5 3 Sy =a, Ven =e, Vex =a. kel kal Huéng dan: 7 = Tur cdc hé thie 6 dau bai suy ra [Ze] = 5 = kx okOx,, tie k=l 1 ksI = Ta: (x) +23x7 +3°x3 +4°x4 45°x5)? = = (XK, + 2x2 +3x5 + 4x4 + 5x5)x(X, +2°X, +3°X; +4°K, +5°XS). Sau khi khai trién cd hai vé ta sé duge ding thiic ma hé s6 cla cdc x} 6 hai vé déu bing nhau, trong khi dé, vdi moi tich x;x; (i #j) thi cdc hé sé cha né 6 vé phai déu 1én hon hé sé cua né & vé trai. Didu do chiing té ring moi tich x;x, (i#j) déu bing 0, tric 14 trong 5 sé x, thi chi cé nhiéu nhat mét sé duong. -Néu moi k, x, =0 thi a=0. - Néu x, >0 thi hé thiic thi nhat (6 dau bai) cho ta x, =a, hé thife thi hai cho x, =a”. Vay a=1, cdc gid tri nay cla x, va a cing thoa man hé thiic thit ba. - Néu x > 0 thi hé the thi nhat cho 2x. =a => x, =a/2, thay vo hé thiic thit hai ta tinh duce a = 4, cdc gia tri a=4 va x) =4:2=2 cing thod man hé thitc thit ba. - Néu x; >0 thi hé thtic thi nhat cho 3x3 =a =>x; =a/3, thay vao hé thtic thif hai ta tim dude a=9, hé thitc thir 3 ciing dude thod man. - Tung tu, néu x, > 0 ta tim dude a=16. -Néu x, >0 ta tim duge a= 25. Bai ton cé cdc nghiém Ja a =0, 1, 4, 9, 16, 25. Bai 79. (1979) Mét con éch nhay tit dinh A dén dinh déi tam E cua mét hinh bat 226 130 Yotaroc ict u. Tai mdi dinh cia bat gidc trit dinh E, con éch cé thé nhay mot buéc téi mét trong hai dinh ké. Dén E thi éch dimg lai va & luén tai dé. Goi a, 18 sé cdc duéng di phn biét cia con éch di tu A dén E bing dung n bude nhay. Ching minh rang: 1 -1 1 a, 1 =0, agg = —=[(2 + V2)" -(2- V2)"1}. 2n-l 5 (¢ ) ¢ | Let A and E be opposite vertices of an octagon. A frog starts at vertex A. From any vertex except E it jumps to one of the two adjacent vertices. When it reaches E it stops. Let a, be the number of distinct paths of exactly n jumps ending at E. Prove that: 47n-1=0, dyq = ple v2y"t -(2-V2y""]. Huéng dan: Mai buéc con éch nhay thuan chiéu kim déng hé ta biéu thi bing miét dau cong va méi buéc con éch nhay ngude chiéu kim déng hé ta biéu thi bing mét d4u trit. Lic d6, mdi dudng di phan biét cia con éch cé n bude nhay sé duge biéu thi bing mét day n du cong va tri. Dé thdy ring méi day déu cé hai tinh chat: 1) Sé ddu cong va trix chénh nhau diing 4. 2) Mét day con gém cae dau thi nhat dén déu thi m, véi m,_, =0, vi véi mot day gdm 2n—1 dau, sé ddu cong va sé ddu trit phi cé mét sé 1é va mét sé chin khong thé chénh nhau diing 4, tiic 1a khong thoa man tinh chit 1). 2. Xét cdc day 2ndéu cé sé dau cong nhiéu hon sé dau trit (ing véi cdc dudng di dén E A D qua doan DE, xem hinh bén), ta goi tat cdc day dé la cdc day céng 2n. B Cc IE Sé cdc day cong 2n 1a bop, hte : ‘Trude hét ta ching minh céng thiic: ban = 4b2n-2 — 2ban-4- “ Dé thay ring b, =0, by =1, bg =4. Do dé ta cé thé vidt by = 4b, —2b2, ttc la cong thiic (*) ding véi n = 3. G Gia sit (*) ding véi n=k—L: by, 7 = 4b 944 —2b 4-6, ta ching k. Cae day cong 2k bao gim cdc day sau day: (a) Cae day cé dau trix cudi cing 6 vi tri thir 2k —2 ; (b) Cac day cé dau trit cudi cing 6 vi tri thit 2k —3; (c) Cac day cé dau trit cudi cing 6 vi tri thit 2k —4; (d) Cac day cé dau trif cuéi cing é vi tri thir 2k —5; (e) Cac day cé dau trix cuéi cing 6 vi tri thir 2k -6; (f) Cac day c6 dau trit cudi cing 6 vi tri thi 2k —7. Ngoai ra khong con day nao khac vi dé thay ring véi mdi day céng thi: - Hai dau cudéi cing phi 1a hai ddu cong ; minh (*) cing sé dung vdi n = - Khéng thé ca 8 ddu cuédi cing 1a dau céng, vi néu vay thi mot trong hai tinh chat 1) hodc 2) sé khéng dude thod man. Ta tinh tung loai day trén: + Méi day cong 2k -2 déu cé dau trix cudi cing 4 trude vi tri thir 2k -3, vi vay tit mdi day cong 2k—2 sé lap duoc mét day cong 2k loai (a) bing cach thém mét c&p + - vao truéc hai d4u cong cuéi cing va lap dude mét day loai (b) bing cach thém mét cap - + vao trudc hai ddu céng cudi cng. Nhu vay cé it nhdt b2, > day loai (a) va b> day loai (b). Ta chimg minh sé day loai (a) dting bing b.,_,. That vay, dé thay ring bat ki day loai (a) nao thi ddu 6 vi tri 2k —3 1a dau cong va dau 6 vi tri 2k —2 la dau tri. Vi vay khi bé c4p + - nay di thi sé due mét day cong 2k —2. Néu sé day loai (a) nhiéu hon b»,_» thi bing cach trén ta sé tim dude nhiéu hon ba, day céng 2k-2, la diéu v6 li. Chimg minh tuong ty ta cé sé day loai (b) ding bing b>,_). + Suy ludn tuong tu nhu trén sé thay sé day loai (c) bing sé day cong 2k-—2 khéng cé dau tris é vi tri 2k —4, tiie 1a sé day loai (c) bing bax-2 —bay-a- + S6 day loai (d) bing sé day cong 2k-—2 khong cé dau tri 6 vi tri 2k—4, 2k—5, tite labing by, 7 —bo,_4 — boy -4 = b24-2 —2by4-4- + 86 day loai (e) bing s6 day cong 2k—2 kh6ng cé ddu tri 6 cdc vi tri 2k-4, 2k-5, 2k-6, bing ba_2 — 2b 24-4 — (bax4 — bay_6) = bax-2 — 3b2x-4 + brx-6- 228 + Sé day loai (f) bing sé day cong 2k—2 khéng cé ddu tri 6 cdc vi tri 2k—-4, 2k-5, 2k—6, 2k-7, bing bax-2 —3bax-4 + b24-6 — (ba4-4 — 2624-6) =bap_-2 — 424-4 + 3b 24-6. Ting 86 cd 6 loai bing: b2, = 6b24_2 —10b2,_4 + 4b, 6 = = Abo, -2 — 2b, -4 +2(b 94-2 — 4b, 4 + 2b-6)- Theo gid thiét quy nap: b2,_» = 4b2,_4—2b2,_¢, nén biéu thiic trong ngoac bing 0. Vay: b2, =4b2,-2 —2b24_4- Ding thtic (*) duge ching minh. Tir (*) suy ra dAng thufe: a = 4424-2 —2a74-4- cy Bay gid ding (**) va cing bing quy nap ta chimg minh: aan = Fpl" 0-4. qa) Dé dang kiém tra duc (1) duing véi n =1,2,3. Gia sit (1) da ding véi n m, 1a nghiém cia phugng trinh (n? -mn~m?)? =1. ©) Ta thit thay (m + n, n) vao phudng trinh: ((n +m)? —(m+n)n—n?)? =(-(n? —mn-m?))? =1. Nhu vay, néu (n, m), véi n > m, la mét nghiém cia phuong trinh (*) ndi trén thi (m + n, n) cing 1a mot nghiém cua phuong trinh (*). Ré rang (2, 1) 1a mét nghiém, nén két qua nay cho ta day nghiém sau day cua (*): (3, 2) ; (5, 3) ; (8, 5) ; (13, 8) ; (21, 13) ; (84, 21) ; (55, 34) ; (89, 55) ; (144, 89) ; (233, 144); (377, 233) ; (610, 377) ; (987, 610) ; (1597, 987) ; (2584, 1597). (**) Tiép tuc, ta di tim nhing nghiém con lai. Gia sit(n, m), vi n > m, la nghiém ctia (*). Ta thi thay (m, n - m) vao phuong trinh: (m? —m(n-m)-(n-m)?)? =(m? +mn-—n?)? =-(n? -mn-m?)? =1. Nhu vay, néu (n, m), véi n > m, 14 mét nghiém cia (*) ndi trén thi (m , n- m) cding 1a mét nghiém ca (*). Mat khac, khi m > 1, ta cé m > n - m (vi néu khéng thi n>2m, do dé n(n—m)> 2m?, suy ra n?-mn-m? >1). Do vay, néu (n, m), véi n> m > 1, 1a mét nghiém cua (*), ta sé cd mét nghiém nhé hon 1a (m, n - m), véi m > n- m. Qua trinh nay duge lap lai va két thiic 4 (n, 1), véin > 1. Nhung theo trén ta chi cé nghiém (2, 1). Vi thé, day nghiém eta (*) la day (**) néi trén. ‘Tém lai, dap sé cho bai todn 1a 1597? +9877. Chi §: Viéc tim nghiém nhuv qua trinh trén xudt phat tit kinh nghiém tyu thanh khi gidi nhiing bai todn lién quan dén day Fibonacci. Xem Day Fibonacci & phan Cac kién thie bé tro. Bai 82. (1981) a) Véi gid tri ndo cia n (n > 2) thi sé tin tai mot tap gdm n 86 nguyén duong lién tiép théa diéu kién: sé 1én nhat trong tap hop nay 1a mét uéc s6 cda béi chung nhé nhét cia n—1 86 con lai? 231 b) Véi gid tri nao cia n (n > 2) thi sé tén tai duy nhdt mot tap hgp cé tinh chat trén? a) For which n (n > 2) is there a set of n consecutive positive integers such that the largest number in the set is a divisor of the least common multiple of the remaining n—1 numbers? 6) For which n (n > 2) is there exactly one set having this property? Huéng * Gid sin = 3 théa diéu kién dé bai. Khi dé, ta xét tap hop gom 3 sé m, m-1,m~2. Vi m-1,m-2 khéng cé thia sé chung nén béi chung nhé nhét cia ching 14 (m-1)(m-2), do dé, m chia hét (m~1)(m-2) = m? -3m+2, suy ra m chia hét 2, diéu nay mau thudn. * Gia stn = 4. Xét tap hgp gim 486 m, m-1,m—2,m-3, mva (m - 1) khong cé thita sé chung, né chi cé thé c6 thita sé chung la 2 véi (m - 2) va chi cé thé cé thita sé chung 1a 3 véi (m - 3). Vi m 1a mét ude sé cia béi chung nhé nhét ciia 3 sé con lai nén suy ra m cé dang 2734. Gia sit B =0. Luc dé, 2 khéng chia hét m - 1 va 2 khéng chia hét m - 3, chimg td m chia hét m - 2, diéu nay mau thudn. Tuong ty, néu @ =0 ta cing cé mau thuan la m chia hét m - 3. Gia sit a, P>1. Lic dé, tuong tu nhu trén, ta cd 2% chia hét m - 2. Dat m—2=27x, tacd 2% x 42=293F ox42ht =38, Tu d6, a =1. Liludn tuong tu véi m - 3 tacé # =1. Suy ra m = 6. Vay khi n = 4, tn tai duy nhdt tap hop {3,4,5,6} théa diéu kién bai toan. *Gid sitn > 4. Xét tap hop gdm cac s6 m, m-1,..., m-(n-3), m-(n- 2), m-(n- 1). Ta muén ring m chia hét bi chung nhé nht cua cdc s6 con lai. D&t m = (n - 1).(n - 2). Khi dé, n - 1 chia hét m - (n - 1) van - 2 chia hét m -(n - 2), do dé (n - 1).(n - 2) chia hét béi chung nhé nhat céa 2 sé nay, vi chting nguyén té cing nhau. Tir d6, m = (n - 1).(n - 2) chia hét béi chung nhé nhit cia cdc sé con lai. Néu dat m = (n - 2)(n - 3), tung ty, ta cing c6 két qua nhu thé. Tém lai, cau tra li cho (a) 1a n> 4 va cho (b) lan = 4. Bai 83. (1981) Cho ham fx, y) théa main cae diéu kién: FO.y)=y 415 f(x +1,0)= f(x); £04 Ly +1) =£(x,£(x +1, y)) vdi moi sé nguyén khong 4m x, y. Tim f(4,1981) . The function f(x,y) satisfies: FO aytls f(x+1,0)= fe); ftlytl= fx, f(e+1y)) for all non-negative integers x, y. Find f(4,1981). Huéng dan: Ta cé: f(I,n) = f(0,f(1,n—1)) =14£(1,n-1). Do dé f(,n)=n+f(1,0)=n+f(0,1)=n+2. Ta lai cé £(2,n) = f(1,f(2,n-1)) = f(2,n-1)+2. Do do f(2,n) = 2n+f(2,0) = 2n+f (1,1) = 2n+3. Bay gid: £(3,n) = £(2,f(3,n-1)) = 2f3,n-1)+3. Dat u, =f(3,n)+3, hic do, u, =2u,_, va up =£(3,0)+3 =f(2,1)+3 =8, do vay: u, =2° va f(3,n)=2"3-3. Ta c6: f(4,n) = £(3,f(4,n—1)) = 204-793 _3, £(4,0) = @G,1)=24 -3=13, £(4,1)=2%4 -3, £(4,2)=27%4 -3. Cuéi cing, bing quy nap ta ching minh duge: £(4,n) =2? -3, trong dé sé mii chifa (n + 2) chit sé 2. Tir dé: 2 £(4,1981)=2? —_-3, véi sé mi chita 1983 chit sé 2. Bai 84. (1982) Cho ham f(n) xéc dinh trén tap hop cdc sé nguyén dugng va nhan gid tri trén t4p hop cdc 56 nguyén khong Am, véi f(2) = 0, (3) > 0, (9999) = 3333 va véi moi m, n: f(m+n)—f(m)-f(n) =0 hay 1. Hay xac dinh f(1982). The function f(n) is defined on the positive integers and takes non- negative integer values, f(2) = 0, f(3) > 0, (9999) - 3333 and for all m, n: 233 S(m+n)— f(m)- f(n)=0 or 1. Determine f(1982). Huéng dan: Ta sé chimg minh ring khi n <9999, ta cé: f(n) = [2 | éday,[.] Ja ki hiéu cua phan nguyén. Truéc hét, ta c6 (3) = 1. That vay, f(1) phai bing 0, vi néu khéng thi f(2)—f(1)—f() 1a sé am. Suy ra foy=Ho09[? =). Nhung {{3) > 0 theo gia thiét, vi vay (3) = 1. Bang quy nap, ta chimg minh duge f(3n)>n. Vi f3n+3)= rotons]? =tcay) hen nia, néu ta cé f(3n) > n thi li luan tugng ty ta dug {(3m) > m véi moi m > n, m&t khac (3.3333) = 3333, do vay f(3n) = n véi moi n< 3333. Bay gid, tit ching minh trén ta cd 0 [n fn +1) = f(3n)+ roo-|} = [Fe : Nhung 3n + 1 = f(9n + 3) > f(6n + 2) + f( 3n + 1) > 3f(3n + 1), do dd f(3n + 1) 1 sao cho 1= x9 2x, 2.2 A 2 xO, XT Xn-1 St Bt 23,999, x X2 Xn b) Tim mét day cé tinh chat (*) nhu trén sao cho i 7 eon xP +4 hcg xp X2 Xn véi moi s6 tu nhién n. 234 Consider infinite sequences {x,,} of positive reals such that we have 1= Xq 2X, >...2%j 2Xj4) 2--- a) Prove that for every such sequence there isan n21 such that: 2 2 x XO, HL, 4 Ft > 3.999, XX Xq 6) Find such a sequence for which: 27 2 x x, 70 tl 24 forall n. A 2 *n Huéng dan: XG Xt Xn a) Xét day vé han ~9 ,. Bai ton sé duge gidi xong x1 _ Xn néu ta ching t6 dude ring téng cia day v6 han trén luén Ién hon hodc bing 4. Goi k 1a cin trén bé nhét ctia cac gidi han cia nhiing day nhu da néi trén. Ro rang k>1. Véi moi ¢ >0 cho true, ta cé thé tim duge mét y {x,,} c6 téng bé hon k + &. Nhung ta cd: = 2 2 xy x2 Xn Oty Bn =a, (") & me ae Xx, X2 x Xy x2 x3 Xadl x) xy x Téng trong ngodc 6 ding thuic trén 14 ting cia mét day khéc nhung cing cé tinh cht cua day dang xét, do dé téng nay >k . Suy ra k+e> a +x,k, x1 digu nay diing véi moi £ > 0, do dé: k>-! +.xyk. Mat Khée, x} ae 22k, x1 do vay k>4. Ta duge diéu phai chimg minh. b) Xét day balvs véi x, = Lic dé, véi moi n ta c6: 2 2 Xan Xt 1 XO ML yg Xl age itt Xp XQ a 2 Bai 86. (1982) Chiing minh ring néu n 1a mét sé nguyén duong sao cho phugng trinh x? —3xy? + y> =Nn c6 mét nghiém nguyén (x, y), thi phudng trinh nay sé tén tai it nhdt 3 nghiém nguyén. Chimg minh ring phuong trinh trén khéng cé nghiém nguyén khi n = 2891. Prove that if n is a positive integer such that the equation x3? +y =n has a solution in integers x, y, then it has at least three such solutions. Show that the equation has no solutions in integers for n = 2891. Huéng dan: ‘Ta viét lai vé tréi cia phuong trinh da cho nhu sau: x3 —3xy? +y? = (y—x)? -3x2y + 2x3 =(y—x)? -3(y—x)x? +(-x)?. Tix dé, néu (x, y) 1a mét nghiém thi (y - x, -x) cing 14 mot nghiém, va bing nhiing bién déi dai sé tuong tu nhu trén, ta ciing c6 (-y, x - y) la mét nghiém cua phuong trinh. Hon nia, 3 nghiém néi trén phan biét nhau, béi néu cé hai trong 3 nghiém dy bing nhau, ta dé dang suy ra x = y = 0, mau thudn véi diéu kién n> 0. Dé ching minh ring phuong trinh da cho khéng cé nghiém nguyén khi n = 2891, ta gid str ngugc lai ring tén tai nghiém trong truéng hop nay, khi dé, ta viét nghiém x va y duéi dang mod 3, va phuong trinh da cho tré thanh x? +y? =-1 (mod 3), suy ra x+y =-1 (mod 3). Do vay ta cé cac truéng hop: (i) x =0 (mod 3) va y =-1 (mod 3); (ii) x =1 (mod 3) va y =1 (mod 3); (iii) x =—-1 (mod 3) va y =0 (mod 3). 6 truéng hgp (i), dt x = 3m, y = 3n - 1 va thay vao phuong trinh da cho ta duge vé trai =-1 (mod 9), trong khi d6, 2981 = 2 (mod 9), mau thudn! Trudng hop (ii) khong xay ra dude, vi theo trén, néu (x, y) 1a mot nghiém thi (y - x, -x) cing 14 mét nghiém, va do do truéng hgp nay dan dén trudng hgp (i). Tuong ty, (iii) ciing dua vé trudng hap (ii). Tém lai, phuong trinh da cho khong thé 6 nghiém nguyén khi n= 2981. Bai 87. (1982) Xét hinh vuéng S cé canh dai 100 (don vi). Goi L 1a mét dung gap khtic nim trong S khong ty ct chinh né, L bao gém cac doan thing AoA, AjA2 5) An-1Aq, Voi Ag khong tring véi A,. Gid sit véi moi diém P nim trén bién (ttic 1a cc canh) cia S déu c6 mét diém trén L sao cho khoang cach tu diém nay dén P khéng lén hon 1/2. Ching minh ring tin tai 2 diém X va Y trén L sao cho khoang cach gitia X va Y khéng Ién hon 1, va dé dai phan dudng gap khic cia L nim gia X va Y khong bé hon 198. Let S be a square with sides length 100. Let L be a path within S which does not meet itself and which is composed of line segments AyA, AAg 5-5 AnAns With Ag does not coincide A,,. Suppose that for every point P on the boundary of S there is a point of L at a distance from P no greater than 1/2. Prove that there are two points X and Y of L such that the distance between X and Y is not greater than 1 and the length of the part of L which lies between X and Y is not smaller than 198. Huéng dan: Nhu lé thudng, ta ki higu d 1a khodng cach Euclide giita hai diém. Khi di doc theo dudng L, ta chon mét diém P, cé khodng cach dén dinh V, cia hinh yuéng khéng qué 1/2. Ta tiép tuc di xa hon cho dén khi tiép cAn vdi mét trong hai dinh ké cia V,, goi dinh nay 1a V,, tuong img ta cing cé P;, va rdi ta cling cé V;, V,, tang ting cé cdc diém Py, Py. Ki hiéu [ L < P,] la phan dudng L tinh tu P, vé truéc va [ L > P, } 1a phan cdn lai. Xét canh V,V,. Dat A= {xevive :4¢atL-

5 ta cé ngay diéu phai ching minh. Vay chi can xét truéng hgp khi \rap|s4. Khi d6 [I 4(S\P)|> 217-4 =213. Diéu nay ching td ring ti tap hop tdt ca cdc 6 khong nguyén 6 trong S (tat cd gém |S\P|=220 sé) cd nhiéu nhat 1a 7 s6 khong nim trong T. Dat : M, = {2.23,3.19,5.17,7.13, 1.1}, Mz ={2.29,3.23,5.19,7.17,11.13}, Mz ={2.31,3.29, 5.23, 7.19, 11.17} ,Mg = {2.37,3.31,5.29, 7.23,11.19}, Mg = {2.41,3.37,5.31, 7.29, 11.23}, Mg = {2.43,3.41,5.37,7.31,13.17}, My = {2.47,3.43,5.41,7.37,13.19}, Mg ={2?, 37, 52, 7°, 13°}. Ré rang M, cSP(i=1,2,....8) va moi phan ti cba Mj doi mot nguyén té cing nhau. Theo Nguyén Ii Dirichlet, tn tai it nhdt 1 chi sé ig (Sig $8) sao cho M,, CT. Ta duge diéu phai ching minh. Bai 127. (1991) Cho G 1a mét dé thi lién thong gsm k canh. Ching minh ring cé thé dénh sé cdc canh bing tat cd cdc sé 1, 2, 3, ..., k sao cho tai méi dinh thuéc vé it nhat hai canh cia dé thi, ta déu 6 ude sé chung lén nhét cua cdc sé nguyén viét trén céc canh cia dinh nay bing 1. [Ta goi dé thi (graph)G 1A mét tap hgp cdc diém, duse goi 1a dinh, cing véi tap hap cac canh néi mét sé cip dinh phan biét véi nhau. Méi c&p dinh thuéc khong qué mét canh. Dé thi G due goi la lién théng (connected graph) néu véi méi cap dinh phan biét x, y déu tén tai mét day cdc dinh x =V,Vj....Vm =Y sao cho mi cAp V;,Vj,, (0Si2. Khi dé tén tai it nhdt mét chu trinh di qua it nhdt hai canh. Ta danh sé cdc canh cia chu trinh dé theo thi ty 1,2,...,m (m 14 sé canh cia chu trinh). Bé cdc canh cua chu trinh nay ra khdi dé thi G, ta duce dé thi G',. Bé tét c& cdc dinh khong lién thuéc G',, ta dude 283 46 thi G,. Néu G, van cé chu trinh chia it nht hai canh thi ta dénh sé céc canh cia mét trong s6 cdc chu trinh nhu vay béi : m+1,m+2,...,m+m, theo thit ty cia dudng di (m, 14 s6 canh cia chu trinh trong G,). Lai loai tt ca cdc canh cia chu trinh nay ra khéi G, ta duge dé thi G2. Tiép tue x4y dung cac chu trinh trong G2 theo céch trén. Tiép tuc nhu vay, sau hitu han budc (do G cé hitu han canh) ta thu duge mét tap hop réng (dé thi khéng dinh, khong canh) hodc thu duge mét dé thi gim cdc chu trinh 1 canh. Trudng hdp dau da dude dinh 86 trong. Déi véi trudng hop thit hai, thi chi viée dién cdc sé con lai mét cdch tuy ¥. Bay gid ta chimg minh ring cach dénh sé trén thod man diéu kién bai todn. Xét mét dinh B tuy y cia dé thi G ma B cé it nhdt hai canh. Theo cach xéy dung thi B phai thuéc it nhdt 1 chu trinh nao dé va khong thé la diém ddu hay diém cuéi cia chu trinh d6. Theo cach dénh s6 é trén thi tai B cé hai canh duge dién hai sé tu nhién lién tiép nén suy ra uéc chung lén nhdt cia tap hgp cdc sé viét trén canh cia dinh B bing 1, ta cé diéu phi ching minh. Bai 128, (1991) Cho sé thyc a>1 tuy ¥. Hay dung day vé han va bi chin x9,x,, X2,.. 8a0 cho ky —Xq||n-m]? >1 vdi moi c&p sé nguyén khong am phan biét m, n. [ Day v6 han cdc sé the X9,x},X2,... dude goi la bi chén (bounded infinitive sequence) néu tn tai mét hing sé C sao cho fx,|SC véi moi n20.] Given any real number a > 1, construct a bounded infinite sequence Xp,X1,X2,...such that |x,—Xp||2—m|">1 for every pair of distinct m, n. [An infinite sequence x9,x,,X2,... of real numbers is bounded if there is a constant C such that |x,|0.] Huéng dan: Céch 1: Dat t = ¥ vi dinh nghia: e=1-—*-. Via> 1 nén 30. Bay gid, cho sé nguyén n > 1 bat ki, ta viét n trong hé nhj phan: n= )°b,2' va dat x, = Dt) ching han, n= 21 = 24+27+2° thi 7 “B50 Xo = E(t 12 419). c Ta sé ching minh ring véi moi m, n khdc nhau ta co ky —Xm{|[n—m[* 21. Khi d6, bai toan dug gidi, vi ré ring diy x, duong vA bi chin : 1 1 béi hing s6 — )'t" =——. 2 oy 1-2 Goi k 1 sé ma cao nhdt ca 2 sao cho 2* chia hét cA m lin n. Lic 46, [n—m|>2*, va trong biéu dién qua hé nhi phan cia m va a, cdc hé sé cia 2°,2',...,2*-! giéng nhau, con hé sé ctia 2" thi khdc nhau. Suy ra dX, — * 4 y;, trong dé y;=0, t' hay -t!. Ta cd: Dae i>k i>k do dé x, ~xalott(i-j5)-0t . Tit dé ta 06 diéu phdi ching minh: bp —%q||m—m]* > 2% =1. Céch 2: V6i aR, ta ki hiéu [a] va {a} lin luot 1a phdn nguyén va phan thap phan cia a. Véi c4p sé nguyén khong 4m phan biét m,n ta 06: {my2}— (nV2} = (mv? ~[mV2]) -(nv2 -[nV2)) = V2¢m—n)-h , trong 46 h =[mV2]-[nV2]. Vi V2 1456 v6 ti, nén 2(m—n)? —h? #0. 2(m—n)? —h? Do vay: {m2} {nv2}| =|¥2¢m-n)-h| a loa=ck ot \¥2¢m—n) +b] ¥2|m —n| +b Mat khde: / [h|=l[mv2]—[nv2| <}mv2 -nv2]+2 = V2|m—n|+2<(2+¥2)}m—n]. b: = fC Do as env2}— fas} > Pima) +2@ + V2)}m —n| (242¥2)}m— |” 285 Suy ra [semv2}—8¢nV2}|m—n| 21. Do vay, ta chon day x, =8{nv2} thi diéu kién 6 dé bai duge théa man vi véi moi n ta 6 |x,|<8 va kn —Xmal [n—mf* > [xq —Xyq||[n—m|>1 véi moi cp sé nguyén khéng m phan biét m, n. Bai 129, (1992) ‘Tim tat cd cdc sé nguyén a, b, c thoa didu kién a3,c24). Hon nifa, d> 1 va néu a>4 thi to t—= 2] 3°45 12 20 15 60 Ti dé, d= 2 hodc d = 3 via = 2 hodc a =3. Ta cé 4 trudng hop dé kiém tra: astetyt ytd ay Pe Truéng hop 1: a = 2 va d = 2. Ta cé: 2be-1 22 > 2be-1= 2(b-1e-1). (b-1e-) Ce! Vé trai 5 phuong trinh trén 1é cdn vé phai chin nén phugng trinh vé nghiém. © Trung hop 2: a= 2 vad =3.Ta cé: 2be — — = 3 9 (b-3Xe-3) =5. -1e-) <= (b-3)(e-3) acl Vib0 tay y, chon z sao cho x =z”, khi dé: F(x +y)=£(2? +y)= f(y) +f7(2) = £00 + £09). Dat y=-x ta nhan duce 0=f(0)=f(x+(-x))=f(x)+f(-x). Suy ra f(—x) =-f(x), didu ndy kéo theo, voi moi x, y: f(x-y) = f(x)-f(y). Bay gid, léy x bat ki, dat y = f(x). Néuy>x, dat z=y—x thi f(@@)=f(y—x)=f(y)- f(x) =x-y =-z. 287 Néuy 0 thi f(z)=-z<0. Bay gid ta chon w sao cho w” =z thi f(z)=f(w?) =f2(w) <0, diéu nay mau thudn. Vay ta phi cé f(x)=x. Bai 131. (1992) Cho S 1a m6t tap hitu han cac diém trong khéng gian 3 chiéu. Goi S, , S, S, lin luot 1a tap gém cac hinh chiéu cdc diém cua S én céc mat phang yOz, zOx, xOy tuong ting. Ching minh ring: ISPS, |-18y 1-18, 1, trong dé, | A| 1a ki hiéu sé tat ca cdc phan tit cua tap hitu han A. Let S be a finite set of points in three-dimensional space. Let S,, S, S, be the sets consisting of the orthogonal projections of the points of S onto the yOz-plane, zOx-plane, xOy-plane respectively. Prove that: ISPS, 1-18, 1.15.1 where |A| denotes the number of points in the set A. Huéng dan: Ta ching minh két qua bing quy nap theo sé cdc diém cua tap hop S. Ré rang két qua dung véi |S|= 1. Gia sit két qua dung khi |S|=n. Ta xét S véi |S|=n+1. Xét mét mat phing song song véi mét trong cac m&t phing toa dé, mat phing nay khong chia bat cit diém nao trong cde diém' cia S, né chia S thanh hai mién khac ring S' va S*. Trong hinh bén, mat phdng nay song song véi_ mat phing xOy. Khi dé: |S |S! |+]$?], 0<|S! |4. b) Tim sé nguyén n sao cho S(n) = n?-14. c) Chimg minh ring tén tai v6 han sé nguyén n sao cho S(n) =n? -14. For each positive integer n, S(n) is defined as the greutest integer such that for every positive integer k < S(n), n’ can be written as the sum 290 or tolanmc et of k positive squares. a) Prove that S(n) 4, ta cé thé lam gidm di k sé hang binh phugng. (Kiém tra: 14=34 34+8,15=3+ 34+3+4 34+3,16=8+8, va cdc gid tri k cao hon dudc thuc hién bing cach thém vao cc béi cia 3, 14, 15 hay 16). Do dé S(n)a7=14, n? =( -15)x? +a? +b? a2 +b? =15, n? =(n?-16)x1? +a? +b? +c? => a +b? +c? =16, n? =(n? -17)x1? +a? +b? +07 +d? => a2 +b? 407 +d? =17, n? =(n?-18)x1? 4a? +b? +07 +d? +e? => a2 +b? +07 +d? +e? =18. Bén triténg hop dau, khong thé cé nghiém nguyén. Véi r>5, ro rang téng cia r sé hang binh phudng Ién hon 13 + r. b) Khéng cé tam gide vuéng nao véi cde canh nguyén ma canh huyén bing 1, 2, 3, 4, 6, 7, 8, 9, 11 hay 12. Nhu thé, ta c6 S(n) = 1 véi cdc gid tri n vita ké. Dé dang kiém tra duoc 25 va 100 cé thé viét thanh ting cia 2.86 binh phuong, nhung khéng thé 1a 3, tix dé: S(5) = S(10) = 2. Ta sé chimg minh n = 13 1a nghiém cua phuong trinh S(n) =n? -14. Ro rang 169 cé thé viét duge thanh ting cua cic binh phuong bAng nhiéu cach, ching han: 169 = 13? = 5? + 12? = 37 + 4? + 12? = 1742? +84 10? = 374 474 424 874.8%, Ta nhan théy (21)? =r? +r? +1? +r? véi moi r 1a sé nguyén, do 46, bang phan tich trén cé thé due tiép tuc: 169 = 37+ 47+ 4748748? (556 hang), 169 = 3° + 47 4 47 4 (47 + 4? + 47 + 4”) + 87(8 56 hang), 169 = 37+ (17+ 1? +...+1?) (161 sé hang). Theo cach trén, 169 cé thé biéu dién duge thanh téng cia 2 + 3t cdc binh phuong (1<1 <53). Tit phan tich 169 = 1? + 2? + 2? + 4? + 4? + 8 + 8? (7 sé hang), tuang ty, ta c6 thé thu duge cdc biéu dién 169 thanh mot téng cdc binh phuong cia 10, 13, 16, ..., 169 sé hang, titc 1a tong cia 1 + 3t cdc s6 hang (20 hay kn?—4 khin > 4, tif trén ta suy ra (2n*) c6 thé duge biéu dién thanh téng cia k sé hang binh phuong, véi 1 C > | 1 | phai 1a béi cua cua 3. Truéc tién, xét trutng Hinh 1. hop n = 2 (xem hinh 1). Phuong phép di chuyén dé gidi bai todn dat trén co sé cdc "b6 ba bude nhay", qua dé, ta di chuyén vé thanh mét hinh chit nhat dat doc theo mét canh cia dé hinh nhu hinh 2. Xét trudng hgp n = 4. Déi véi trudng hgp nay, mot day cdc bd ba buéc nhdy sé dua vé trutng hgp 2x2 (n = 2) (xem hinh 3) 293 Hinh 2. Hinh 3. : Véi trudng hgp 5x5, ta cing dua vé 2x2 bing mot 2 day céc bé ba bude nhay. Cac 6 3x1 dude dai chd 7 nhu hinh 4. 314 6 Khi n2>6, mét hinh vuéng nxn luén cé thé dude rit 5 gon vé hinh vuéng (n—3)x(n—3) bing cach dii Hinh 4. ché cdc hinh chi L véi 2n—3 bé ba bude nhay, ta minh hoa nhu hinh 5, voi n = 7. ile Diéu nay ching té ring néu n = 4, 7, [ 10, ... hay 2, 5, 8, 11, .... thi hinh vuéng nxn ; c6 thé gidi duge. 4 ___,Diéu gi xdy ra kbi n la béi cia 3? Cé i thé kiém tra ring truéng hop 3x3 khong gidi 7 10 du¢c, nhung nhu thé chua du, ta phai chimg 6! Is 9 minh trong trudng hop téng quat, bai todn Hinh 5. khong cé léi gidi khi n 1a béi cua 3. Ta té cdc hinh vuéng bing 3 mau dé (D), D{XIV{DIXLV{D | xanh (X) va vang (V) nhw hink 6. X|V|D|X|V|D/X | Néu n Ja boi eda 3 thi mét hinh vudng gdm V{D|XIVIDIXIV | nxn 6 té mau nhu trén sé c6 sé 6 méi mau D{X|v[D[ xl V|D | yang aha. Hinh 6. Gia stn = 3m. Ta xudt phat ti 6 dd. Goi A at cd cdc nude di dén 6 da, B 1a sé tat cd cdc nuéc di dén 6 xanh va C la s6 tt cd cdc nude di dén 6 vang. Mot nuéc di dén 6 dé sé lam ting 86 cdc 6 dé lén 1, lam gidm sé cdc 6 xanh di 1 va ciing lam gidm sé cdc 6 294 vang di 1. Ban dau ta cé m 6 dé, m 6 xanh, m 6 vang. Nhu thé ta dugc: -A+B+C=m-1; A-B+C=m;A+B-C=m, SuyraA=m,B=m —}=C. Nhung efe 56 A, B, © phi nguyén, mau thuan nay cho ta diéu phai ching minh. Bai 135. (1993) 2 Cé tén tai hay khong mét ham f tit tap cdc sé nguyén dusng vao chinh né sao cho véi moi n, ta c6: £()=2 £(f(n)) =f(n) +n ? f(n) r—2, béi néu thé ta sé c6 n2u, +u,)=U,,)- Tit do, thém u, vao khai trién cia n—u, sé cho ta mét khai trién theo dang trén céa n, chting minh hoan tat theo quy nap. Tiép dén, ta cing ding quy nap dé chimg minh ring U,+U,_2 +U,_4+..=Upyy—l. Hién nhién diéu nay ding véi r = 1 va 2. Gid sit né dung véi r—1, khi dé tacéd: Upp +Uy yt... =Upyg “Up FU _y + Uy 3 +... =Uyy2 —U, +U, —1=u, 42-1. Vay cong thitc dting véi r + 1, suy ra né dting véi moi r. Tiép dén, ta chimg minh ring biéu dién cia n 1a duy nhdt. Hién nhién digu nay dung voi n = 1. Gia st n6 ding véi moi s6 nhé hon n, nhung biéu dién cho n lai khéng duy nhat. Nhu thé ta sé cb n = u,+...= u, +.... Néur=s thi biéu dién cho n—u, khong duy nhat, trai gid thiét quy nap. Gid sitr > s. Nhung sé hang u,,; —1 trong biéu dién thit hai lic 6 lai bé hon u,. Vay biéu dién cho n phai duy nh&t, ménh dé duge ching minh theo quy nap. Bay gid, ta gid st n=b,u, +b,_U,_1 +... + bot, dat f(n) =b,b,_1..-boug0. Hién nhién khi n = 1 = up thi ff) = u, =2. Néu n=u,, +..+U,, thi f(n)=ug,41+--+Ua 4 Va f(F(n)) = Ug, 42 +--+ Ug 425 do dé f(n)+n= (Ug, + 4a, 41) +--+ Ua, +a, 1) = f(f(n)). Vay tén tai ham fnhu trén thod man dé bai. Bai 136. (1993) C6 n ngon dén Lo, Ly, .... Lia (n> 1) duge dat trén mét dudng tron. Ta cing ding L,,, va hiéu dé 1a Ly. 6 moi thai diém, méi ngon dén cé thé séng hodc bi tit. Ban ddu tat cd ching déu dude bat sang. Ta biéu thi cdc buéc so,s), ... nhzf sau: tai bude s,, néu L;_, da tit thi ta tat L; (néu L;dang sang) hoe bat L; séng lén (néu L; dang tit), cbn néu Lj_) dang sang thi ta khong lam gi ca. Chiing minh ring: a) Tén tai sé nguyén duong M(n) sao cho sav M(n) bude thi tat cd cdc ngon dén cing sang lén tré lai. b) Néu nn = 2* thi ta c6 thé chon M(n) = n?-1. o) Néun = 2**", ta cé thé chon M(n) = n? -n +1. There are n > 1 lamps Lz. Ly ..- Ly, in a circle, We use Lys, 10 mean L. A lamp is at all times either on or off. Initially they are all on. Perform steps 59,8}, ... as follows: at step sj, if L,. is lit, then switch L, from on to off or vice versa, otherwise do nothing. Show that: 206 a) There is a positive integer M(n) such that after M(n) steps all the lamps are on again; b) Ifn= 2*, then we can take M(n) = n? -1. o) Ifn= 2", then we can take M(n) = n? ~n+1. Huéng dan: a) Ta biéu thi tinh trang sing téi cia cdc ngon dén Lo,..., Ly béi vector V=(Vo,¥j,--;Vn-1)> Voi V; = 0 néu L, tét va v; = 1 néu L, 45. Theo dé bai, tinh trang ban dau cla cdc ngon dén 1a vector” e=(LLL...,1). Buée s; la buée chuyén trang thdi ti vector (Vo.Vio-Vn-1) Sang vector (Vo,Vj5.-5Vj1sVj-itVjo-sVnt)> Voi phép cong 6 day trong co sé 2. Van dé dit ra la xdc dinh gid tri r sao cho 5,5,_1...5)89 =¢. Ta ki hiéu R 1 todn tit chuyén vector (vg ,Vj ,---»Vq_1) Sang vec- tor (Vj ,V25-~sVq_1>¥0)» hi dé: 5; =RsgRJ. Ta 6: 1Sp = (R“spR'(RYsgR™)...(R25gR2(R'5gR_ So = R“'so(Rso)" = Ro 'sp(Rsp)"*!. Do dé 5,5,_1..5159 =€ <> (Rsp)"1e=R"e=e. DE ¥ ring chi cé mét sé hitu han cdc trang théi khdc nhau cha nhiing ngon dén (chinh xdc 14 2"), do vay, phai cd su lip lai tai mét giai doan nao dé cia day e, Rsye,(Rsp)"e,... . Vi thé, véi m, n nado dé ma m < n ta c6 (Rsp)"e=(Rso)"e. Do Rsp 14 song anh, ta 6 (Rsg)" “e=e, nhu thé a) dude chimg minh. Vp 0-5 Vn—1) = (V1 > V2 0-2Vn-1> Vat + Vo) va da thiic tudng tig véi Rspv la Q(X) = Vp-p FVp-aXt Vg 3X2 Het VX 2 + (V yy EQ) (48 ¥ phép cong trong hé cs s6 2). Ta cing 6: Q(x) = xP(x) (mod x" — x! Ty. Vi thé, vAn dé tim r dé (Rsp)'e =e tuong duong vii viée tim r 46: x" =1(mod x"-x"™!-1). Gid st n=2". Khi dd x™ =(x")" = (x14) =x 1 vin 1a lof thita cla 2 va tat cd cdc hé s6, ngoai tri hé sé dau tién va cudi cing cta khai trién theo co s6 2, déu chin, tic 1 tring véi 0 (mod 2). Tit 6 ta duge: x™ —x™—* 2} x™ M(x!) 21> x" “| SH, do vay sau n? ~1 bude tat cd cdc ngon dén sé déng loat séng lén, cu b) duge ching minh. Gid sit n=2* +1, khi dé: x@el =(xmtlye a(x" 4x) = xm yt vi n—1 1a luy tha cta 2 va tat ca cdc hé 36, ngoai tri bé sé dau tién va cuéi cing cia khai trién theo co sé 2, déu chin, tic 14 tring véi 0 (mod 2). Tu dé ta duge: xMt_ ymin-1) x"! suy ra xm mnt a on, x" ax xX "lV ytex"!_1),dodé x"! 1, Vay sau n? —n+1 bude tat cd cic ngon dén sé ding loat sang én, cAu c) dude ching minh. Chi: §: (*) G day, ta hiéu mét vector 14 mét bé c6 thit tun sé. Ban doc théy ré ring, thuc chat, mét vector trong khéng gian 2, 3 chiéu (mit phdng, khong gian) tuong tng 14 mét bé cé thif tu gdm 2, 3 sé. MS réng ra, sau nay cac ban sé thdy ring trong mét khong gian (tuyén tinh) n chiéu, ngudi ta con goi méi phan ti cla khong gian dé (tite mot b6 n 6) la mét vector. Bai 137, (1995) Gia sit a, b vac la cdc sé thc duong théa man didu kién abe = 1. Chiing minh ring: =~ +4123. a(b+c) b’(cta) c(atb) 2 Let a, b, c be positive real numbers with abe = 1. (x" Prove that: —-1—+-,1 4,133. @(bt+c) W(cta) c(a+b) 2 Huéng dan: pat L=x,t=y,)=z, thi x>0,y>0,2>0,xy2=1. c ab Theo bat ddng thiic gitta trung binh cong va trung binh nhan thi xeytz23 va-% 47 42 93) ytz zt+x x+y 2 Khi dé bat ding thi cdn ching minh cé dang 7 2 2 a Ya a y+Z Z+x xty 2 Khong mat tinh téng quat cé thé coi x > y > z > 0. Khi dé: Se ytz Z+x x+y Do vay, theo bat ding thiic Trébusép thi a: - - Soe (2) se yt+z z+x y+z 3 yt+z z+x x+y) 32 Dau dang thiic xay ra khi va chi khix = y=z=1haya=b=c=1. Bai 138. (1995) Tim gia tri lén nhdt cia x, dé tén tai day cdc 6 thuc duong xp, x, --+» Xjg95 théa main hai diéu kid i) Xp = Xio95 5 ii) x; +2 =2x, + vai moi i=1, 2, ... 1995. Xit Xj >0. Find the maximum value of xo for which there exists a sequence Xe Xp +++» Xj995 Of positive reals with xo = xXj995 such that for i= 1, .., 1995: x,4+—2-=2x, +. il %i Huéng dan: Xét day hitu han {x;}, i=0,1,2,...,1995 thda man ca hai diéu ign (i) vA Gi). Tir gid thiét x;_)+2-=2x, +1, vi=1,2,...1995 suy ra Xi-t Xj i ring vi moi i=1,2,...,1995 ta cd: : Xi-t Bang quy nap, ta sé ching minh céng thtic n; € {-i, -i+1,...,-1, 0,1,..., iI}, va m; € {-L}}, Vi=1,2,..., 1995. 299 That vay, gid stt cong thiic trén dung véi i, ta c6: 1 [i = 2M = 2M xmitt 1 Xian =22xq7™ = 2A A! Xie oe nj €{-i,-i+l,..,i-1} Ngoai ra, ti gia thiet quy nap tasuyra m, € {-1, 1} {rere rend mj, €{-1L}} , Vay khang dinh dung véi i + 1. Céng thitc trén ding véi moi i theo nguyén li quy nap. Dat j=max{ili=l,..., 1995; x;=2" Xo}. Dé ¥ ring chi sé j luén luén tén tai. Ta xét hai kha nang: 1) j= 1995. .Khi dé xj995 = 2" xo! =x,. Vay x9 = 271995 <2!9 hay x, < 297. Dau dang thiic xdy ra khi x; 2) j < 1995. Khi d6 myo, = 1 va sé lan u cua day m,, my, ..., Moos 14 mét 86 chin (tic 14 tén tai mét sé chiin cdc c&p sé (x,.,, x), i= 1, «++» 1995 sao cho x; = x; 1994 va xjo95 = Xj404- Néu x; =x; thin, , vin, cing tinh chin I8, cbn néu x; =5KH thi n,, va n, sé khéc vé tinh chin 1é, diéu dé mau thudn véi gid thiét Xy995 = 275 Xo (Myg95 = Ny = 0). Vay kha nang (2) khong xdy ra. Tom lai, gid tri lén nhdt cha x, bing 2””, Bai 139. (1995) Cho p 1a mét sé nguyén té lé. Tim sé cdc tap con A cia tap hop (1, 2, ..., 2p), biét ring: i) A chifa dung p phan ti; ii) tng tat cd cde phan ti ctta A chia hét cho p. Let p be an odd prime number. How many p-element subsets A of {1, 2, ..., 2p} are there, the sum of whose elements is divisible by p 2 Huéng dan: «+; P, ta goi A; 1a tap hop bao gim cac bé sé ‘Véi moi i = 1, (@j.-0a) (mma ta khong dé ¥ thif tu), véi a, € {1,..., 2p}, ay +...+ap chia hét cho p va trong bé (aj,...,a)) chi c6 dting mét s6 xudt hién i lan, cdc sé con lai xudt hién ding mét lan. Khi dé A, chinh la tap con A cia {1, ..., 2p} théa man: i) A chia p phan tis, ii) téng tt ca cdc phan tit cha A chia hét cho p. Ki A; 1a tap nhan ti A; bing cach trong méi bé (a),...,2p) ta loai i s6 giéng nhau, i= 1, ..., p. A, 14 tap nhan tir A, bing cdch trong méi bé (a),...,a,) ta loai i—1 sé giéng nhau, i = 2, ..., p. Ta cing ki hiéu |A|| 1 86 cdc phan tit cia tap A;. Khi d6 \Ail+Aisal=2C5,', 1=2,...p—1 @ plAy|+|A2|=2C5,, That vay, ta xét cdc bé sé (khéng thit tu) (b),...,bp_;), b; #b;, véi ixj. Dat S=b, +..+b,;. Khi dé tin tai duy nhdt r théa man 0 (8,3) > (16,6) > (11,1) -> (19,4) > 7) > (19,10) > (16,2) —> (8,5) — (16,8) > (19,0). c) Trudng hgp r = 97, dé nghi ban doc tiép tuc chimg minh ring truéng hgp nay bai toan khéng giai dude. Bai 141. (1996) Goi S la tap cdc sé nguyén khéng am. Tim tat cd cdc ham di ti S vo chinh né sao cho véi moi m, n ta cé: f(m+f(n)) = f(F(m))+f(n). Let S be the set of non-negative integers. Find all functions f from S to itself such that f(m-+ f(n))= f(f(m))+ f(n) for all m, n. Huéng dan: Cho m = n = 0, dang thiic 6 dé bai tré thanh: £(0+ £(0)) = £(F(0)) + £0), suy ra £(0)=0, ti dé f(f(0))=0. Cho m = 0, ta cé: f(f(n)) = f(n), do dd dang thie 6 dé bai cé thé viét thanh: f(m+f(n)) =f(m)+f(n). Theo trén, f(n) 1a diém bat déng (ta néi x 1 diém bat dong cia ham f néu f{x) = x). Goi k 14 diém bat dong khdc 0 bé nht cia ham f. Néu nhu k khéng tén tai, thi f{n) = 0 vi moi n, khi dy, ham f(n) = 014 mét nghiém cua bai todn. Néu tén tai k, thi bing quy nap, ban doc dé dang chimg minh duge f(qk)=qk véi q 14 s6 nguyén khong am tuy y. Bay gid, gid str f cé diém bat dong n khac, ta viét: n = kq +r, voi Or=0. Nhu thé, diém bat dong cia him sé 1 béi cia k. Tuy nhién, véi moi n, f{n) 14 diém bat dong, do dé f(n) 1a béi cha k véi moi n. Ta ldy cde sé nguyén khéng 4m nj,n3,...,ny_, va chon ng =0, thé thi ham téng quat théa man diéu kién cia bai todn 1A f(qk+1)=qk +n,k, véi O---» X1)- Néu |S(zo)| <1 hode |S(#)|r va |S(Z)|>r. DEY: S(19) +S) = (xy +2xq +..+ 1X _) + (Kq +2Xy_] +... +X}) = (n41)(x,+xX24+...4X,), ta suy ra |S(z9)+S(z)| =n + 1 = 2r. Nhung vi [S(zq)|>r va [S(t)|>r 309 nén ta phi e6 S(9) va S() trai déu nhau, nghia a mét sé thi lén hon r, con mét sé thi nhé hon —r. Ti zo, ta cd thé thu duge 2,,= 7 bing cdch hodn chuyén hai phan tit ké nhau. Noi cach khac, tén tai mdt day cdc hodn vi 19, 7, 5 Hm 8a0 cho 7,,= # va véi méi i (i =0, 1, ..., m—1), hodn vi 7, c6 dude tit z, bang cach hoan chuyén hai sé hang lién tiép. Diéu nay c6 nghia Fling néu 7 = (1, ¥2+—-+ Yn)» Mist = (215225 Zn) thi tén tai mot chi 86 ke fl, 2,..,n—I} sao cho 2 =Yus1s Zest =Yko2j=Yjo J#k, k+l. Do gia thiét |x; | n. Nhu thé, cit méi mét nxn- ma tran bac ta lai xay dung duge mét 2nx2n- ma tran bac. Didu nay ob nghia tén tai vo han sé cdc ma tran bac. Bai 147. (1997) Tim cac c&p sé nguyén duong (a, b) thod mana” =b*. Find all pairs (a, b) of positive integers that satisfy: a°” = 6”. Hung dan: Trude hét, 48 y ring néu ta cé a™ =b" thi sé tn tai cdc 86 nguyén duéng c, e, f,d dé a=c°, b=c!, véi m= ed, n= fd, va d la uée chung Ién nhét cia m va n. Diéu nay cé thé dude ching minh bing cdch biéu dién a va b nhu la tich cdc thiia sé nguyén té. Tré lai bai todn, goi d 1A vée chung1én nhét cua a va b?, dat a= de, b?= df. Khi dé, tén tai c d8 a=c*,b=c". Suy ra fe°=ec”". Ta khong thé cé e = 2f vi néu e = 2f xay ra, bY trén ta sé 06 e = f, mau thudn! Gia str2f>e, khi dé f=ec7"*, suyrae=1va f=c™!. Néu c= 1, ta c6 f= 1 va duge nghiém a = b = 1. Néu c>2, tacé c7*-'>2! > £, bai todn vé nghiém. Sau cing, gid si 2f 2° >¢ véi e>5, do dé tacé e = Shay 4 (e> 2f= 2). e=3 cho ta nghiém a = 27, b = 3; e =4 cho ta a = 16, b= 2. Tém lai, nghiém cia bai todn Ia cdc cp (1, 1), (16, 2), (27, 3). Bai 148. (1997) Véi méi sé nguyén duong n, ta ki hiéu f(n) 1a sé tt cd cdc cach bigu dién n nhw mét téng cdc lug thita cia 2 véi sé ma nguyén va khong am. Cac biéu dién khée nhau vé thit ty sfp xép cdc sé hang cia téng dude xem nhu giéng nha. Vi du, f(4) = 4, vi 4227 =2' 4.2! 22! 429429 = 29429 4.29429, Ching minh ring véi n 23, ta cé: 318 a w 24 3, 24 (2"-' +1)f(2"). Ta sé ching minh [(2")<2? bing guy nap. Ban doc ty kiém tra ring bat ding thic dung véi n = 3 (khong ding véi n = 1 va n = 2). Gia str bat dang thuic ding vi n, tic 1 nw £(2")<22, két hop véi bat ding thitc da thiét lap 6 trén ta dude nw n?+2n41 (n+1? £2") <2922 <2 2 =2 2 Vay bat dang thiic ding véi moi n. 313 Ap dung (*) nhiéu lan ta cé: £(2"*") = £(2") + £(2" I +..4fG)+f(2)+£C)+1. (#4) 2 Dé ching minh bit ding thie 24 f(2)+ f(2r—1) > F)+ f(2r—2)>...> f+ f(r +1). Néu k chin thi f(k) = f(k +1) va f(2r—k)=f(2r+1—k), do dé £(k)+f(2r+1—k) = f(k +1) +f(Qr—k). Néuk Ié thi f(k +1) = r09.+4(*) va Farsi) = Fert) 42 eet), nhung theo trén, f 1a ham don digu tang nén (tsa 0 dé ta suy ra f(k)+f(2r+1-k)2 f(k +1)+f(2r-k). Tu cde bat ding thie vita chimg minh, ta suy ra bé d3 duge ching minh. Ap dung két qua é bé dé trén cho (**) ta dugc: £(2"*) > ntl £2"). (#4) a Ta (***), bing quy nap ta c6 thé chimg minh duge 24 2"412 4 =2 4 >2 4 , tie 1a bat ding thtic trén diing véi moi n. Bai todn dai duge gidi xong. Bai 149, (1998) Trong mét cuéc thi, cé a thi sinh va b gidm khio, véi b 1a sé nguyén duong 1é va b >3. Méi gidm khao sé danh gid méi thi sinh theo 2 miic “rét” va "dau". Goi k 14 mét sé nguyén dung sao cho néu lay 2 gidm khdo tuy y thi daénh gid cua 2 vi gidm khdo nay c6 két qua tring nhau nhiéu nhit la cho k thi sinh. Chimg minh ring: Keb. a 314 In a competition, there are a contestants and b judges, where b >3 is an odd integer. Each judge rates each contestant as either “pass” or “fail”. Suppose k is a number such that, for any two judges, their ratings coincide for at most k cor.testants. Prove that ‘ > eo Huéng dén: Goi N 1a sé tat cd cde bé ba (gidm khdo, gidm khdo, thi sinh) sao cho: - hai gidm khdo néi dén trong bé 3 dé 14 khac nhau; - hai gidm khdo dé c6 cing mét danh gid (rét ho&c dau) cho thi sinh nhc téi trong bé 3 46. Ro rang c6 tat ca ~~ oe ) cp gidm khéo chon ti b vi giém khio, va theo gid thiét, méi c&p gidm khdo nay cé két qua dénh gid giéng nhau cho k thi sinh 1a nhiéu nhat, do dé: N <<“ seed a wD Bay gid, ta xét mét thi sinh cé dinh X nao Po va ta tinh sé cae c4p gidm khio c6 cing danh gid véi X. Gid stt c6 x gidm khdo cho X dau, khi dé, c6 x(x—1) (b= x)(b—: x)(b-x-1) 2 2 cho X rét. Do dé, ting s6 cdc c4p gidm khdo cé cimg danh gid cho thi sinh X 1a; —D+O—MO— XD Nruing ta ob: cap gidm khdo cho X dau va c6 c&p giém khao 2 x(x=1)+(b-x)(b—x-1) _ 2x?-2bx+b?—b 2 2 : xt) bbb? b_(o-i? 1 2) 94 °2°4 2° 4 4 2 Ngoai ra, ea 1a sé nguyén (vi b 18), do vay, s6 bé nhat cdc cAp gidm _p2 _12 khéo c6 cing dénh gid choxta: © 2 _Tirdé: nese" @ k=) | Tw (1) va (2) ta nhan duge — > a 2b Bai 150. (1998) Véi moi s6 nguyén duong n, ta ky hiéu din) 1a sé tat ca cdc ude 86 315 duong cia n (ké c& 1 va n). Hay xdc dinh tat cd cdc sé nguyén dudng k sao cho d(n”) =kd(n), vain nguyén dudng nao dé. For any positive integer n, let d(n) denote the number of positive divisors of n (including 1 and n itself). Determine all positive integers k such that d(n”) = kd(n) for some n. Huéng dén: Gia st khi phan tich thanh thita sé nguyén té, sé n cé dang n=pj! py? -Prt Khi dé: d(n) = (a; +1)(az +1)...(a, +1) ,d(n”) = 2a, +1)(2a, +1)...2a, +1). Nhu vay, dé c6 d(n”) = kd(n), ta phai chon cdc sé a; sao cho (2a, +1)(2ay +1)...(2a, +1) =k(ay + (az + 2)...(a, +1). Tit dé, cdc s6 (2a; +1) phai la cdc sé lé, suy ra k phai la 6 18. Ta sé chimg minh dao lai ring véi sé k 1é bat ki, ta cé thé tim dude cdc sé a; thoa diéu kién bai ton. @) Ta st dung phuong phap quy nap theo k. Dau tién, diéu nay dting cho k =1 (khi dé, n =1). Tiép theo, ta ching minh ring néu phat biéu (+) ding cho k thi né cling dting cho 2™k ~1. Laic dé, phat biéu (*) ciing sé ding véi moi k nguyén duong, béi vi bat ctf sé 18 nao cing cé thé duge viét duéi dang 2™k -1, véi k 14 86 1é nhé hon sé lé dé. Dat a, =2'(2™ -1)k -1) vdi i=0,1,...,m—1. Khi dé, 2a; +1 =2'1(2™ —1yk —(2'*! 1) va a, +1=2'(2™ -k-(2!-1). Do vay, tich cia céc sé (2a; +1) chia hét cho tich cia cac sé (a; +1) khi 2™(2™ —1)k—(2™ 1) chia hét cho (2™-1)k hay ome) Nhu thé, néu ta chon cac a; véi k da cho thi phat biéu (*) ding cho 2™k —1. Suy ra diéu phai ching minh. Bai 151. (1998) Hay xc dinh tat cA cdc c&p sé nguyén dudng (a,b) sao cho a”b+a+b chia hét cho ab? +b+7. Determine all pairs (a, b) of positive integers such that ab? +b+Tdivides a’b+a+b. 316 Huéng dan: ¢ Néuaa+l,dodé: ab? +b+7>ab?+b2>(a+l)(ab+1) =a2b+a+ab>a*b+atb. Nhu vay, ta khong tim dude (a,b) thoa diéu kién bai todn trong trudng hgp nay. a’btatb .. © Gidst a>b Dat k=————_, gid st k 1a sé nguyén duong. ab? +b+7 , Ta co: Sa Lab? +b 7) =ab? +a4ab472 4741 >ab? 4a. Suy b b bb a4. . 7 rak3 thi Ce > 0, suy ra: (2-i ab? +b +7) = ab? +a— b—2)-1-7 cab? +a #1. , ail ail . + Néu =-— b thi sé mii cao nhat cua p chia hét f{t). Lic dé a>ofirt) véi sé n nao d6. Nhung na>(n+1)b, do dé k" n Khéng chia hét f"*'(t). Digu nay mau thudn. Bay gid ta dat aA EN’, lie dé: F(t £(s)) = £(t7kg(s)) = kF(?g(s)) =k? g(07e()), sf?(t)=k?sf7 (0), suy ra g(t7g(s))=sg?(t). Nhw thé g cing 1a him théa man céc diéu kién ca dé bai ma hién nhién né c6 cac gia tri bé hon f (véi k > 1). Ta cing cé g(1) = 1. Do ta muén tim gia tri nhé nhat cha £(1998) nén ta sé chi y dén nhiing ham f théa main f(1) = 1. Ta dacé f(f(t))=t va f(t?) =f7(1), tirdé: £7 (st) = (87?) = (S7F(F(U?))) = £7 (S)E(W?) = £7 (9)E7 (0), suy ra f(st)=f(s)f(t). Gia st p nguyén t6 va f{p) = m.n. Khi dé f(m)f(n) = f(mn) = F(F(p)) =p, nhu thé mét trong céc sé f(m), f(n) phai bing 1. Néu f(m)=1 thi m=f(f(m))=f()=1. Tom lai f(p) 1a sé nguyén té. Néu f(p)=q thi f(q) =p. Bay gid ta xc dinh cdc ham f trén tap cdc s6 nguyén t6 thda min 318 cdc diéu kign: anh cia sé nguyén té 14 mét sé nguyén té va néu f(p)= thi f(q)=p. Gid st s=p)’...p?* va f(p,)=q;, gid st t=qP wg? (néu sé t cé nhiéu hon r nhan ti thi ta cd thé thém vao s cdc nhan ti p véi sé miu bing 0), khi dé ta suy ra: 12(s) =a? +a) ghee : £(F(9) =p?" peter : Bay gid, dé ¥ ring 1998 =2.3°37, va f(e)=3, f(3)=2, £37)=5, ta suy ra gid tri nhé nhat ma £1998) dat duge la: £(1998) = 3.27.5 = 120. Bai 153. (1999) Xéc dinh tt cA cdc tap hop hitu han S gm nhiing diém trong m&t phing sao cho: © Scéit nhdt 3 diém; ¢ véi moi A, B khdc nhau thuéc S, duéng trung truc cla AB 1a mét truc déi xting cia cdc diém trong S. Determine all finite sets S of at least three points in the plane which satisfy the following condition: for any two distinct points A and B in S, the perpendicular bisector of the line segment AB is an axis of symmetry for S. Huéng dan: Cac tap hap S cAn tim 1a tat cd nhiing da gidc déu n dinh, véi n > 2. That vay, xét tap S gém nhiing diém thod méin yéu cdu bai ton. Gia st A,, Ag, Ay, ... Ay 1a nhitng dinh cia bao léi cua S. Ta sé ching té nhiing dinh nay tao thanh da gidc déu k canh. A;,, phai nim trén trung truc cua A,A,,2 (néu khong, anh déi xing cia né qua trung truc ctia A,A,,. 86 nim ngoai bao Idi). Suy ra ring tat cA cdc canh cia da gidc 46 bing nhau. Tudng tu, A,,, va A... phai la anh déi xtmg cia nhau qua trung truc cia A,A,,s (néu khong, mét trong hai diém dé sé nim ngoai bao léi). Do dé, tat cA cdc géc déu bing nhau. Vay A,A,...Ay la da gidc déu. Bay gid, bat ki truc déi xtmg nao cho cac diém thuéc S ciing phai 1a mét truc déi xing cho cdc diém A,, i = 1, ..., k, ta suy ra ring truc nay phdi di qua tam C cia da gidc déu k canh A,A,...A,. Gid st X Ja mét diém tuy ¥ nim bén trong da gidc A,A,...A,. Khi dé, X phai nim bén trong hoc ngoai tam gidc A,A,,C. C phai la tam ving tron ngoai tiép 319 AA,X (vi C 1a giao diém 3 duéng trung tryc cua tam gidc 46, méi ding trung truc nay 14 mét truc déi ximg cia S), do vay, X phéi nim trén vang tron tam C, ving tron nay qua A, va Aja. ‘Thé nhung, tat cd cdc diém cua tam gidc A,A,,,X thuc su nim bén trong duéng tron, ngoai trit hai diém A, va A,,,. Vi vay, X khéng thé nado nim bén trong da gidc 1di AyAp...Ay. Ta cé diéu phai chiing minh. Bai 154. (1999) Cho n 1a mét sé nguyén duong c6 dinh, n> 2. a) Hay tim hing sé C bé nhat sao cho ta cé Vx? +x;7) 2. a) Determine the least constant C such that the inequality Dxix jx? + x7) ,...,X, tacd (Fx) (ae Ean) = p> i 1. Gia sin > 1, c&p (n, p) 1a nghiém cia bai toan. Goi q 1a thita sé nguyén té bé nhat cua n. Dat x 1a sé nguyén dung nhé nhat sao cho (p-1)* =-1 (mod q), va y la s6 nguyén dung nhé nhat sao cho (p~1)¥ =1 (mod q). 86 y chic chdn tén tai va ta c6 y 1, néu (a, p) 14 nghiém ca bai todn thi p 1a wéc sé nguyén t6 321 Telarc get nhé nh&t cha n. Ma theo gia thiét n<2p nén hodc p =n, hodc p=2van = 4. Dé thay p = 2 van = 4 khong thoa man, do dé ta cé n = p. Ta ciing dé dang kiém tra duge n = p = 2 van = p = 3 thod man diéu kién cia bai todn. ; Xét trudng hop n > 3. Dé ¥ ring ta c6 (-1)? =-1 nén ding nhi thiic Newton dé khai trién ta duge: 1 1 (P-P 41=14(-1) +P? — pp—Dp? + P(P—INP—2)p? ~~ Cac hé sé nhi thtic ding truée p', (i> 3), trong khai trién trén, hién nhién chia hét cho p°. Tit dé, ting 5 vé phai cia ding thie trén 1a bing p? cong véi mét béi sé cha p?. Nhu thé, téng nay khong chia hét cho p>. Mat khic, vi p > 3 nén p?~! chia hét cho p®, do dé p?-! khong thé chia hét (p—1)? +1, néi cach kha, bai toan v6 nghiém khi n > 3. * Tom lai, cc cp (n, p) tim dude theo yéu cau cia bai todn 1a (2, 2), (2, 3) va (1, p), vai p 1a sé nguyén té tuy ¥. Bai 156. (1999) Tim tat cd cdc ham f:R——>R sao cho: f(x-f(y)) =f(f(y)) + xf(y) + f£Q)-1,Vx,yeR. Determine all functions f :R——>R such that SE- FON) = FSO) + FO) + F@)-1 for all real numbers x, y. Huéng dan: Dat c = f(0) va goi A= f(R). Néu ac A, dat a = fly) vax =a, ta cd f(a—a) = f(a) +a” +f(a)-1, suy ra: l+c a2 f(a)=-2¢-2_., # (a) 29 @) Tiép dén, ta ching minh A-A=R, é day, taki hiéu A-A={x|xeR,3a,beA:x=a—b}. Truéc hét, ta dé ¥ c khac 0, vi néu thé thi dat y = 0, ta cé: f(x—c)=f(c)+xc+f(x)-1, C) suy ra f(0) = f(c) = 1, mau thudn! Tw (**) ta cing c6: 322 Telnet f(x-c)-f(x) = xe+(f(c)-1), ma x chay ty do trén R nén xc+(f(c)—1) cing nhan gid tri bat ki trén R. Nhu vay, moi x thudc R, ta cé thé tim dugc a, b thudc A sao cho x=a-b, suy ra: f(x) =f(a—b) =f(b)+ab+f(a)—1. Do d6, dang (*) ta di dén: f(x) =c— Bab 2 2 2 Dac biét, diéu nay dung cho moi x thudc A. So sanh véi (*) ta suy 2 c-~_,WxeR. 2 2 rac = 1. Do vay, moi x thuéc R ta c6: fog=1. Sau cing, dé dang kiém tra duge ring ham f x4c dinh nhu trén théa man cdc gia thiét cia bai todn, do vay, f 1a nghiém duy nhat cha Bai 157. (2000) Cho 3 sé thuc duong a, b, c cé tich bing 1. Ching minh ring (a-1+t}fo-14 fete} 1. b c a a, b, c are positive reals with product 1. Prove that (a1 plo tffete ts. b c a Huéng dan: 1 11 ft Diy —=—*- =a, tacé:| b-1+— =fo-fs L}-ifisat), be abe c bbe b suy ra ee an ga) ie Thuc hién tuong ty véi hai thita s6 kia va tit dé: 1 1 1\P (a1 E}(o-1+Z}fe-1+2] 0, va ta cing cé b> 1 nén a (o-1+4}>0. c Cho k 1a mét sé thuc duong, n 14 mét sé nguyén Ién hon 1. Con diém nim trén mét duing thing sao cho ching khéng tring tdt cd thanh mét diém. Xét hai diém bat ki A, B khong trang nhau. Gia sit A nim bén phai B. Khi dé, ta néi m6t nuéc di 14 mét su chuyén ché dude thuc hién nhu sau: thay B béi diém B' nim bén phai A sao cho AB' = KAB, Tim tt ca cde gid tri k dé véi cdc nude di (hitu han) nhu vay, ta c6 thé chuyén toan bé cdc diém trén dudng thing néi trén vé bén phai cha mét diém méc tuy ¥ nao dé. Bai 158. (2000) k is a positive real. n is an integer greater than 1. n points are placed on a line, not all coincident. A move is carried out as follows. Pick any two points A and B which are not coincident. Suppose that A lies to the right of B. Replace B by another point B’ to the right of A such that AB’ = kBA. For what values of k can we move the points arbitarily far to the right by repeated moves? Huéng dan: Cau tra 18i 14 k thod man: ket, c= That vay, ta gid sit k<—!_ 5 Khi d6- ky =f-(a->0. = Goi X la téng tat cd cdc khodng cach tir nhing diém trén duéng thing dén diém dau mit bén phai. Néu ta thye hién mot nude di ma khéng lam thay déi ddu mut bén phai, dé thdy ring X bi gidm di. Néu ddu mut phai bi dich chuyén vé phia phai mét khoang cach z, thi X bi gidm bét it nhat la: Loan =koz. X khong thé triét tigu duge. Do 324 46, tng cdc khoang céch ma ddu mit phai bi dich chuyén nhidu nhit 1a Xe, vai Xo 1A gid tri ban ddu cia X. 0 Nhu vay, bai todn Ichong gidi duse khi k <<. = Dio lai, khi k>—!—, ta cd ky =(n—-1)—-1 20. n-1 k Ta luén cé thé dich chuyén diém mit bén trai, Nuéc di nay lam dai chd diém mit bén phi mét doan z > 0 va lam ting X len: (Ie 7 = ky220. Xo Nhu thé X khong thé bj gidm di, Nhung 22k =k >0. Vithé ta = c6 thé chuyén dich diém mit bén phai va xa mét cach tuy ¥ vé phia phai (va do dé, t&t cd cde diém, béi vi n - 1 nude di khdc sé lam dich chuyén cdc diém khac vé bén phai cia diém mit bén phi dé). ‘Tém lai, ede gid tri k cdn tim Ia: ket. ae Bai 159. (2000) C6 100 tém thé khdc nhau duge danh sé tix 1 dén 100, toan b6 100 tém thé nay duge dit trong 3 cdi hép phan biét nhau (mdi hép phai chita it nhdt 1 tm thé). Héi cé bao nhiéu cach sip xép 100 tm thé dy vao trong 3 hép néi trén dé cho néu ta chon ngau nhién 2 hop, réi lai rut ngau nhién 1 tdm thé & mdi hop duge chon, thi ti téng cia 2 thé nay (tic 14 téng céa 2 86 duge dénh trén ching) ta cé thé suy ra dude hop thit ba khong duge chon? 100 cards are numbered 1 to 100 (each card different) and placed in 3 boxes (at least one card in each box). How many ways can this be done so that if two boxes are selected and a card is taken from each, then ihe knowledge of their sum alone is always sufficient to identify the third box? Huéng dén: Dé gidn tién, khi ta néi A chifa n, c6 nghia 1a ta néi A chita tim thé c6 danh sé n. Goi A, B, C theo this ty 1a ba chiéc hp da cho, ta xét 4 trudng hgp sau: 325 © Truéng hgp 1: A chia 1, B chiéa 2, C chia 3. © Trudng hgp 2: A chita 1 va 2. ° Truéng hop 3: A chia 1 va 3, B chita 2. © Truéng hop 4: A chita 1, B chtta 2 va chita 3. Ta sé ching minh ring cac truéng hop 1 va 4 thod man yéu cau dé bai, con cdc truéng hgp 2 va 3 thi khong. * Xét trudng hgp 1, trong trung hgp nay, tuy theo phan du (1, 2, 0 - mod 3) cha n ma ta sé dit tm thé cé sé n vao cdc hép tuong ing A (chita tim thé 1), B (chiia tam thé 2) va C (chifa t4m thé 3). Ta cé thé chiing minh bing quy nap ring phai thuc hién nhu thé thi bai todn mdi cé thé giai duge. That vay, xét tim thé n + 1, vi (n+1)+(7-2)=n+(n-I) nén trong trudng hgp nay ta phai dat t4m thé (n + 1) vao cing hop véi t4m thé (n—2), tic la hép c6 cing phan du (mod 3) (béi néu khong nhu thé, tric 1a gid sit tim thé (n + 1) d&t khac hép véi tim thé(n—2), thi theo trén ta cé hai c&p sé tit hai c&p hop khéc nhau ma cé cing téng - khéng dodn dinh dude nhw dé bai doi héi). Mat khéc, ta cing thdy ring néu dat nhu thé thi bai todn gidi duge. Qua vay, rit hai tim thé bat ki tir hai hép khac nhau, dem téng cdc s6 trén hai tam thé chia cho 3, néu sé du a 0 thi hép thit 3 (khong st dung dé rt thé) 14 C, néu sé du 1a 1 thi hop thif 3 1a A, néu sé du la 2 thi hop tit 31a B. DE ¥ ring, vi cé 6 cach sip xép céc s6 1, 2, 3 vio trong 3 hop ma méi hép chi chiia 1 sé, do dé trong trudng hep 1 ta c6 6 cach khac nhau. * Xét trudng hgp 2, trong trudng hop nay, ta goi n 14 sé nhé nhat (ghi trén thé) khéng chifa trong hp A. Gid str thé néi trén cha trong hép B. Goi m 1a sé nhé nhit (ghi trén thé) chiia trong Lp thi 3 (C). Nhu thé t4m thé (m—1) khéng nim trong C. Néu t4m thé (m—1) nay nim trong A thi do ta co m+(n—1)=(m-1)+n nén din dén diéu mau thudn. That vay, ta cd m 6 trong C, (n-1) 6 trong A, tang hai thé cho ta két lun hép khéng dusc chon 1a B; mt khac, (m—1) nim trong A, con n nim trong B, ma cing véi cing téng dé cho ta két Juan hop khong dugc chon 1a C: nhu thé, ta khong thé suy dodn dude theo yéu cdu dé bai. Tit d6, do thé (m~1) khéng nim trong A duc, né phai nim trong 326 B. Nhung (m—1)+2=m+1 nén ta cing dé thay diéu mau thudn (Ii Juan tuong tu nhu trén). Tém lai, trugng hgp 2 khéng xay ra, nghia 1a néu sép xép nhu trutng hgp 2 thi bai todn khéng gidi duge. + Xét trudng hop 3, trong trudng hgp nay, ta goi n 1a sé nhé nhat (ghi trén the) chtta trong hép C. Tir dé thé (n—1) phai nim trong A hay B. Néu (n—1)ndm trong B, thi do (n—1)+2=n+2 nén ta c6 diéu mau thudn (mét téng ciia hai thé trong A va B bing mét ting ctia hai thé trong C va A). Néu thé(n—1)nim trong B, thi do (n—1)+3=n+2 ta cing c6 diéu mau thudn (mé¢t téng cda hai thé trong B va A bing mét téng cia hai thé trong C va B). Nhu vay, trutng hgp 3 khong thé xay ra dug. * Xét trudng hop 4, trong trutng hop nay, ta goi n 1a sé nhé nhat (ghi trén thé) chia trong hép C. Nhu thé thé (n—1) khong nim trong A dude (bdi néu khéng thi do ta cd (n—1)+3=2+n nén mot tong hai thé chtia trong A va B bing mot téng cia hai thé trong B va C: mau thudn). Vay thé (n —1) phai nim trong B. Bay gid, xét dén thé (n + 1), thé nay khong thé nim trong A duge (béi néu khéng thi do ta cé (n+1)+2=3+n nén mét téng hai thé chia trong A va B bing mét téng ciia hai thé trong B va C: mau thudn). Thé (n + 1) nay cing khéng thé nim trong B hay C du¢c (béi néu khéng thi do ta cd 14+(n+1)=2+n nén mot téng hai thé chiia trong A va B (hay C) bing mét téng cia hai thé trong B va C: mau thudn). Vay (n + 1) khéng tén tai, suy ra ta cé n = 100. Bing quy nap, dé dang ching minh ring khi dé, dé bai ton cé thé gidi dude, tat cd cdc thé 4, 5, ..., 98 déu phai nim trong B. Qua vay, gia sit thé m (3< m < 98) nim trong B, néu thé (m + 1) nim trong A, do ta c6100+ im =99 +(m+1) nén mét téng hai thé chita trong C va B bing mot téng cia hai thé trong B va A: mau thudn. Nhu vay, & tring hop 4, ta cé su sip xép 1 trong A, 2 dén 99 trong B, 100 trong C, va su sip xép nay gitip cé thé gidi duge bai todn. That vay, hé chon hai thé cé téng tit 3 dén 100 thi ta suy ra ngay hép khéng due chon 1a C, hé chon hai thé cé téng 14 101 thi ta suy ra ngay hop khong duge chon 1a A. 327 Dé ¥ ring, cing nhu é trudng hop 1, ta cé 6 cach sap xép nhu thé khi thay déi vj tri 3 hop. Tém lai, dé gidi dude bai todn, ta c6 cd thay 12 cach sip xép 100 tdm thé vao 3 hép. Bai 160. (2000) Liéu c6 thé tim duge sé ty nhién N cé ding 2000 uéc sé nguyén t6 khac nhau va N chia hét 2" +1 hay khong? Can we find N divisible by just 2000 different primes, so that N divides 2" +12 Huéng dan: Dé ¥ ring véi sé b Ié ta 6 2 4.1 = (28 41)(22O-Y 20> gad, nén (2° +1) 1a mot thita sé cla 2° +1. Dé cé thé tim N nhv dé bai ddi héi, ta tim m sao cho: (2) m chi cé vai wée s6 nguyén té phan biét; (2) 2™ +1 06 86 16n cdc ude 56 nguyén t6 phan biét; (3) 2" +1 chia hét cho m. Qua vay, néu tim dude m nhu thé, ta 6 thé chon k bing tich cia mét sé cae thita sé nguyén t6 cia 2™+1 (hdc m), 48 cho km c6 ding 2000 thita sé nguyén t6. Khi dé, 2" +1 van chia hét cho km va suy ra 2” +1 chia hét cho m. Trudng hgp don gidn nhat, m chi cé duy nbat 1 thita s6 nguyén t6 P, néi cach khc, m 1a béi cia p. Nhung néu m nguyén t6 thi 2? -2 chia hét cho p, do dé sé p duy nhat ma p chia hét 2° +1 143. Nhu thé, cau héi dat ra la: (1a, =2™ +1 06 chia hét cho m=3" hay khéng; (2)a, =2™ +16 duge di nhiéu cdc thita sé aguyén té hay khong? Ta cé apy =a,(22™-2™ +1), vi m=3", Nhung 2™ =a), —1, do 46 a4) =a) (a? —3a,, +3). Ta a? =9, ding quy nap, ta dé dang ching minh dude a, chia hét cho 3+), nhw thé ta cé khing dinh cho c4u héi (1). Hon nia, vi a, 14 mét thita sé cia a,,, nén bat ki thaa sé nguyén 16 nao ciia a), cing 1a mét thita sé nguyén té cla a,,). Dat a, =3"* by, khi dd b, 4) =b, 328"? -3"*2b, +1). Ta 06 (71h? 325, +1) >1 nén né phai cé mét thiza sé nguyén té p lén hon 1. Vi (3"*"p? -3'*?p, +1) 1a béi cia 3b, cong thém 1 nén p la 3 hoac 1a sé chia hét b,. Tu d6, bj, c6 it nhdt 1 thita sé nguyén té p lon hon 3 ma p khéng chia hét b,. Nhu vay b,,, c6 it nh&t h thita sé nguyén té phan biét 1én hon 3, ta cing cé khiing dinh cho cau héi (2). Do la tt cd nhiing gi ta cdn dé tré Ii cau héi cho bai todn. Ta cd thé chon m nhu da néi & doan truéc dé 06 dude 320: (1) m chi cé 1 thiza sé nguyén té, (2) 2™41=37'b, 9906 it nh&t 1999 thita sé nguyén t6 phan biét lén hon 3, (3) m chia hét 2" 41. Chon k 1 tich cia 1999 thita sé nguyén té phan biét chia hét by999- Khi d6, N = km ]a sé phai tim, N cé dung 2000 uéc sé nguyén té khac nhau va N chia hét 2% +1. Bai 161. (2001) Cho cac sé nguyén duong a, b, c, d véi a>b>c>d>0. Gid st ac+ bd =(b+d+a—c)(b+d-a+c). Ching minh ring ab+cd khong phai 1a sé nguyén 6. Let a, b, c, d be integers with a > b > c > d > 0. Suppose that ac + bd =(b+d +a—c)(b+d-—a+c). Prove that ab+cd is not prime. Gidi. Dang thiic da cho twong duong véi a? ~ac+c” =b’ +bd+d?. a) Xét ti gide ABCD véi AB = a, BC = d, CD =b, AD=c, BAD =60°, BCD =120°, BD?= a? -ac+c? =b? +bd+d’, mot tif gidc nhu thé ré rang tén tai trén co sé cé (1) va Dinh Ii ham cosin. Dat ABC =a, suy ra CDA =180° -@. Dinh li ham cosin trong cdc tam gidc ABC va ACD cho ta: a? +d? —2adcosa = AC? =b? +c? +2becosa. Tu do, reosaa2 te —b =e va ad+be a? +d?—b’-c? _ (ab+cd)(ac +bd) adtbo tind tC AC? =a? +d? —ad Vi ABCD néi tiép nén Dinh li Ptolémé cho ta (AC.BD)? = (ab+cd)?, suyra: (ac + bd)(a? ~ ac +c”) = (ab + cd)(ad + be). (2) Tiép theo, tit gid thiét, ta cd (a—d)(b—c)>0, (a—b)(c—d) >0, titi bit dling thie miy dé ding suy ra duge ab+cd >ac+ bd >ad+be. (3) oo gia stt ngugc lai ring ab+cd 14 sé nguyén té. Khi dé, tix (3), suy ra ring ab+cd va ac+bd nguyén t6 cing nhau. Do vay, (2) cho ta két Iuan ad+be chia hét cho ac+bd, diéu nay khong thé xdy ra vi da cé (3). Ta c6 dpem. Phu luc I MOT SO DE THI VO DICH TOAN CAC NUGC VA KHU VUC Phan Sé hoe, Dai sé, Giai tich, Hinh hoc té hép 331 Bai 1. (Olympic Todén Bungary, 1999) Cho day cdc sé nguyén aj, a9,...,4,,... thoa man: (n—Na,4; =(n+Da, —2(n-1) véi moi n > 1. Néu 2000 chia hét ajggg, hay tim sé n nhé nhit, véi n > 2, sao cho 2000 chia hét a, . Hung dan: Hién nhién, ti ding thu & dé bai, ta cé a, =0, va khin > 2 thi: Do dé, day da cho dusc x4c dinh mét cdch duy nhat béi a. Ngoai ra, ta cd.a, = (n—I)(cn+2), véi e= 2-1 1a mét sé thye tay ¥, day a, théa min ding thitc 6 diéu kign ca bai todn. ‘Tat cA cdc day a, théa man diéu kién cia bai ton déu cé dang nhu thé. Vi tat cd cdc s6 hang cua day déu 1a cdc s6 nguyén va 2000 chia hét ajggg nén ta dé thdy ring c 1a sé nguyén va c = 1000m + 2. Nhu thé, suy ra 2000 chia hét a, khi va chi khi 1000 chia hét (n—1)(n +1). Ta dé, n= 2k +1 vak(k + 1) chia hét cho 250=5°.2. Vik va (k + 1) nguyén t6 cing nhau nén ta suy ra sé n nhé nhat, n > 2, 1a: 2 x 124+ 1= 249. Bai 2. (Olympic Toén Bungary, 1999) Tim tat cd cdc cp sé nguyén (x, y) théa man: x? = y> +2y? +1. Huéng dan: Ré rang ta c6 x > y. Mat khac, ta cé: xy 3 +2y?2 41 y(y+3)>0. Do vay, néu y > 0 ho&c y <-3 thi bai todn vé nghiém. Thi truc tiép céc c&p (x, y) vao ding thie x? =y?+2y? +1 ta dude cdc c&p sé nguyén théa man dé bai la: (-2,—3), (1,2), (1,0). Bai 3. (Olympic Todn nuée Ao, 1982) Tim tt cd cac sé d(0;1) cé tinh chat: Néu fx) 1a ham sé tuy ¥ lién tuc, xdc dinh véi xe[0;1], ngoai ra f(0)=f(1), thi tn tai sé, Xq €[0;1—d] 48 sao cho f(x9) = f(x9 +4). 332 Huéng dan: Ta chiing minh ring bat ki 66 d=2, bday kN, déu thod man diéu kién bai todn. Cé thé léy tuy ¥ ham sé f(x) lién tuc va sé k>1 (sé d=1 thoa man diéu kién, vi £(0)= f(1)). Xét ham sé: a(x) = a(x + i). £(%) xée dinh trén doan [ok Vi ting cde 06 2r=(2}-rl2))-€)-- te 0-2) bing 0, nén gitta chting cé s6 khong duong va ciing c6 sé khong am. Béi vi ham sé g(x) lin tuc, nén tén tai sé x, dé sao cho o(xo)=0, nghia la: {xorg }- f(x). Bay gid gid sit cho s6 de[0;1] khong bing z véi bat cif gid tri nao cla ke N. Léy sé keN sao cho kd<1<(k+I}d, va xét him sé lién tuc f(x) tuy y, xdc dinh trén [0;d] va thod man cdc dang thtc £(0) = 0,f(1—kd) = -k, f(d@) =1. Tiép tuc xét ham sé nay trén doan [0;1] sao cho véi méi x < [d;1] ta cé ding thite f(x)= f(x—d)+1. Ta nhan dugc ham lién tue, hon nia f(1) = f(1—d) +1 =f(1—2d) +2 =...f(1-kd) +k = 0 = f(0) va véi bat ki gid tri x € [0;1—d] ta c6 hé thie: f(x +d) = f(x) +1 # f(x). Bai 4. (V6 dich todn Ba Lan, 1989 - 1990) Tim tt cd cdc ham f théa man phuong trinh ham. (x= y)E(+ y)— (x + y)F(X—y) = 4xy(x? — y?). Huéng dan: . fxty)_ F&-y) _ x+y x-y Tit dé bai ta cé: xy. Dat 20) = 2), ie dé: g(x + y)— g(x — y) = 4xy. Lai dat x= — = , tasuyra g(at+h)-g(a)= dart ae. Tit day ta duige: see) =2ath. Cho h -> 0, ta c6: g'(a)=2a. Do dé g(x) = x? +O, flx) = x° + Cx, véi C 1a hing s6 tiy ¥. Bai 5. (Olympic 30 - 4, Viét Nam, dé dé nghi, 2000) Tim a dé phuong trinh sau cé 3 nghiém phan biét: 3 2 x +l x +1 x41 +21 4q—ayX tty ga—6=0. xvx x vx Huéng dan: pat t= Vx+—L, diu kign t> 2, phwong trinh tré thanh vx P+ 2(a—It? +(1-4a)t—-2=0 © (t-2)(t? + 2at + =0 (2 10} 2 [isomer=o. 2) Phuong trinh (1) cho ta mét nghiém x = 1. Nhan xét: (2) <> (vx)? -tvx+1=0 (3). Phuong trinh (3) c6. A=t?—4 , néu t > 2 thi (3) cé hai nghiém duong phan biét. Suy ra ring phugng trinh da cho cé thém hai nghiém phan biét khéc 1. Mat khdc, phuong trinh (2) c6 P = 1, do dé phuong trinh nay khéng thé cé ding thdi hai nghiém phan biét lén hon 2. Vi vay, dé phuong trinh d4 cho cé ba nghiém phan biét thi phuong trinh (2) phai cé hai nghiém t,, t. thoa diéu kién ty Qsao cho f thod man diéu kién £(1) = 2 va déng nhét thite: f(xy) =f(O0f(y)-f(x+y) +L xy EQ. Huéng dan: Cho trong déng nhat thic gid tri y =1, ta nhan duge déng nhét thie: 334 f(x) = f(x) £()-f+)+1 (x EQ), hay f(x+) = f(x) 41. Tw day, véi moi xe Q, ne Z tacé: f(x+n)=f(x)+n, f(n)=f(I)+n-1=n+1, Sau d6, cho ding nhit thie ede gid tri x=1,y=n véi neZ, ta n nahin duge: {ia)-s(! it) (2 +a}, suy ra fo {2) dn fn Cudi ciing, cho gia tri x = py = v6 peZqeN: {ot}e0)ort} ta suy ra {? }-ou tt}-bep=Ba. q q Nhu vay chi cé mét ham sé f(x)=x-+1 1A thuc sy thod man tét cA céc diéu kién cua bai toan. Bai 7. (Thi chon hoe sinh gidi toén Bucharest, 1999) Cho cdp 6 cong sau: 308, 973, 1638, 2302, 2968, 3633, 4298. Hay xéc dinh cap s6 nhén b,, by, b;, by, bs, bg sao cho 308 < b, < 973 . Gia tri lén nhdt cho b, 1 972, va gid tri nhé nhdt cho bg 14 3634. Nhu thé ta duge: 3634 < 972.x*° => 1,301 < x. Mat khdc, tir gid tri nhé nhdt 14 2304 cia by va gid tri Ién nhdt 1a 4297 cia bg, ta cing c6: 4297 > 2304.x? => x < 1,87. Vay: 1,301 < x < 1,37. (b) Tiép tue diing by = by-x° va (b) d8 Ki luda, ta tim duge x = ‘. 2 Khi 46 by = »(5) . Ma 3°=243 nén b, phai la béi cia 243, nhung b, nim gitia 309 va 972 nén b, = 486, 729 hay 972. Két hgp véi b,.x = bz > 973, ta tim dude b, = 972. Cuéi cing, tit (a), cde 86 by, b2, b3,"by, bs, bg thoa man diéu kién cilia bai todn 1a: 972, 1296, 1728, 2304, 3072, 4096. Cube thi USAMTS chon tai ning Toin hoc cia Mi (USA Mathematical ‘Talent Search) due gido su George Bezsenyi té chic lin dau tién vio nim 1969, dudi si tai try cia Hoc viga Ki thudt RoseHiulman va COMAP (Consortium for Mathematics and It's Applications - Hgi Todn hoc vi tng dung), Day li mit chuyén muc thuding xuyén cia t3 bio mang tén Consortium, duge COMAP xuat ban. Dudi diy, ching ti trich gidi thigu cing cde ban m@t sé bai todn thuje cufe thi USA Mathematical Talent Search. Bai 8. (USAMTS, 2000 - 2001) Hay tim sé du khi chia sé 1776!” cho 2000. Huéng dan: Cach 1: Tacé: 1776! =1776 (mod 2000), 76(mod 2000), 1776° = 576 (mod 2000), 17764 =976(mod 2000), —_1776° =1376(mod 2000), 776 (mod 2000), 1776” =176(mod 2000), va tiép tuc nhu vay. Ti 1776° = 1776! (mod 2000), ta duge 1776" =1776"~> (mod 2000), véi moi n > 5. Do vay ta sé xét phan du ctia sé mi khi chia cho 5. Dé thdy 1492! chia hét cho 5 nén: 1776! =1776° =1376(mod 2000). Vay khi chia sé 1776'4™ cho 2000 ta duoc s6 du la 1376. Cach 2: Truéc hét, ta ding quy nap dé ching minh bé dé sau: Bé dé: Véi moi sé nguyén duong n, ta cé: 1376" = 1376 (mod 2000). Chitng minh Bé dé: Dé thdy 1376! =1376 (mod 2000) va1376? = 1893376 = 1376 (mod 2000). Gid sé ménh dé trén dung cho n= k, ta sé ching minh 1376**! =1376 (mod 2000). Dé lam diéu nay, ta thyc hién nhan hai vé cua phuong trinh ding du 1376* =1376 (mod 2000) cho 1376 va dugc 1376**! = 1376? (mod 2000). Vi 1376? =1376(mod 2000) nén ti day ta cd 1376" =1376 (mod 2000) va Bé 48 duge ching minh. Tré lai bai todn: Ta cé 1776° =1376(mod 2000). Vi sé 5 chia hét 1492! 14921 1492! nén ta duge 1776'4 =(1776°) 5 =1376 5 (mod 2000). 14921 Ap dung bé dé trén ta c6 1376 5 =1376(mod 2000), tix 46, khi chia sé 1776! cho 2000 ta duge sé dul 1376. Cach 3: Dinh li Euler cho ta: a! =1(mod 125), véi moi a thod man (a, 125) = 1. Ta c6 1611776 nén 1776'4 =0(mod 16). Bay gid, xét 86 du cia 1776'*™ Ichi chia cho 125, vi (125, 1776) = 1 va 51492! nén theo Dinh li Euler ta c6: 1776!4™ = 1 (mod 125). Tiép dén, gidi hé phwong trinh déng du n=l (mod 125) fr =1 (mod 16) bing phuong phap thit chon, ta dugc nghiém duy nht 1376. Vay 1776'4% = 1376 (mod 2000). Bai 9. (USAMTS, 2000 - 2001) Xac dinh sé nguyén duong N bé nhat gdm 5 chit sé sao cho 2N ciing 1a sé cé 5 chit sé va tat cd cdc chit sé 0, 1, 2, ..., 9 déu cé mat trong 2 sé N va 2N. Hung dén: Ta gid sit N 1a sé ABCDE va 2N 1a s6 FGHIJ, trong dé A, B, C, D, E, F, G, H, I, J 1a nhing chit sé kha nhau lay tit cde chit s6 0, 1, 2, 3..., 9. Aro rang khéng thé la s6 0, sé nhé nhdt ma A cé thé nh4n duge 1a 1. Ta sé gidi bai todn tit nhiing sé bé nhat cé thé nh4n, cho dén khi nao gap didu vé Ii thi théi. Xét trudng hgp A = 1. Khi a6 F = 2. Sé nbd nhdt ma B 6 thé nhan 1a 3 (B khéng thé bing 0 vi hoac sé bi tring 2 l4n s6 0, hodc bi tring 2 lin sé 1), hic dé s6 nhé nht ma C cé thé nhén 1a 4, tuong ting ta 337 Rodarte dude G = 6. 1) Bay gid, sé nhé nhdt ma D cé thé nhan 1a 5. Suy ra H = 9, vi H = 2C + 1 (nbd). Tiép theo, sé duy nhat ma E sé nh4n 1a 8 (vi néu E = 0,7 thi bi tring sé), nhung khi E = 8 thi 2D cong véi 1 nhé nia 1a 11, tute I = 1,voliviA=1. 2) Nhu thé D khéng thé bing 5. Gi st D nhan sé bé nhat tiép theo 1a 7 (do da cé G = 6). Lic 46, H = 9 (= 2C + 1), nén E = 8 nhu trén. Nhung bay gid ta lai gip mau thudn vi J = 6, tring sé. 3) Tiép dén, gid sit D = 8, suy ra H = 9. Lic nay, sé nho nhét ma E cé thé nhan 1a 5. Ta c6 2E = 10 nén J = 0, ty dé 1 = 7 vi 2D + 1 (nhd) = 17. Tom lai, ta dudc s6 N théa man dé bai la N = 13485 (2N = 26970). Chit §: Dr. Béla Bajnok, DH Gettysburg, cng tac vién cia USAMTS, ngudi da dit ra dé todn nay, cho biét ring cdc sé N nhé nhdt ké tiép theo (sau 13485) la: 13548, 13845, 14583. Bai 10. (USAMTS, 2000 - 2001) Biét ring cé mét cach thit dé xét xem mét sé A cé chia hét cho 19 hay khéng nhu sau: Gia six ta c6 s6 A, bé di chit s6 cudi cig cia 86 A, ta duge sé B, dem B céng véi 2 1dn chit sé (cudi cing) da bé, ta duge sé C. Lap lai qua trinh trén véi sé C, réi tiép tuc nhw thé cho dén chimg nao thu duoc sé Mohd hon 20. Khi dé, A chia hét cho 19 néu va chi néu M = 19. Vi du, s6 67944 chia hét cho 19, cdn 44976 thi khéng. Ta thyc hién nhu sau: 67944 44978 8 12 6802 4508 4 18 684 468 8 16 16 4 12 4 19 10 7B nocd Hay tim cach thi tuong tu dé xét xem mét sé A c6 chia hét cho 29 hay khéng. Hiuéng dan: Phuong phap thi: Gia si ta cé 6 A, bé di chit s6 cudi cing cia s6 A, ta duc sé B, dem B cong véi 3 lan chi sé (cudi cing) da bé, ta dude sé. C. Lap lai qua trinh trén véi sé C, réi tiép tue nhu thé cho dén ching nao thu duge sé M nhé hon 30. Khi dé, A chia hét cho 29 néu va chi néu M=29. Chiing minh: Ta viét A = 10a + b, véi b 1a chit sé cudi cing cia sé A. Khi bé b di (due sé B = a) va cong véi 3 lin chit sé b, ta dude sé C =a + 3b. Bay gid, dé y ring 10a + b chia hét cho 29 khi va chi khi 10a + b + 29b chia hét cho 29. Ta lai cé 10a + b + 29b = 10(a + 3b), s6 nay chia hét cho 29 khi va chi khi a + 3b chia hét cho 29, sé di cé diéu nay vi 10 khong chia hét cho 29 va 29 1a sé nguyén t6. Vay A chia hét cho 29 khi va chi khi C chia hét cho 29. Qua trinh nay lap lai nhw thé cho dén khi dugc sé M < 30. Hon nia, 29 1a sé duy nhat gitia 0 va 30 chia hét cho 29, do vay phép thi cia ching ta 1a ding. Chi §: Ban doc hay ching minh ring phép thi tuong tu nhu thé sé dude thuc hién véi cac phép chia cho 39, 49, 59, 69... Bai 11. (USAMTS, 2000 - 2001) Gia str da thie p(x) = x5 +x? +1 c6 nim nghiém 1,1) .....15. Dat q(x) =x? -2. Hay xde dinh tich q(r,)q(r2)...4(ts). Huéng dén: Ta c6: p(x) = (x — HX —1)...(X = 5) va a(n )a(r2)--a(t5) = (FP - 203 -2)...(15 2). V6i moi i = 1, 2, ..., 5, 1? -2=(V2—5)(-V¥2 —,) nén q(1,)q(r))-.-a(t5) = = (V2 =n )(V2 -1y)..V2 = 15 (V2 - 4 (V2 ~1)..(-V2 = 15) = p(v2)p(—V2) = [(V2)° + (V2)? +1). V2)? +( V2? +1). Vay q(r)q(r))-.-q(ts) = — 23. Bai 12, (USAMTS, 2000 - 2001) Xac dinh tat ca cdc sé nguyén duong N sao cho khi sé N duge viét trong hé thap phn thi N sé lén hon ting cdc binh phuong nhiing chi sé 339 cua né 1 don vi. Huéng dan: Gia si N = ...asa4a3a7a,49 , trong dé, cdc chit s6 a; nhan gid tri tir 0 dén 9. Theo gia thiét: lt+aQ +a? taz+az+aj +...= a9 +a10! +4510? +a310° +a4104 +..., hay 0=1+a9(a9 —1)+a;(a, —10)+a(ay —100) +a3(a3 1000) +a4(a4 —10000)+... hay a;(a, —10)+ay(az —100) +a3(a3 —1000)+a4(a4 —10000)+... = —1—ag(ag -1) 2 -1-9x8 =-73. ) DE y ring cdc a; 1a cdc cha sé tir 0 dén 9, do dé ta suy ra ay =a; =a, =a5 0, vi néu khong nhu thé thi ting 6 vé trai cia (*) sé nhé hon —73. Vay: g(a —1)+1=a,(10-a,). (*) Cho ag cdc gid tri tit 0 dén 9, vé trai cua (**) nhAn cc gid tri tuong tng 1a 1, 3, 7, 13, 21, 31, 43, 57, 73. Tuong tu, cho a, cac gid tri tit 0 dén 9, vé phai ciia (**) nhan cdc gid tri tuang ting 14 0, 9, 16, 21, 24, 25, 21, 16, 9, 0. Tw dé, ta dude ay = hay 7. Hai sé tim duge 14 35 va 75, thi lai, ching théa man yéu cau cua dé bai. Cie bai sau diy due trich tit ede ki thi William Lowell Putnam ti 1997 dén nim 2000, Day Ia cuge thi dinh cho sink vién ede mide Mi va Canada, nhuing nhitng bai trich d dy phi hop véi hoe sinh gioi Todn phé thing @ mide ta. Bai 13. (Putnam, 1995) Ta n6i mét tap hop M dong véi phép nhan néu Va, beM: abeM. Cho S la tp cdc sé thc dong véi phép nhan (phép nhén & day la phép nhan théng thudng), gid sti S 14 hgp cua 2 tap rdi nhau T va U. Biét ring tich cua 3 phén ttt bat ki (khong cdn phan biét) cda T la mot phan ti cua T, va tich 3 cua phn ti bat ki cua U 14 mot phan tit ca U. Chiing minh ring it nhat mét trong hai tap T, U déng véi phép nhan. Huéng dan: Gia st ngugc lai ring ca hai tap T, U déu khong déng véi phép nhan. Lic dé, tén tai a,b € T vac, d © U sao cho ab¢T ,cd¢U. 340 Nhung do S déng véi phép nhan nén ab Svacd c S,maSla hgp hgp cta 2 tap rdi nhau T va U nén ta suy ra ab € U vacd T. Bay gid, cling tix gia thiét ta duge: (ab)ed € U va (cd)ab € T. Ma phép nhan & day la phép nhan thong thudng nén 2 phan tif (ab)cd va (cd)ab gidng nhau, suy ra U va T khong réi nhau, mau thudn nay cho ta diéu phai ching minh. Bai 14, (Putnam, 1996) Xe dinh sé A nhé nhat sao cho véi hai hinh vuéng bit ki cé ting dién tich bing 1, t6n tai mét hinh chit nhat cé dién tich bing A thod man diéu kién: hai hinh vuéng néi trén cé thé duge xép nim hoan toan trong hinh chit nh4t ma phdn trong cia chung khéng dé lén nhau, va cdc canh cia 2 hinh vuéng thi song song véi cdc canh cua hinh cha nhat. Huéng dan: V6i 2 hinh vuéng cé téng dién tich bing 1 nhu da néi 6 da bai, goi x la d6 dai canh céa mét hinh, ta cd: z BA x2 aie. Khi d6, hinh chit nhat nhé nhat chita ching nhw da néi sé cé cdc canh lax va x+V1—x2 vacé dién tich: £(x)= x? +xV1-x2 . Tacé: f'(x) = 2x+V1-x2 = Tud6,x= i = | thay vo ta duge gid tri cdn tim: 0)=50+V2). Bai 15. (Putnam, 1997) Cho day sé a, xdc dinh béi a, =2, a,,,=2"". Ching minh ring a, =a,_; (mod n) véin > 2. Huéng dan: Dé thay ring két qué sau day manh hon két qué cdn chimg minh: a, =a,_; (mod m) véi moi m < n. Ta sé ching minh két qua nay bing quy nap. Két qua hién nhién ding véi n = 2. Gid st né ding véi moi k < n. Ta phai ching minh a, n— (mod m) véi moi m < n, nghia B41 a (») Ts "4 RE ‘Upp Bune, “™ PH 194 BT WON] uoNy Up Suga qune Suny f-Unz-un oo z *@ & wrout oa va p75 n= tape tn "uD (666r ‘wousnzy ‘LT pg U 98 ‘Bupnyd quryo 98 B] reyd Buoyy o+ug+ e+ ou Oyo ows u Suonp Upgdndu 98 O6np may 943 99 & 9 ‘q @ uedngu 98 tout toa ‘Rey wos, ‘Buonyd Yury 98980 BT nap (py (ey eu (y BS 918 Tey upnyy nyu apa * “484 DUN “(p pow) 9 = C+47 Bh Suonyd yum og °F BI (ey Ba (Ty pu UeU (> pout) z +4925 (D5~(¢)y ser FROBN ‘(5 pout) ga 4@ Fy} Supnyd HUPGO 98 O89 Br (py Ba (2X nou ‘op op Pow) 92 = (b)3~(z)3 490 aq ‘Bupnyd quryo 98 OB BY ep (py (eM (ay “(LY oyp oes 9 ‘q ‘8 uedngu 9s O80 Tey ug Supa Pe] opngu 2S VIB By ‘or 4eg ‘( pow) ae (¥)z 90 Teyd 23 BD 9m ‘Supnyd TMD 98 39ur ey O+Ug+ ue + c= (UI ne vty ‘upp Sugny qUr 9s yy Rud Suoyy 04 Uq+ ,ue+ c4 OY ows u Suonp uedngu 98 OOnp uy 9UF 90 By ‘9 ‘q ‘8 ueénsu 98 tour pa Bupr yurar Sunyo (866 “wnugng) ‘OT tog “Fura Supyo wyd REI Bz dng TS Wow ipa (wu POW) ou Z = bug? ‘OP Op ‘neyu Suno 93 Ushngu st BA(T + 3) Sunyy Gs? Pow) ize bul WRU 2052] 67 BA byl 52 99 84 ware up T+38 pow) oz rd Ba Ans ((p4 3Z)P pour) 2-Hy 48 :09 2 dey 4nb 9973 Bs OR US WS THIZS (14 12)6 OPT + ag pow) p= G+sqyp% 8D HT Yuig OOM “OP PEL (1+87),2 = WE eS w rou ips ca POU) enteZ = sugZ Bf 2 = 2 Khi d6, ding thie u, = SUatUa3~8tn-1Un-2 arise vidi don Un-2Un-3 gian thanh: v, =6v,_,—8v,_>. Bang quy nap, ta dé dang ching minh dude v,, c6 dang v, = A2"+B4". Nhung ta cé: vy =42=2, vy= 3 =12, yy U2 nén ta duc v,,; =4" —2". Suy ra u, = (481 —2-Fyqn-? 29-2), (4-2), Bay gid, voi moi sé p nguyén t6, ta co 4?~ P-! (mod p), do dé p chia hét 4°"! 2°"! va p chia hét 4° — 28, véi moi sé s 1a béi cla p-1. Néu p' chia hét n, thi tén tai it nhdt r béi sé cia p—1 nhé hon n, do dé p’ chia hét u,, . Tix dé ta suy ra n chia hét u,. WMSETS (Winoonsin Mathematics Science & Engineering Talent Search) li ki thi chon tai nding toin hoe cia tring Dai hye Winconsin Madison, USA, gc sinh moi muée dda ob thé tham gia dé nhin he bing 24000 di-la cho 4 nim hoe. Hai bai dui dy duge trich ti dd thi nim hoe 2000 - 2001. Bai 18. (WMSETS, 2000 - 2001) Goi S 1a tap hgp gim 100 sé nguyén duong bé hon 290. Ching minh ring tén tai mdt tap khac ring T con cia S sao cho tich tat cA cdc phan ti ca T a mét sé chinh phuong. Huéng dan: ‘V6i méi tap con Z khac réng cia S, ta ki hiéu P(Z) la tich tat cd cdc phan tit cia Z va mz 1a 86 sao cho P(Z) = c?mz, trong dé c” 1a sé chinh phusng 1én nhat chia hét P(Z). Nhu thé, mi sé m,1a m6t tich cua cae s6 nguyén té khac nhau, méi sé nguyén té nay bé hon 20; hic nay, do chi cé it hon 100 sé nguyén 16 nhu thé, ta thdy ring nhiéu lm 1a cb 2” kha ning cho m,. Hon nila, 86 cdc tap Z cé thé cé 1 sé tit cd cdc tap con khac réng cia S, nghia 1a c6 thé c6 2'—1 tap Z. Vi 2! -1 > 2” nén chic chin phai tin tai hai tap con khac rng X va Y cia S sao cho my =my. Bay gid, gid sit P(X) = a’my va PCY) = b’my =b’my, thi ta cd P(X).PCY) = a?m,.b’m,, day 1a sé chinh phuong. Ta goi T 1 tap hop 343 con cua S gém cdc phan tit hoac thuéc X, hoac thuéc Y nhung khéng thudc ca hai (T = KUY \ (KAY). Do X khdc Y nén T khae rong. Mat khac, méi s6 cua XNY da xudit hién 2 lan trong tich P(X).P(Y), do dé ta 66 PCX).PCY) = PCT) (PK NY). Vi P(X).PCY) 1a sé chinh phuong theo trén nén ta suy ra P(T) cing 1a sé chinh phuong, diéu phai ching minh. Bai 19. (WMSETS, 2000 - 2001) Gia string 2 > X> A véi a, b, ¢, d, x, y 14 cae 86 nguyén khong y b am. Néu ad — be =1, hay ching minh x >a+ceva y>b+d. Huéng dan: Dey a x_ay—bx o<2-%-9°-™* by by do dé ta cé ay —bx =r (*) 1.86 nguyén duong. Tuong tu, dx —cy =s (**) cing 14 sé nguyén duong. Str dung gid thiét ad—be =1, nhan 2 vé cia (*) cho c va nhan 2 vé cua (**) cho a réi cong chting lai ta duoc x =as+cr; nhan 2 vé cia (*) cho d va phan 2 vé ciia (**) cho b rdi cong chting lai ta duge y = bs +dr. Vir,s21,tadidén x=as+cr>at+c va y=bst+dr>b+d. Bai 20. (Olympic Ton hoc Canada, 2001) Randy: "Chao Rachel, cdu da viét ra m6t phuong trinh bac hai thi vi that. Thé nghiém cia ching 1a gi nao?" Rachel: "Nghiém 1a hai sé nguyén duong. Mét trong hai nghiém 1a tudi cua té, nghiém kia 1a tudi céa thing Jim- -my em té.” Randy: "Thé thi cing don gidn thoi! Dé xem t6 cé thé tim ra tudi céa c4u va Jimmy khéng nao. Nghe cing ching khé my, vi cdc hé sé déu la sé nguyén ca. Tién thé, té cing tinh dude téng ctia 3 hé sé 1a mot sé nguyén 6. Rachel: "Nghe kha day. Réi, néi tudi ciia té dil.” Randy: "Tu tu, dé t6 dodn... Xem nao, té dodn tudi c4u, sau d6 * thay vao x trong phuong trinh... quy that, sao lai cho két qua -55 ma khéng 18 0 nhi?" Rachel: "Théi dep di, dé t6 yén nado!” B44 a) Hay ching minh tudi cia Jimmy 1a 2. b) Xac dinh tudi ctia Rachel. Randy: "Hi Rachel, that's an interesting quadratic equation you have written down. What are its roots? Rachel: "The roots are two positive integers. One of the roots is my age, and the other root is the age of my youn- -ger brother, Jimmy.” Randy: "That is very neat! Let me see if I can figure out how old you and Jimmy are. That shouldn't be too difficult since all of your coefficients are integers. By the way, I notice that the sum of the three coefficients is a prime number.” Rachel: "Interesting. Now figure out how old I am." Randy: "Instead, I will guess your age and subtitute it for x in your quadratic equation ... darn, that gives me -55, and not 0.” Rachel: “Oh, leave me alone!" a) Prove that Jimmy is two years old. b) Determine Rachel's age. Huéng dan: Goi R 1a sé tudi cha Rachel, J 1a sé tudi cha Jimmy. Dang bac hai ma Rachel dua ra 1a a(x - Rx -J)= vdi a 1a mét sé nao dé. Theo gia thiét, a 1a sé nguyén. Téng cdc hé sé 1a a~ a R+J)+ RI =a RWI 1). ax? a(R + J)x+aRJ, Vi téng trén 1a sé nguyén té nén hai trong ba sé a; R - 1; J - 1 phai bing 1. Ta lai cé R > J > 0, nén chic chin duge a =1,J=2,R-1 la sé nguyén té, va dang bac hai tré thanh (x— R)(x—2). Mat khdc, theo lai déi thoai, dang bac hai nhan gid tri - 55 = - 5.11 voi sé nguyén x nao dé. Vi R > 2 nén nhan tu thi nhat, (x — R), phai bang 1. Ta cé bén trudng hep nhu sau: ©) x-R=-55 va x-2=1, suyrax=3, R= 58. -Il va x =5,suyrax=7,R = 18. va x~2=11, suy rax = 13, R = 18. 7, R = 58. Do R-1 la sé nguyén t6, ta loai 2 trudng hop sau, vay R =18 va J =2. ~I va x-2=53, suy rax 345 Phu luc IT DE THI VA HUONG DAN GIAI IMO 2001, 2002. (Bé sung cho Lin tai ban thi hai) DIEU LE CUA CUOC THI OLYMPIC TOAN QUOC TE (IMO) DANH SACH CAC NUGC CHU NHA DANG CAI IMO TU 1959 DEN 2004 THANH TICH CUA DOL TUYEN VIET NAM TAI CAC Ki THI IMO TU 1974 DEN 2002 DE THI VA HUONG DAN GIAI IMO LAN THU 42 (2001) IMO LAN THU 43 (2002) (B6 sung cho lan tdi bdn thit hai) 424 IMO, Washington, D.C, USA 8-9 July, 2001 Bai 1. (2001) Cho tam gidc nhon ABC co O 1a tam vong tron ngoai tiép. Goi P 1a chan dudng cao ha tit A xuéng canh BC. Gia sitring BCA > ABC + 30°. Ching minh CAB+ COP < 90°. Let ABC be an acute-angled triangle with circumcentre O. Let P on BC be the foot of the altitude from A. Suppose that BCA > ABC + 30°. Prove that CAB+ COP < 90°. Huéng dan: Goi D 1 mét diém trén dudng tron ngoai tiép tam gidc ABC sao cho AD song song véi BC. Ta cd CBD =BCA, do d6 ABD> 30°. Suy ra ring AOD> 60°. Bay gid, gid sir Z Ja trung diém AD va Y JA trung diém BC. Khi dé, AZ > z, véi R 1a ban kinh dung trdn ngoai tiép tam gide ABC. Nhung do AZYP 1a hinh chit nhat nén ta cé AZ = YP. Ta khong thé cé O tring véi Y, vi néu ngusc lai, géc A sé bing 90° va tam gidéc ABC khéng nhon nhu gid thiét. Do vay: 347 oP> YP 23. Mi PC = ¥C- YP PC. Suy ra, ta cé6 COP< OCP. Goi CE 1 duéng kinh cia dudng tron ngoai tiép tam gidc ABC, ta cé OCP=ECB. Nhung ECB=EAB, EAB + BAC = EAC = 90°, vi EC 1a dudng kinh, nén ta duge COP + BAC < 90°. Cach khée: Dat a =CAB, B =ABC, y= BCA va 5 =COP. Goi K va Q lin lugt la cdc diém d6i xing cha A va P qua trung truc cia BC. Goi R 1a ban kinh dudng tron ngoai tiép ca AABC. Thé thi OA = OB = OC = OK=R. Hon nita, ta c6 QP = KA vi KQPA la hinh chi nhat. Bay gid, dé y ring: AOK = AOB- KOB= AOB - AOC= 2y - 28> 60°. Tu dé, va do OA = OB = R, ta suy ra ring KA > R va QP > R. Cho nén, bang cach sit dung bat dang thie tam gidc, ta cé: OP +R=0Q+0C>QC=QP+PC > R+PC. Suy ra OP > PC, va nhu thé, trong ACOP, ta cé PCO> 8. Bay gid, do a= ; BOC = 5180 - 2PCO)=909-PCO, ta suy ra duge a +8 < 90°. Bai 2. (2001) Cho cac sé thuc ducng a,b,c. Ching minh ring Prove that a b c = ee Va? +8be Vb? +8ca Ve? +8ub for all positive real numbers a, b and e. Huéng dén: Truéc tién, ta chu y bat ding thitc Cauchy - Schwarz sau: Bay gid, ta dat 7 =o Oe Va? +8be Vb? +8ca ve? + 8ab e 4 yi =aal, yZ=bd', YS =ee', va suy ra x1), =, X7¥7 =b, x3y3 =c. Khi dé, tir bat ding thiic Cauchy - Schwarz ndi trén ta dudc: (xf 43 +3)? + 93 +93) 2 Oy tay +2593)? ab ec. (a+b+e)* Ca be : oe : Tiép dén, ta lai dat X,=Va, X,=Vb. X3=Ve, ¥ =Vaa', Y= V6.5, ¥3= Veo’, va cing dp dung bat dang thtic Cauchy - Schwarz dé cé: (XK + X2¥o + X35)? S(XP XG + XD HB +B) © (aa'xbb'4cc') < Jat b+ eVaa® +56 400" . Céec két qua (1) va (2) cho ta: abe, arbre) ae Va+b+cvaa?+bb' +0 3 abe (at+b+c)? eae ae ee aR eC aa?+bb? +6 Mat khac, ta lai co qa) (2) (3) 349 3 (at+b+c)2 > Vaa"?-+bb? +c? . (4) ‘That vay, (4) tudng duong véi (a+ b+c)° > aa?+bb+cc, tit bat ding thiic nay, thay a’. ',c’ vao réi khai trién va bién déi ta due bat ding thiic tuong duong 1a 3(ab? + ac? + ba? + be? + ca? +cb) > Babe © alb—c + (ca)? +ce(a—b) 20, bat ding thiic hién nhién sau cing nay chiing t6 (4) ding. Sai hét, (3) va (4) cho ta abc =+—+-—210 ———— oa ape Te +8be Vata +8ca ‘aa A dé thay du dang thitc xdy ra khi a =b =c =1. Chi §: Cung la dp dung Bat déng thite Cauchy, nhung ta c6 thé trinh bay theo cdch sau: Truéc hét ta sé ching minh ring 1 3 Team 2 a hay Bbe | a+b4c3 I 1 1 2 (a3 +b3 +03)? >a3 (a? + Bbc). Bat ding thie Cauchy cho ta: por 1 foie itis (a3 +63 +03)?-(a3)? =(63 +c3)@3 +a} +63 +c) gee 2 ie ee > 2b3¢3. 4a3 633 = 8a be. Nhu vay: Hoe ! 2 2 (a3 +63 +03)? > @3)? +8a3be=a3(a” + 860), tir dé ta c6: 1 a eo a+b3+c3 Tung tu, ta cé: 350 3 So ¢ a 1 +¥° + 8ab ve : B+ +03 Cong cac bat ding thitc nay lai thi duge: b + bo? + 8ca Bai 3. (2001) Hai mudi mét ni sinh va hai muoi mét nam sinh cing tham gia mét ki thi Todn hoe. Biét ring: - Méi thi sinh gidi duoc nhiéu nhit 6 bai toan. - Véi méi cép bat ki gdm mét nd sinh va mét nam sinh, c4 hai ngudi nay sé gidi dugc it nhdt mét bai todn. Chting minh ring tén tai mét bai ton sao cho cé it nhdt 3 nit sinh gidi duoc bai nay, va cing cé it nhat 3 nam sinh gidi duge nd. Twenty-one girls and twenty-one boys took part in a mathematic- -al contest. - Each contestant solved at most six problems. - For each girl and each boy, at least one problem was solved by both of them. Prove that there was a problem that was solved by at least three girls and at least three boys. Huéng dan: Ta sé ding cdc ki hiéu sau: G 1a tap hgp cdc nit sinh cudc thi va B 1a tap hgp cac nam sinh, P 1a tap hgp cac bai ton, P(g) 1a tap hep cae bai todn duce gidi bdi_geG, va P(b) 1a tap hgp cdc bai ton duge giai bdi b € B. Sau cing, G(p) la tap hgp cdc nif sinh gidi bai toan p € P va Bip) 1a tap hop cdc nam sinh giai bai toan p € P. Theo cdc ki higu nay, ta 6 véi moi g « Gvab € B: @ IP@| < 6 1PO)| <6, @)PE)rPO)+9, 351 6 day, ki hié ‘Ta mu6n chitng minh rang tén tai p € P thoa man |G(p)|> 3 |.| chi sé cdc phan tit cua mét tap hop hitu han. val B(p)|> 3. Dé lam diéu dé, ta sé gid sit ring diéu nguge lai xAy ra va di dén mau thudn bing cach dém (theo hai cach) tat cd cdc bé ba (p. a, 2) sao cho p € P(g) 0 Pb). Véi T = {(p,q.r): p © P(g) 0 Pb) }, diéu kign (b) cho ta: ItTl= YS ]Pey.o Poy > iG].|B] = 21%, qa geGbeR GiA sit khéng c6 p € P nao thoa man|G(p)|> 3 val Be)! > 3. Ta bat ddu bang cach chi y rang: YiG@)| = YP} < 6iGl va YIBpy< 6|Bl. (2) peP geG peP Cha y: Dang thuc trong (2) cé duge bai ki thuat sau: Goi x(p, q) = 1 néu g giai bai toan p va (p,q) = 0 néu nguic lai, va thay déi tha tu cha dau tong trong >) x(q.p). Dat pePgeG P.=tpe P: [Gl 231}, P.=tpeP: 1G@)| <2}. Ménhdé: Y|G(pz1Gl,dods Yicpi 4 nén p € P, cho nén m6i ni sinh giai it nhat mét bai toan trong P_. Dé dé Yic@) > ial. @) peP_ Theo (2) va (3) ta c6é Yew] = Tic@) = Licw) < sial. pePy peP. pe? Cung thé, mai nam sinh gidi mét bai toan ma bai nay duge gidi béi it nb&t bén ni sinh, cho nén mai nam sinh gidi mt bai toan peP,. Nhu thé 7/B(p)| > !B(p)|, va sau d6, tién hanh phép tinh toan nhu per, trén hing vAch sit dung (2). 352 St dung ménh dé vita chiing minh trén, ta thay Itl= Y|G(p}-1B@ l= Y|G(p.1Be! + Y|G@).1 Be)! peP peP, peP_ <2 [Gp] +2 YIB(p < 101G] + 10] B] =20 x 21. pePy peP_ Diéu nay mau thudn véi (1), nén ta cé diéu phai ching minh. Cach 2: ‘ qua doi héi & dé toan ding trong trudng hdp 20 ni sinh va 20 nam sinh tham gia cuée | thi, néi ném na, bai todn dung cho mét khéi 20 x 20. ‘That vay, ta tao 20 hinh chi nhat, méi hinh 6 kich thude 2x 10 va danh sé 1, 2, ... , 20. Ta chia khéi 20 x 20 nay thanh 4 Khéi (mdi khéi 14 10 x 10). G bén trén va phia trai, ta dit nim ngang 5 hinh chi nhat 2 x 10, cing dit nhu thé véi bén dudi va phia phai. Con & bén trén, phia phai va bén duéi phia trai, ta dit méi khéi nhu thé gdm 5 hinh chi nbat 2 x 10 theo chiéu dung (xem hinh) . Bay gid, 6 méi hang sé gdm phan giao cia 5 hinh chit nhat dimg va 1 hinh chit nhat ngang. Néi cach khdc, méi hang chiia 6 sé khdc. nhau. Tuong tu nhu thé, méi cét cing chiia 6 sé khac nhau. Nhung mdi sé bat ki da cho hoac 1a sé nim trong 10 hang va 2 cét, hodc 1 ngude lai, vi thé khong 6 sé nao nim trong 3 hang va 3 cét. Diéu nay ching td ring bai ton cho mot khéi 20 x 20. | | Truéc tién, ta chi ¥ ring két ‘Tré lai trudng hop khéi 21 x 21, ta gid sti ring mét su sdp xép dude goi la chdp nhén due néu khong cé sé nguyén nao nim 6 it nhat 3 hang va ft nhat 3 cot. Ta t6 mau trdng mét 6 vudng nhé néu 6 nay chiia sé nguyén xudt hién 6 téi thiéu 3 hang, va t6 mau den mét 6 vung nhd néu 6 nay chia sé nguyén xudt hién 6 chi 1 hode hai hang. Ta dém cic 6 vuéng mau tring va mau den. AA Toa boc ct 353 MGi hang cé 21 6 va chiia nhiéu nit 1a 6 sé nguyén khac nhau. Ta c6 6 x 3 < 21, vi vay, trong mdi hang phai cd chita mét sé ma sé nay xudt hién t6i thiéu 3 lan; suy ra sé dé nim 6 qua lim 14 2 hang. Do dé, c6 téi da 5 s6 khac nhau trong méi hang ma mdi mot trong nhiing sé nay xuat hién é it nhat 3 hang. Mai sé nguyén nay cé thé xudt hién téi da 1A da 5 x 2= 10 6 mau tring trong mét hang. Diéa:nay ding cho tat cd cdc hang, do vay téng cong cé téi da 210 6 dude t6 mau trang. 2 14n trong cing mét hang, vi thé cd Tuong tu, trong bat ki mét cdt da cho, cé nhiéu nhat 6 sé nguyén khac nhau, va do 46, phai tén tai mét sé xudt hién it nhat 3 lan. Vi vay, c6 nhiéu nbat 1a 5 sé khéc nhau ma mii sé xudt hién 6 tdi da 2 cot. Mai sé nhuw thé cé thé chiém nhiéu nhat 2 6 vudng trong mii cét, do dééntong mii cét, co nhidu nhat 5 x 2 = 10 6 dude t6 mau den. Diu nay dig cho tit cd cdc cét, do vay tang cong cé tdi da 210 6 duce td mau den. Dén day, ta cé didu mau thudn, vi 210 + 210 < 441. Nhén xét: Trong céch gidi thi hai ndy, bai todn vé céc thi sinh dé chuyén vé bai todn t6 mau trén nhiing 6 vuéng. Cach gidi nay mang tinh téng quat hon, vé néu theo doi ki ting diém mét trong li luén, céch gidi dep hon vd dé hiéu hon. Déng khdc, tt li ludn dé, ban doc c6 thé téng qudt hod bai todn nhu sau: Trong mét khéi 4N+1 x 4N+1 dude danh sé (nhu trén), sao cho trong mdi hang va méi cot cé nhiéu nhat 1a N+1 sé khdc nhau, sé tin tai mét sé nao dé xudt hién 6 it nh4t 3 hang va it nhat 3 cot. Tuy nhién, vé mat téng quat, diéu néy khéng dung cho truing hop 4N x 4N (N la sé tue nhién). Bai 4. (2001) Cho n 1A sé nguyén Ié Ién hon 1 va goi ky, kp,...,k, 1A cae so nguyén. Véi méi mét trong n! hoan vi a=(a,@),..., 4,) cua tap {1, 2, ... nj, ta dat. S(a) = ay x ky tay x ky t...+4, xky- Chiang minh ring tén tai cac hoan vi a, b cha tap hop {1, 2, ..., n} sao cho n! 1A mét uéc s6 cua S(a) ~ S(b). Let n be an odd integer greater than 1, and let ky, ky, ..-s ky be Db Telarc ede given integers. For each of the n! permutations a=(d,, 4), ... dy) of (1, 2, ..., nl, let S(a) =a, xky tay ky +...44, Xk, Prove that there are iwo permutations a and b, such that n! is a divisor of S(a) ~S(b). Huéng dan: Goi XS(@) 1a téng cia S(@) khi a chay trén tat cn! hoan vi a=(a,4,...,4,). Trong S(a), ky dude nhan véi mai h « {1, ..., n} cd thay (n - 1)! ln, mét ln v6i méi hoan vi cha {1, ..., n} trong d6 a, = n, Nhu thé hé so cia k, trong ES(a) 1a (n-I (+ 24... +n) = (n+ DI. Diéu tudng tu cing dung véi moi k;, cho nén ta co: 2s(a)= C2 9. a) 1 Néu n! khéng phai la ude cha S(a)—S(b) véi moi hoén via + b, thi mdi S(a) phai c6 n sé du khac nhau (mod n!). Vi c6 n! hon vi, nén nhiing 86 du nay phai chinh la 0, 1, 2, ..., n! - 1. Do dé =S(a) = aoe (mod n!) . (2) Két hop (1) va (2), ta 06: Gs! Sk (GEE a 8) ae 2 Bay gid, vin le, vé trai cha (3) sé dng du véi 0 theo mod n!, trong khi véin > 1 thi vé phai khong déng du véi 0 (n! - 11a 18). Voi n >1 va lé, ta gap phai diéu mau thuan. Bai 5. (2001) Goi ABC 1a mét tam giac c6 BAC = 60°. Goi AP a phan gidc géc BAC va BQ la phan gide géc ABC sao cho P é trén BC, Q 6 trén AC. Gia st AB + BP=AQ+ QB. Hi sé do cua cac géc tam gidc ABC ? Ina triangle ABC, let AP bisect BAC, with P on BC, and let BQ bisect ABC, with Q on CA. It is known that BAC = 60° and that AB + BP =AQ + QB. 355 What are the possible angles of triangle ABC ? Pr Huéng dan: Ki higu cée géc cia ABC béi «= 60°, B vay. Kéo dai AB tdi P’ sao cho BP’ = BP va dung P” trén AQ sao cho AP” = AP’. Thé thi BP’P 14 mét tam gidc cén c6 géc 6 day bing B/2. Do AQ + BP’ = AB + BP’ = AB + BP = AQ+ QB, suy ra rng QP” = QB. Vi AP’P” 1a mét tam giée déu va AP la phan gidc géc A, nén ta cd PP’= PP”. B6 dé: Cac diém B, P, P” thang hang, do dé P” tring véi C. Chitng minh. Gia sit nguge lai ring A B Pp BPP” 1a mét tam giac khong suy bién. Ta c6 ZPBQ = ZPP’B = ZPP"Q = pi2. Hinh vé bén cho thay P va Q é cing mét phia déi véi BP”, ngoai ra, con trudng hgp PG phia bén kia cia BP”. Q Trong mdi trudng hgp, gia thiét BPP” 8 khéng suy bién déu din dén BP = PP” = PP’, nhu thé dua dén két Juan rang BPP’ 1a mat tit gidc va rot cuc di d&n nghich Ii: B/2 = 60°, cho ta a +B =60° + 120° = 180°. Do dé, cdc diém B, P, P” thing hang va P” = C nhu dai héi. Vi tam gidc BCQ cfin, nén 120°- B=y =/2, ta suy ra B= 80° va y = 40°. Vay ABC la tam giac cé cdc géc 14 60", 80° va 40°. Céch 2: Khéng mit tinh ting quat, cé thé cho AB = 1. Dat géc ABQ = x. Chu y rang bay gid ta cé thé gidi cho hai tam gidc APB va AYB. Dac biét, dang Dinh li ham sin, ta c6 _ sin 60° © sin(120° ~ x)" Su dung cac cong thiic higng gidc nhu sin(a + 5), v. v..., va dat s = sinx, c = cosx, ta dude: 203 s%e- 4sc-2V3 c3 + 2V3c? + 6sc - 2s - V3 =O hay - V3 (4c - 2c - 2¢ + 1) = 2s.(2e? -3e + 1) = 2s.(2e- 1c - D. i Tu dé c=— ho: 2 oe -V3 (2c? - 1) = 2s.(e- 1). @) Néu c= 3, ta duge x= 60° hay gée B = 120°, Nhung trong triding hgp nay cdc canh déi dién cua hai dinh A va B song song nhau nén tam gide ABC suy bién. Do vay, ta cé (*), binh phuong hai vé cia (*) va dé y ring s” = 1-c?, ta nhan duge: 16c' - 8c? - 12c” + Be - 1=0. Vé trai ding thitc nay cé nhan tit 2c - 1. Do dé ta cé: 8c3 - Ge + 1=0. Nhung vi 4c" - 3e = cos3s, nén ta duge costs = > Tw dé ta duce cdc nghiém x = 40°, 80°, 160°, 200°, 280°, 320°. Nhung ta phai c6 x < 60° dé tam gidc ABC khong suy bién. Suy ra goc B bang 2x = 80°. Bai 6. (2001) Cho cdc s6 nguyén duong a, b, c,d véi a>b>c>d>0. Gia st ac + bd =(b+d+a—c)(b+d-atc). Chiing minh ring ab +cd khong phai 1a sé nguyén té. Let a, 5, c, d be integers witha >b >c>d>0. Suppose that ac+bd =(b+d+a-—c)(b+d-atc). Prove that ab+cd is not prime. Huéng dén: Dang thiic da cho tuong duong véi @ acre =b? +bd ta’. a Xét taf gide ABCD voi AB =a, BC =d, CD =b, AD =e, BAD =60°, BCD =120°, BD?= a? —ac+c? =b? +bd +d’, 357

rps 3}. Chi y ring tit gia thiét taco AUB=V,V,. Hon nia, ta ciing cb ANB #9. ' 1 Chon ze AMB. Ly Xe[L , cing vay, ta chon diém Ye[L > P,]. Khi d6 d(X, Y) <1 va phan dutng gdp khuc gida X va Y sé cé gid tri bé nhat 1a 198. 237 Bai 88. (1983) Tim tat cd nhimg ham f :(0, 0) —> (0, 0) thod man déng thdi hai diéu kién sau: i) £(xf(y)) = yF&), Vx,y €(0, 0); ii) f(x) >. khi x 900. Find all functions f defined on the set of positive reals which take positive real values and satisfy: D fOf(») = WH), Vx.y EO, 0); ii) f(x) > © while x0. Huéng dan: Ta c6 £?(y) = yf(I), va do f(1)# 0 nén fa song anh. Ti dé, ton tai y dé fly) = 1. Két hop diéu nay véi (i) khi cho x = 1 ta dude f(.1)=f0)=yf(), va ti f(1) > 0, ta cé y = 1, suy ra f(1) =1. Cho y =x trong (i) ta cb f(xf(x)) = xf(x) véi moi x > 0. Suy ra xflx) la diém bét déng cia f. Bay gid, néu ca x va y 1a cde diém bat déng cua f thi cing tu (i) ta due flxy) = yx, do dé xy ciing la diém bat déng. Nhu vay, tap cdc diém bat dong la déng véi phép nhan. Hon nita, néu x la diém bat dong, ta c6 1=fW)= iG £00] . (2) x x nén (2). nghia 1a | cing 1a diém bét dong. Néi cach khéc, tap x x 7 cac diém bit dong déng vdi phép nghich dao. Nhu vay, ngoai diém 1 ra, néu f cé diém bat dong khdc, thi hoc diém nay Ién hon 1, hoic nghich dao cua né 1én hon 1, do dé, luy thita nhiéu ln cua diém Ién hon 1 nay (theo trén, cing 14 diém bat déng) sé Ién tuy y, diéu nay tréi véi diéu kién (ii). Vay 1 1a diém bat dong duy nht cua f, va do xf{x) 1a diém bat déng vdi moi x > 0 nén ham duy nhat thod man didu kign eda bai ton 1a f(x) = :. Ghi chi: +x la diém bdt déng cia ham f néu f(x) = x. G chuong trinh Toan chuyén nganh bac Dai hoc va sau Dai hoc, ban doc sé thudng xuyén gip 238 khai niém nay. Diém mau chét céa ching minh trén 1a dua vao cac diéu kign dé bai dé ching td ham f cin tim cé diém bat déng duy nhdt. Mot dinh li déng vai tro hét stic quan trong trong Li thuyét Phuong trinh vi phan, Phuong trinh tich phan, Bat ddng thic bién phan, ..., dé 1a Nguyén li dnh xa co Banach, nguyén li nay néi rang trong mét sé diéu kién (khd r6ng rai), ham f sé cé duy nht diém bat dong (xem, ching han, Phan Diic Chinh, Gidi tich ham, DH va THCN, 1976). + Ta néi tap S la déng déi véi phép todn * néu Vx, yeS,x*yeS. Bai 89. (1983) Cho cae sé nguyén dugng a, b, c sao cho khong cé 2 sé nao trong chung cé uéc chung 1én hon 1 (nguyén té cing nhau). Ching minh ring 2abe —ab—be—ca 1a sé nguyén Ién nhdt khong thé biéu dién duge duéi dang xbe + yca + zab, vdi x, y, z la cac s6 nguyén khong am. Let a, b and c be positive integers, no two of which have a common divisor greater than 1. Show that 2abc—ab-bc-ca is the largest integer which cannot be expressed in the form xbe + yca+zab, where x, y, z are non-negative integers. Huéng dan: Ta sé chimg minh ring: a) 2abe —ab—be—ca khong thé biéu dién duéi dang xbe + yca + zab, véi x, y, z 14 cdc sé nguyén khéng am. b) Néu N 1 sé nguyén sao cho N > 2abe — ab be —ca thi N c6 thé dude biéu dién duéi dang xbc+yca+zab, véi x, y, z la cdc sé nguyén khéng am. Ching minh a): Gia sit tén tai cdc sé nguyén khong am x, y, z sao cho 2abe—ab—be~ca = xbe+yca+zab. Khi dé, vi vé trai chia hét cho c nén & vé phai, s6 hang (z + 1) ab phai chia hét cho c. Nhung theo gia thiét thi cdc c&p sé (a, c) va (b, c) khong cé ude chung nao Ién hon 1 nén suy ra z + 1 chia hét choc. Doz + 1 #0 vaz + 1 chia hét choc nén z+ 12 c. Chimg minh tuong tu tacdz+12>¢,x+12a, y+12b. Tir dé, ta c6 diéu mau thudn. Chiing minh b): Bé dé: Gid sita va b 1a hai sé tu nhién va nguyén té cing nhau. Khi dé téng 6 dang mb + na, trong dé m va n Ja cae sé nguyén khong 4m khéng lén hon a - 1 va b - 1 tuong ting, sé nhén hét moi sé du cé thé cua phép chia cho ab. Chuing minh: Khi chia cho ab ta c6 sé du khéc nhau. Vi m nhan a gid tri va n nhan b gia tri nén tap hop cac téng dang mb + na sé gsm ab phan ti. Vay chi con phai ching minh ring néu cdc c&p cé thi tu (m,, n,) va (m,, n,) 1a khac nhau thi cdc s6 rajb+nja va m3b+n a khi chia cho ab, sé cho cac sé du khdc nhau. Ban doc dé dang chtmg minh diéu nay bang phan ching, véi chu y a va b nguyén té cing nhau. Ap dung bé dé trén, vi hai sé ab va c nguyén té cing nhau nén ta suy ra ring cdc sé dang kbe + mac nhan hét moi sé du cha phép chia cho ab, véi 0 2abe — ab—be—ca thi sé z, 1a khong 4m. That vay, do 0kgbc +moac (Didu nay hién nhién trong trudng hop N > abe - ab - be - ca. Néu 2abc -ab—be—ca < N 2abe—ab—be-ca déu biéu dién duoc dudéi dang N = k,bc + m,ca + z,ab, trong dé k,, m,, z, 1a nhing sé nguyén khéng 4m. Bai todn dén day da dugc gidi quyét. Bai 90. (1983) Cé thé chon dude hay khéng 1983 sé nguyén duong khdc nhau sao cho tat ca cdc sé dy déu nhé hon hode bing 100000 va khéng cé bat cit 3 86 nao trong chung tao thanh mét sé cong? Is it possible to choose 1983 distinct positive integers, all less than 240 or equal to 10000, no three of which are consecutive terms of an arithme- -tie progression? Huéng dén: Ta sé xAy dung mét tap hop T 1én hon, gdm 2047 sé nguyén, tat cA déu nhé hon 100000 va khéng cé bat ci 3 sé nao trong ching tao thanh mot cdp sé cong. Goi T 1a tap hop bao gém cdc sé nguyén duong dude biéu dién theo hé co sé 3, sao cho mii sé cé nhiéu nhat 11 chit s6, mdi chit s6 1a 0 hodc 1. Vay T c6 2'' —1 = 2047 > 1983 86 nhu thé, sé Ién nat 1a 11111111111 = 1+ 1.34 13° +... + 1.3" = 88573 < 100000. Bay gid, gid si cé x, y, z eT sao cho x +z = 2y. Sé 2y chi gim cdc chi sé 0 va 2, tit dé ta suy ra dude x = y = z. Diéu nay c6 nghia khong c6 bat ci 3 sé nao trong T tao thanh mét cdp sé cong. Bai 91. (1983) Cho tam gidc déu ABC va goi S 1a tap tat cd nhing diém nim trén cdc canh AB, BC, CA (gom ca 3 diém A, B, C). Hay xdc dinh xem diéu nay cé ding khong: vdi moi phan hogch S thanh 2 tép con réi nhau thi it nhdt mét trong hai tép dé sé chita 3 dinh cia mét tam gidc vuéng? Let ABC be an equilateral triangle and S the set of all points contained in the three segments AB, BC and CA (including A, B and C). Determine whether, for every partition of S into two disjoint subsets, at least one of the two subsets contains the vertices of a right- angled triangle. Huéng dan: Gia str nguoc lai, tn tai S 14 mét phan hoach nhu da néi é dé bai ma khéng cé tap nao dudc phan ra cé chifa 3 dinh cia mét tam gidc vuéng. Té mau tat cA cdc phan tu cia hai tap dude phan ra véi hai mau a6 va xanh tuong ting. Gia suf ring mdi tam gidc vuéng déu c} hai mau. A Xét tam gide DEF sao cho spaces E BC CA AB 3 nhu hinh vé. Dé thay cdc tam gide DEC, EAF, FBD vuéng tai D, E, F tuong ting. B Cc Khéng mat tinh ting quat, ta gid sur F va E 241 wertearecedett 6 mau dé. Khi dé, toan bé canh AC, tri diém E, c6 mau xanh. Suy ra D c6 mau dé. Li ludn tuong ty, ta cing c6 cd 3 diém D, E, F déu c6 mau xanh. Diéu mau thudn trén cho ta két ludn cia bai toan. Bai 92. (1983) Cho a, b, ¢ 1a dé dai cdc canh ca mét tam gidc. Ching minh ring a7b(a—b)+b2c(b—c) + c7a(c -a) >0. Hay xdc dinh xem khi nao ddu ding thtic xdy ra. Let a, b and c be the lengths of the sides of a triangle. Prove that a’b(a-—b) + b’c(b—c) +c?a(c—a) > 0. Determine when equality occurs. Huéng dan: Dita=y+z,b=z+x,c=x+y. Lic dé, diéu kién cdc canh cla mét tam gidc ddi hdi x, y, z > 0. Sau nhiing bién déi cdn thiét, bat ding thiic cdn chimg minh trd thanh: xy? + yz? + zx? > xy2(x-+y +z). Ap dung bat ding thiic Cauchy ta duge: (xy? + yz? + 2x7 )(x + y +z) > xyz(x ty +z), tit dé, suy ra bat dang thtic can ching minh 1a ding, déu ding thic xdy ra khi va chi khi BE ae, Zz x Tré lai véia=y+2z,b=z+x,c=x+y, taco déu ding thiic xdy ra khi va chi khi a = b = c, tiic 14 tam gidc ABC déu. Bai 93. (1984) Ching minh ring 0< yet ant xy—2ayes trong d6, x, y, z 1a cdc sé thyc khéng 4m théa man diéu kién x + y +z = 1. Prove that 0< yz+zx+xy—2xyz < 7/217, where x, y and z are non-negative real numbers satisfying x + y +z = 1. Hung dan: Ta c6: (1—2x)(1—2y)(1— 2z) —2(x+y +z)+ 4(yz+ zx + xy) — 8xyz =4(yz+zx + xy)—8xyz-1, do gid thiét x + y + z = 1. Tirdé ta suy ra: 242 1 tance YZ+ 2x + xy — 2xyz= ja —2x)(I-2y)(1 - 22) + ‘. Mat khac, theo Dinh li trung binh s6 hoc - hinh hoc (xem phan hién thie bé trg) ta cing c6: (1-28) 2y (122) <2 2x 41-294 1-22) = 1 2807, Do vay, ta suy ra yz+zx + xy —2xyz <—.— = beetle O84 07 7, Ngoai ra, do gid thiét x + y + z= 1, ta cé: ye + 2x + xy —2xyz = xy(1~ z) + x2z(1- y)+ yz 20 nén suy ra diéu phai chting minh. Bai 94. (1984) Tim hai sé nguyén dugng a, b théa man 2 diéu kién: i) ab(a +b) khéng chia hét cho 7; ii) (a+b)! —a7 —b’ chia hét cho 7’. Find one pair of positive integers a, b such that ab(a+b) is not divisible by 7, but (a+b)! -a’ —b’ is divisible by 7”. Huéng dén: Ta cé: (a+b)7 —a7 —b7 = 7a% + 21a%b? + 35a4b? +35a°b4 + 21a7b* + 7ab® = Tab(a? + 3a‘b + Sa>b? +5a7b? +3ab* +b) = Tab(a + b)(a* + 2ab +3a7b? + 2ab? +b") = Tab(a + b)(a? +ab+b?)?. Tu day, két hop cdc diéu kién (i) va (ii) ta c6 a? +ab+b? chia hét cho 7°. Ta c6 (a+b)? >a? +ab+b? > 7° = 343, suy ra: a+b2[v343]=18. ‘Thi cho a = 1, b = 18, ta thay hai diéu kién cia dau bai duge théa man. Vay c4p sé nguyén duong can tim 1a (a, b) = (1, 18). Bai 95. (1984) Trong mat phing, cho 2 diém phan biét O va A. Véi moi diém X (khac O) trong mat phang nay, ta ki hiéu a(X) 1a sé do cua gc gitta OA va OX, tinh ti OA, theo chiéu ngudc kim ding hé, 0 < a(X) < 27. Goi aX) C(X) la dutng tron tam O ban kinh OX + —— Mai diém trén mat phing dugc t6 bing mét trong mét sé hitu han cae mau s&c khac nhau. Ching minh ring tén tai mét diém Y ma a(¥) > 0 sao cho tén tai mét diém trén bién (trén chu vi) cua dudng tron C(Y) c6 cung mau véi Y. Given points O and A in the plane. Every point in the plane is colored with one of a finite number of colors. Given a point X in the plane, let a(X) be angle AOX, which is measured in radians in the range [0, 2x) counter-clockwise. (X) The circle C(X) has center O and radius OX + x” Prove that we can find a point Y, not on OA (a(Y) > 0), such that its color appears on the circumference of the circle C(Y). Huéng dan: Gia str cdc diém cia mat phdng dugc t6 béi n mau. Trén mét dudng tron bat ki, c6 thé cé dén 2" —1 t hop mau khac nhau. Suy ra ring, néu ta lay 2" dudng tron ma méi dudng tron nay déu cé tam O va ban kinh bé hon 1, thi phai c6 2 trong sé nhing duéng tron dé cé cing mét sé mau. Ta goi r, s 1a ban kinh hai du@ng tron nay (0 3. Goi p 1a chu vi da gidc dé. Ki hiéu nhu thudng Ié, [ x ] la phan nguyén cia sé thuc x, hay ching minh ring: Let d be the sum of the lengths of all the diagonals of a plane convex polygon with n > 3 vertices. 244 Let p be its perimeter. Prove that: a llet n-3< < p l2jl 2 where [x] denotes the greatest integer not exceeding x. Huéng dan: Xét dudng chéo AX, ta goi B la dinh ké tiép cuia da gidc ké tit A, ngude chiéu kim ding hé va Y la dinh ké tiép cia da gidc ké tir X, nguge chiéu kim déng hé. Khi dé, ta goi K 1a giao diém cia AX va BY. Vi AK + KB > AB va KX + KY > XY nén AX+BY>AB+XY. — (*) Git diém A cé dinh, cho diém X thay déi qua cdc dinh, ta thdy (n - 3) trutng hgp xay ra, nghia la ta thiét lap duge tat cd (n - 3) bat ding thie nhu (*). Trong qua trinh thiét lap cac bat d&ng thiic nay, 6 vé trai, mai duéng chéo xudt hién 4 lan, con é vé phai, méi canh xudt hién 2(n - 3) lin, do dé, khi ldy téng cdc bat ding thtic nay vé theo vé ta dusc: 4d > 2(n-3)p <> n-3 * Goi d, 1a téng céc dutng chéo néi hai dau mut cia duing gép khic gim k canh lién tiép cua da gidc, ta cé: dy at+d>b+t+c. Vaym>n. Bay gid, ta c6 b(2” —b) = be = ad = a(2™ —a), suy ra 2"(b-a2™") =b2" —a2™ =b? -a? =(b-a)(b+a), va do dé (b—a)(b+a) chia hét cho2". Mat khac, néu cé6 b+a=0 (mod 4) thi b =0 (mod 2), diéu nay mau thuan, vib la sé 1é. Do vay, ta suy ra, hoc (b - a) chia hét cho 2""!, hoa (b + a) chia hét cho 2""!. Ta goi x 14 mét trong 2 86 (b + a) va (b - a) thi On. Trong trudng hop (a), do diéu kién thi hai cia dau bai, m, cé cing mau véi |m, -k|=m,_,. Trong trudng hgp (b), ciing do diéu kién thi hai ma m, 6 cing mau véi |m, ~k|=n-m,_;, va do diéa kién thit nhit, né lai cimg mau véi m,..;. Tom lai, tat ca cdc phan té cha M déu c6 cing mau. Bai 99. (1985) Cho da thitc véi hé sé nguyén P(x) = ag ayx +. tayx®. Taki hiéu sé cdc hé sé 1é cia da thite nay 1a o(P). Véi i = 0, 1, 2, 3, ..., ta dat Q;(x) = (1+x)!. Ching minh ring néu i, ,i2,...,i, 14 cdc 85 nguyén théa man diéu kién 0 m. Dat A=Q) +Qi, +Qj, +--+ Qi, (I+ x)" = Qi + Qi FQ, t+,» suy ra A va B 6 bac nhé hon m. Lic dé: 0(Q) = 0(A +(1+x)™B) =0(A+B+x™B)=0(A+B)+0(B). Bay gid, ta cd o(A-B) > 0(A—B+B)=0(A), béi vi mét hé sé cia Ala lé khi va chi khi mét trong hai hé sé tuong img cua A - B va cua B 1a 1é. Nhung ta cé 0(A —B) = 0(A +B), vi cdc hé sé tuong img cua A- B va cia A + B hoic bing nhau hodc cing chin cing 1é. Tit dé 0(A +B)+0(B) 2 0(A). Ta eiing c6 o(A) > 0(Q;,) theo gid thiét quy nap. Tom lai, bat ding thiic ciing dting cho truéng hap 2. Bai 100. (1985) Xét tap hop M gdm 1985 sé nguyén dugng phan biét, sao cho khéng 248 c6 sé nao cé ude 86 nguyén té 1én hon 23. Ching minh ring M chia mét tap con gém 4 phan ti ma tich cia 4 phan tt nay Ia lay thiia bac 4 cha mét sé nguyén. Given a set M of 1985 distinct positive integers, none of which has a prime divisor greater than 23, prove that M contains a subset of 4 elements whose product is the 4th power of an integer. Huéng dan: Vi chi cé 9 sé nguyén t6 khong Idn hon 23 nén mdi mét phan tit trong 1985 phan tit k cua tap hop M cé thé phan tich dugc thanh thita sé nguyén té trong dé cé nhiéu nhat 1a 9 sé nguyén té phan biét: k= pt! py?--Py?, “ trong d6 k; 20 va k; nguyén. Véi méi sé hang cia M ta gan tuong ing véi mét vector (x, X», ...., Xe) trong dé: x, = 0 néu sé mii k; eda p, trong (*) 1a chin va x, = 1 néu k, 14 16. Nhu thé ta cé dude 2° vector phan biét. Theo nguyén tic Dirichlet, moi tap hdp con cia 2° + 1 phan ti cia M chita it nhdt 2 sé nguyén phan biét, ching han a, va b,, véi cing vectd lay thita. Suy ra tich ca chting 1a mét sé chinh phugng: a,b, =c?. Khi ta léy di mét cp nhu thé tir tap hop M thi cdn lai 1985-2 >2° +156, dp dung lin nita nguyén tc Dirichlet va tiép tuc ldy di nhimg c4p nhu thé cho téi khi it nhat con lai 2°+1 sé trong M. Vi 1985 > 3(2° +1) = 1539 nén ta cé thé ldy di 2° + 1 cp a,, b, va sé c6 1985—2(2? +1) =959 > 2? +1=513 sé cdn lai trong M. Bay gid ta nhin lai 2° + 1 cp ldy di va ly can bac hai cla ¢, (véi ¢, 1a tich ajb; =c? céa ching), tric la ¢; = Jajb; . Sé c, kkéng thé chifa cdc thita sé nguyén té khéc py, Pa,..--P2, cho nén it nhdt cé mét c&p ¢, ¢; vai cing vects lay thita, va ce, = d?, dla sé nguyén bat ki. Suy ra d‘ =c?c} =ajbja;b;, voi aj,b;,a),b; bat ki trong M. Bai 101. (1985) Véi x, 14 mot sé thuc tay y, ta xdy dung day 86 x1,X,...,Xq > bing cach dat x, ral +4), moi n = 2, 3, .... Chimg minh ring n 249 tén tai duy nhat mét gid tri x, sao cho 01 véi moi n> 1. Dé thay S, (x) cé cdc hé sé khong 4m, do dé day sé S,,(x) tang thuc su trén doan [0, 1]. Ti dé, tén tai duy nhdt cdc nghiém a, ,b, thda man: 1 Sn(@n)=1- = Sn(ba) =I. Ta cd: Sp4y(a,)=Sq(q){ Sy(aq) +2 |=1-2 <1-—L, do 48 eet a hot n\4n nen n = a ntl’ ay t, do 46 b, >b,4;- Nhu vay, {a,} la day ting, con {b,} 1a day gidm va tat cd cde a, déu nhé hon tat ca cdc by. Tw dé, ta c6 thé tim duge sé x, sao cho x, Ién hon tat cd cdc a, va nhé hon tat ca cdc b, . Suy ra ring véi moi n ta c6: 1d Xs. Cuéi cing, hién nhién n n x,, > 0. Tém lai, tén tai x, dé cho 00. D&t X = (x1 ,%2,%3,%4,%5) va £(X) = £(%),,%2,%3,%4.%5) = (Xp —X5)? + (3 — 1)? + (Kg HQ)? HOK5 — 3)? HK] 4)? Ro rang f (X) > 0 va f (X) khéng thay déi khi thit tu cdc x, thay déi. Khi x3 <0, thi todn tit xdc dinh nhu da néi khi téc dong lén X sé bién X thanh Y = (x, ,x7—X3,—X3,%4 —X3,X5)- Tu dé, bing cdc tinh ton don gidn ta thu duge £(Y)—f(X) = 2x3(%) +X2 +x3 +X4 4X5) = 2x3 85-2. Néi cach khac, ett méi ln todn tit da cho tac dong lén X thi gia tri ham f (X) sé gidm di it nhdt 2 don vi. Vi vay, qua trinh nay phai dimg sau mét sé hitu han bude. Chui §: Van dé mau chit dé gidi bai todn & day 1a xdc dinh mét ham 5 bién cé gid tri bi chan duéi, ham nay phai déc lap véi cde hoan vi ctia 5 bién, va sau dé, dudi tac dong cda mét todn tit nao dé lén cac bién, gid tri ham sé gidm di. Khi dé, do gid tri cua ham bi chin dudi, thuat toan bit buéc phai dimg sau hitu han buféc. Ré rang Ja viéc xéc dinh mét ham nhuw thé khéng don gidn. Tuy nhién, true gide cho thay ring mét ham véi nhiing doi héi nhu trén 06 thé dude tim thdy nhimg Dang todn phuong (Quadratic form, cdc ban hoc sinh phé théng sé gp & Dai hoc - cu thé, 8 mén Dai sé tuyén tinh). 252 John Webb (Khoa Ton va Toan ting dung, Dai hoc Cape Town, Nam Phi - cf. South Africa and the 36” International Muthematical Olympiad) da dua ra dang toan phuong sau: £(% 4.9.3 5%4.%5)= D QUIN; éday a khii=j Qi =4b khi|i-jl=1 c trong cdc trudng hop khac. Va sau nhimng tinh todn, rét dai dong, dé dang toan phuong trén tat cho viée gidi bai toan, ta cAn chon a, b, c mt cach thich hop, u thé: a=1,b=Ovac=-I. Bai 104. (1986) Goi n 1a mét sé nguyén duong, n> 5, xét da gidc déu tam O, n dinh. Cho A, B la hai dinh ké nhau cua da gidc. Mét tam giac XYZ chuyén déng trong mat phing nhung luén bing tam gidc OAB, cé vi tri ban dau tring véi tam gidc OAB, sao cho hai diém Y va Z thi chay trén khdp ca bién (cdc canh) ctia da giac, con X thi nim bén trong da gidc. Tim quj tich cua X. Let A, B be adjacent vertices of a regular n-gon (n> 5) with center O. A triangle XYZ, which is congruent to and initially coincides with OAB, moves in the plane in such a way that Y and Z each trace out the whole boundary of the polygon, with X remaining inside the polygon. Find the locus of X. Huéng dan: Cho AB = 2 va M Ia trung diém AB. Ta lap hé truc toa do vuéng géc c6 géc tai A, truc x 1a truc AB (huéng duong tix A dén B), truc Ay cé hudng duong hudéng vao bén trong da gidc. Gia sit diém Z chuyén dong doc theo AB, huéng ti B dén A. Dat t 1a goc YZA, goi (x, y) 1a toa do diém X. Ta c6: yix=5-*, do dé XZ= vay = Xisin( ) a 2 on 7 x sint+cotg—cost . sin " Talaics BYsin2™ = YZsint = 2sint, MX = cotg”, tirdé suy ra n n 253 x= MY cost— BY cos +MXsint n = cost +(cotg -2cotg int = cost+tgsint = ytg™. n n n n Tit d6, ta suy ra quy tich diém X 1a mét hinh sao cdu tao béi n doan thing bit nguén tir diém O. X di chuyén tu O dén ddu mit kia cla mét doan thing réi quay tré lai O, réi di doc theo doan thing khac, tiép tuc nhu thé. Ta cé: 1 x+y? = + cos?| t+— |, 2% 2 sin COs’ n n au 1-cos vi vay, 46 dai mdi doan thang néi trén 14 7 : . a sin .cos - n n Bai 105. (1986) Ki hiéu R* 1a tap cdc 6 thc kh6ng am, hay xdc dinh tat cd cdc ham f: R* -> R* théa man cdc diéu kién sau: a) {(2) = 0; b) flx) #0, v6i0 < x <2; o) f(xf(y))f(y) = f(x+y), Vx, yeR*. Find all functions f, defined on the non-negative real numbers and taking non-negative real values, such that: a) f(2) = 0; b) fix) #0, where 0 < x< 2; ) Lf MSOM)= f+ y), Vx, YER*. Huéng dan: Ta c6 0 = f(xf(2))f(2) =£(x+2) nén suy ra véi moi x > 2: flx) = 0. Ta cing 06 f(y)f((2—y)F(y)) = £(2) = 0, do dé, néu y <2 thi £(2-y)f(y)) =0, suy ra Q-wly22ety25% Bay gid, gid su tin tai yy nado dé dé f(y) > Faz Mic dé, ta 06 =o thé tim y, sao cho y,>yq va y, <2 4é cho: 254 2 f(y) =5——. oy Dat x; =2—yj, ta cd f(x;F(¥o)) = £(2) =0, do dé: f(x, + yg) = 0. Nhumg mat khac ta lai cé x, + yg <2. Ré rang mau thudn da xy ra. Vay khong thé tén tai yp ndo dé dé f(yg)> 2-yq Tom lai, néu cé mot ham f théa main cdc diéu kién cia dé bai thi fix) = 0 vi moi x > 2 va véi moi x <2 ta 66: (x)= 5. -x Dao lai, mét ham f dude xac dinh nhu trén sé théa man cac diéu kién 6 dé bai. That vay, dé thay f(2) = 0 va flx) #0 véi 0 2, dodo -y fix +y) =O va tirdé suy ra f(xf(y))f(y) = F(x + y) nhu dé bai dai hai. Néu 2% <2 thita ciing 06 2-y 2 2 2 f(xf(y))f(y) = a =f(x+y). (xf(yF) W% Q-y 2-x-y (x+y) 2-y Tom lai, ham duy nhdt thoa man diéu kién bai toan 1a 0 = khix>2 f(x) = OO") 2 iogx <2. -x Bai 106. (1986) Trong mat phdng toa dé, cho mét tap hiu han cac diém cé toa dé nguyén. Hi ring, cé phai ta luén luén cé thé t6 mau dé mét sé diém cda tap hgp nay, va sé con lai duge t6 mau xanh, sao cho véi bat ki dutng thang L nao song song véi mét trong hai truc toa dé thi sy khac nhau (vé gid tri tuyét déi) cia sé diém mau xanh va sé diém mau dé trén L sé khong én hon 1 ? Hay ching minh cho cau tr ldi cua ban. One is given a finite set of points in the plane, each point having integer coordinates. Is it always possible to color some of the points in the 255 set red and the remaining points green in such a way that for any straight line L parallel to either one of the coordinate axes the difference (in absolute value) between the numbers of green point and red points on L is not greater than 1? Prove for your answer. Huéng dan: Goi T la tap hitu han cac diém cé toa dé nguyén da cho 6 dé bai. Xét mot dudng thing L tay ¥ song song véi mét trong cac truc toa dé va cdt tap hop T theo thit tu tai céc diém Ay, Ay, ..., Ay (thit tof ti trai sang phi, hode tit dudi lén trén). Néi A, va Ap, Ay va Ay... . Caing lam nhu vay véi mét dudng thing L khac. Khi dé, ta duge mét ho cdc doan thing va méi diém cia T déu thudc khéng qua hai doan. Vi vay, ta duige cdc dudng gap khuic khéng c6 dinh chung. Cac dung gap khiic déng nay gdm mét sé chin cdc doan. Ta c6 thé t6 mau xen ké: dé, xanh, dé, xanh... déi véi mdi dudng gap khic. Céc diém rdi rac khdc khong thuéc dudng gp khtic nao thi ta t6 mau tay y. Ta dugc mét cach té mau théa man diéu kién dau bai vi cdc diém nim trén céc dudng song song véi cac truc toa dé duge néi véi nhau béi cdc doan ma cde dau mut dau va cudi cé mau khac nhau. Tém lai, ta cé thé t6 mau nhu bai todn dai héi. Bai 107. (1987) Cho S la tap hep {1,2,...,n},n 21. Ta goi p,(k) 14 sé cde hodn vi a cia S cé ding k diém cé dinh. Ching minh ring: )'k.p,(k) =n! k=0 Let p,(k) be the number of permutations of the set (1, 2, 3, ... , n} 0 which have exactly k fixed points. Prove that )’ kp, (k) =n!. k=0 Huéng dan: Truéc hét, nhc lai ring mét hodn vi cia tap {1,2,....n},n>1 1a mét song anh f: {1,2,....n} > {1,2,...,n}, va néu f ck diém bat dong, ta néi hoan vi dé cé k diém cé dinh. Ngoai ra, ta cdn c6 céng thie nck} =kCK, tirdé xom =Ck néuk # 0. 1 Tré lai bai toan, ta dé y ring: p,(k)=Cfp,,(0), )ipa(k) =n! . k=0 Tw do suy ra: n n n n-l Dk pak) = LKCHP p10) =P DCH Pa 4 (0) = Ch pPp-n-1(0) k=0 k=0 =0 k=0 al =n Ypy-s(k) =n(n -1)!=n! . k=0 Céch khdc: Ung véi méi hoan vi ta viét vects (d), di, ..., d,) ao cho d, = 1 néu i thuéc S 14 diém cé dinh cha hodn vi da cho va sao cho d, = 0 trong trudng hgp trai lai. Vi sé hon vi 1a n! nén ta viét due 1! vects. Ta dém sé don vi ting quat trong tat cd cdc vecto nay bing hai cach khaéc nhau. Sé vects, trong cach viét ma c6 dting k don vi, bing p,(I0), vi thé sé : don vi téng quat trong tat ca cac vects bing kp, (k). k=0 Mat khac, sé vects irong dé c6 don vi vi tri thiti, bing (n - I! . Vay sé don vi 6 vi tri thiti trong tat ca cdc vectd bing (n - 1)! va sé tong quat tat ca don vi trong tat ca cac vectd sé bing n(n - 1)! = n!. a Vay )ik.pa(k) =n! . k=0 Bai 108. (1987) Cho x, ,X9,....X, 1a cde sé thue thod man diéu kién x? 4x3 tt x? =1. Ching minh ring véi mi sé nguyén k, véi k > 2, luén tén tai cdc sé nguyén aj ,a2,...,a, khong dong thii bang 0 sao cho véi moi i = 1, 2, ..., vn 1 nta cd |a;|Sk—1 va [ape tax) +..+a,x, [S(kK-)— kee ae 2 Let x, ,X2,...., be real numbers satisfying x} +x} +...+x? =1. Prove that for every integer k > 2, there are integers a, ,d),...4,, not all 0, such that |a,|ox? va gid thiét i-t isl x? +x3 +...4x2 =1 ta dé dang ching minh dude [xy 4 1x9 [4..41x, 1s Va. Bay gid, véi cdc b; nhan gid tri nguyén thudc doan [0,k -1], ta : xét k" gid tri cd dang "b,x; . Mdi gid tri dé phai nim trong doan isl [0, (k -1)vn]. Ta chia doan nay thanh k" —1 doan con cé dé dai bing vn k"-1" tri néi trén roi vao cing mét doan con. Cu thé, néu hai gid tri do la 0 a DLbjx; va } bfx; thi ta phai 6 a Dlbj -b); nhau la (k-1) Khi do, theo Nguyén tac Dirichlet, phai c6 2 gid S(k-I a a =| bia; — bis; isl isl suy ra diéu phai ching minh. Bai 109. (1987) Ching minh ring khéng thé tén tai mét ham f nao tit tap cdc sé nguyén khéng 4m vao chinh né sao cho véi moi n: f(f(n)) =n + 1987. Prove that there is no function f from the set of non-negative inte- -gers into itself such that fif{n)) = n + 1987 for all n. Huéng dan: Gia str nguoc lai, tén tai mét ham f nhu da néi 6 dé bai. Khi do, néu f(n) = flm) thi n + 1987 = f?(n) = £?(m) = f(n +1987), do dé, m =n, nghia la f don anh. Ta cé: £3(n) = f(£2(n)) = f(n +1987), £?(n) = £2(£(n)) = f(n) + 1987, suy ra f(n +1987) = f(n) +1987. @) M&t khac, bing quy nap, ta dé dang chimg minh duge £(n+1987q) = f(n) +1987q (q > 0). oo) Dat A= 10, 1, 2, ..., 1986 |. Bay gid, chon N. D&t {(n) =1987q +r, véi re N,0 n + 1987 = 1987q + flr) = 1987q, diéu nay ching td q € { 0, 1}. Vay fn) < 1986 + 1987 véi moi n. Tiép dén, ta dat A, ={neA :0< f(n) <1986}, Az ={ne A : 1987 < f(n) < 1986 +1987 }. Khi dé, tacé6 A= A, UA va Ay NA, =2. Vi Acé sé phan ti 1é, nén dén day, gid sit ban dau cia ching ta sé mau thudn séu ta thiét lap duoc mét song anh gita A, va A2. (Aj va A, cé sé phan ti chin 1é khac nhau. (Xem thém 4 phin Kién thite b6 trg - Ban sé (cardinality) cia tép hgp). Xét thu hep cua flén Ay, ta 6: f: Ay —> Az 1a mot song anh. That vay, truéc hét, néun ¢ A, thi f(n) € A va f{f{n)) = n + 1987 nén f{n) € A, nhu thé, dinh nghia nh xa trén 1a hop li. Ngoai ra, theo chiing minh trén, thu hep nay 14 mét den anh. Dé két thuc bai todn, ta sé ching minh ring f 1a toan anh. Cho ne A, tuyy, khi dé fln) -1987 eA. TH (*) ta duge f(m) = f(m—1987) +1987, véi m2 1987 va tif fln)> 1987 ta duge: f(f(n) — 1987) = f(f(n)) —1987 =n. Vay f(n)—1987 € A, va anh cla né qua f 1a n, suy raf: Aj >A, la toan anh, diéu phai chtmg minh. Bai 110. (1887) Cho n la mét sé nguyén lén hon hoc bing 3. Ching minh ring tén tai mét tap hop gm n diém trong mat phing sao cho khodng cach gidia hai diém tuy ¥ trong tap hgp nay 14 mét sé v6 ti va bat ki 3 diém nao trong chung cing xdc dinh mét tam gidc khéng suy bién (non- degenerate) cé dién tich hitu ti. Let n be an integer greater than or equal to 3. Prove that there is a set of n points in the plane such that the distance between any two points is irrational and each set of 3 points determines a non-degenerate triangle with rational area. Huéng dén: Trong mat phing, ta goi x,, 1a diém cé toa dé (n,n”), voin = 1, 259 2, 3, .... Ta sé ching minh riing khong céch gida hai diém bat ki nh thé 14 mot sé v6 ti, va tam gid duge xdc dinh bdi 3 diém bat ki trong chung 1a mét sé hitu ti khac 0. Lay n > m. Ta cé |x, —X,,| 1 dé dai canh huyén cia mét tam gidc cé hai canh géc vuéng 14. n—m va n?—m? =(n~m)(n+m), do dd [Xp —Xm | = (n—m)yl-+ (+m)? . Ta lai ed: (n+m)? <(n+m)? +1<(n+m+41)?=(n+m)? +14 2(n +m) nén (n+m)? +1 khéng phai 1a sé chinh phuong, va do dé cin ca né 1a sé v6 ti, . Dé thdy diéu vita néi trén, ta gid sti M khong phai 1a sé chinh phuong, va gid stt JM 1A sé hitu ti. Vi M khong chinh phuong nén ta cd thé tim due sé nguyén té p sao cho p’**! chia hét M nhung p2**? khéng chia hét M, via = 0. reat = © enikni as YN =". cing 1a 6 hit th. Do a5, P P tin tai sé nguyén té q sao cho q chia hét N, nhung q” khéng chia hét N. r. Cho VN =", véi x va s la hai sé nguyén té cing nhau, ta 06 s7N s Bay gid, q chia hét r, suy ra q” chia hét r”, do dé q chia hét s?. Tir dé, q chia hét s. Do vay r va s cé thiza sé chung, diéu nay mau thudn. Tém lai, sé khong chinh phuong khong thé cé c&in bac 2 1a sé hitu ti. Dé tiép tuc gidi bai todn, ta ldy 3 sé a, b, csao cho a 2. Ching minh ring néu k? +k +n 1a mét sé nguyén té6 véi moi sé nguyén k thod man osks thi k? +k +n sé 1a sé nguyén té véi moi k thoa man 0 0, sao cho 2c < b. Lic dé, s6 2!*'c 14 mét trong cdc sé b + 1, b + 2, ..., 2b - 1, 2b, do vay 2i*le nguyén té cing nhau véi m, va suy ra c ciing thé. Néu ta cé (2b +1)? > m thi ta cé thé két Iudn ring m 1a sé nguyén té, bai vi néu nd 1 hgp sé thi né sé cé mét thita sé < Vm. Dat n= 3r? +h, void N, va N thi nguyén t cing nhau véi tat cd cdc sr ++ 1r+s+2,...,2rt2s. Ta cd (2r+2s +1)? = 4r? +8184 4s” + 4r +4541. Vir,s > 1nén tacéd4s+1>s+2, 4s” >s? va Grs > 3r. Suy ra (2r+2s +1)? > 4r? + 21s+s¢4+7r+s+2 =3r2 +6r+24(rts\(r+stl2N. Bay gid, néu N cé mét thita s6 chia hét 2r—i, vdi i thude cde 86 ti —2s dén r—s—1 thi: N-(i+2s+1)(2r—i) =n + (r-i-s—Iy(r-i-s), tric la 6 dang n+s'(s't1), véi s' la sé tx 0 dén r+s—-1. Nhung n+s‘(s'+1) 14 s6 nguyén té theo ching minh quy nap, do 46, cach duy nhdt dé'né cé mét thiza sé chia hét 2r—i 14 chinh né phai chia hét 2r—i. Mat khac, ta cd 261 2r-i< 2r+2s < 2n—4<2n, n+s\(s'+1)>N, do vay, néu n+s'(s'+1) c6 thita sé chung véi 2r—i thi nd sé bing 2r—i =s+r+1+s'. Suyra s =s—(n-r—1)<0, diéu nay khong thé xdy ra duge. nu thé, ta di dén két luan ring N nguyén té cing nhau véi tat cd cdc sér+s +1, ..., 2r + 2s, va do vay N 1a sé nguyén té, diéu phai ching minh. Bai 112. (1988) Cho n la sé nguyén duong va A;, Az, ..., Az, ,1 1a cdc tap hgp con cua tap hop B. Gia si ring: i) Méi tap hop A, (i = 1, 2,..., 2n + 1) chia duing 2n phan tit. ii) Véi méi cp tap hop khae nhau A, va A,, Aj OA, chi chita ding mét phan ti. iii) Mdi phn tu cua B thuéc it nhdt hai tap hop A, (i = 1, 2...., 2n+1). V6i nhing gi tri nao cia n thi ta cé thé gin cho méi phan tit cia B gid tri 0 hodc 1 sao cho trong mai tap hgp A, cé ding n phan tir duge gan gid tri 0? Let n be a positive integer and let A, Az, ..., Am. 1 be subsets of a set B. Suppose that i) Each A, has exactly 2n elements, ii) Each A, A;, 1> is a union of disjoint intervals, the x-l x-2° x-3 x-70° 4 sum of whose lengths is 1988. Huéng dén: 7 1 2 3 7 5 Taco: —~—+—~—+—— +...+ == x-l x-2 x-3 x-70 4 70 7 Dk T]«-) k=1 1k T]&-)-5 T]@-) k=I 1 0, p(69) = 4.69.(69 — 1)(69 — 2)...(69 — 68)(69 - 70) <0, p(3) = 4.3.3 —1)3 —2)(3 — 4)... - 69)(3 - 70) > 0, p(2) = 4.2.(2—1)(2-3)(2- 4)...(2— 69)(2- 70) > 0, p(l) = 4.1.(1-2)(1-3)(1 — 4)... - 69)(1- 70) < 0. Tit d6 Dinh li gid tri trung gian cho ta 69 khéng diém cia p(x); 265 ngoai ra, vi p(x) co hé sé cia hang ti chia sé mi lén nhat la —5 nén né c6 thém mét khéng diém khéc > 70. Bay gid, ta c6 es 20 khi va chi khi p(x) va q(x) cing ddu, diéu nay xay ra 6 cdc khoang (t6 dam) trén dé thi nhu sau, khi ta vé p(x) va q(x) trén cing mét hé truc: Dé thuan tién, taki hiéu S=14+2+...470. Téng dé dai cdc khong (t6 dam) bing téng cua céc diém 6 dau mit bén phai cia ching tri di téng cia cdc diém & dau mit bén trdi, tic 1a tng ede khong diém cia p(x) trit cho S. Dé thy ting cde khéng diém cia p(x) li =. Vi vay, tng dé dai cdc khoang can tinh 1a 98 _¢_45_ 4 70.71 _ diéu phai chimg minh. Bai 115. (1988) Cho a va b 1a hai sé nguyén dugng sao cho ab + 1 chia hét 2 2 a’ +b ab+ Let a and b be positive integers such that ab + 1 divides a” +b”. a+b? a? +b”. Chimg minh ring 1a mét sé chinh phuong. Show that is the square of an integer. ab+ Huéng dan: Gia str ngugce lai ring Daghbe ~ab+l khéng phai 1a sé chinh phuong. Bién déi (*) ta duce: a? +b? —kab=k. (**) @) 266 Vi a va b trong ding thitc (**) c6 vai tro déi xing nén khong mat tinh téng quat, ta gid stra > b. Xem (**) nhu 1A mét phuong trinh bac hai déi véi a, né cé hai nghiém Ia a va a,. Ta cé a, cing la sé nguyén vi theo Dinh li Viet a + a, = kb, trong dé a va kb 1a hai sé nguyén. Néu a, = 0 thi k = b’, trai véi diéu gid sit trén. Néu a, < 0 thi vé trai cuia ding thtic (**) khi thay a bing a, sé lén hon vé phai. Vay a, > 0, 2 25 theo Dinh li Viet ta cé: a, = 2 <9 = ca Chob,=b, ta duge mot a a nghiém mdi (a,, b,) ma a, + b, a,+b,>a,+b,>.... Didéu nay mau thudn vi né khong thé xay ra trong pham vi 36 tu nhién. Vay k phai la sé chinh phuong, diéu phai ching minh. Bai 116. (1989) Chimg minh ring tap hop {1,2,3,...,.1989} 06 thé duge viét thanh hdi cua cdc tap con réi nhau Ay, Az,...,A};7 sao cho mo A; 1, 2, ..., 117, déu c6 chifa 17 phan ti va téng gid tri cdc phan ti cla nhimg A, aéu bing nhau. Prove that the set {1,2,3,...,1989 } can be expressed as the disjoint union of subsets A, Ay ....Ay17 such that: i) Each A, conta‘ns 17 elements; ii) The sum of all the elements in each A, is the same. Huéng dan: Trude hét, ta x4y dung 117 tap hgp gém 3 sé sao cho téng cia 3 sé dé trong m3i tap déu bing 0 va chung ri nhau ting déi mét nhu sau. Ti tap {1,2,3,...,1989 }, ta tao thanh tap gdm 348 sé: M = {- 994, - 993,...,993, 994}, tap Egp nay c6 dude bing cach lay ting sé hang cia di 995. Khi dé, ta tao 116 tap hgp gm 3 s6 néi trén la: Ap hgp da cho trit 267 N, = {993,-496,—497}, N2 = {-993,496,497}, Nays, = {993 — 4k ,2k — 496, 2k — 497}, Nox+2 = {-993+ 4k ,.— 2k + 496,— 2k +497}, Nuys = {665.-382,—383}, Nyy = {- 665,382,383}. Ngoai ra, ta dat N,,7 ={—-1,0,1}. Tat cd 117 tap hop trén déu réi nhau ting déi mét. That vay, trong méi tap, do cdc phan ti thi hai déu chin nén cdc phan tit tht hai cua cdc tap hop N,,..., Nye khong thé tring véi céc phan ti thi nhat hodc thi ba cia nhung tap hop nay, tat ca cac phan tit thi nhdt cia nhing tap hop nay cé gid tri tuyét déi lon hon tat cd phan ti thi ba, thanh thi cdc tap hop N, réi nhau ting doi mét. Ngoai ra, néu s6 x nao dé 1a phan tit cia mot trong cdc tap hop N, thi sé ( - x) cing 1a phan ti cia mot trong cdc tap hop N,. Dé y ring 14.117 phan ti cia tap hop M, khéng thuéc vé mét trong cdc tap hop N,, dude chia thanh 7.117 c4p sé véi déu ddi nhau. Bang cach tay y ta thém 7 cdp sé phan biét vao tap hgp N, da chon & trén, ta sé chia ducc tap hop M thanh 117 tap hop con tig cap khong giao nhau. Cuédi cing dé thod man yéu cau cia bai todn, ta chi can xay dung 117 tap A, bing cach cong 995 vao ting phan tit cia cdc tap N, tuong ting. Chi: §: Thong thudng, khong phai chi vdi cdc ki thi IMO ma ca véi nhing ki thi v6 dich todn khac trén thé gidi, nguéi ta co khuynh huéng thiét lép cdc dé todn ma cde con sé cé gén vdi ném thi (ching han, nam 2000 thi c6 sé 2000, nim 1978 thi cé sé 1978). Tat nhién khéng phai bao gid cang vay, nhung ngudi ta sé lam diéu dé néu nhu cé thé duge. Xem nhu thé, thi cic dé todn c6 gén vdi nhing sé ndém thi tugng trung kia thong thuting duoc dac biét hod tit mét bai todn téng quat hon. Vé phia ching ta, néu tim ra bai todn téng quat ti cdc dé todn nay thi do 1a viée 1am v6 cing thii vi va bé ich. Trong truéng hgp nay, ta c6 bai toan téng quat sau: Chuing minh rang tap hop {1,2,3,....n} c6 thé duge viét thank hi ctia ede tap rdi nhau Ay, A2.....Am, vdi m Id ude $6 ctia n, sao cho 1, 2 phén tt cia nhitng A, déu bang nhau. moi Aj, i m, déu cé chifa n/m phén ni vd tong gid tri cde 268 Bai 117. (1989) Cho n va k 1a cdc sé nguyén duong, S 1a tap n diém trong mat phang thoa man cac diéu kién: i) khéng cé 3 diém nao trong S thing hang; ii) véi moi diém P thuéc S, tén tai it nhdt k diém trong S cach déu P. Ching minh ring k < ; +V2n. Let n and k be positive integers and let S be a set of n points in the plane such that i) no three points of S are collinear, and ii) for any point P of S there are at least k points of S equidistant from P. Prove that: k < ; +V2n. Huéng dan: Ta gid sit nguge lai ring k > 3 Pn. Lay mét diém P trong S, hic dé, tin tai it nhdt k diém trong S cdch déu P, Nhu vay, tén tai it nhdt Cz cap diém A, B ma AP = BP. Vi digu nay xdy ra véi moi P thuéc S, nén cd it nhdt nCj cAp diém (A, B) cd thif ty sao cho phat biéu sau day 1a ding: trén dudng trung truc cia AB c6 it nhdt mét diém ciia S. Ta cb: act = n =D 23(van +4] van—2) 1 1 =3(2-1) _ o{n= 3) >ma=0 =2C8. Vi C? 1a sé tat cd cae ep diém cia S, cn 2C? 1a sé tat ca cae c&p diém cé thif tu cua tap hgp S (xem lai chinh hop - arrangement) nén tit bit ding thic trén, theo Nguyén téc Dirichlet, suy ra ring bit buéc phai tén tai mét cp diém A, B va cac diém P,, P,, P; sao cho: AP, = BP, i= 1, 2,3. Nhumg khi dé 3 diém P,, P,, P; thing hang, diéu nay mau thudn véi gid thiét 4 dé bai. 269 Bai 118. (1989) Chiing minh ring vi moi sé nguyén dutong n, ton tai n sé nguyén duong lién tiép sao cho khéng cé sé nao trong chung 1a sé nguyén to hoac luy thita cua mét sé nguyén t6. Prove that for each positive integer n there exist n consecutive positive integers none of which is a prime or a prime power. Huéng dan: Cho truéc sé nguyén duong n, ta hay tim cach chon sé N thich hgp sao cho N + 1,N+2,..., N+n.c6 tinh chat nhu dé bai dai hdi. Muén vay, ta dat N= ((n+I)!)? +1. Lic dé, dé thdy 86.14 51a mét uéc sé thuc su cia N + j, véi moi j = 1, 2, .... nnénnsoN + 1,N+2, ., N+ n khong phi la sé nguyén Bay gid, gia sit N +j = p™, véi p nguyén t6 va m > 1. Lic dé, ton tai sé nguyén duong r, vdi 1 < r < m sao cho 1+ j=p". Nhung khi do ta 6 pl! | (n+1)!)? va p'*! | p™. Vi vay: p'"1(N-1) va pi (N+ j). Tu dé ta suy ra p™*"|(1+ j), diéu nay mau thudn, ta c6 diéu phai ching minh. Bai 119. (1989) Mét hodn vj { x;, X2y---) Xo } cba tap hgp {1, 2,..., 2n} dude goi 1a c6 tinh chét P, trong dé n 1a mét sé nguyén duong, néu |x; - x;,,|=n véi it nhat mét i thuéc {1,2,...,2n —1}. Ching minh ring véi mai n, sé cdc hodn vi cé tinh chat P 1én han sé cdc hon vi khong c6 tinh chat d6. A permutation { x) X2...; X2n | of the set (1, 2, ..., 2n} where n isa positive integer is said to have property P if |x, — x,,,|=1 for at least one i in (1, 2, ..., 2n-1]. Show that for each n there are more permutations with property P than without. Huéng dan: Ta chia cdc sé 1, 2, ..., 2n thanh tig cap nhu sau: (1, n+ D, (2,1 + 2),..., (n, 2n). Bay gid ta thiét lap mot tuong tng tu tap hop cdc hoan vi khong cé tinh chat P vao tap hgp cac hodn vi cé tinh chat P bing cach sau day: Gia Sif (X10) Xe-15 Xey Xk +9 > Xn | 1 hodin vi bat ki khéng c6 tinh chat P, va gid st x, 1a sd thudc cing mét cip véi s6 x,,, k <2n-2. Khi dé 270 ~ hodn vi khéng cé tinh chdt P nay sé dude dat tuong tng vai hodn vi cé “tinh chdt PAA ( X,y +5 %y-1+ Xi Xam» Kon a9 -- Xe v1}. Ta goi f la tong ting xéc dinh nhu thé. Dé dang thy rang tuong ting f 1a mét anh xa. Ban doc tu chig minh hai kh4ng dinh sau: Véi anh xa f nay, cde hoan vi khéng cé tinh chdt P ma khéc nhau sé bién thanh cac hoan vi cé tinh chdt P cing khac nhau, diéu nay co nghia f la don anh. Mat khac, khong phai hodn vi co tinh chdt P nao cing 1a anh qua f cha hon vi khéng cé tinh chdt P. Néi cdch khac, f khong phai la toan anh. Diéu dé chimg td ring sé cdc hodn cé tinh chdt P 16n hon sé cdc hoan vj khéng cé tinh chdt P. Chit 5: 1) Xem thém Ban sé (cardinality) cia tap hop 6 phan Kién thie 66 trg cho huéng dan trén. 2) Ki higu 1A1 1a ban sé cia tap hgp A (khi A hau han, {Al 1a sé tt cA cdc phan tit cia A). Huéng dan gidi sau day dude trich tit Arthur Engel, Problem-Solving Strategies, Springer 1998 [ Problem books in mathematics series ]: Goi Aj, 1a tap tat ca nhiing hon vi sao cho trong cac hoan vi dé, hai phan tit k va k + n dig ké nhau. Goi A 1a tap tat cd nhing hodn vi cé tinh chat P. Dé thay A= UA . Khi do: IAI ZIAI-ZIAC Ault DIAL OAR Am I=. « k SO" suy ra diéu phai chimg minh. Bai 120. (1990) Cho sé nguyén n > 3 va xét tap hop E gom 2n-1 diém phan biét nim trén mét dung tron. Gid sit trong sé cdc diém nay c6 ding k diém 271 dugc t6 mau den. Mét cach t6 mau nhu thé duge goi 1a hidu t6 t6t néu tén tai it nhdt mét c&p diém mau den sao cho phan trong céa mot trong hai cung tao béi hai diém nay chia ding n diém cia tap E. (Ta néi phan trong - interior - cha mot cung la cac diém cua cung dé trit hai diém dau cung). Hay xdc dinh gid tri k nhé nhat sao cho moi cach to mau k diém céa E nhu thé déu #é¢. Take n > 3 and consider a set E of 2n-1 distinct points on a circle. Suppose that exactly k of these points are to be colored black. Such a coloring is “good” if there is at least one pair of black points such that the interior of one of the arcs between them contains exactly n points from E. Find the smallest value of k so that every such coloring of k points of E is good. Huéng dan: Theo chiéu kim ding hé trén dudng tron, ta ki hiéu 2n - 1 diém da cho lan luot béi 0, 1, 2, ..., 2n - 2. Dat K = (0, 1, 2, ..., 2n - 2}. Véi m nguyén ta déng nhdt m véi i thuéc K khi_m =i (mod 2n - 1). Ta dinh nghia quan hé i * j déi véi hai phan tit i va j (thudc K) néu ta cd n+l Ré rang ta cé i * j khi va chi khi mot trong hai phan trong cua cung tao thanh chifa ding n diém cia E. Néu k la mét sé théa man tinh chat cha dau bai thi ta néi don gidn ring k Jd t6t. Tit d6, k 14 t6¢ khi va chi khi véi moi tap con H cua K gém k phan tu, ton tai hai phan ti cia H cé quan hé * véi nhau. Ta sé chimg minh: a) sén la tt, b) néu 2n - 1 khong chia hét cho 3 thi n - 1 khéng phi tét, c) néu 2n - 1 chia hét cho 3 thi n - 118 ét. Chitng minh (a) : Xét tap con H cia K gém n phan tit. Gid sit trong H khéng cé cp phan ttt nao cé quan hé * véi nhau. Véi j thuée H, ta ki hiéu K(j) la tap hop tat ca cdc phan tu (cua K) cé quan hé * véi j. ‘Thé thi K(j) va H khong giao nhau véi moi j thudc H. Vi méi phan tit cd m&t nhiéu nhat 6 trong hai K(j) nén hgp cdc K(j) (véi j chay khdp H) cé it nhat n phan té, diéu nay mau thudn vdi viée H cé n phan tt. Chuing minh (b): D& ching minh khang dinh (b), ta chi can xay dung mét tap con L cua K sao cho L gém n - 1 phan tit va khéng c6 cip 272 nao cé quan ké * voi nhau. Xét cdc tap L = {3k|k =0.1,....n-2} M = {3k +n-2/k =0.1,...n-1}= {3(n-1-k) +n-2/k =0,1,....9-1} = {4n-5-3k|k =0,1,....n—1}= {3(-1-k)[k = 0,1,....n-1}. Vi 2n- 1 khong chia hét cho 3 nén ta suy ra LUM = {3k|k =-n,-n4l,...n-2} =K. Ti do ta cé L gém n - 1 phan tu, M gém n phan tii, L va M cé giao bing rang. Dé ¥ riing véi j thudc L, j = 3k trong dé 0 2. Thé thi 2n- 1 = 3(2m- 1), n-2=3(m- 1) van+ 1 = 3m, Néui va j (thudc K) cé quan hé * véi nhau thi i va j déng du véi nhau theo modulo 3. Ta viét: K, ={3k +r]k =0,1,...,.2m—2}(r=0,1,2). Nhu thé, K duge phan thanh ba tap con khéng giao nhau nhu K,, K,, Ky. Véi méi i thudc K, thi tiri thuée K, ta suy ra hai phan tit cé quan hé * véi i cing thuéc K,. Ta ching minh ring 3m - 2 1a dt. ‘That vay, néu A la tap con cua K va gém 3m - 2 phan ti thi theo nguyén tdc Drichlet, mot tap con dang ANK, cé it nhdt m phan tu. Tuong tu nhu ching minh 6 phan (a) ta di dén két ludn ring tén tai mét cap phan ti cé quan hé * véi nhau trong ANK, CA. Bay gié ta chting minh ring 3(m - 1) khong #6t. Muén thé, ta chi cAn xay dung mét tap con gom (m - 1) phan tit cia K, ma khong cé cip phan ti nao cé quan hé * véi nhau. Dat M, ={3k +r: k=0,1,2,...,m—2}, vir =0, 1, 2. Dé y ring i va j cé quan hé * véi nhau khi va chi khi __.,_[3¢m-) li-jl= [* , ta thay ngay khéng cé cp phan ti nao cé quan hé * véi nhau trong méi tap M,, M,, Mp. Tom lai, tit nhing diéu trén, ta suy ra gid tri k nhd nhat (Kimi) sao cho moi cach t6 mau k diém cia E nhu da néi 6 dé bai déu #60 1a: a enact 273 0 1 (mod3) n khi 2n-1¥3m,meN n khin ‘min = n-1 khi 2n-1=3m,meN~ ' n-1 khi n=2 (mod3) Bai 121. (1990) 241... Xéc dinh tat cA cdc sé nguyén n Ién hon 1 sao cho —;— 1a mét sé nguyén. . : "+1 Determine all integers n greater than 1 such that —z ~isan integer. Huéng dan: Gia sitn 1a sé théa man yéu cau cua dé bai. Vi 2" +1 1a sé 1é nén sé n thoa man dé bai phai 1a sé 1é. Goi p 14 ude sé nguyén té nhé nhat cia n. Cho x 1a sé nguyén dudng nhé nhat sao cho 2* =-1 (mod p), vay 1a sé nguyén duong nhé nhat sao cho 2° =1 (mod p). Sé y nhu thé chic chan tén tai va ta cé y < p, vi: 2° =1 (mod p), va x ciing tin tai vi 2" =-1 (mod p). Ta cé thé viét n = ys +r, voi < r0. Khi dé, néu h chin, ta gip mau thudn vi tinh bé nhat cia y; cing vay, néu h Jé thi do x bé nhat nén mau thudn. Vay ta cé k = 0, va x chia hét n. Nhung x < p, con p lai ld sé nguyén té nhé nhat chia hét n nén ta phai cé x = 1. Suy ra 2=—1 (mod p) va tu dé p = 3. Bay gid, ta gia str ring 3™ 1a ldy thita lén nhat cia 3 chia hét n. Ta sé ching minh ring m = 1. That vay, ding khai trién nhj thie Newton cho (3-1)" +1 ta dude: 1-14n3-1 n(n—193? +..=3n— Pa ng? +... 2 2 Hién nhién 3n chia hét cho3™*', nhung khong chia hét cho 3". Sé hang ting quat la trong khai trién trén 14 C2, 3°, 6 day ta viét n = 3™a vi theo trén ta cé 3 1a lay thia Idn nhdt cua 3 chia hét n, va ta 274 aetanecese xét khi b > 3. Cac hé sé nhi thiic déu 1a cdc sé nguyén nén sé hang tong quat nay chdc chan chia hét cho 3"* khi b > m+ 2. Ta cé thé viét hé sé nhi thtc dudi dang: gma 3h a1 3M =2 3M=-3 IP ~(b-1) bo 2 3 b-1 Khi b khong phai béi cda 3, sé hang dau tién trong tich trén c6 dang (*) 3" 5 , vi 3 héng chia hat ¢ hodc d, va cdc sé hang cdn lai thi cd dang 5 , véi 3 khéng chia hét ¢ hode d. Tit dé, néu b khéng phai béi céa 3 thi hé sé nhi thiic sé chia hét cho 3", vi b > 3, diéu nay cé nghia la sé hang (*) ndi trén chia hét cho 3"*?. Tuong tu, ta cing phan tich nhu thé khi b 1a béi ctia 3, va cudi cang, dé yb > 3, ta di dén két ludn ring sé hang (*) chia hét cho 3"*?. Tit phan tich trén, ta suy ra 3*" 1a lay thita cao nhat eda 3 chia nét 2" +1. Nhung 2" +1 chia hét cho n? va do vay chia hét cho 3°", tix dé ta cé m= 1. Tiép dén, ta cé thé kiém tra dé dang ring n = 3 14 mot dap s6 cia bai toan. Néu n > 3, ta dat n = 3t (theo trén, ta da chimg minh dude ring 3! la lay thita ln nhdt cia 3 chia hét n). Goi q 1a ude s6 nguyén t6 nhd nhit eta t. Gia st w 1a sé nguyén dudng nhé nhat sao cho 2“ =-1 (mod q) va v 1a sé nguyén duong nhé nhat sao cho 2° =1 (mod q). Sé v chdc chin tan tai va v 3, mau thudn! ‘Tom lai, ta da chimg minh duoc ring n = 3 1a sé nguyén duy nhat 2°41 n2 lén hon 1 dé cho 1a mét sé nguyén. Bai 122. (1990) Goi Q* 1a tap hop tat cA cac sé hitu ti duong. Hay xay dung mot ham sé f: Q* ——> Q* sao cho véi moi x, y € Q’: F(xf(y) = £2 y Construct a function from the set of positive rational numbers into itself such that f(x fly) = fx) /y for all x, y. Huéng dan: Truéc tién, ta ching minh f(1) = 1. That vay, lay x = y = 1 tac f(f())=1, suy ra f(1) =f(F(D) = FAf(F))) = ie . Tiép dén, ta sé ching minh ring f(xy) = f(x)f(y). Véi moi y € @. ta cd {un) 1=f()= Ld eo y= a : do dé, néu z= ~ thi f(z) = y. Tu do suy ra: (y) Flay) = fextte)) = #2 - goat). Sau cing, ta 6 f(f(x)) = F(1E(x)) = © au. x Bai todn khéng doi héi chting ta tim td¢ cd cdc ham ma chi yéu cdu tim mét ham théa man cdc diéu kién da cho. Do vay, ta phan tap cac sé nguyén té thanh 2 tap vo han: S = {p),p)....}, T= {q).42,-..}- Ta bat dau xay dung ham f théa man yéu cau bai todn bing dinh nghia 1 f(Pr)=an» f(Qa)=— in Sau d6, ching ta mé rng dinh nghia nay trén tap cdc sé hau ti bing cach dung cac hé thu f(xy) = f(x)f(y): Pi,Pi -_)_ PmPm, 7 Px, Pk2-~-Am,4mz--) Pj, Pj, Ak, Ik ‘Thi lai, ta thay ham vita xay dung théa man: f foxy) = = : Bai 123. (1990) Cho truéc mét sé nguyén n, >! khéi dau. Hai ngudi A va B choi mét trd chdi, ho luan phién chon cdc sé nguyén nj,n2,Nn3,... theo quy luat nhu sau: Truéc tién, A chon n, théa man diéu kién ng 1. Téng quat, ta cé trinh ty tiép theo: Néu biét ny, 14 sé nguyén vita duc B chon thi A chon sé nguyén tay ¥ n,,) nao dé théa man diéu kién ny, \, two players A and B choose integers m,n, ,ny,...alternately according to the following rules: Knowing ny, A chooses any integer nx,, such that ny, Sny,, Sry. Knowing he MK Nx%1, B chooses any integer ny,,. such that =p’ for some prime p and integer r >1. Player A wins the game by choosing the number 1990; player B wins by choosing the number 1. For which no does a) A have a winning strategy? b) B have a winning strategy? ©) Neither player have a winning strategy? Huéng dan: Cau tra ldi la: a) néu ng > 8 thi A thing; b) néu np = 2, 3, 4,5 thi B thing; c) néu ng = 6, 7 thi hai déi thi héa nhau. a) Chién luge cua A trong viéc chon sé khi biét truéc sé n nhu sau: (1) néu n €[8, 11], A chon sé 60; (2)néu n €[12, 16], A chon sé 140; (3) néu n €[17, 22], A chon sé 280; (4) néu n €[23, 44], A chon sé 504; (5) néu n €[45, 1990], A chon sé 1990; (6) néu n = 1991 = 11.181 (181 14 sé nguyén té), A chon sé 1991; (7) néu n € (LI A8L+ LI LIST. voi r > 0, A chon sé HIT! 181. Ro Néu Ub xdy ra, sau (4), B chi c6 thé chon duoc mét trong cac sé 56, 63. 72, 168, vi vay. A thang 6 buéc (5). Néu (3) xdy ra, sau (3), B chi c6 thé chon dude mét trong cac sé 35, 10, 56, 70, 140. vi vay. A chic chan thing béi bude (4) va (5). Néu (2) xy ra, sau (2), B chi c6 thé chon duoc mét trong cac sé 20, 28, 35, 70, vi vay, A chac chan thang bdi cae bude (3), (4) va (5). Néu (1) xay ra, sau (1), B chi cé thé chon dude mét trong cac sé 12, 15, 20, 30, vi vay, A che chin thing béi cdc buéc (2), (3), (4) va (5). ang néu c6 (5), A lap tuc thing cudc. Néu B duge biét sé 11'"!.181 sau khi A da chon sé nay, B chi c6 thé chon dude mét trong cdc sé 181, 11.181, ..., II" 181 va LIT", tat ca nhing sé nay déu < 11'.181. Vi vay, néu A duoc biét cac sé n nhu trong (6) va (7), thi chién thuat 6 (6) hodc (7) cho thay A da quay lai chon sé bé hon n(va > 11), do dé A sé thang cuéc sau mét sé hitu han bude quay lai. b) Néu B duoc biét sé bé hon 6, anh ta sé cé thé chon dusc sé 1 va thing cudec. Ti do, néu ban dau A biét sé 2. anh ta thua ngay, béi vi anh ta sé phai chon sé < 5. Néu B duoc biét sé < 11, anh ta sé c6 thé chon duge sé 1 hode 2 va thang cuéc. Tu do, néu ban dau A biét sé 3. anh ta thua ngay, béi vi anh ta sé phai chon sé < 10. Néu B due biét sé < 19, anh ta sé c6 thé chon duc sé 1 hodc 2 hodc 3 va thang cuéc. Ti dé, néu ban dau A biét sé 4, anh ta thua ngay, béi vi anh ta sé phai chon s6 < 17. Néu B duoc biét cae sé < 29, anh ta sé cé thé chon due sé 1, 2, 3 hodc 4 va thang cuéc. Tw dé, néu ban dau A biét sé 5, anh ta thua ngay, 278 béi vi anh ta sé phai chon sé bé hon 25. c) Bay gid, xét truéng hgp A nhan dude cdc sé 6 hodc 7 (n, = 6 hodc n, = 7), ngudi chai A cé thé chon n, = 30 = 2. 3. 5 hode n, = 42 = 2. 3. 7, néu khong chon thé thi B sé thing (ching han, khi A chon 31, 32, 33, 34, 35 hay 36 thi B sé chon tuong img 1, 1, 3, 2, 5, 4 va B thing). Khi B déi dién véi sé 30 ma A chon, B phai chon mét trong 3 sé 6, 10, 15. Ta da biét, theo trén, ring B sé thua néu anh ta chon 10, 15, va thé thi ngudi choi B chon n, = 6. Can khi B déi dign véi sé 42 ma A chon, B phai chon mét trong 3 sé 6, 14, 21. Ta da biét, theo trén, ring B sé thua néu anh ta chon 14, 21, va thé thi ngudi chdi B chon n, = 6. Sau dé cd hai ngudi choi A va B luan phién chon cac sé 30, 6, 30, 6, 30, 6,... dé tranh bj thua va ho sé héa. Bai 124. (1990) Chiing minh ring tén tai mét da gidc 1éi 1990 canh ma tat ca cdc géc cla né déu bing nhau va cdc canh cé dé dai 1a nhiing sé 17, 2?,..., 19907 dude sip xép theo mét trat tu nao dé. Prove that there exists a convex 1990-gon such that all its angles are equal and the lengths of the sides are the numbers 1" , 2?,...,19907 in some order. Huéng dé Trong mat phing phi, ta cé thé biéu dién sé do cdc canh nhu da néi & dé bai bing cdc sé p2w", trong dé, p,, vain = 1,2, ..., 1990, 1a mot hodn vi cua céc sé 1, 2, .... 1990 va w IA cin bac 1990 cia don vi (xem muc sé phtic 6 phan Céc hién thitc bé tro). Dé ¥ ring 1990 1a tich cia nhiéu hon 2 sé nguyén té phan biét: 1990 = 2.5.1999. Vi thé, ta cé thé viét w = —1.a.b, trong dé —1 la cin bac 2 cia don vi, ala cin bac 5 ciia don vi va b la can bac 199 cia don vi. Bay gid, vdi mét trong cac cin bac 1990 cua don vi ndi trén, ta cd thé viét thanh (-1)'ai.b*, vi 00 thi n phai 1a sé nguyén t6 hodc la liy tha cua 2. Let n > 6 be an integer and let ay,d,...,44 be all the positive integers less than n and relatively prime ton. If dy - ay = a - a) = a, — ay, >0, prove that n must be either a prime number or a power of 2. Huéng dén: Ta co ngay a, =1 va a =n-1. Ta cing cé a =p vdi p 14 sé nguyén té nho nhat ma p khong la uéc cia n. Vay céng sai cia cdp sé cong {a,} la d=p-l. Néu n 1é thi a) =2 va d=1. Suy ra n nguyén té cing nhau véi moi s6 tu nhién nhé thua n. Vay n nguyén té. Néu n chin thi p>3. Tir do: * Khi p=3 thi d=2 ta dude cdp sé 1,3,5,...n—1. Vi véi moi s6 18 q 3 thi n chia hét cho 3 va a, =a)+d(k-I) nén n-1=1+(p-I)(k~1). Do vay n—2 chia hét cho p—1. Gid sit q 1a uéc nguyén té cua p—1 thi q cing la uéc cia sé n—2 vi q

You might also like